NCLEX Miscellaneous 10

Ace your homework & exams now with Quizwiz!

For which patient can the nurse safely delegate morning care to the nursing assistive personnel (NAP)? Assume an experienced NAP, and base your decision on patient condition. Assume there are no complications other than the conditions stated. 1) 32-year-old admitted with a closed head injury 2) 76-year-old admitted with septic shock 3) 62-year-old who underwent surgical repair of a bowel obstruction 2 days ago 4) 23-year-old admitted with an exacerbation of asthma with dyspnea on exertion

*3) 62-year-old who underwent surgical repair of a bowel obstruction 2 days ago* Morning care for the patient who underwent surgical repair of a bowel 2 days ago can be safely delegated to the nursing assistive personnel because the patient should be stable. The patient who sustained a closed head injury may develop increased intracranial pressure during care. Therefore, he requires the critical thinking skills of a registered nurse to perform his morning care safely. The patient admitted with septic shock may easily become unstable with care; therefore, a registered nurse is required to provide his morning care safely. The patient admitted with an exacerbation of asthma who becomes short of breath with activity also requires the critical thinking skills of a registered nurse to detect respiratory compromise quickly.

Which is a major factor regulating sleep? 1) Electrical impulses transmitted to the cerebellum 2) Level of sympathetic nervous system stimulation 3) Amount of sleep a person has become accustomed 4) Amount of light received through the eyes

*3) Amount of sleep a person has become accustomed to*

Comparing the changes in vital signs as a person ages, which statement(s) is/are correct? (Select all that apply.) 1) Blood pressure decreases less than heart rate and respiratory rate. 2) Respiratory rate remains fairly stable throughout a person's life. 3) Blood pressure increases; heart rate and respiratory rate decline. 4) Men have higher blood pressure than women until after menopause.

*3) Blood pressure increases; heart rate and respiratory rate decline.* *4) Men have higher blood pressure than women until after menopause.*

The nurse is teaching a client who sustained an ankle injury about cold application. Which instruction should the nurse include in the teaching plan? 1) Place the cold pack directly on the skin over the ankle. 2) Apply the cold pack to the ankle for 30 minutes at a time. 3) Check the skin frequently for extreme redness. 4) Keep the cold pack in place for at least 24 hours.

*3) Check the skin frequently for extreme redness.*

The patient is diagnosed with obstructive sleep apnea. Identify the symptoms you would expect the client to exhibit. Choose all that apply. 1) Bruxism 2) Enuresis 3) Daytime fatigue 4) Snoring

*3) Daytime fatigue* *4) Snoring*

What position should the patient assume before the nurse inserts an indwelling urinary catheter? 1) Modified Trendelenburg 2) Prone 3) Dorsal recumbent 4) Semi-Fowler's

*3) Dorsal recumbent* The nurse should have the patient lie supine with knees flexed, feet flat on the bed (dorsal recumbent position). If the patient is unable to assume this position, the nurse should help the patient to a side-lying position. Modified Trendelenburg position is used for central venous catheter insertion. Prone position is sometimes used to improve oxygenation in patients with adult respiratory distress syndrome. Semi-Fowler's position is used to prevent aspiration in those receiving enteral feedings.

Which of the following goals is appropriate for a patient with a nursing diagnosis of Constipation? The patient increases the intake of: 1) Milk and cheese. 2) Bread and pasta. 3) Fruits and vegetables. 4) Lean meats.

*3) Fruits and vegetables.* The nurse should encourage the patient to increase his intake of foods rich in fiber because they promote peristalsis and defecation, thereby relieving constipation. Low-fiber foods, such as bread, pasta, and other simple carbohydrates, as well as milk, cheese, and lean meat, slow peristalsis.

From what stage of sleep are people typically most difficult to arouse? 1) NREM, alpha waves 2) NREM, sleep spindles 3) NREM, delta waves 4) REM

*3) NREM, delta waves*

The nurse is caring for a patient who underwent a bowel resection 2 hours ago. His urine output for the past 2 hours totals 50 mL. Which action should the nurse take? 1) Do nothing; this is normal postoperative urine output. 2) Increase the infusion rate of the patient's IV fluids. 3) Notify the provider about the patient's oliguria. 4) Administer the patient's routine diuretic dose early.

*3) Notify the provider about the patient's oliguria.* 50 mL in two hours is not normal output. The kidneys typically produce 60 ml of urine per hour. Therefore, the nurse should notify the provider when the patient shows diminished urine output (oliguria). Patients who undergo abdominal surgery commonly require increased infusions of IV fluid during the immediate postoperative period. The nurse cannot provide increased IV fluids without a provider's order. The nurse should not administer any medications before the scheduled time without a prescription. The provider may hold the patient's scheduled dose of diuretic if he determines that the patient is experiencing deficient fluid volume.

A client's axillary temperature is 100.8°F. The nurse realizes this is outside normal range for this client, and that axillary temperatures do not reflect core temperature. What should the nurse do to obtain a good estimate of the core temperature? 1) Add 1°F to 100.8°F to obtain an oral equivalent. 2) Add 2°F to 100.8°F to obtain a rectal equivalent. 3) Obtain a rectal temperature reading. 4) Obtain a tympanic membrane reading.

*3) Obtain a rectal temperature reading.*

Which expected outcome is best for the patient with a nursing diagnosis of Acute Pain related to movement and secondary to surgical resection of a ruptured spleen and possible inadequate analgesia? 1) The patient will verbalize a reduction in pain after receiving pain medication and repositioning. 2) The patient will rest quietly when undisturbed. 3) On a scale of 0 to 10, the patient will rate pain as a 3 while in bed or as a 4 during ambulation. 4) The patient will receive pain medication every 2 hours as prescribed.

*3) On a scale of 0 to 10, the patient will rate pain as a 3 while in bed or as a 4 during ambulation.*

A patient with tuberculosis is scheduled for computed tomography (CT). How should the nurse proceed? (Select all that apply.) 1) Question the order because the patient must remain in isolation. 2) Place an N-95 respirator mask on the patient and transport him to the test. 3) Place a surgical mask on the patient and transport him to CT lab. 4) Notify the computed tomography department about precautions prior to transport.

*3) Place a surgical mask on the patient and transport him to CT lab.* *4) Notify the computed tomography department about precautions prior to transport.*

A patient in the emergency department is angry, yelling, cursing, and waving his arms when the nurse comes to the treatment cubicle. Which action(s) by the nurse are advisable? 1) Reassure the patient by entering the room alone. 2) Ask the patient if he is carrying any weapons. 3) Stay between the patient and the door; keep the door open. 4) Make eye contact while stating firmly "I will not tolerate cursing and threats."

*3) Stay between the patient and the door; keep the door open.* Make sure you do not get trapped. You should never enter the room alone if someone is threatening, the nurse must be calm and reassuring. Asking about weapons and setting limits may escalate the situation.

Which of the following statements made by a nurse reflects the best understanding of the role of the bath in the nursing assessment process? 1. "I work with my ancillary staff to be able to determine what is abnormal." 2. "The skin is easy to observe for abnormalities when you are giving the bath." 3. "I use the time to really look at my clients and determine what's normal and what's not." 4. "Bath time is an excellent time to get to know your clients and form that nurse-client relationship."

*3. "I use the time to really look at my clients and determine what's normal and what's not."* Take this time to identify abnormalities and initiate appropriate actions to prevent further injury to sensitive tissues. It also provides an opportunity to assess other systems (e.g., circulatory, respiratory) and client behaviors as well. While the nurse is responsible for determining abnormalities, the ancillary staff should be instructed to report any suspicious factors they note. Answer 3 is the most thorough statement regarding the question.

The nurse should describe pain that is causing the client a "burning sensation in the epigastric region" as: 1. Referred 2. Radiating 3. Deep or visceral 4. Superficial or cutaneous

*3. Deep or visceral* Deep or visceral pain is diffuse and may radiate in several directions. Visceral pain may be described as a burning sensation. Referred pain is felt in a part of the body separate from the source of pain, such as with a myocardial infarction, in which pain may be referred to the jaw, left arm, and left shoulder. Radiating pain feels as though it travels down or along a body part, such as low back pain that is accompanied by pain radiating down the leg from sciatic nerve irritation. Superficial or cutaneous pain is of short duration and is localized as in a small cut.

Which of the following symptoms would the nurse expect with a client who is experiencing acute pain? 1. Bradycardia 2. Bradypnea 3. Diaphoresis 4. Decreased muscle tension

*3. Diaphoresis* An expected assessment finding of a client experiencing acute pain would be diaphoresis resulting from sympathetic nerve stimulation. Additional assessment findings of a client experiencing acute pain would be an increased heart rate, respiratory rate, and muscle tension.

Following an assessment of the client, the nurse identifies the nursing diagnosis activity intolerance related to increased weight gain and inactivity. An outcome identified by the nurse should be: 1. Resting heart rate will be 90 to 100 beats/minute 2. Blood pressure will be maintained between 140/80 and 160/90 mm Hg 3. Exercise will be performed 3 to 4 times over the next 2 weeks 4. Achievement of a rating of 3 for activity endurance

*3. Exercise will be performed 3 to 4 times over the next 2 weeks* An appropriate outcome for activity intolerance related to increased weight gain and inactivity is that the client will perform exercise 3 to 4 times over the next 2 weeks. This outcome is realistic, measurable, and addresses the problem. A resting heart rate of 90-100 beats/minute is too high, and it does not address the need to increase activity. This outcome does not state whether this blood pressure is at rest or after exercising. It also does not address the need to increase activity. A more appropriate outcome is that the client will increase his or her activity (over the next 2 weeks).

While ambulating in the hallway of a hospital, the client complains of extreme dizziness. The nurse, alert to a syncopal episode, should first: 1. Support the client and walk quickly back to the room 2. Lean the client against the wall until the episode passes 3. Lower the client gently to the floor 4. Go for help

*3. Lower the client gently to the floor* If the client has a syncopal episode or begins to fall, the nurse should assume a wide base of support with one foot in front of the other, supporting the client's weight, and then extend the leg, allowing the client to slide against the leg while gently lowering the client to the floor and protecting the client's head. The nurse should not attempt to walk the client quickly back to the room. The nurse should not lean the client against a wall as the client may fall. The nurse should not leave the client alone and go for help.

A priority nursing intervention when caring for a client who is receiving an epidural infusion for pain relief is to: 1. Use aseptic technique 2. Label the port as an epidural catheter 3. Monitor vital signs every 15 minutes 4. Avoid supplemental doses of sedatives

*3. Monitor vital signs every 15 minutes* When clients are receiving epidural analgesia, monitoring occurs as often as every 15 minutes, including assessment of respiratory rate, respiratory effort, and skin color. Complications of epidural opioid use include nausea and vomiting, urinary retention, constipation, respiratory depression, and pruritus. A common complication of epidural anesthesia is hypotension. Assessing vital signs is the priority nursing intervention. Because of the catheter location, strict surgical asepsis is needed to prevent a serious and potentially fatal infection. To reduce the risk for accidental epidural injection of drugs intended for IV use, the catheter should be clearly labeled "epidural catheter." Supplemental doses of opioids or sedative/hypnotics are avoided because of possible additive central nervous system adverse effects.

Which of the following is most appropriate when the nurse assesses the intensity of the client's pain? 1. Ask about what precipitates the pain. 2. Question the client about the location of the pain. 3. Offer the client a pain scale to objectify the information. 4. Use open-ended questions to find out about the sensation.

*3. Offer the client a pain scale to objectify the information.* Descriptive scales are a more objective means of measuring pain intensity. Asking the client what precipitates the pain does not assess intensity, but rather it is an assessment of the pain pattern. Asking the client about the location of pain does not assess the intensity of the client's pain. To determine the quality of the client's pain, the nurse may ask open-ended questions to find out about the sensation experienced.

A nurse who is caring for postoperative clients on a surgical unit knows that for 24 to 48 hours postoperatively, clients who have undergone general anesthesia may experience: 1. Colitis 2. Stomatitis 3. Paralytic ileus 4. Gastrocolic reflex

*3. Paralytic ileus*

Discussing the client's follow-up dietary needs immediately after the surgery when the client is experiencing discomfort is an error in: 1. Pacing 2. Intonation 3. Timing and relevance 4. Denotative meaning

*3. Timing and relevance* Discussing follow-up dietary needs immediately after surgery when the client is experiencing discomfort is an error in timing and relevance. The client is less likely to be able to pay attention and comprehend instruction when in pain, and immediately after surgery, discussing follow-up dietary needs would seem irrelevant. Pacing has to do with the speed of conversation. This is not an example of an error in pacing. Intonation is the tone of voice used. This is not an example of an error in intonation. Denotative meaning is when a single word can have several meanings. This is not an example of an error in denotative meaning.

Which urine specific gravity would be expected in a patient admitted with dehydration? 1) 1.002 2) 1.010 3) 1.025 4) 1.030

*4) 1.030* Normal urine specific gravity ranges from 1.010 to 1.025. Specific gravity less than 1.010 indicates fluid volume excess, such as when the patient has fluid overload (too much IV fluid) or when the kidneys fail to concentrate urine. Specific gravity greater than 1.025 is a sign of deficient fluid volume that occurs, for example, as a result of blood loss or dehydration.

The nurse notes that the electrical cord on an IV infusion pump is cracked. Which action by the nurse is best? 1) Continue to monitor the pump to see if the crack worsens. 2) Place the pump back on the utility room shelf. 3) A small crack poses no danger so continue using the pump. 4) Clearly label the pump and send it for repair.

*4) Clearly label the pump and send it for repair.* Label it and take it out of service - all organizations have labels which indicate the equipment is not working. Evaluate the policy to determine if Clinical engineering or biomed needs to be contacted.

Which patient teaching would be most therapeutic for someone with sleep disturbance? 1) Give yourself at least 60 minutes to fall asleep. 2) Avoid eating carbohydrates before going to sleep. 3) Catch up on sleep by napping or sleeping in when possible. 4) Do not go to bed feeling upset about a conflict.

*4) Do not go to bed feeling upset about a conflict.*

The nurse is removing personal protective equipment (PPE). Which item should be removed first? 1) Gown 2) Gloves 3) Face shield 4) Hair covering

*4) Hair covering*

The nurse assists a surgeon with central venous catheter insertion. Which action is necessary to help maintain sterile technique? 1) Closing the patient's door to limit room traffic while preparing the sterile field 2) Using clean procedure gloves to handle sterile equipment 3) Placing the nonsterile syringes containing flush solution on the sterile field 4) Remaining 6 inches away from the sterile field during the procedure

*4) Remaining 6 inches away from the sterile field during the procedure*

A patient with a colostomy complains to the nurse, "I am having really bad odors coming from my pouch." To help control odor, which foods should the nurse advise him to consume? 1) White rice and toast 2) Tomatoes and dried fruit 3) Asparagus and melons 4) Yogurt and parsley

*4) Yogurt and parsley* Yogurt, cranberry juice, parsley, and buttermilk may help control odor. White rice and toast (also bananas and applesauce) help control diarrhea. Asparagus, peas, melons, and fish are known to cause odor. Tomatoes, pears, and dried fruit are high-fiber foods that might cause blockage in a patient with an ostomy.

The nurse is completing an assessment of the client's sleep patterns. A specific question that the nurse should ask to determine the potential presence of sleep apnea is: 1. "How easily do you fall asleep?" 2. "Do you have vivid, lifelike dreams?" 3. "Do you ever experience loss of muscle control or falling?" 4. "Do you snore loudly or experience headaches?"

*4. "Do you snore loudly or experience headaches?"* To assess for sleep apnea (unlike assessing for narcolepsy or insomnia), the nurse may ask, "Do you snore loudly?" and "Do you experience headaches after awakening?" A positive response may indicate the client experiences sleep apnea.

An adult client reports to the nurse that she has been experiencing constipation recently and is interested in any suggestions regarding dietary changes she might make. Which of the following suggestions provided by the nurse is most likely to minimize the client's complaint? 1. "Have you tried foods like prunes and bran?" 2. "You might find the new flavored bulk laxatives helpful." 3. "What have you tried in the past that hasn't been helpful?" 4. "Increase your fluid intake; have some juice with breakfast."

*4. "Increase your fluid intake; have some juice with breakfast."*

The nurse is visiting the client who has a nursing diagnosis of urinary retention. Upon assessment the nurse anticipates that this client will exhibit: 1. Severe flank pain and hematuria 2. Pain and burning on urination 3. A loss of the urge to void 4. A feeling of pressure and voiding of small amounts

*4. A feeling of pressure and voiding of small amounts*

The nurse recognizes that a client recovering from anesthesia required for surgical repair of a fractured ulna is likely to experience difficulty urinating primarily because of: 1. The impaired cognitive state the client will experience as the effects of the anesthesia wear off 2. The decreased volume of orally ingested fluids before, during, and after the surgical procedure 3. The length of time the client was under the effects of general anesthesia required for the surgical procedure 4. The effects of the anesthetic on the nerves and muscles controlling the relaxation of the urinary bladder

*4. The effects of the anesthetic on the nerves and muscles controlling the relaxation of the urinary bladder*

When obtaining a sterile urine specimen from an indwelling urinary catheter the nurse should: 1. Disconnect the catheter from the drainage tubing 2. Withdraw urine from a urinometer 3. Open the drainage bag and removing urine 4. Use a needle to withdraw urine from the catheter port

*4. Use a needle to withdraw urine from the catheter port*

Immobilizing Interventions: Observing Bucks Extension Traction

*Inspect the skin of the limb in traction (beneath velcro boot) at least once q 8hrs for irritation and inflamation *Do NOT release the weights of the traction *No pin care required *If burning sensation present in foot= NOTIFY PHYSICIAN

173.) A nurse reviews the medication history of a client admitted to the hospital and notes that the client is taking leflunomide (Arava). During data collection, the nurse asks which question to determine medication effectiveness? 1. "Do you have any joint pain?" 2. "Are you having any diarrhea?" 3. "Do you have frequent headaches?" 4. "Are you experiencing heartburn?"

1. "Do you have any joint pain?" Rationale: Leflunomide is an immunosuppressive agent and has an anti-inflammatory action. The medication provides symptomatic relief of rheumatoid arthritis. Diarrhea can occur as a side effect of the medication. The other options are unrelated to medication effectiveness.

127.) The nurse provides medication instructions to an older hypertensive client who is taking 20 mg of lisinopril (Prinivil, Zestril) orally daily. The nurse evaluates the need for further teaching when the client states which of the following? 1. "I can skip a dose once a week." 2. "I need to change my position slowly." 3. "I take the pill after breakfast each day." 4. "If I get a bad headache, I should call my doctor immediately."

1. "I can skip a dose once a week." Rationale: Lisinopril is an antihypertensive angiotensin-converting enzyme (ACE) inhibitor. The usual dosage range is 20 to 40 mg per day. Adverse effects include headache, dizziness, fatigue, orthostatic hypotension, tachycardia, and angioedema. Specific client teaching points include taking one pill a day, not stopping the medication without consulting the health care provider (HCP), and monitoring for side effects and adverse reactions. The client should notify the HCP if side effects occur.

31.) A community health nurse visits a client at home. Prednisone 10 mg orally daily has been prescribed for the client and the nurse reinforces teaching for the client about the medication. Which statement, if made by the client, indicates that further teaching is necessary? 1. "I can take aspirin or my antihistamine if I need it." 2. "I need to take the medication every day at the same time." 3. "I need to avoid coffee, tea, cola, and chocolate in my diet." 4. "If I gain more than 5 pounds a week, I will call my doctor."

1. "I can take aspirin or my antihistamine if I need it." Rationale: Aspirin and other over-the-counter medications should not be taken unless the client consults with the health care provider (HCP). The client needs to take the medication at the same time every day and should be instructed not to stop the medication. A slight weight gain as a result of an improved appetite is expected, but after the dosage is stabilized, a weight gain of 5 lb or more weekly should be reported to the HCP. Caffeine-containing foods and fluids need to be avoided because they may contribute to steroid-ulcer development.

179.) A nurse provides medication instructions to a client who had a kidney transplant about therapy with cyclosporine (Sandimmune). Which statement by the client indicates a need for further instruction? 1. "I need to obtain a yearly influenza vaccine." 2. "I need to have dental checkups every 3 months." 3. "I need to self-monitor my blood pressure at home." 4. "I need to call the health care provider (HCP) if my urine volume decreases or my urine becomes cloudy."

1. "I need to obtain a yearly influenza vaccine." Rationale: Cyclosporine is an immunosuppressant medication. Because of the medication's effects, the client should not receive any vaccinations without first consulting the HCP. The client should report decreased urine output or cloudy urine, which could indicate kidney rejection or infection, respectively. The client must be able to self-monitor blood pressure to check for the side effect of hypertension. The client needs meticulous oral care and dental cleaning every 3 months to help prevent gingival hyperplasia.

77.) Phenytoin (Dilantin), 100 mg orally three times daily, has been prescribed for a client for seizure control. The nurse reinforces instructions regarding the medication to the client. Which statement by the client indicates an understanding of the instructions? 1. "I will use a soft toothbrush to brush my teeth." 2. "It's all right to break the capsules to make it easier for me to swallow them." 3. "If I forget to take my medication, I can wait until the next dose and eliminate that dose." 4. "If my throat becomes sore, it's a normal effect of the medication and it's nothing to be concerned about."

1. "I will use a soft toothbrush to brush my teeth." Rationale: Phenytoin (Dilantin) is an anticonvulsant. Gingival hyperplasia, bleeding, swelling, and tenderness of the gums can occur with the use of this medication. The client needs to be taught good oral hygiene, gum massage, and the need for regular dentist visits. The client should not skip medication doses, because this could precipitate a seizure. Capsules should not be chewed or broken and they must be swallowed. The client needs to be instructed to report a sore throat, fever, glandular swelling, or any skin reaction, because this indicates hematological toxicity.

220.) A adult client with muscle spasms is taking an oral maintenance dose of baclofen (Lioresal). The nurse reviews the medication record, expecting that which dose should be prescribed? 1. 15 mg four times a day 2. 25 mg four times a day 3. 30 mg four times a day 4. 40 mg four times a day

1. 15 mg four times a day Rationale: Baclofen is dispensed in 10- and 20-mg tablets for oral use. Dosages are low initially and then gradually increased. Maintenance doses range from 15 to 20 mg administered three or four times a day.

115.) A client received 20 units of NPH insulin subcutaneously at 8:00 AM. The nurse should check the client for a potential hypoglycemic reaction at what time? 1. 5:00 PM 2. 10:00 AM 3. 11:00 AM 4. 11:00 PM

1. 5:00 PM Rationale: NPH is intermediate-acting insulin. Its onset of action is 1 to 2½ hours, it peaks in 4 to 12 hours, and its duration of action is 24 hours. Hypoglycemic reactions most likely occur during peak time.

10.) The clinic nurse is performing an admission assessment on a client. The nurse notes that the client is taking azelaic acid (Azelex). Because of the medication prescription, the nurse would suspect that the client is being treated for: 1. Acne 2. Eczema 3. Hair loss 4. Herpes simplex

1. Acne Rationale: Azelaic acid is a topical medication used to treat mild to moderate acne. The acid appears to work by suppressing the growth of Propionibacterium acnes and decreasing the proliferation of keratinocytes. Options 2, 3, and 4 are incorrect.

145.) A nurse has a prescription to give a client albuterol (Proventil HFA) (two puffs) and beclomethasone dipropionate (Qvar) (nasal inhalation, two puffs), by metered-dose inhaler. The nurse administers the medication by giving the: 1. Albuterol first and then the beclomethasone dipropionate 2. Beclomethasone dipropionate first and then the albuterol 3. Alternating a single puff of each, beginning with the albuterol 4. Alternating a single puff of each, beginning with the beclomethasone dipropionate

1. Albuterol first and then the beclomethasone dipropionate Rationale: Albuterol is a bronchodilator. Beclomethasone dipropionate is a glucocorticoid. Bronchodilators are always administered before glucocorticoids when both are to be given on the same time schedule. This allows for widening of the air passages by the bronchodilator, which then makes the glucocorticoid more effective.

26.) Glimepiride (Amaryl) is prescribed for a client with diabetes mellitus. A nurse reinforces instructions for the client and tells the client to avoid which of the following while taking this medication? 1. Alcohol 2. Organ meats 3. Whole-grain cereals 4. Carbonated beverages

1. Alcohol Rationale: When alcohol is combined with glimepiride (Amaryl), a disulfiram-like reaction may occur. This syndrome includes flushing, palpitations, and nausea. Alcohol can also potentiate the hypoglycemic effects of the medication. Clients need to be instructed to avoid alcohol consumption while taking this medication. The items in options 2, 3, and 4 do not need to be avoided.

100.) Saquinavir (Invirase) is prescribed for the client who is human immunodeficiency virus seropositive. The nurse reinforces medication instructions and tells the client to: 1. Avoid sun exposure. 2. Eat low-calorie foods. 3. Eat foods that are low in fat. 4. Take the medication on an empty stomach.

1. Avoid sun exposure. Rationale: Saquinavir (Invirase) is an antiretroviral (protease inhibitor) used with other antiretroviral medications to manage human immunodeficiency virus infection. Saquinavir is administered with meals and is best absorbed if the client consumes high-calorie, high-fat meals. Saquinavir can cause photosensitivity, and the nurse should instruct the client to avoid sun exposure.

202.) A nurse is collecting data from a client about medications being taken, and the client tells the nurse that he is taking herbal supplements for the treatment of varicose veins. The nurse understands that the client is most likely taking which of the following? 1. Bilberry 2. Ginseng 3. Feverfew 4. Evening primrose

1. Bilberry Rationale: Bilberry is an herbal supplement that has been used to treat varicose veins. This supplement has also been used to treat cataracts, retinopathy, diabetes mellitus, and peripheral vascular disease. Ginseng has been used to improve memory performance and decrease blood glucose levels in type 2 diabetes mellitus. Feverfew is used to prevent migraine headaches and to treat rheumatoid arthritis. Evening primrose is used to treat eczema and skin irritation.

156.) A nurse is reviewing the laboratory results for a client receiving tacrolimus (Prograf). Which laboratory result would indicate to the nurse that the client is experiencing an adverse effect of the medication? 1. Blood glucose of 200 mg/dL 2. Potassium level of 3.8 mEq/L 3. Platelet count of 300,000 cells/mm3 4. White blood cell count of 6000 cells/mm3

1. Blood glucose of 200 mg/dL Rationale: A blood glucose level of 200 mg/dL is elevated above the normal range of 70 to 110 mg/dL and suggests an adverse effect. Other adverse effects include neurotoxicity evidenced by headache, tremor, insomnia; gastrointestinal (GI) effects such as diarrhea, nausea, and vomiting; hypertension; and hyperkalemia.

Cane walking?

1. COAL C - cane O - opposite A - affected L - leg

32.) Desmopressin acetate (DDAVP) is prescribed for the treatment of diabetes insipidus. The nurse monitors the client after medication administration for which therapeutic response? 1. Decreased urinary output 2. Decreased blood pressure 3. Decreased peripheral edema 4. Decreased blood glucose level

1. Decreased urinary output Rationale: Desmopressin promotes renal conservation of water. The hormone carries out this action by acting on the collecting ducts of the kidney to increase their permeability to water, which results in increased water reabsorption. The therapeutic effect of this medication would be manifested by a decreased urine output. Options 2, 3, and 4 are unrelated to the effects of this medication.

105.) A nurse is collecting data from a client and the client's spouse reports that the client is taking donepezil hydrochloride (Aricept). Which disorder would the nurse suspect that this client may have based on the use of this medication? 1. Dementia 2. Schizophrenia 3. Seizure disorder 4. Obsessive-compulsive disorder

1. Dementia Rationale: Donepezil hydrochloride is a cholinergic agent used in the treatment of mild to moderate dementia of the Alzheimer type. It enhances cholinergic functions by increasing the concentration of acetylcholine. It slows the progression of Alzheimer's disease. Options 2, 3, and 4 are incorrect.

33.) The home health care nurse is visiting a client who was recently diagnosed with type 2 diabetes mellitus. The client is prescribed repaglinide (Prandin) and metformin (Glucophage) and asks the nurse to explain these medications. The nurse should reinforce which instructions to the client? Select all that apply. 1. Diarrhea can occur secondary to the metformin. 2. The repaglinide is not taken if a meal is skipped. 3. The repaglinide is taken 30 minutes before eating. 4. Candy or another simple sugar is carried and used to treat mild hypoglycemia episodes. 5. Metformin increases hepatic glucose production to prevent hypoglycemia associated with repaglinide. 6. Muscle pain is an expected side effect of metformin and may be treated with acetaminophen (Tylenol).

1. Diarrhea can occur secondary to the metformin. 2. The repaglinide is not taken if a meal is skipped. 3. The repaglinide is taken 30 minutes before eating. 4. Candy or another simple sugar is carried and used to treat mild hypoglycemia episodes. Rationale: Repaglinide is a rapid-acting oral hypoglycemic agent that stimulates pancreatic insulin secretion that should be taken before meals, and that should be withheld if the client does not eat. Hypoglycemia is a side effect of repaglinide and the client should always be prepared by carrying a simple sugar with her or him at all times. Metformin is an oral hypoglycemic given in combination with repaglinide and works by decreasing hepatic glucose production. A common side effect of metformin is diarrhea. Muscle pain may occur as an adverse effect from metformin but it might signify a more serious condition that warrants health care provider notification, not the use of acetaminophen.

116.) A nurse administers a dose of scopolamine (Transderm-Scop) to a postoperative client. The nurse tells the client to expect which of the following side effects of this medication? 1. Dry mouth 2. Diaphoresis 3. Excessive urination 4. Pupillary constriction

1. Dry mouth Rationale: Scopolamine is an anticholinergic medication for the prevention of nausea and vomiting that causes the frequent side effects of dry mouth, urinary retention, decreased sweating, and dilation of the pupils. The other options describe the opposite effects of cholinergic-blocking agents and therefore are incorrect.

193.) Sodium hypochlorite (Dakin's solution) is prescribed for a client with a leg wound containing purulent drainage. The nurse is assisting in developing a plan of care for the client and includes which of the following in the plan? 1. Ensure that the solution is freshly prepared before use. 2. Soak a sterile dressing with solution and pack into the wound. 3. Allow the solution to remain in the wound following irrigation. 4. Apply the solution to the wound and on normal skin tissue surrounding the wound.

1. Ensure that the solution is freshly prepared before use. Rationale: Dakin solution is a chloride solution that is used for irrigating and cleaning necrotic or purulent wounds. It can be used for packing necrotic wounds. It cannot be used to pack purulent wounds because the solution is inactivated by copious pus. It should not come into contact with healing or normal tissue, and it should be rinsed off immediately if used for irrigation. Solutions are unstable and the nurse must ensure that the solution has been prepared fresh before use. **Eliminate options 2 and 3 first because they are comparable or alike. It makes sense to ensure that the solution is freshly prepared; therefore, select option 1**

112.) A hospitalized client is started on phenelzine sulfate (Nardil) for the treatment of depression. The nurse instructs the client to avoid consuming which foods while taking this medication? Select all that apply. 1. Figs 2. Yogurt 3. Crackers 4. Aged cheese 5 Tossed salad 6. Oatmeal cookies

1. Figs 2. Yogurt 4. Aged cheese Rationale: Phenelzine sulfate (Nardil) is a monoamine oxidase inhibitor(MAOI). The client should avoid taking in foods that are high in tyramine. Use of these foods could trigger a potentially fatal hypertensive crisis. Some foods to avoid include yogurt, aged cheeses, smoked or processed meats, red wines, and fruits such as avocados, raisins, and figs.

93.) The client who is human immunodeficiency virus seropositive has been taking stavudine (d4t, Zerit). The nurse monitors which of the following most closely while the client is taking this medication? 1. Gait 2. Appetite 3. Level of consciousness 4. Hemoglobin and hematocrit blood levels

1. Gait Rationale: Stavudine (d4t, Zerit) is an antiretroviral used to manage human immunodeficiency virus infection in clients who do not respond to or who cannot tolerate conventional therapy. The medication can cause peripheral neuropathy, and the nurse should monitor the client's gait closely and ask the client about paresthesia. Options 2, 3, and 4 are unrelated to the use of the medication.

91.) Cyclobenzaprine (Flexeril) is prescribed for a client to treat muscle spasms, and the nurse is reviewing the client's record. Which of the following disorders, if noted in the client's record, would indicate a need to contact the health care provider regarding the administration of this medication? 1. Glaucoma 2. Emphysema 3. Hyperthyroidism 4. Diabetes mellitus

1. Glaucoma Rationale: Because this medication has anticholinergic effects, it should be used with caution in clients with a history of urinary retention, angle-closure glaucoma, and increased intraocular pressure. Cyclobenzaprine hydrochloride should be used only for short-term 2- to 3-week therapy.

125.) A nurse is preparing to administer digoxin (Lanoxin), 0.125 mg orally, to a client with heart failure. Which vital sign is most important for the nurse to check before administering the medication? 1. Heart rate 2. Temperature 3. Respirations 4. Blood pressure

1. Heart rate Rationale: Digoxin is a cardiac glycoside that is used to treat heart failure and acts by increasing the force of myocardial contraction. Because bradycardia may be a clinical sign of toxicity, the nurse counts the apical heart rate for 1 full minute before administering the medication. If the pulse rate is less than 60 beats/minute in an adult client, the nurse would withhold the medication and report the pulse rate to the registered nurse, who would then contact the health care provider.

158.) A client with chronic renal failure is receiving epoetin alfa (Epogen, Procrit). Which laboratory result would indicate a therapeutic effect of the medication? 1. Hematocrit of 32% 2. Platelet count of 400,000 cells/mm3 3. White blood cell count of 6000 cells/mm3 4. Blood urea nitrogen (BUN) level of 15 mg/dL

1. Hematocrit of 32% Rationale: Epoetin alfa is used to reverse anemia associated with chronic renal failure. A therapeutic effect is seen when the hematocrit is between 30% and 33%. The laboratory tests noted in the other options are unrelated to the use of this medication.

167.) A nurse prepares to reinforce instructions to a client who is taking allopurinol (Zyloprim). The nurse plans to include which of the following in the instructions? 1. Instruct the client to drink 3000 mL of fluid per day. 2. Instruct the client to take the medication on an empty stomach. 3. Inform the client that the effect of the medication will occur immediately. 4. Instruct the client that, if swelling of the lips occurs, this is a normal expected response.

1. Instruct the client to drink 3000 mL of fluid per day. Rationale: Allopurinol (Zyloprim) is an antigout medication used to decrease uric acid levels. Clients taking allopurinol are encouraged to drink 3000 mL of fluid a day. A full therapeutic effect may take 1 week or longer. Allopurinol is to be given with or immediately following meals or milk to prevent gastrointestinal irritation. If the client develops a rash, irritation of the eyes, or swelling of the lips or mouth, he or she should contact the health care provider because this may indicate hypersensitivity.

113.) A nurse is reinforcing discharge instructions to a client receiving sulfisoxazole. Which of the following would be included in the plan of care for instructions? 1. Maintain a high fluid intake. 2. Discontinue the medication when feeling better. 3. If the urine turns dark brown, call the health care provider immediately. 4. Decrease the dosage when symptoms are improving to prevent an allergic response.

1. Maintain a high fluid intake. Rationale: Each dose of sulfisoxazole should be administered with a full glass of water, and the client should maintain a high fluid intake. The medication is more soluble in alkaline urine. The client should not be instructed to taper or discontinue the dose. Some forms of sulfisoxazole cause the urine to turn dark brown or red. This does not indicate the need to notify the health care provider.

43.) A histamine (H2)-receptor antagonist will be prescribed for a client. The nurse understands that which medications are H2-receptor antagonists? Select all that apply. 1. Nizatidine (Axid) 2. Ranitidine (Zantac) 3. Famotidine (Pepcid) 4. Cimetidine (Tagamet) 5. Esomeprazole (Nexium) 6. Lansoprazole (Prevacid)

1. Nizatidine (Axid) 2. Ranitidine (Zantac) 3. Famotidine (Pepcid) 4. Cimetidine (Tagamet) Rationale: H2-receptor antagonists suppress secretion of gastric acid, alleviate symptoms of heartburn, and assist in preventing complications of peptic ulcer disease. These medications also suppress gastric acid secretions and are used in active ulcer disease, erosive esophagitis, and pathological hypersecretory conditions. The other medications listed are proton pump inhibitors. H2-receptor antagonists medication names end with -dine. Proton pump inhibitors medication names end with -zole.

109.) A client taking buspirone (BuSpar) for 1 month returns to the clinic for a follow-up visit. Which of the following would indicate medication effectiveness? 1. No rapid heartbeats or anxiety 2. No paranoid thought processes 3. No thought broadcasting or delusions 4. No reports of alcohol withdrawal symptoms

1. No rapid heartbeats or anxiety Rationale: Buspirone hydrochloride is not recommended for the treatment of drug or alcohol withdrawal, paranoid thought disorders, or schizophrenia (thought broadcasting or delusions). Buspirone hydrochloride is most often indicated for the treatment of anxiety and aggression.

12.) A nurse is caring for a client who is receiving an intravenous (IV) infusion of an antineoplastic medication. During the infusion, the client complains of pain at the insertion site. During an inspection of the site, the nurse notes redness and swelling and that the rate of infusion of the medication has slowed. The nurse should take which appropriate action? 1. Notify the registered nurse. 2. Administer pain medication to reduce the discomfort. 3. Apply ice and maintain the infusion rate, as prescribed. 4. Elevate the extremity of the IV site, and slow the infusion.

1. Notify the registered nurse. Rationale: When antineoplastic medications (Chemotheraputic Agents) are administered via IV, great care must be taken to prevent the medication from escaping into the tissues surrounding the injection site, because pain, tissue damage, and necrosis can result. The nurse monitors for signs of extravasation, such as redness or swelling at the insertion site and a decreased infusion rate. If extravasation occurs, the registered nurse needs to be notified; he or she will then contact the health care provider.

235.) A tricyclic antidepressant is administered to a client daily. The nurse plans to monitor for the common side effects of the medication and includes which of the following in the plan of care? 1. Offer hard candy or gum periodically. 2. Offer a nutritious snack between meals. 3. Monitor the blood pressure every 2 hours. 4. Review the white blood cell (WBC) count results daily.

1. Offer hard candy or gum periodically. Rationale: Dry mouth is a common side effect of tricyclic antidepressants. Frequent mouth rinsing with water, sucking on hard candy, and chewing gum will alleviate this common side effect. It is not necessary to monitor the blood pressure every 2 hours. In addition, it is not necessary to check the WBC daily. Weight gain is a common side effect and frequent snacks will aggravate this problem.

18.) The nurse is reviewing the history and physical examination of a client who will be receiving asparaginase (Elspar), an antineoplastic agent. The nurse consults with the registered nurse regarding the administration of the medication if which of the following is documented in the client's history? 1. Pancreatitis 2. Diabetes mellitus 3. Myocardial infarction 4. Chronic obstructive pulmonary disease

1. Pancreatitis Rationale: Asparaginase (Elspar) is contraindicated if hypersensitivity exists, in pancreatitis, or if the client has a history of pancreatitis. The medication impairs pancreatic function and pancreatic function tests should be performed before therapy begins and when a week or more has elapsed between administration of the doses. The client needs to be monitored for signs of pancreatitis, which include nausea, vomiting, and abdominal pain. The conditions noted in options 2, 3, and 4 are not contraindicated with this medication.

171.) A nurse is preparing to administer furosemide (Lasix) to a client with a diagnosis of heart failure. The most important laboratory test result for the nurse to check before administering this medication is: 1. Potassium level 2. Creatinine level 3. Cholesterol level 4. Blood urea nitrogen

1. Potassium level Rationale: Furosemide is a loop diuretic. The medication causes a decrease in the client's electrolytes, especially potassium, sodium, and chloride. Administering furosemide to a client with low electrolyte levels could precipitate ventricular dysrhythmias. Options 2 and 4 reflect renal function. The cholesterol level is unrelated to the administration of this medication.

30.) A client with diabetes mellitus visits a health care clinic. The client's diabetes mellitus previously had been well controlled with glyburide (DiaBeta) daily, but recently the fasting blood glucose level has been 180 to 200 mg/dL. Which medication, if added to the client's regimen, may have contributed to the hyperglycemia? 1. Prednisone 2. Phenelzine (Nardil) 3. Atenolol (Tenormin) 4. Allopurinol (Zyloprim)

1. Prednisone Rationale: Prednisone may decrease the effect of oral hypoglycemics, insulin, diuretics, and potassium supplements. Option 2, a monoamine oxidase inhibitor, and option 3, a β-blocker, have their own intrinsic hypoglycemic activity. Option 4 decreases urinary excretion of sulfonylurea agents, causing increased levels of the oral agents, which can lead to hypoglycemia.

152.) Intravenous heparin therapy is prescribed for a client. While implementing this prescription, a nurse ensures that which of the following medications is available on the nursing unit? 1. Protamine sulfate 2. Potassium chloride 3. Phytonadione (vitamin K ) 4. Aminocaproic acid (Amicar)

1. Protamine sulfate Rationale: The antidote to heparin is protamine sulfate; it should be readily available for use if excessive bleeding or hemorrhage occurs. Potassium chloride is administered for a potassium deficit. Vitamin K is an antidote for warfarin sodium. Aminocaproic acid is the antidote for thrombolytic therapy.

102.) A client with human immunodeficiency virus is taking nevirapine (Viramune). The nurse should monitor for which adverse effects of the medication? Select all that apply. 1. Rash 2. Hepatotoxicity 3. Hyperglycemia 4. Peripheral neuropathy 5. Reduced bone mineral density

1. Rash 2. Hepatotoxicity Rationale: Nevirapine (Viramune) is a non-nucleoside reverse transcriptase inhibitors (NRTI) that is used to treat HIV infection. It is used in combination with other antiretroviral medications to treat HIV. Adverse effects include rash, Stevens-Johnson syndrome, hepatitis, and increased transaminase levels. Hyperglycemia, peripheral neuropathy, and reduced bone density are not adverse effects of this medication.

90.) A nurse is reviewing the record of a client who has been prescribed baclofen (Lioresal). Which of the following disorders, if noted in the client's history, would alert the nurse to contact the health care provider? 1. Seizure disorders 2. Hyperthyroidism 3. Diabetes mellitus 4. Coronary artery disease

1. Seizure disorders Rationale: Clients with seizure disorders may have a lowered seizure threshold when baclofen is administered. Concurrent therapy may require an increase in the anticonvulsive medication. The disorders in options 2, 3, and 4 are not a concern when the client is taking baclofen.

53.) Rifabutin (Mycobutin) is prescribed for a client with active Mycobacterium avium complex (MAC) disease and tuberculosis. The nurse monitors for which side effects of the medication? Select all that apply. 1. Signs of hepatitis 2. Flu-like syndrome 3. Low neutrophil count 4. Vitamin B6 deficiency 5. Ocular pain or blurred vision 6. Tingling and numbness of the fingers

1. Signs of hepatitis 2. Flu-like syndrome 3. Low neutrophil count 5. Ocular pain or blurred vision Rationale: Rifabutin (Mycobutin) may be prescribed for a client with active MAC disease and tuberculosis. It inhibits mycobacterial DNA-dependent RNA polymerase and suppresses protein synthesis. Side effects include rash, gastrointestinal disturbances, neutropenia (low neutrophil count), red-orange body secretions, uveitis (blurred vision and eye pain), myositis, arthralgia, hepatitis, chest pain with dyspnea, and flu-like syndrome. Vitamin B6 deficiency and numbness and tingling in the extremities are associated with the use of isoniazid (INH). Ethambutol (Myambutol) also causes peripheral neuritis.

92.) In monitoring a client's response to disease-modifying antirheumatic drugs (DMARDs), which findings would the nurse interpret as acceptable responses? Select all that apply. 1. Symptom control during periods of emotional stress 2. Normal white blood cell counts, platelet, and neutrophil counts 3. Radiological findings that show nonprogression of joint degeneration 4. An increased range of motion in the affected joints 3 months into therapy 5. Inflammation and irritation at the injection site 3 days after injection is given 6. A low-grade temperature upon rising in the morning that remains throughout the day

1. Symptom control during periods of emotional stress 2. Normal white blood cell counts, platelet, and neutrophil counts 3. Radiological findings that show nonprogression of joint degeneration 4. An increased range of motion in the affected joints 3 months into therapy Rationale: Because emotional stress frequently exacerbates the symptoms of rheumatoid arthritis, the absence of symptoms is a positive finding. DMARDs are given to slow progression of joint degeneration. In addition, the improvement in the range of motion after 3 months of therapy with normal blood work is a positive finding. Temperature elevation and inflammation and irritation at the medication injection site could indicate signs of infection.

221.) A nurse is reviewing the health care provider's prescriptions for an adult client who has been admitted to the hospital following a back injury. Carisoprodol (Soma) is prescribed for the client to relieve the muscle spasms; the health care provider has prescribed 350 mg to be administered four times a day. When preparing to give this medication, the nurse determines that this dosage is: 1. The normal adult dosage 2. A lower than normal dosage 3. A higher than normal dosage 4. A dosage requiring further clarification

1. The normal adult dosage Rationale: The normal adult dosage for carisoprodol is 350 mg orally three or four times daily.

21.) A nurse is assisting with caring for a client with cancer who is receiving cisplatin. Select the adverse effects that the nurse monitors for that are associated with this medication. Select all that apply. 1. Tinnitus 2. Ototoxicity 3. Hyperkalemia 4. Hypercalcemia 5. Nephrotoxicity 6. Hypomagnesemia

1. Tinnitus 2. Ototoxicity 5. Nephrotoxicity 6. Hypomagnesemia Rationale: Cisplatin is an alkylating medication. Alkylating medications are cell cycle phase-nonspecific medications that affect the synthesis of DNA by causing the cross-linking of DNA to inhibit cell reproduction. Cisplatin may cause ototoxicity, tinnitus, hypokalemia, hypocalcemia, hypomagnesemia, and nephrotoxicity. Amifostine (Ethyol) may be administered before cisplatin to reduce the potential for renal toxicity.

213.) A client is admitted to the hospital with complaints of back spasms. The client states, "I have been taking two or three aspirin every 4 hours for the past week and it hasn't helped my back." Aspirin intoxication is suspected. Which of the following complaints would indicate aspirin intoxication? 1. Tinnitus 2. Constipation 3. Photosensitivity 4. Abdominal cramps

1. Tinnitus Rationale: Mild intoxication with acetylsalicylic acid (aspirin) is called salicylism and is commonly experienced when the daily dosage is higher than 4 g. Tinnitus (ringing in the ears) is the most frequently occurring effect noted with intoxication. Hyperventilation may occur because salicylate stimulates the respiratory center. Fever may result because salicylate interferes with the metabolic pathways involved with oxygen consumption and heat production. Options 2, 3, and 4 are incorrect.

3.) Salicylic acid is prescribed for a client with a diagnosis of psoriasis. The nurse monitors the client, knowing that which of the following would indicate the presence of systemic toxicity from this medication? 1. Tinnitus 2. Diarrhea 3. Constipation 4. Decreased respirations

1. Tinnitus Rationale: Salicylic acid is absorbed readily through the skin, and systemic toxicity (salicylism) can result. Symptoms include tinnitus, dizziness, hyperpnea, and psychological disturbances. Constipation and diarrhea are not associated with salicylism.

110.) A client taking lithium carbonate (Lithobid) reports vomiting, abdominal pain, diarrhea, blurred vision, tinnitus, and tremors. The lithium level is checked as a part of the routine follow-up and the level is 3.0 mEq/L. The nurse knows that this level is: 1. Toxic 2. Normal 3. Slightly above normal 4. Excessively below normal

1. Toxic Rationale: The therapeutic serum level of lithium is 0.6 to 1.2 mEq/L. A level of 3 mEq/L indicates toxicity.

8.) A client with severe acne is seen in the clinic and the health care provider (HCP) prescribes isotretinoin. The nurse reviews the client's medication record and would contact the (HCP) if the client is taking which medication? 1. Vitamin A 2. Digoxin (Lanoxin) 3. Furosemide (Lasix) 4. Phenytoin (Dilantin)

1. Vitamin A Rationale: Isotretinoin is a metabolite of vitamin A and can produce generalized intensification of isotretinoin toxicity. Because of the potential for increased toxicity, vitamin A supplements should be discontinued before isotretinoin therapy. Options 2, 3, and 4 are not contraindicated with the use of isotretinoin.

239.) Which of the following precautions will the nurse specifically take during the administration of ribavirin (Virazole) to a child with respiratory syncytial virus (RSV)? 1. Wearing goggles 2. Wearing a gown 3. Wearing a gown and a mask 4. Handwashing before administration

1. Wearing goggles Rationale: Some caregivers experience headaches, burning nasal passages and eyes, and crystallization of soft contact lenses as a result of administration of ribavirin. Specific to this medication is the use of goggles. A gown is not necessary. A mask may be worn. Handwashing is to be performed before and after any child contact.

79.) Ibuprofen (Advil) is prescribed for a client. The nurse tells the client to take the medication: 1. With 8 oz of milk 2. In the morning after arising 3. 60 minutes before breakfast 4. At bedtime on an empty stomach

1. With 8 oz of milk Rationale: Ibuprofen is a nonsteroidal anti-inflammatory drug (NSAID). NSAIDs should be given with milk or food to prevent gastrointestinal irritation. Options 2, 3, and 4 are incorrect.

24.) A nurse is reinforcing teaching for a client regarding how to mix regular insulin and NPH insulin in the same syringe. Which of the following actions, if performed by the client, indicates the need for further teaching? 1. Withdraws the NPH insulin first 2. Withdraws the regular insulin first 3. Injects air into NPH insulin vial first 4. Injects an amount of air equal to the desired dose of insulin into the vial

1. Withdraws the NPH insulin first Rationale: When preparing a mixture of regular insulin with another insulin preparation, the regular insulin is drawn into the syringe first. This sequence will avoid contaminating the vial of regular insulin with insulin of another type. Options 2, 3, and 4 identify the correct actions for preparing NPH and regular insulin.

195.) A nurse is caring for a client who is taking metoprolol (Lopressor). The nurse measures the client's blood pressure (BP) and apical pulse (AP) immediately before administration. The client's BP is 122/78 mm/Hg and the AP is 58 beats/min. Based on this data, which of the following is the appropriate action? 1. Withhold the medication. 2. Notify the registered nurse immediately. 3. Administer the medication as prescribed. 4. Administer half of the prescribed medication.

1. Withhold the medication. Rationale: Metoprolol (Lopressor) is classified as a beta-adrenergic blocker and is used in the treatment of hypertension, angina, and myocardial infarction. Baseline nursing assessments include measurement of BP and AP immediately before administration. If the systolic BP is below 90 mm/Hg and the AP is below 60 beats/min, the nurse should withhold the medication and document this action. Although the registered nurse should be informed of the client's vital signs, it is not necessary to do so immediately. The medication should not be administered because the data is outside of the prescribed parameters for this medication. The nurse should not administer half of the medication, or alter any dosages at any point in time.

234.) A hospitalized client is started on phenelzine sulfate (Nardil) for the treatment of depression. At lunchtime, a tray is delivered to the client. Which food item on the tray will the nurse remove? 1. Yogurt 2. Crackers 3. Tossed salad 4. Oatmeal cookies

1. Yogurt Rationale: Phenelzine sulfate is a monoamine oxidase inhibitor (MAOI). The client should avoid taking in foods that are high in tyramine. These foods could trigger a potentially fatal hypertensive crisis. Foods to avoid include yogurt, aged cheeses, smoked or processed meats, red wines, and fruits such as avocados, raisins, or figs.

Match the breath sound with the appropriate description. 1) High-pitched sound heard on inspiration in infants 2) High-pitched, continuous musical sound 3) High-pitched popping or low-pitched bubbling sounds 4) Low-pitched continuous sounds that clear with coughing 5) Labored, snoring sound a. Crackles b. Rhonchi c. Stridor d. Wheezes e. Stertor

1. c. High-pitched sound heard on inspiration in infants: Stridor 2. d. High-pitched, continuous musical sound: Wheezes 3. a. High-pitched popping or low-pitched bubbling sounds: Crackles 4. b. Low-pitched continuous sounds that clear with coughing: Rhonchi 5. e. Labored, snoring sound: Stertor

Developmental stages

2-3 months: turns head side to side 4-5 months: grasps, switch & roll 6-7 months: sit at 6 and waves bye-bye 8-9 months: stands straight at eight 10-11 months: belly to butt (phrase has 10 letters) 12-13 months: twelve and up, drink from a cup

mastitis

2-4wks tx: support bra, cabbage, ABX: pcn pain meds, heat!

183.) A client who received a kidney transplant is taking azathioprine (Imuran), and the nurse provides instructions about the medication. Which statement by the client indicates a need for further instructions? 1. "I need to watch for signs of infection." 2. "I need to discontinue the medication after 14 days of use." 3. "I can take the medication with meals to minimize nausea." 4. "I need to call the health care provider (HCP) if more than one dose is missed."

2. "I need to discontinue the medication after 14 days of use." Rationale: Azathioprine is an immunosuppressant medication that is taken for life. Because of the effects of the medication, the client must watch for signs of infection, which are reported immediately to the HCP. The client should also call the HCP if more than one dose is missed. The medication may be taken with meals to minimize nausea.

194.) A nurse provides instructions to a client regarding the use of tretinoin (Retin-A). Which statement by the client indicates the need for further instructions? 1. "Optimal results will be seen after 6 weeks." 2. "I should apply a very thin layer to my skin." 3. "I should wash my hands thoroughly after applying the medication." 4. "I should cleanse my skin thoroughly before applying the medication."

2. "I should apply a very thin layer to my skin." Rationale: Tretinoin is applied liberally to the skin. The hands are washed thoroughly immediately after applying. Therapeutic results should be seen after 2 to 3 weeks but may not be optimal until after 6 weeks. The skin needs to be cleansed thoroughly before applying the medication.

78.) A client is taking phenytoin (Dilantin) for seizure control and a sample for a serum drug level is drawn. Which of the following indicates a therapeutic serum drug range? 1. 5 to 10 mcg/mL 2. 10 to 20 mcg/mL 3. 20 to 30 mcg/mL 4. 30 to 40 mcg/mL

2. 10 to 20 mcg/mL Rationale: The therapeutic serum drug level range for phenytoin (Dilantin) is 10 to 20 mcg/mL. ** A helpful hint may be to remember that the theophylline therapeutic range and the acetaminophen (Tylenol) therapeutic range are the same as the phenytoin (Dilantin) therapeutic range.**

215.) A client with rheumatoid arthritis is taking acetylsalicylic acid (aspirin) on a daily basis. Which medication dose should the nurse expect the client to be taking? 1. 1 g daily 2. 4 g daily 3. 325 mg daily 4. 1000 mg daily

2. 4 g daily Rationale: Aspirin may be used to treat the client with rheumatoid arthritis. It may also be used to reduce the risk of recurrent transient ischemic attack (TIA) or brain attack (stroke) or reduce the risk of myocardial infarction (MI) in clients with unstable angina or a history of a previous MI. The normal dose for clients being treated with aspirin to decrease thrombosis and MI is 300 to 325 mg/day. Clients being treated to prevent TIAs are usually prescribed 1.3 g/day in two to four divided doses. Clients with rheumatoid arthritis are treated with 3.6 to 5.4 g/day in divided doses. **Eliminate options 1 and 4 because they are alike**

29.) A client is taking Humulin NPH insulin daily every morning. The nurse reinforces instructions for the client and tells the client that the most likely time for a hypoglycemic reaction to occur is: 1. 2 to 4 hours after administration 2. 4 to 12 hours after administration 3. 16 to 18 hours after administration 4. 18 to 24 hours after administration

2. 4 to 12 hours after administration Rationale: Humulin NPH is an intermediate-acting insulin. The onset of action is 1.5 hours, it peaks in 4 to 12 hours, and its duration of action is 24 hours. Hypoglycemic reactions most likely occur during peak time.

64.) Nalidixic acid (NegGram) is prescribed for a client with a urinary tract infection. On review of the client's record, the nurse notes that the client is taking warfarin sodium (Coumadin) daily. Which prescription should the nurse anticipate for this client? 1. Discontinuation of warfarin sodium (Coumadin) 2. A decrease in the warfarin sodium (Coumadin) dosage 3. An increase in the warfarin sodium (Coumadin) dosage 4. A decrease in the usual dose of nalidixic acid (NegGram)

2. A decrease in the warfarin sodium (Coumadin) dosage Rationale: Nalidixic acid can intensify the effects of oral anticoagulants by displacing these agents from binding sites on plasma protein. When an oral anticoagulant is combined with nalidixic acid, a decrease in the anticoagulant dosage may be needed.

88.) Dantrolene sodium (Dantrium) is prescribed for a client experiencing flexor spasms, and the client asks the nurse about the action of the medication. The nurse responds, knowing that the therapeutic action of this medication is which of the following? 1. Depresses spinal reflexes 2. Acts directly on the skeletal muscle to relieve spasticity 3. Acts within the spinal cord to suppress hyperactive reflexes 4. Acts on the central nervous system (CNS) to suppress spasms

2. Acts directly on the skeletal muscle to relieve spasticity Rationale: Dantrium acts directly on skeletal muscle to relieve muscle spasticity. The primary action is the suppression of calcium release from the sarcoplasmic reticulum. This in turn decreases the ability of the skeletal muscle to contract. **Options 1, 3, and 4 are all comparable or alike in that they address CNS suppression and the depression of reflexes. Therefore, eliminate these options.**

86.) A nurse is reinforcing discharge instructions to a client receiving baclofen (Lioresal). Which of the following would the nurse include in the instructions? 1. Restrict fluid intake. 2. Avoid the use of alcohol. 3. Stop the medication if diarrhea occurs. 4. Notify the health care provider if fatigue occurs.

2. Avoid the use of alcohol. Rationale: Baclofen is a central nervous system (CNS) depressant. The client should be cautioned against the use of alcohol and other CNS depressants, because baclofen potentiates the depressant activity of these agents. Constipation rather than diarrhea is an adverse effect of baclofen. It is not necessary to restrict fluids, but the client should be warned that urinary retention can occur. Fatigue is related to a CNS effect that is most intense during the early phase of therapy and diminishes with continued medication use. It is not necessary that the client notify the health care provider if fatigue occurs.

199.) A nurse is applying a topical glucocorticoid to a client with eczema. The nurse monitors for systemic absorption of the medication if the medication is being applied to which of the following body areas? 1. Back 2. Axilla 3. Soles of the feet 4. Palms of the hands

2. Axilla Rationale: Topical glucocorticoids can be absorbed into the systemic circulation. Absorption is higher from regions where the skin is especially permeable (scalp, axillae, face, eyelids, neck, perineum, genitalia), and lower from regions where penetrability is poor (back, palms, soles). **Eliminate options 3 and 4 because these body areas are similar in terms of skin characteristics**

123.) A nurse is planning to administer amlodipine (Norvasc) to a client. The nurse plans to check which of the following before giving the medication? 1. Respiratory rate 2. Blood pressure and heart rate 3. Heart rate and respiratory rate 4. Level of consciousness and blood pressure

2. Blood pressure and heart rate Rationale: Amlodipine is a calcium channel blocker. This medication decreases the rate and force of cardiac contraction. Before administering a calcium channel blocking agent, the nurse should check the blood pressure and heart rate, which could both decrease in response to the action of this medication. This action will help to prevent or identify early problems related to decreased cardiac contractility, heart rate, and conduction. **amlodipine is a calcium channel blocker, and this group of medications decreases the rate and force of cardiac contraction. This in turn lowers the pulse rate and blood pressure.**

224.) Neuroleptic malignant syndrome is suspected in a client who is taking chlorpromazine. Which medication would the nurse prepare in anticipation of being prescribed to treat this adverse effect related to the use of chlorpromazine? 1. Protamine sulfate 2. Bromocriptine (Parlodel) 3. Phytonadione (vitamin K) 4. Enalapril maleate (Vasotec)

2. Bromocriptine (Parlodel) Rationale: Bromocriptine is an antiparkinsonian prolactin inhibitor used in the treatment of neuroleptic malignant syndrome. Vitamin K is the antidote for warfarin (Coumadin) overdose. Protamine sulfate is the antidote for heparin overdose. Enalapril maleate is an antihypertensive used in the treatment of hypertension.

20.) The client with metastatic breast cancer is receiving tamoxifen. The nurse specifically monitors which laboratory value while the client is taking this medication? 1. Glucose level 2. Calcium level 3. Potassium level 4. Prothrombin time

2. Calcium level Rationale: Tamoxifen may increase calcium, cholesterol, and triglyceride levels. Before the initiation of therapy, a complete blood count, platelet count, and serum calcium levels should be assessed. These blood levels, along with cholesterol and triglyceride levels, should be monitored periodically during therapy. The nurse should assess for hypercalcemia while the client is taking this medication. Signs of hypercalcemia include increased urine volume, excessive thirst, nausea, vomiting, constipation, hypotonicity of muscles, and deep bone and flank pain.

170.) Atenolol hydrochloride (Tenormin) is prescribed for a hospitalized client. The nurse should perform which of the following as a priority action before administering the medication? 1. Listen to the client's lung sounds. 2. Check the client's blood pressure. 3. Check the recent electrolyte levels. 4. Assess the client for muscle weakness.

2. Check the client's blood pressure. Rationale: Atenolol hydrochloride is a beta-blocker used to treat hypertension. Therefore the priority nursing action before administration of the medication is to check the client's blood pressure. The nurse also checks the client's apical heart rate. If the systolic blood pressure is below 90 mm Hg or the apical pulse is 60 beats per minute or lower, the medication is withheld and the registered nurse and/or health care provider is notified. The nurse would check baseline renal and liver function tests. The medication may cause weakness, and the nurse would assist the client with activities if weakness occurs. **Beta-blockers have "-lol" at the end of the medication name**

34.) A client with Crohn's disease is scheduled to receive an infusion of infliximab (Remicade). The nurse assisting in caring for the client should take which action to monitor the effectiveness of treatment? 1. Monitoring the leukocyte count for 2 days after the infusion 2. Checking the frequency and consistency of bowel movements 3. Checking serum liver enzyme levels before and after the infusion 4. Carrying out a Hematest on gastric fluids after the infusion is completed

2. Checking the frequency and consistency of bowel movements Rationale: The principal manifestations of Crohn's disease are diarrhea and abdominal pain. Infliximab (Remicade) is an immunomodulator that reduces the degree of inflammation in the colon, thereby reducing the diarrhea. Options 1, 3, and 4 are unrelated to this medication.

146.) A client has begun therapy with theophylline (Theo-24). The nurse tells the client to limit the intake of which of the following while taking this medication? 1. Oranges and pineapple 2. Coffee, cola, and chocolate 3. Oysters, lobster, and shrimp 4. Cottage cheese, cream cheese, and dairy creamers

2. Coffee, cola, and chocolate Rationale: Theophylline is a xanthine bronchodilator. The nurse teaches the client to limit the intake of xanthine-containing foods while taking this medication. These include coffee, cola, and chocolate.

63.) A client with coronary artery disease complains of substernal chest pain. After checking the client's heart rate and blood pressure, a nurse administers nitroglycerin, 0.4 mg, sublingually. After 5 minutes, the client states, "My chest still hurts." Select the appropriate actions that the nurse should take. Select all that apply. 1. Call a code blue. 2. Contact the registered nurse. 3. Contact the client's family. 4. Assess the client's pain level. 5. Check the client's blood pressure. 6. Administer a second nitroglycerin, 0.4 mg, sublingually.

2. Contact the registered nurse. 4. Assess the client's pain level. 5. Check the client's blood pressure. 6. Administer a second nitroglycerin, 0.4 mg, sublingually. Rationale: The usual guideline for administering nitroglycerin tablets for a hospitalized client with chest pain is to administer one tablet every 5 minutes PRN for chest pain, for a total dose of three tablets. The registered nurse should be notified of the client's condition, who will then notify the health care provider as appropriate. Because the client is still complaining of chest pain, the nurse would administer a second nitroglycerin tablet. The nurse would assess the client's pain level and check the client's blood pressure before administering each nitroglycerin dose. There are no data in the question that indicate the need to call a code blue. In addition, it is not necessary to contact the client's family unless the client has requested this.

84.) Baclofen (Lioresal) is prescribed for the client with multiple sclerosis. The nurse assists in planning care, knowing that the primary therapeutic effect of this medication is which of the following? 1. Increased muscle tone 2. Decreased muscle spasms 3. Increased range of motion 4. Decreased local pain and tenderness

2. Decreased muscle spasms Rationale: Baclofen is a skeletal muscle relaxant and central nervous system depressant and acts at the spinal cord level to decrease the frequency and amplitude of muscle spasms in clients with spinal cord injuries or diseases and in clients with multiple sclerosis. Options 1, 3, and 4 are incorrect.

209.) A client with multiple sclerosis is receiving diazepam (Valium), a centrally acting skeletal muscle relaxant. Which of the following would indicate that the client is experiencing a side effect related to this medication? 1. Headache 2. Drowsiness 3. Urinary retention 4. Increased salivation

2. Drowsiness Rationale: Incoordination and drowsiness are common side effects resulting from this medication. Options 1, 3, and 4 are incorrect.

133.) A nurse is monitoring a client receiving desmopressin acetate (DDAVP) for adverse effects to the medication. Which of the following indicates the presence of an adverse effect? 1. Insomnia 2. Drowsiness 3. Weight loss 4. Increased urination

2. Drowsiness Rationale: Water intoxication (overhydration) or hyponatremia is an adverse effect to desmopressin. Early signs include drowsiness, listlessness, and headache. Decreased urination, rapid weight gain, confusion, seizures, and coma also may occur in overhydration. **Recall that this medication is used to treat diabetes insipidus to eliminate weight loss and increased urination.**

240.) A client with Parkinson's disease has been prescribed benztropine (Cogentin). The nurse monitors for which gastrointestinal (GI) side effect of this medication? 1. Diarrhea 2. Dry mouth 3. Increased appetite 4. Hyperactive bowel sounds

2. Dry mouth Rationale: Common GI side effects of benztropine therapy include constipation and dry mouth. Other GI side effects include nausea and ileus. These effects are the result of the anticholinergic properties of the medication. **Eliminate options 1 and 4 because they are comparable or alike. Recall that the medication is an anticholinergic, which causes dry mouth**

41.) The client has been taking omeprazole (Prilosec) for 4 weeks. The ambulatory care nurse evaluates that the client is receiving optimal intended effect of the medication if the client reports the absence of which symptom? 1. Diarrhea 2. Heartburn 3. Flatulence 4. Constipation

2. Heartburn Rationale: Omeprazole is a proton pump inhibitor classified as an antiulcer agent. The intended effect of the medication is relief of pain from gastric irritation, often called heartburn by clients. Omeprazole is not used to treat the conditions identified in options 1, 3, and 4.

131.) The nurse is reinforcing medication instructions to a client with breast cancer who is receiving cyclophosphamide (Neosar). The nurse tells the client to: 1. Take the medication with food. 2. Increase fluid intake to 2000 to 3000 mL daily. 3. Decrease sodium intake while taking the medication. 4. Increase potassium intake while taking the medication.

2. Increase fluid intake to 2000 to 3000 mL daily. Rationale: Hemorrhagic cystitis is a toxic effect that can occur with the use of cyclophosphamide. The client needs to be instructed to drink copious amounts of fluid during the administration of this medication. Clients also should monitor urine output for hematuria. The medication should be taken on an empty stomach, unless gastrointestinal (GI) upset occurs. Hyperkalemia can result from the use of the medication; therefore the client would not be told to increase potassium intake. The client would not be instructed to alter sodium intake.

101.) Ketoconazole is prescribed for a client with a diagnosis of candidiasis. Select the interventions that the nurse includes when administering this medication. Select all that apply. 1. Restrict fluid intake. 2. Instruct the client to avoid alcohol. 3. Monitor hepatic and liver function studies. 4. Administer the medication with an antacid. 5. Instruct the client to avoid exposure to the sun. 6. Administer the medication on an empty stomach.

2. Instruct the client to avoid alcohol. 3. Monitor hepatic and liver function studies. 5. Instruct the client to avoid exposure to the sun. Rationale: Ketoconazole is an antifungal medication. It is administered with food (not on an empty stomach) and antacids are avoided for 2 hours after taking the medication to ensure absorption. The medication is hepatotoxic and the nurse monitors liver function studies. The client is instructed to avoid exposure to the sun because the medication increases photosensitivity. The client is also instructed to avoid alcohol. There is no reason for the client to restrict fluid intake. In fact, this could be harmful to the client.

162.) Carbamazepine (Tegretol) is prescribed for a client with a diagnosis of psychomotor seizures. The nurse reviews the client's health history, knowing that this medication is contraindicated if which of the following disorders is present? 1. Headaches 2. Liver disease 3. Hypothyroidism 4. Diabetes mellitus

2. Liver disease Rationale: Carbamazepine (Tegretol) is contraindicated in liver disease, and liver function tests are routinely prescribed for baseline purposes and are monitored during therapy. It is also contraindicated if the client has a history of blood dyscrasias. It is not contraindicated in the conditions noted in the incorrect options.

89.) A nurse is reviewing the laboratory studies on a client receiving dantrolene sodium (Dantrium). Which laboratory test would identify an adverse effect associated with the administration of this medication? 1. Creatinine 2. Liver function tests 3. Blood urea nitrogen 4. Hematological function tests

2. Liver function tests Rationale: Dose-related liver damage is the most serious adverse effect of dantrolene. To reduce the risk of liver damage, liver function tests should be performed before treatment and periodically throughout the treatment course. It is administered in the lowest effective dosage for the shortest time necessary. **Eliminate options 1 and 3 because these tests both assess kidney function.**

161.) A nurse is caring for a client with severe back pain, and codeine sulfate has been prescribed for the client. Which of the following would the nurse include in the plan of care while the client is taking this medication? 1. Restrict fluid intake. 2. Monitor bowel activity. 3. Monitor for hypertension. 4. Monitor peripheral pulses.

2. Monitor bowel activity. Rationale: While the client is taking codeine sulfate, an opioid analgesic, the nurse would monitor vital signs and monitor for hypotension. The nurse should also increase fluid intake, palpate the bladder for urinary retention, auscultate bowel sounds, and monitor the pattern of daily bowel activity and stool consistency (codeine can cause constipation). The nurse should monitor respiratory status and initiate breathing and coughing exercises. In addition, the nurse monitors the effectiveness of the pain medication.

204.) A client receives a dose of edrophonium (Enlon). The client shows improvement in muscle strength for a period of time following the injection. The nurse interprets that this finding is compatible with: 1. Multiple sclerosis 2. Myasthenia gravis 3. Muscular dystrophy 4. Amyotrophic lateral sclerosis

2. Myasthenia gravis Rationale: Myasthenia gravis can often be diagnosed based on clinical signs and symptoms. The diagnosis can be confirmed by injecting the client with a dose of edrophonium . This medication inhibits the breakdown of an enzyme in the neuromuscular junction, so more acetylcholine binds to receptors. If the muscle is strengthened for 3 to 5 minutes after this injection, it confirms a diagnosis of myasthenia gravis. Another medication, neostigmine (Prostigmin), also may be used because its effect lasts for 1 to 2 hours, providing a better analysis. For either medication, atropine sulfate should be available as the antidote.

227.) When teaching a client who is being started on imipramine hydrochloride (Tofranil), the nurse would inform the client that the desired effects of the medication may: 1. Start during the first week of administration 2. Not occur for 2 to 3 weeks of administration 3. Start during the second week of administration 4. Not occur until after a month of administration

2. Not occur for 2 to 3 weeks of administration Rationale: The therapeutic effects of administration of imipramine hydrochloride may not occur for 2 to 3 weeks after the antidepressant therapy has been initiated. Therefore options 1, 3, and 4 are incorrect.

169.) Insulin glargine (Lantus) is prescribed for a client with diabetes mellitus. The nurse tells the client that it is best to take the insulin: 1. 1 hour after each meal 2. Once daily, at the same time each day 3. 15 minutes before breakfast, lunch, and dinner 4. Before each meal, on the basis of the blood glucose level

2. Once daily, at the same time each day Rationale: Insulin glargine is a long-acting recombinant DNA human insulin used to treat type 1 and type 2 diabetes mellitus. It has a 24-hour duration of action and is administered once a day, at the same time each day.

47.) A client has been taking isoniazid (INH) for 2 months. The client complains to a nurse about numbness, paresthesias, and tingling in the extremities. The nurse interprets that the client is experiencing: 1. Hypercalcemia 2. Peripheral neuritis 3. Small blood vessel spasm 4. Impaired peripheral circulation

2. Peripheral neuritis Rationale: A common side effect of the TB drug INH is peripheral neuritis. This is manifested by numbness, tingling, and paresthesias in the extremities. This side effect can be minimized by pyridoxine (vitamin B6) intake. Options 1, 3, and 4 are incorrect.

228.) A client receiving an anxiolytic medication complains that he feels very "faint" when he tries to get out of bed in the morning. The nurse recognizes this complaint as a symptom of: 1. Cardiac dysrhythmias 2. Postural hypotension 3. Psychosomatic symptoms 4. Respiratory insufficiency

2. Postural hypotension Rationale: Anxiolytic medications can cause postural hypotension. The client needs to be taught to rise to a sitting position and get out of bed slowly because of this adverse effect related to the medication. Options 1, 3, and 4 are unrelated to the use of this medication.

50.) A nurse has given a client taking ethambutol (Myambutol) information about the medication. The nurse determines that the client understands the instructions if the client states that he or she will immediately report: 1. Impaired sense of hearing 2. Problems with visual acuity 3. Gastrointestinal (GI) side effects 4. Orange-red discoloration of body secretions

2. Problems with visual acuity Rationale: Ethambutol causes optic neuritis, which decreases visual acuity and the ability to discriminate between the colors red and green. This poses a potential safety hazard when a client is driving a motor vehicle. The client is taught to report this symptom immediately. The client is also taught to take the medication with food if GI upset occurs. Impaired hearing results from antitubercular therapy with streptomycin. Orange-red discoloration of secretions occurs with rifampin (Rifadin).

25.) A home care nurse visits a client recently diagnosed with diabetes mellitus who is taking Humulin NPH insulin daily. The client asks the nurse how to store the unopened vials of insulin. The nurse tells the client to: 1. Freeze the insulin. 2. Refrigerate the insulin. 3. Store the insulin in a dark, dry place. 4. Keep the insulin at room temperature.

2. Refrigerate the insulin. Rationale: Insulin in unopened vials should be stored under refrigeration until needed. Vials should not be frozen. When stored unopened under refrigeration, insulin can be used up to the expiration date on the vial. Options 1, 3, and 4 are incorrect.

186.) A nurse prepares to administer sodium polystyrene sulfonate (Kayexalate) to a client. Before administering the medication, the nurse reviews the action of the medication and understands that it: 1. Releases bicarbonate in exchange for primarily sodium ions 2. Releases sodium ions in exchange for primarily potassium ions 3. Releases potassium ions in exchange for primarily sodium ions 4. Releases sodium ions in exchange for primarily bicarbonate ions

2. Releases sodium ions in exchange for primarily potassium ions Rationale: Sodium polystyrene sulfonate is a cation exchange resin used in the treatment of hyperkalemia. The resin either passes through the intestine or is retained in the colon. It releases sodium ions in exchange for primarily potassium ions. The therapeutic effect occurs 2 to 12 hours after oral administration and longer after rectal administration.

40.) The client who chronically uses nonsteroidal anti-inflammatory drugs has been taking misoprostol (Cytotec). The nurse determines that the medication is having the intended therapeutic effect if which of the following is noted? 1. Resolved diarrhea 2. Relief of epigastric pain 3. Decreased platelet count 4. Decreased white blood cell count

2. Relief of epigastric pain Rationale: The client who chronically uses nonsteroidal anti-inflammatory drugs (NSAIDs) is prone to gastric mucosal injury. Misoprostol is a gastric protectant and is given specifically to prevent this occurrence. Diarrhea can be a side effect of the medication, but is not an intended effect. Options 3 and 4 are incorrect.

48.) A client is to begin a 6-month course of therapy with isoniazid (INH). A nurse plans to teach the client to: 1. Drink alcohol in small amounts only. 2. Report yellow eyes or skin immediately. 3. Increase intake of Swiss or aged cheeses. 4. Avoid vitamin supplements during therapy.

2. Report yellow eyes or skin immediately. Rationale: INH is hepatotoxic, and therefore the client is taught to report signs and symptoms of hepatitis immediately (which include yellow skin and sclera). For the same reason, alcohol should be avoided during therapy. The client should avoid intake of Swiss cheese, fish such as tuna, and foods containing tyramine because they may cause a reaction characterized by redness and itching of the skin, flushing, sweating, tachycardia, headache, or lightheadedness. The client can avoid developing peripheral neuritis by increasing the intake of pyridoxine (vitamin B6) during the course of INH therapy for TB.

23.) A client who has been newly diagnosed with diabetes mellitus has been stabilized with daily insulin injections. Which information should the nurse teach when carrying out plans for discharge? 1. Keep insulin vials refrigerated at all times. 2. Rotate the insulin injection sites systematically. 3. Increase the amount of insulin before unusual exercise. 4. Monitor the urine acetone level to determine the insulin dosage.

2. Rotate the insulin injection sites systematically. Rationale: Insulin dosages should not be adjusted or increased before unusual exercise. If acetone is found in the urine, it may possibly indicate the need for additional insulin. To minimize the discomfort associated with insulin injections, the insulin should be administered at room temperature. Injection sites should be systematically rotated from one area to another. The client should be instructed to give injections in one area, about 1 inch apart, until the whole area has been used and then to change to another site. This prevents dramatic changes in daily insulin absorption.

216.) A nurse is caring for a client with gout who is taking Colcrys (colchicine). The client has been instructed to restrict the diet to low-purine foods. Which of the following foods should the nurse instruct the client to avoid while taking this medication? 1. Spinach 2. Scallops 3. Potatoes 4. Ice cream

2. Scallops Rationale: Colchicine is a medication used for clients with gout to inhibit the reabsorption of uric acid by the kidney and promote excretion of uric acid in the urine. Uric acid is produced when purine is catabolized. Clients are instructed to modify their diet and limit excessive purine intake. High-purine foods to avoid or limit include organ meats, roe, sardines, scallops, anchovies, broth, mincemeat, herring, shrimp, mackerel, gravy, and yeast.

114.) A postoperative client requests medication for flatulence (gas pains). Which medication from the following PRN list should the nurse administer to this client? 1. Ondansetron (Zofran) 2. Simethicone (Mylicon) 3. Acetaminophen (Tylenol) 4. Magnesium hydroxide (milk of magnesia, MOM)

2. Simethicone (Mylicon) Rationale: Simethicone is an antiflatulent used in the relief of pain caused by excessive gas in the gastrointestinal tract. Ondansetron is used to treat postoperative nausea and vomiting. Acetaminophen is a nonopioid analgesic. Magnesium hydroxide is an antacid and laxative.

188.) The nurse should anticipate that the most likely medication to be prescribed prophylactically for a child with spina bifida (myelomeningocele) who has a neurogenic bladder would be: 1. Prednisone 2. Sulfisoxazole 3. Furosemide (Lasix) 4. Intravenous immune globulin (IVIG)

2. Sulfisoxazole Rationale: A neurogenic bladder prevents the bladder from completely emptying because of the decrease in muscle tone. The most likely medication to be prescribed to prevent urinary tract infection would be an antibiotic. A common prescribed medication is sulfisoxazole. Prednisone relieves allergic reactions and inflammation rather than preventing infection. Furosemide promotes diuresis and decreases edema caused by congestive heart failure. IVIG assists with antibody production in immunocompromised clients.

147.) A client with a prescription to take theophylline (Theo-24) daily has been given medication instructions by the nurse. The nurse determines that the client needs further information about the medication if the client states that he or she will: 1. Drink at least 2 L of fluid per day. 2. Take the daily dose at bedtime. 3. Avoid changing brands of the medication without health care provider (HCP) approval. 4. Avoid over-the-counter (OTC) cough and cold medications unless approved by the HCP.

2. Take the daily dose at bedtime. Rationale: The client taking a single daily dose of theophylline, a xanthine bronchodilator, should take the medication early in the morning. This enables the client to have maximal benefit from the medication during daytime activities. In addition, this medication causes insomnia. The client should take in at least 2 L of fluid per day to decrease viscosity of secretions. The client should check with the physician before changing brands of the medication. The client also checks with the HCP before taking OTC cough, cold, or other respiratory preparations because they could cause interactive effects, increasing the side effects of theophylline and causing dysrhythmias.

45.) A client has a prescription to take guaifenesin (Humibid) every 4 hours, as needed. The nurse determines that the client understands the most effective use of this medication if the client states that he or she will: 1. Watch for irritability as a side effect. 2. Take the tablet with a full glass of water. 3. Take an extra dose if the cough is accompanied by fever. 4. Crush the sustained-release tablet if immediate relief is needed.

2. Take the tablet with a full glass of water. Rationale: Guaifenesin is an expectorant. It should be taken with a full glass of water to decrease viscosity of secretions. Sustained-release preparations should not be broken open, crushed, or chewed. The medication may occasionally cause dizziness, headache, or drowsiness as side effects. The client should contact the health care provider if the cough lasts longer than 1 week or is accompanied by fever, rash, sore throat, or persistent headache.

214.) A health care provider initiates carbidopa/levodopa (Sinemet) therapy for the client with Parkinson's disease. A few days after the client starts the medication, the client complains of nausea and vomiting. The nurse tells the client that: 1. Taking an antiemetic is the best measure to prevent the nausea. 2. Taking the medication with food will help to prevent the nausea. 3. This is an expected side effect of the medication and will decrease over time. 4. The nausea and vomiting will decrease when the dose of levodopa is stabilized.

2. Taking the medication with food will help to prevent the nausea. Rationale: If carbidopa/levodopa is causing nausea and vomiting, the nurse would tell the client that taking the medication with food will prevent the nausea. Additionally, the client should be instructed not to take the medication with a high-protein meal because the high-protein will affect absorption. Antiemetics from the phenothiazine class should not be used because they block the therapeutic action of dopamine. **eliminate options 3 and 4 because they are comparable or alike**

57.) A nurse is monitoring a client who is taking propranolol (Inderal LA). Which data collection finding would indicate a potential serious complication associated with propranolol? 1. The development of complaints of insomnia 2. The development of audible expiratory wheezes 3. A baseline blood pressure of 150/80 mm Hg followed by a blood pressure of 138/72 mm Hg after two doses of the medication 4. A baseline resting heart rate of 88 beats/min followed by a resting heart rate of 72 beats/min after two doses of the medication

2. The development of audible expiratory wheezes Rationale: Audible expiratory wheezes may indicate a serious adverse reaction, bronchospasm. β-Blockers may induce this reaction, particularly in clients with chronic obstructive pulmonary disease or asthma. Normal decreases in blood pressure and heart rate are expected. Insomnia is a frequent mild side effect and should be monitored.

28.) The health care provider (HCP) prescribes exenatide (Byetta) for a client with type 1 diabetes mellitus who takes insulin. The nurse knows that which of the following is the appropriate intervention? 1. The medication is administered within 60 minutes before the morning and evening meal. 2. The medication is withheld and the HCP is called to question the prescription for the client. 3. The client is monitored for gastrointestinal side effects after administration of the medication. 4. The insulin is withdrawn from the Penlet into an insulin syringe to prepare for administration.

2. The medication is withheld and the HCP is called to question the prescription for the client. Rationale: Exenatide (Byetta) is an incretin mimetic used for type 2 diabetes mellitus only. It is not recommended for clients taking insulin. Hence, the nurse should hold the medication and question the HCP regarding this prescription. Although options 1 and 3 are correct statements about the medication, in this situation the medication should not be administered. The medication is packaged in prefilled pens ready for injection without the need for drawing it up into another syringe.

165.) The client has been on treatment for rheumatoid arthritis for 3 weeks. During the administration of etanercept (Enbrel), it is most important for the nurse to assess: 1. The injection site for itching and edema 2. The white blood cell counts and platelet counts 3. Whether the client is experiencing fatigue and joint pain 4. A metallic taste in the mouth and a loss of appetite

2. The white blood cell counts and platelet counts Rationale: Infection and pancytopenia are adverse effects of etanercept (Enbrel). Laboratory studies are performed before and during treatment. The appearance of abnormal white blood cell counts and abnormal platelet counts can alert the nurse to a potential life-threatening infection. Injection site itching is a common occurrence following administration of the medication. In early treatment, residual fatigue and joint pain may still be apparent. A metallic taste and loss of appetite are not common signs of side effects of this medication.

83.) The client has been on treatment for rheumatoid arthritis for 3 weeks. During the administration of etanercept (Enbrel), it is most important for the nurse to check: 1. The injection site for itching and edema 2. The white blood cell counts and platelet counts 3. Whether the client is experiencing fatigue and joint pain 4. A metallic taste in the mouth, with a loss of appetite

2. The white blood cell counts and platelet counts Rationale: Infection and pancytopenia are side effects of etanercept (Enbrel). Laboratory studies are performed before and during drug treatment. The appearance of abnormal white blood cell counts and abnormal platelet counts can alert the nurse to a potentially life-threatening infection. Injection site itching is a common occurrence following administration. A metallic taste with loss of appetite are not common signs of side effects of this medication.

51.) Cycloserine (Seromycin) is added to the medication regimen for a client with tuberculosis. Which of the following would the nurse include in the client-teaching plan regarding this medication? 1. To take the medication before meals 2. To return to the clinic weekly for serum drug-level testing 3. It is not necessary to call the health care provider (HCP) if a skin rash occurs. 4. It is not necessary to restrict alcohol intake with this medication.

2. To return to the clinic weekly for serum drug-level testing Rationale: Cycloserine (Seromycin) is an antitubercular medication that requires weekly serum drug level determinations to monitor for the potential of neurotoxicity. Serum drug levels lower than 30 mcg/mL reduce the incidence of neurotoxicity. The medication must be taken after meals to prevent gastrointestinal irritation. The client must be instructed to notify the HCP if a skin rash or signs of central nervous system toxicity are noted. Alcohol must be avoided because it increases the risk of seizure activity.

82.) A client is receiving meperidine hydrochloride (Demerol) for pain. Which of the following are side effects of this medication. Select all that apply. 1. Diarrhea 2. Tremors 3. Drowsiness 4. Hypotension 5. Urinary frequency 6. Increased respiratory rate

2. Tremors 3. Drowsiness 4. Hypotension Rationale: Meperidine hydrochloride is an opioid analgesic. Side effects include respiratory depression, drowsiness, hypotension, constipation, urinary retention, nausea, vomiting, and tremors.

135.) A nurse reinforces medication instructions to a client who is taking levothyroxine (Synthroid). The nurse instructs the client to notify the health care provider (HCP) if which of the following occurs? 1. Fatigue 2. Tremors 3. Cold intolerance 4. Excessively dry skin

2. Tremors Rationale: Excessive doses of levothyroxine (Synthroid) can produce signs and symptoms of hyperthyroidism. These include tachycardia, chest pain, tremors, nervousness, insomnia, hyperthermia, heat intolerance, and sweating. The client should be instructed to notify the HCP if these occur. Options 1, 3, and 4 are signs of hypothyroidism.

7.) Isotretinoin is prescribed for a client with severe acne. Before the administration of this medication, the nurse anticipates that which laboratory test will be prescribed? 1. Platelet count 2. Triglyceride level 3. Complete blood count 4. White blood cell count

2. Triglyceride level Rationale: Isotretinoin can elevate triglyceride levels. Blood triglyceride levels should be measured before treatment and periodically thereafter until the effect on the triglycerides has been evaluated. Options 1, 3, and 4 do not need to be monitored specifically during this treatment.

14.) The client with acute myelocytic leukemia is being treated with busulfan (Myleran). Which laboratory value would the nurse specifically monitor during treatment with this medication? 1. Clotting time 2. Uric acid level 3. Potassium level 4. Blood glucose level

2. Uric acid level Rationale: Busulfan (Myleran) can cause an increase in the uric acid level. Hyperuricemia can produce uric acid nephropathy, renal stones, and acute renal failure. Options 1, 3, and 4 are not specifically related to this medication.

68.) Bethanechol chloride (Urecholine) is prescribed for a client with urinary retention. Which disorder would be a contraindication to the administration of this medication? 1. Gastric atony 2. Urinary strictures 3. Neurogenic atony 4. Gastroesophageal reflux

2. Urinary strictures Rationale: Bethanechol chloride (Urecholine) can be harmful to clients with urinary tract obstruction or weakness of the bladder wall. The medication has the ability to contract the bladder and thereby increase pressure within the urinary tract. Elevation of pressure within the urinary tract could rupture the bladder in clients with these conditions.

238.) Ribavirin (Virazole) is prescribed for the hospitalized child with respiratory syncytial virus (RSV). The nurse prepares to administer this medication via which of the following routes? 1. Orally 2. Via face mask 3. Intravenously 4. Intramuscularly

2. Via face mask Rationale: Ribavirin is an antiviral respiratory medication used mainly in hospitalized children with severe RSV and in high-risk children. Administration is via hood, face mask, or oxygen tent. The medication is most effective if administered within the first 3 days of the infection.

CPR per infant breath rate

20 per min mouth to nose

SSRI's (antidepressants) take about _____to work.

3 weeks

Cranial nerves for Assessing extraocular eye movements?

3,4,6

128.) A nurse is providing instructions to an adolescent who has a history of seizures and is taking an anticonvulsant medication. Which of the following statements indicates that the client understands the instructions? 1. "I will never be able to drive a car." 2. "My anticonvulsant medication will clear up my skin." 3. "I can't drink alcohol while I am taking my medication." 4. "If I forget my morning medication, I can take two pills at bedtime."

3. "I can't drink alcohol while I am taking my medication." Rationale: Alcohol will lower the seizure threshold and should be avoided. Adolescents can obtain a driver's license in most states when they have been seizure free for 1 year. Anticonvulsants cause acne and oily skin; therefore a dermatologist may need to be consulted. If an anticonvulsant medication is missed, the health care provider should be notified.

164.) A client receives a prescription for methocarbamol (Robaxin), and the nurse reinforces instructions to the client regarding the medication. Which client statement would indicate a need for further instructions? 1. "My urine may turn brown or green." 2. "This medication is prescribed to help relieve my muscle spasms." 3. "If my vision becomes blurred, I don't need to be concerned about it." 4. "I need to call my doctor if I experience nasal congestion from this medication."

3. "If my vision becomes blurred, I don't need to be concerned about it." Rationale: The client needs to be told that the urine may turn brown, black, or green. Other adverse effects include blurred vision, nasal congestion, urticaria, and rash. The client needs to be instructed that, if these adverse effects occur, the health care provider needs to be notified. The medication is used to relieve muscle spasms.

11.) The health care provider has prescribed silver sulfadiazine (Silvadene) for the client with a partial-thickness burn, which has cultured positive for gram-negative bacteria. The nurse is reinforcing information to the client about the medication. Which statement made by the client indicates a lack of understanding about the treatments? 1. "The medication is an antibacterial." 2. "The medication will help heal the burn." 3. "The medication will permanently stain my skin." 4. "The medication should be applied directly to the wound."

3. "The medication will permanently stain my skin." Rationale: Silver sulfadiazine (Silvadene) is an antibacterial that has a broad spectrum of activity against gram-negative bacteria, gram-positive bacteria, and yeast. It is applied directly to the wound to assist in healing. It does not stain the skin.

42.) A client with a peptic ulcer is diagnosed with a Helicobacter pylori infection. The nurse is reinforcing teaching for the client about the medications prescribed, including clarithromycin (Biaxin), esomeprazole (Nexium), and amoxicillin (Amoxil). Which statement by the client indicates the best understanding of the medication regimen? 1. "My ulcer will heal because these medications will kill the bacteria." 2. "These medications are only taken when I have pain from my ulcer." 3. "The medications will kill the bacteria and stop the acid production." 4. "These medications will coat the ulcer and decrease the acid production in my stomach."

3. "The medications will kill the bacteria and stop the acid production." Rationale: Triple therapy for Helicobacter pylori infection usually includes two antibacterial drugs and a proton pump inhibitor. Clarithromycin and amoxicillin are antibacterials. Esomeprazole is a proton pump inhibitor. These medications will kill the bacteria and decrease acid production.

141.) The nurse has reinforced instructions to a client who has been prescribed cholestyramine (Questran). Which statement by the client indicates a need for further instructions? 1. "I will continue taking vitamin supplements." 2. "This medication will help lower my cholesterol." 3. "This medication should only be taken with water." 4. "A high-fiber diet is important while taking this medication."

3. "This medication should only be taken with water." Rationale: Cholestyramine (Questran) is a bile acid sequestrant used to lower the cholesterol level, and client compliance is a problem because of its taste and palatability. The use of flavored products or fruit juices can improve the taste. Some side effects of bile acid sequestrants include constipation and decreased vitamin absorption. **Note the closed-ended word "only" in option 3**

219.) A health care provider instructs a client with rheumatoid arthritis to take ibuprofen (Motrin). The nurse reinforces the instructions, knowing that the normal adult dose for this client is which of the following? 1. 100 mg orally twice a day 2. 200 mg orally twice a day 3. 400 mg orally three times a day 4. 1000 mg orally four times a day

3. 400 mg orally three times a day Rationale: For acute or chronic rheumatoid arthritis or osteoarthritis, the normal oral adult dose is 400 to 800 mg three or four times daily.

191.) A child is brought to the emergency department for treatment of an acute asthma attack. The nurse prepares to administer which of the following medications first? 1. Oral corticosteroids 2. A leukotriene modifier 3. A β2 agonist 4. A nonsteroidal anti-inflammatory

3. A β2 agonist Rationale: In treating an acute asthma attack, a short acting β2 agonist such as albuterol (Proventil HFA) will be given to produce bronchodilation. Options 1, 2, and 4 are long-term control (preventive) medications.

120.) A client is taking lansoprazole (Prevacid) for the chronic management of Zollinger-Ellison syndrome. The nurse advises the client to take which of the following products if needed for a headache? 1. Naprosyn (Aleve) 2. Ibuprofen (Advil) 3. Acetaminophen (Tylenol) 4. Acetylsalicylic acid (aspirin)

3. Acetaminophen (Tylenol) Rationale: Zollinger-Ellison syndrome is a hypersecretory condition of the stomach. The client should avoid taking medications that are irritating to the stomach lining. Irritants would include aspirin and nonsteroidal antiinflammatory drugs (ibuprofen). The client should be advised to take acetaminophen for headache. **Remember that options that are comparable or alike are not likely to be correct. With this in mind, eliminate options 1 and 2 first.**

233.) Diphenhydramine hydrochloride (Benadryl) is used in the treatment of allergic rhinitis for a hospitalized client with a chronic psychotic disorder. The client asks the nurse why the medication is being discontinued before hospital discharge. The nurse responds, knowing that: 1. Allergic symptoms are short in duration. 2. This medication promotes long-term extrapyramidal symptoms. 3. Addictive properties are enhanced in the presence of psychotropic medications. 4. Poor compliance causes this medication to fail to reach its therapeutic blood level.

3. Addictive properties are enhanced in the presence of psychotropic medications. Rationale: The addictive properties of diphenhydramine hydrochloride are enhanced when used with psychotropic medications. Allergic symptoms may not be short term and will occur if allergens are present in the environment. Poor compliance may be a problem with psychotic clients but is not the subject of the question. Diphenhydramine hydrochloride may be used for extrapyramidal symptoms and mild medication-induced movement disorders.

35.) The client has a PRN prescription for loperamide hydrochloride (Imodium). The nurse understands that this medication is used for which condition? 1. Constipation 2. Abdominal pain 3. An episode of diarrhea 4. Hematest-positive nasogastric tube drainage

3. An episode of diarrhea Rationale: Loperamide is an antidiarrheal agent. It is used to manage acute and also chronic diarrhea in conditions such as inflammatory bowel disease. Loperamide also can be used to reduce the volume of drainage from an ileostomy. It is not used for the conditions in options 1, 2, and 4.

207.) A client is suspected of having myasthenia gravis, and the health care provider administers edrophonium (Enlon) to determine the diagnosis. After administration of this medication, which of the following would indicate the presence of myasthenia gravis? 1. Joint pain 2. A decrease in muscle strength 3. An increase in muscle strength 4. Feelings of faintness, dizziness, hypotension, and signs of flushing in the client

3. An increase in muscle strength Rationale: Edrophonium is a short-acting acetylcholinesterase inhibitor used as a diagnostic agent. When a client with suspected myasthenia gravis is given the medication intravenously, an increase in muscle strength would be seen in 1 to 3 minutes. If no response occurs, another dose is given over the next 2 minutes, and muscle strength is again tested. If no increase in muscle strength occurs with this higher dose, the muscle weakness is not caused by myasthenia gravis. Clients receiving injections of this medication commonly demonstrate a drop of blood pressure, feel faint and dizzy, and are flushed.

139.) Prednisone is prescribed for a client with diabetes mellitus who is taking Humulin neutral protamine Hagedorn (NPH) insulin daily. Which of the following prescription changes does the nurse anticipate during therapy with the prednisone? 1. An additional dose of prednisone daily 2. A decreased amount of daily Humulin NPH insulin 3. An increased amount of daily Humulin NPH insulin 4. The addition of an oral hypoglycemic medication daily

3. An increased amount of daily Humulin NPH insulin Rationale: Glucocorticoids can elevate blood glucose levels. Clients with diabetes mellitus may need their dosages of insulin or oral hypoglycemic medications increased during glucocorticoid therapy. Therefore the other options are incorrect.

107.) A client receiving a tricyclic antidepressant arrives at the mental health clinic. Which observation indicates that the client is correctly following the medication plan? 1. Reports not going to work for this past week 2. Complains of not being able to "do anything" anymore 3. Arrives at the clinic neat and appropriate in appearance 4. Reports sleeping 12 hours per night and 3 to 4 hours during the day

3. Arrives at the clinic neat and appropriate in appearance Rationale: Depressed individuals will sleep for long periods, are not able to go to work, and feel as if they cannot "do anything." Once they have had some therapeutic effect from their medication, they will report resolution of many of these complaints as well as demonstrate an improvement in their appearance.

174.) A client with portosystemic encephalopathy is receiving oral lactulose (Chronulac) daily. The nurse assesses which of the following to determine medication effectiveness? 1. Lung sounds 2. Blood pressure 3. Blood ammonia level 4. Serum potassium level

3. Blood ammonia level Rationale: Lactulose is a hyperosmotic laxative and ammonia detoxicant. It is used to prevent or treat portosystemic encephalopathy, including hepatic precoma and coma. It also is used to treat constipation. The medication retains ammonia in the colon (decreases the blood ammonia concentration), producing an osmotic effect. It promotes increased peristalsis and bowel evacuation, expelling ammonia from the colon.

96.) The nurse is caring for a postrenal transplant client taking cyclosporine (Sandimmune, Gengraf, Neoral). The nurse notes an increase in one of the client's vital signs, and the client is complaining of a headache. What is the vital sign that is most likely increased? 1. Pulse 2. Respirations 3. Blood pressure 4. Pulse oximetry

3. Blood pressure Rationale: Hypertension can occur in a client taking cyclosporine (Sandimmune, Gengraf, Neoral), and because this client is also complaining of a headache, the blood pressure is the vital sign to be monitoring most closely. Other adverse effects include infection, nephrotoxicity, and hirsutism. Options 1, 2, and 4 are unrelated to the use of this medication.

153.) A client is diagnosed with pulmonary embolism and is to be treated with streptokinase (Streptase). A nurse would report which priority data collection finding to the registered nurse before initiating this therapy? 1. Adventitious breath sounds 2. Temperature of 99.4° F orally 3. Blood pressure of 198/110 mm Hg 4. Respiratory rate of 28 breaths/min

3. Blood pressure of 198/110 mm Hg Rationale: Thrombolytic therapy is contraindicated in a number of preexisting conditions in which there is a risk of uncontrolled bleeding, similar to the case in anticoagulant therapy. Thrombolytic therapy also is contraindicated in severe uncontrolled hypertension because of the risk of cerebral hemorrhage. Therefore the nurse would report the results of the blood pressure to the registered nurse before initiating therapy. The findings in options 1, 2, and 4 may be present in the client with pulmonary embolism.

69.) A nurse who is administering bethanechol chloride (Urecholine) is monitoring for acute toxicity associated with the medication. The nurse checks the client for which sign of toxicity? 1. Dry skin 2. Dry mouth 3. Bradycardia 4. Signs of dehydration

3. Bradycardia Rationale: Toxicity (overdose) produces manifestations of excessive muscarinic stimulation such as salivation, sweating, involuntary urination and defecation, bradycardia, and severe hypotension. Treatment includes supportive measures and the administration of atropine sulfate subcutaneously or intravenously.

49.) A client has been started on long-term therapy with rifampin (Rifadin). A nurse teaches the client that the medication: 1. Should always be taken with food or antacids 2. Should be double-dosed if one dose is forgotten 3. Causes orange discoloration of sweat, tears, urine, and feces 4. May be discontinued independently if symptoms are gone in 3 months

3. Causes orange discoloration of sweat, tears, urine, and feces Rationale: Rifampin should be taken exactly as directed as part of TB therapy. Doses should not be doubled or skipped. The client should not stop therapy until directed to do so by a health care provider. The medication should be administered on an empty stomach unless it causes gastrointestinal upset, and then it may be taken with food. Antacids, if prescribed, should be taken at least 1 hour before the medication. Rifampin causes orange-red discoloration of body secretions and will permanently stain soft contact lenses.

38.) An older client recently has been taking cimetidine (Tagamet). The nurse monitors the client for which most frequent central nervous system side effect of this medication? 1. Tremors 2. Dizziness 3. Confusion 4. Hallucinations

3. Confusion Rationale: Cimetidine is a histamine 2 (H2)-receptor antagonist. Older clients are especially susceptible to central nervous system side effects of cimetidine. The most frequent of these is confusion. Less common central nervous system side effects include headache, dizziness, drowsiness, and hallucinations.

16.) The clinic nurse is reviewing a teaching plan for the client receiving an antineoplastic medication. When implementing the plan, the nurse tells the client: 1. To take aspirin (acetylsalicylic acid) as needed for headache 2. Drink beverages containing alcohol in moderate amounts each evening 3. Consult with health care providers (HCPs) before receiving immunizations 4. That it is not necessary to consult HCPs before receiving a flu vaccine at the local health fair

3. Consult with health care providers (HCPs) before receiving immunizations Rationale: Because antineoplastic medications lower the resistance of the body, clients must be informed not to receive immunizations without a HCP's approval. Clients also need to avoid contact with individuals who have recently received a live virus vaccine. Clients need to avoid aspirin and aspirin-containing products to minimize the risk of bleeding, and they need to avoid alcohol to minimize the risk of toxicity and side effects.

150.) A client complaining of not feeling well is seen in a clinic. The client is taking several medications for the control of heart disease and hypertension. These medications include a β-blocker, digoxin (Lanoxin), and a diuretic. A tentative diagnosis of digoxin toxicity is made. Which of the following assessment data would support this diagnosis? 1. Dyspnea, edema, and palpitations 2. Chest pain, hypotension, and paresthesia 3. Double vision, loss of appetite, and nausea 4. Constipation, dry mouth, and sleep disorder

3. Double vision, loss of appetite, and nausea Rationale: Double vision, loss of appetite, and nausea are signs of digoxin toxicity. Additional signs of digoxin toxicity include bradycardia, difficulty reading, visual alterations such as green and yellow vision or seeing spots or halos, confusion, vomiting, diarrhea, decreased libido, and impotence. **gastrointestinal (GI) and visual disturbances occur with digoxin toxicity**

148.) A client is taking cetirizine hydrochloride (Zyrtec). The nurse checks for which of the following side effects of this medication? 1. Diarrhea 2. Excitability 3. Drowsiness 4. Excess salivation

3. Drowsiness Rationale: A frequent side effect of cetirizine hydrochloride (Zyrtec), an antihistamine, is drowsiness or sedation. Others include blurred vision, hypertension (and sometimes hypotension), dry mouth, constipation, urinary retention, and sweating.

85.) A nurse is monitoring a client receiving baclofen (Lioresal) for side effects related to the medication. Which of the following would indicate that the client is experiencing a side effect? 1. Polyuria 2. Diarrhea 3. Drowsiness 4. Muscular excitability

3. Drowsiness Rationale: Baclofen is a central nervous system (CNS) depressant and frequently causes drowsiness, dizziness, weakness, and fatigue. It can also cause nausea, constipation, and urinary retention. Clients should be warned about the possible reactions. Options 1, 2, and 4 are not side effects.

138.) A daily dose of prednisone is prescribed for a client. A nurse reinforces instructions to the client regarding administration of the medication and instructs the client that the best time to take this medication is: 1. At noon 2. At bedtime 3. Early morning 4. Anytime, at the same time, each day

3. Early morning Rationale: Corticosteroids (glucocorticoids) should be administered before 9:00 AM. Administration at this time helps minimize adrenal insufficiency and mimics the burst of glucocorticoids released naturally by the adrenal glands each morning. **Note the suffix "-sone," and recall that medication names that end with these letters are corticosteroids.**

71.) After kidney transplantation, cyclosporine (Sand immune) is prescribed for a client. Which laboratory result would indicate an adverse effect from the use of this medication? 1. Decreased creatinine level 2. Decreased hemoglobin level 3. Elevated blood urea nitrogen level 4. Decreased white blood cell count

3. Elevated blood urea nitrogen level Rationale: Nephrotoxicity can occur from the use of cyclosporine (Sandimmune). Nephrotoxicity is evaluated by monitoring for elevated blood urea nitrogen (BUN) and serum creatinine levels. Cyclosporine is an immunosuppressant but does not depress the bone marrow.

244.) A client has a prescription for valproic acid (Depakene) orally once daily. The nurse plans to: 1. Administer the medication with an antacid. 2. Administer the medication with a carbonated beverage. 3. Ensure that the medication is administered at the same time each day. 4. Ensure that the medication is administered 2 hours before breakfast only, when the client's stomach is empty.

3. Ensure that the medication is administered at the same time each day. Rationale: Valproic acid is an anticonvulsant, antimanic, and antimigraine medication. It may be administered with or without food. It should not be taken with an antacid or carbonated beverage because these products will affect medication absorption. The medication is administered at the same time each day to maintain therapeutic serum levels. **Use general pharmacology guidelines to assist in eliminating options 1 and 2. Eliminate option 4 because of the closed-ended word "only."**

154.) A nurse is reinforcing dietary instructions to a client who has been prescribed cyclosporine (Sandimmune). Which food item would the nurse instruct the client to avoid? 1. Red meats 2. Orange juice 3. Grapefruit juice 4. Green, leafy vegetables

3. Grapefruit juice Rationale: A compound present in grapefruit juice inhibits metabolism of cyclosporine. As a result, the consumption of grapefruit juice can raise cyclosporine levels by 50% to 100%, thereby greatly increasing the risk of toxicity. Grapefruit juice needs to be avoided. Red meats, orange juice, and green leafy vegetables are acceptable to consume.

97.) Amikacin (Amikin) is prescribed for a client with a bacterial infection. The client is instructed to contact the health care provider (HCP) immediately if which of the following occurs? 1. Nausea 2. Lethargy 3. Hearing loss 4. Muscle aches

3. Hearing loss Rationale: Amikacin (Amikin) is an aminoglycoside. Adverse effects of aminoglycosides include ototoxicity (hearing problems), confusion, disorientation, gastrointestinal irritation, palpitations, blood pressure changes, nephrotoxicity, and hypersensitivity. The nurse instructs the client to report hearing loss to the HCP immediately. Lethargy and muscle aches are not associated with the use of this medication. It is not necessary to contact the HCP immediately if nausea occurs. If nausea persists or results in vomiting, the HCP should be notified. **(most aminoglycoside medication names end in the letters -cin)**

60.) A nurse is planning to administer hydrochlorothiazide (HydroDIURIL) to a client. The nurse understands that which of the following are concerns related to the administration of this medication? 1. Hypouricemia, hyperkalemia 2. Increased risk of osteoporosis 3. Hypokalemia, hyperglycemia, sulfa allergy 4. Hyperkalemia, hypoglycemia, penicillin allergy

3. Hypokalemia, hyperglycemia, sulfa allergy Rationale: Thiazide diuretics such as hydrochlorothiazide are sulfa-based medications, and a client with a sulfa allergy is at risk for an allergic reaction. Also, clients are at risk for hypokalemia, hyperglycemia, hypercalcemia, hyperlipidemia, and hyperuricemia.

122.) A client who has begun taking fosinopril (Monopril) is very distressed, telling the nurse that he cannot taste food normally since beginning the medication 2 weeks ago. The nurse provides the best support to the client by: 1. Telling the client not to take the medication with food 2. Suggesting that the client taper the dose until taste returns to normal 3. Informing the client that impaired taste is expected and generally disappears in 2 to 3 months 4. Requesting that the health care provider (HCP) change the prescription to another brand of angiotensin-converting enzyme (ACE) inhibitor

3. Informing the client that impaired taste is expected and generally disappears in 2 to 3 months Rationale: ACE inhibitors, such as fosinopril, cause temporary impairment of taste (dysgeusia). The nurse can tell the client that this effect usually disappears in 2 to 3 months, even with continued therapy, and provide nutritional counseling if appropriate to avoid weight loss. Options 1, 2, and 4 are inappropriate actions. Taking this medication with or without food does not affect absorption and action. The dosage should never be tapered without HCP approval and the medication should never be stopped abruptly.

5.) Mafenide acetate (Sulfamylon) is prescribed for the client with a burn injury. When applying the medication, the client complains of local discomfort and burning. Which of the following is the most appropriate nursing action? 1. Notifying the registered nurse 2. Discontinuing the medication 3. Informing the client that this is normal 4. Applying a thinner film than prescribed to the burn site

3. Informing the client that this is normal Rationale: Mafenide acetate is bacteriostatic for gram-negative and gram-positive organisms and is used to treat burns to reduce bacteria present in avascular tissues. The client should be informed that the medication will cause local discomfort and burning and that this is a normal reaction; therefore options 1, 2, and 4 are incorrect

243.) A hospitalized client is having the dosage of clonazepam (Klonopin) adjusted. The nurse should plan to: 1. Weigh the client daily. 2. Observe for ecchymosis. 3. Institute seizure precautions. 4. Monitor blood glucose levels.

3. Institute seizure precautions. Rationale: Clonazepam is a benzodiazepine used as an anticonvulsant. During initial therapy and during periods of dosage adjustment, the nurse should initiate seizure precautions for the client. Options 1, 2, and 4 are not associated with the use of this medication.

230.) A client is placed on chloral hydrate (Somnote) for short-term treatment. Which nursing action indicates an understanding of the major side effect of this medication? 1. Monitoring neurological signs every 2 hours 2. Monitoring the blood pressure every 4 hours 3. Instructing the client to call for ambulation assistance 4. Lowering the bed and clearing a path to the bathroom at bedtime

3. Instructing the client to call for ambulation assistance Rationale: Chloral hydrate (a sedative-hypnotic) causes sedation and impairment of motor coordination; therefore, safety measures need to be implemented. The client is instructed to call for assistance with ambulation. Options 1 and 2 are not specifically associated with the use of this medication. Although option 4 is an appropriate nursing intervention, it is most important to instruct the client to call for assistance with ambulation.

241.) A client with a history of simple partial seizures is taking clorazepate (Tranxene), and asks the nurse if there is a risk of addiction. The nurse's response is based on the understanding that clorazepate: 1. Is not habit forming, either physically or psychologically 2. Leads to physical tolerance, but only after 10 or more years of therapy 3. Leads to physical and psychological dependence with prolonged high-dose therapy 4. Can result in psychological dependence only, because of the nature of the medication

3. Leads to physical and psychological dependence with prolonged high-dose therapy Rationale: Clorazepate is classified as an anticonvulsant, antianxiety agent, and sedative-hypnotic (benzodiazepine). One of the concerns with clorazepate therapy is that the medication can lead to physical or psychological dependence with prolonged therapy at high doses. For this reason, the amount of medication that is readily available to the client at any one time is restricted. **Eliminate options 2 and 4 first because of the closed-ended word "only"**

52.) A client with tuberculosis is being started on antituberculosis therapy with isoniazid (INH). Before giving the client the first dose, a nurse ensures that which of the following baseline studies has been completed? 1. Electrolyte levels 2. Coagulation times 3. Liver enzyme levels 4. Serum creatinine level

3. Liver enzyme levels Rationale: INH therapy can cause an elevation of hepatic enzyme levels and hepatitis. Therefore, liver enzyme levels are monitored when therapy is initiated and during the first 3 months of therapy. They may be monitored longer in the client who is greater than age 50 or abuses alcohol.

151.) A client is being treated for acute congestive heart failure with intravenously administered bumetanide. The vital signs are as follows: blood pressure, 100/60 mm Hg; pulse, 96 beats/min; and respirations, 24 breaths/min. After the initial dose, which of the following is the priority assessment? 1. Monitoring weight loss 2. Monitoring temperature 3. Monitoring blood pressure 4. Monitoring potassium level

3. Monitoring blood pressure Rationale: Bumetanide is a loop diuretic. Hypotension is a common side effect associated with the use of this medication. The other options also require assessment but are not the priority. **priority ABCs—airway, breathing, and circulation**

201.) A nurse is preparing to administer eardrops to an infant. The nurse plans to: 1. Pull up and back on the ear and direct the solution onto the eardrum. 2. Pull down and back on the ear and direct the solution onto the eardrum. 3. Pull down and back on the ear and direct the solution toward the wall of the canal. 4. Pull up and back on the ear lobe and direct the solution toward the wall of the canal.

3. Pull down and back on the ear and direct the solution toward the wall of the canal. Rationale: When administering eardrops to an infant, the nurse pulls the ear down and straight back. In the adult or a child older than 3 years, the ear is pulled up and back to straighten the auditory canal. The medication is administered by aiming it at the wall of the canal rather than directly onto the eardrum.

37.) The client has begun medication therapy with pancrelipase (Pancrease MT). The nurse evaluates that the medication is having the optimal intended benefit if which effect is observed? 1. Weight loss 2. Relief of heartburn 3. Reduction of steatorrhea 4. Absence of abdominal pain

3. Reduction of steatorrhea Rationale: Pancrelipase (Pancrease MT) is a pancreatic enzyme used in clients with pancreatitis as a digestive aid. The medication should reduce the amount of fatty stools (steatorrhea). Another intended effect could be improved nutritional status. It is not used to treat abdominal pain or heartburn. Its use could result in weight gain but should not result in weight loss if it is aiding in digestion.

137.) A nurse is reinforcing instructions for a client regarding intranasal desmopressin acetate (DDAVP). The nurse tells the client that which of the following is a side effect of the medication? 1. Headache 2. Vulval pain 3. Runny nose 4. Flushed skin

3. Runny nose Rationale: Desmopressin administered by the intranasal route can cause a runny or stuffy nose. Headache, vulval pain, and flushed skin are side effects if the medication is administered by the intravenous (IV) route.

111.) A client arrives at the health care clinic and tells the nurse that he has been doubling his daily dosage of bupropion hydrochloride (Wellbutrin) to help him get better faster. The nurse understands that the client is now at risk for which of the following? 1. Insomnia 2. Weight gain 3. Seizure activity 4. Orthostatic hypotension

3. Seizure activity Rationale: Bupropion does not cause significant orthostatic blood pressure changes. Seizure activity is common in dosages greater than 450 mg daily. Bupropion frequently causes a drop in body weight. Insomnia is a side effect, but seizure activity causes a greater client risk.

95.) The nurse is reviewing the results of serum laboratory studies drawn on a client with acquired immunodeficiency syndrome who is receiving didanosine (Videx). The nurse interprets that the client may have the medication discontinued by the health care provider if which of the following significantly elevated results is noted? 1. Serum protein 2. Blood glucose 3. Serum amylase 4. Serum creatinine

3. Serum amylase Rationale: Didanosine (Videx) can cause pancreatitis. A serum amylase level that is increased 1.5 to 2 times normal may signify pancreatitis in the client with acquired immunodeficiency syndrome and is potentially fatal. The medication may have to be discontinued. The medication is also hepatotoxic and can result in liver failure.

98.) The nurse is assigned to care for a client with cytomegalovirus retinitis and acquired immunodeficiency syndrome who is receiving foscarnet. The nurse should check the latest results of which of the following laboratory studies while the client is taking this medication? 1. CD4 cell count 2. Serum albumin 3. Serum creatinine 4. Lymphocyte count

3. Serum creatinine Rationale: Foscarnet is toxic to the kidneys. Serum creatinine is monitored before therapy, two to three times per week during induction therapy, and at least weekly during maintenance therapy. Foscarnet may also cause decreased levels of calcium, magnesium, phosphorus, and potassium. Thus these levels are also measured with the same frequency.

211.) A client with epilepsy is taking the prescribed dose of phenytoin (Dilantin) to control seizures. A phenytoin blood level is drawn, and the results reveal a level of 35 mcg/ml. Which of the following symptoms would be expected as a result of this laboratory result? 1. Nystagmus 2. Tachycardia 3. Slurred speech 4. No symptoms, because this is a normal therapeutic level

3. Slurred speech Rationale: The therapeutic phenytoin level is 10 to 20 mcg/mL. At a level higher than 20 mcg/mL, involuntary movements of the eyeballs (nystagmus) appear. At a level higher than 30 mcg/mL, ataxia and slurred speech occur.

187.) A clinic nurse prepares to administer an MMR (measles, mumps, rubella) vaccine to a child. How is this vaccine best administered? 1. Intramuscularly in the deltoid muscle 2. Subcutaneously in the gluteal muscle 3. Subcutaneously in the outer aspect of the upper arm 4. Intramuscularly in the anterolateral aspect of the thigh

3. Subcutaneously in the outer aspect of the upper arm Rationale: The MMR vaccine is administered subcutaneously in the outer aspect of the upper arm. The gluteal muscle is most often used for intramuscular injections. The MMR vaccine is not administered by the intramuscular route.

46.) A postoperative client has received a dose of naloxone hydrochloride for respiratory depression shortly after transfer to the nursing unit from the postanesthesia care unit. After administration of the medication, the nurse checks the client for: 1. Pupillary changes 2. Scattered lung wheezes 3. Sudden increase in pain 4. Sudden episodes of diarrhea

3. Sudden increase in pain Rationale: Naloxone hydrochloride is an antidote to opioids and may also be given to the postoperative client to treat respiratory depression. When given to the postoperative client for respiratory depression, it may also reverse the effects of analgesics. Therefore, the nurse must check the client for a sudden increase in the level of pain experienced. Options 1, 2, and 4 are not associated with this medication.

245.) A client taking carbamazepine (Tegretol) asks the nurse what to do if he misses one dose. The nurse responds that the carbamazepine should be: 1. Withheld until the next scheduled dose 2. Withheld and the health care provider is notified immediately 3. Taken as long as it is not immediately before the next dose 4. Withheld until the next scheduled dose, which should then be doubled

3. Taken as long as it is not immediately before the next dose Rationale: Carbamazepine is an anticonvulsant that should be taken around the clock, precisely as directed. If a dose is omitted, the client should take the dose as soon as it is remembered, as long as it is not immediately before the next dose. The medication should not be double dosed. If more than one dose is omitted, the client should call the health care provider.

229.) A client who is taking lithium carbonate (Lithobid) is scheduled for surgery. The nurse informs the client that: 1. The medication will be discontinued a week before the surgery and resumed 1 week postoperatively. 2. The medication is to be taken until the day of surgery and resumed by injection immediately postoperatively. 3. The medication will be discontinued 1 to 2 days before the surgery and resumed as soon as full oral intake is allowed. 4. The medication will be discontinued several days before surgery and resumed by injection in the immediate postoperative period.

3. The medication will be discontinued 1 to 2 days before the surgery and resumed as soon as full oral intake is allowed. Rationale: The client who is on lithium carbonate must be off the medication for 1 to 2 days before a scheduled surgical procedure and can resume the medication when full oral intake is prescribed after the surgery. **lithium carbonate is an oral medication and is not given as an injection**

80.) A nurse is caring for a client who is taking phenytoin (Dilantin) for control of seizures. During data collection, the nurse notes that the client is taking birth control pills. Which of the following information should the nurse provide to the client? 1. Pregnancy should be avoided while taking phenytoin (Dilantin). 2. The client may stop taking the phenytoin (Dilantin) if it is causing severe gastrointestinal effects. 3. The potential for decreased effectiveness of the birth control pills exists while taking phenytoin (Dilantin). 4. The increased risk of thrombophlebitis exists while taking phenytoin (Dilantin) and birth control pills together.

3. The potential for decreased effectiveness of the birth control pills exists while taking phenytoin (Dilantin). Rationale: Phenytoin (Dilantin) enhances the rate of estrogen metabolism, which can decrease the effectiveness of some birth control pills. Options 1, 2, are 4 are not accurate.

129.) Megestrol acetate (Megace), an antineoplastic medication, is prescribed for the client with metastatic endometrial carcinoma. The nurse reviews the client's history and contacts the registered nurse if which diagnosis is documented in the client's history? 1. Gout 2. Asthma 3. Thrombophlebitis 4. Myocardial infarction

3. Thrombophlebitis Rationale: Megestrol acetate (Megace) suppresses the release of luteinizing hormone from the anterior pituitary by inhibiting pituitary function and regressing tumor size. Megestrol is used with caution if the client has a history of thrombophlebitis. **megestrol acetate is a hormonal antagonist enzyme and that a side effect is thrombotic disorders**

22.) A nurse is caring for a client after thyroidectomy and notes that calcium gluconate is prescribed for the client. The nurse determines that this medication has been prescribed to: 1. Treat thyroid storm. 2. Prevent cardiac irritability. 3. Treat hypocalcemic tetany. 4. Stimulate the release of parathyroid hormone.

3. Treat hypocalcemic tetany. Rationale: Hypocalcemia can develop after thyroidectomy if the parathyroid glands are accidentally removed or injured during surgery. Manifestations develop 1 to 7 days after surgery. If the client develops numbness and tingling around the mouth, fingertips, or toes or muscle spasms or twitching, the health care provider is notified immediately. Calcium gluconate should be kept at the bedside.

27.) Sildenafil (Viagra) is prescribed to treat a client with erectile dysfunction. A nurse reviews the client's medical record and would question the prescription if which of the following is noted in the client's history? 1. Neuralgia 2. Insomnia 3. Use of nitroglycerin 4. Use of multivitamins

3. Use of nitroglycerin Rationale: Sildenafil (Viagra) enhances the vasodilating effect of nitric oxide in the corpus cavernosum of the penis, thus sustaining an erection. Because of the effect of the medication, it is contraindicated with concurrent use of organic nitrates and nitroglycerin. Sildenafil is not contraindicated with the use of vitamins. Neuralgia and insomnia are side effects of the medication.

81.) A client with trigeminal neuralgia is being treated with carbamazepine (Tegretol). Which laboratory result would indicate that the client is experiencing an adverse reaction to the medication? 1. Sodium level, 140 mEq/L 2. Uric acid level, 5.0 mg/dL 3. White blood cell count, 3000 cells/mm3 4. Blood urea nitrogen (BUN) level, 15 mg/dL

3. White blood cell count, 3000 cells/mm3 Rationale: Adverse effects of carbamazepine (Tegretol) appear as blood dyscrasias, including aplastic anemia, agranulocytosis, thrombocytopenia, leukopenia, cardiovascular disturbances, thrombophlebitis, dysrhythmias, and dermatological effects. Options 1, 2, and 4 identify normal laboratory values.

Common S/S of TEF?

4 C'S- Coughing, Choking, Cyanosis and continuous droolings

118.) A nurse is caring for an older client with a diagnosis of myasthenia gravis and has reinforced self-care instructions. Which statement by the client indicates that further teaching is necessary? 1. "I rest each afternoon after my walk." 2. "I cough and deep breathe many times during the day." 3. "If I get abdominal cramps and diarrhea, I should call my doctor." 4. "I can change the time of my medication on the mornings that I feel strong."

4. "I can change the time of my medication on the mornings that I feel strong." Rationale: The client with myasthenia gravis should be taught that timing of anticholinesterase medication is critical. It is important to instruct the client to administer the medication on time to maintain a chemical balance at the neuromuscular junction. If not given on time, the client may become too weak to swallow. Options 1, 2, and 3 include the necessary information that the client needs to understand to maintain health with this neurological degenerative disease.

54.) A nurse reinforces discharge instructions to a postoperative client who is taking warfarin sodium (Coumadin). Which statement, if made by the client, reflects the need for further teaching? 1. "I will take my pills every day at the same time." 2. "I will be certain to avoid alcohol consumption." 3. "I have already called my family to pick up a Medic-Alert bracelet." 4. "I will take Ecotrin (enteric-coated aspirin) for my headaches because it is coated."

4. "I will take Ecotrin (enteric-coated aspirin) for my headaches because it is coated." Rationale: Ecotrin is an aspirin-containing product and should be avoided. Alcohol consumption should be avoided by a client taking warfarin sodium. Taking prescribed medication at the same time each day increases client compliance. The Medic-Alert bracelet provides health care personnel emergency information.

61.) A home health care nurse is visiting a client with elevated triglyceride levels and a serum cholesterol level of 398 mg/dL. The client is taking cholestyramine (Questran). Which of the following statements, if made by the client, indicates the need for further education? 1. "Constipation and bloating might be a problem." 2. "I'll continue to watch my diet and reduce my fats." 3. "Walking a mile each day will help the whole process." 4. "I'll continue my nicotinic acid from the health food store."

4. "I'll continue my nicotinic acid from the health food store." Rationale: Nicotinic acid, even an over-the-counter form, should be avoided because it may lead to liver abnormalities. All lipid-lowering medications also can cause liver abnormalities, so a combination of nicotinic acid and cholestyramine resin is to be avoided. Constipation and bloating are the two most common side effects. Walking and the reduction of fats in the diet are therapeutic measures to reduce cholesterol and triglyceride levels.

62.) A client is on nicotinic acid (niacin) for hyperlipidemia and the nurse provides instructions to the client about the medication. Which statement by the client would indicate an understanding of the instructions? 1. "It is not necessary to avoid the use of alcohol." 2. "The medication should be taken with meals to decrease flushing." 3. "Clay-colored stools are a common side effect and should not be of concern." 4. "Ibuprofen (Motrin) taken 30 minutes before the nicotinic acid should decrease the flushing."

4. "Ibuprofen (Motrin) taken 30 minutes before the nicotinic acid should decrease the flushing." Rationale: Flushing is a side effect of this medication. Aspirin or a nonsteroidal anti-inflammatory drug can be taken 30 minutes before taking the medication to decrease flushing. Alcohol consumption needs to be avoided because it will enhance this side effect. The medication should be taken with meals, this will decrease gastrointestinal upset. Taking the medication with meals has no effect on the flushing. Clay-colored stools are a sign of hepatic dysfunction and should be immediately reported to the health care provider (HCP).

218.) A film-coated form of diflunisal has been prescribed for a client for the treatment of chronic rheumatoid arthritis. The client calls the clinic nurse because of difficulty swallowing the tablets. Which initial instruction should the nurse provide to the client? 1. "Crush the tablets and mix them with food." 2. "Notify the health care provider for a medication change." 3. "Open the tablet and mix the contents with food." 4. "Swallow the tablets with large amounts of water or milk."

4. "Swallow the tablets with large amounts of water or milk." Rationale: Diflunisal may be given with water, milk, or meals. The tablets should not be crushed or broken open. Taking the medication with a large amount of water or milk should be tried before contacting the health care provider.

198.) Coal tar has been prescribed for a client with a diagnosis of psoriasis, and the nurse provides instructions to the client about the medication. Which statement by the client indicates a need for further instructions? 1. "The medication can cause phototoxicity." 2. "The medication has an unpleasant odor." 3. "The medication can stain the skin and hair." 4. "The medication can cause systemic effects."

4. "The medication can cause systemic effects." Rationale: Coal tar is used to treat psoriasis and other chronic disorders of the skin. It suppresses DNA synthesis, mitotic activity, and cell proliferation. It has an unpleasant odor, can frequently stain the skin and hair, and can cause phototoxicity. Systemic toxicity does not occur. **The name of the medication will assist in eliminating options 2 and 3**

87.) A client with acute muscle spasms has been taking baclofen (Lioresal). The client calls the clinic nurse because of continuous feelings of weakness and fatigue and asks the nurse about discontinuing the medication. The nurse should make which appropriate response to the client? 1. "You should never stop the medication." 2. "It is best that you taper the dose if you intend to stop the medication." 3. "It is okay to stop the medication if you think that you can tolerate the muscle spasms." 4. "Weakness and fatigue commonly occur and will diminish with continued medication use."

4. "Weakness and fatigue commonly occur and will diminish with continued medication use." Rationale: The client should be instructed that symptoms such as drowsiness, weakness, and fatigue are more intense in the early phase of therapy and diminish with continued medication use. The client should be instructed never to withdraw or stop the medication abruptly, because abrupt withdrawal can cause visual hallucinations, paranoid ideation, and seizures. It is best for the nurse to inform the client that these symptoms will subside and encourage the client to continue the use of the medication.

226.) A client receiving lithium carbonate (Lithobid) complains of loose, watery stools and difficulty walking. The nurse would expect the serum lithium level to be which of the following? 1. 0.7 mEq/L 2. 1.0 mEq/L 3. 1.2 mEq/L 4. 1.7 mEq/L

4. 1.7 mEq/L Rationale: The therapeutic serum level of lithium ranges from 0.6 to 1.2 mEq/L. Serum lithium levels above the therapeutic level will produce signs of toxicity.

163.) A client with trigeminal neuralgia tells the nurse that acetaminophen (Tylenol) is taken on a frequent daily basis for relief of generalized discomfort. The nurse reviews the client's laboratory results and determines that which of the following indicates toxicity associated with the medication? 1. Sodium of 140 mEq/L 2. Prothrombin time of 12 seconds 3. Platelet count of 400,000 cells/mm3 4. A direct bilirubin level of 2 mg/dL

4. A direct bilirubin level of 2 mg/dL Rationale: In adults, overdose of acetaminophen (Tylenol) causes liver damage. Option 4 is an indicator of liver function and is the only option that indicates an abnormal laboratory value. The normal direct bilirubin is 0 to 0.4 mg/dL. The normal platelet count is 150,000 to 400,000 cells/mm3. The normal prothrombin time is 10 to 13 seconds. The normal sodium level is 135 to 145 mEq/L.

75.) A client with myasthenia gravis becomes increasingly weak. The health care provider prepares to identify whether the client is reacting to an overdose of the medication (cholinergic crisis) or increasing severity of the disease (myasthenic crisis). An injection of edrophonium (Enlon) is administered. Which of the following indicates that the client is in cholinergic crisis? 1. No change in the condition 2. Complaints of muscle spasms 3. An improvement of the weakness 4. A temporary worsening of the condition

4. A temporary worsening of the condition Rationale: An edrophonium (Enlon) injection, a cholinergic drug, makes the client in cholinergic crisis temporarily worse. This is known as a negative test. An improvement of weakness would occur if the client were experiencing myasthenia gravis. Options 1 and 2 would not occur in either crisis.

181.) A client is taking ticlopidine hydrochloride (Ticlid). The nurse tells the client to avoid which of the following while taking this medication? 1. Vitamin C 2. Vitamin D 3. Acetaminophen (Tylenol) 4. Acetylsalicylic acid (aspirin)

4. Acetylsalicylic acid (aspirin) Rationale: Ticlopidine hydrochloride is a platelet aggregation inhibitor. It is used to decrease the risk of thrombotic strokes in clients with precursor symptoms. Because it is an antiplatelet agent, other medications that precipitate or aggravate bleeding should be avoided during its use. Therefore, aspirin or any aspirin-containing product should be avoided.

56.) Heparin sodium is prescribed for the client. The nurse expects that the health care provider will prescribe which of the following to monitor for a therapeutic effect of the medication? 1. Hematocrit level 2. Hemoglobin level 3. Prothrombin time (PT) 4. Activated partial thromboplastin time (aPTT)

4. Activated partial thromboplastin time (aPTT) Rationale: The PT will assess for the therapeutic effect of warfarin sodium (Coumadin) and the aPTT will assess the therapeutic effect of heparin sodium. Heparin sodium doses are determined based on these laboratory results. The hemoglobin and hematocrit values assess red blood cell concentrations.

242.) A client who was started on anticonvulsant therapy with clonazepam (Klonopin) tells the nurse of increasing clumsiness and unsteadiness since starting the medication. The client is visibly upset by these manifestations and asks the nurse what to do. The nurse's response is based on the understanding that these symptoms: 1. Usually occur if the client takes the medication with food 2. Are probably the result of an interaction with another medication 3. Indicate that the client is experiencing a severe untoward reaction to the medication 4. Are worse during initial therapy and decrease or disappear with long-term use

4. Are worse during initial therapy and decrease or disappear with long-term use Rationale: Drowsiness, unsteadiness, and clumsiness are expected effects of the medication during early therapy. They are dose related and usually diminish or disappear altogether with continued use of the medication. It does not indicate that a severe side effect is occurring. It is also unrelated to interaction with another medication. The client is encouraged to take this medication with food to minimize gastrointestinal upset. **Eliminate options 2 and 3 first because they are comparable or alike and because of the word "severe" in option 3**

178.) Methylergonovine (Methergine) is prescribed for a client with postpartum hemorrhage caused by uterine atony. Before administering the medication, the nurse checks which of the following as the important client parameter? 1. Temperature 2. Lochial flow 3. Urine output 4. Blood pressure

4. Blood pressure Rationale: Methylergonovine is an ergot alkaloid used for postpartum hemorrhage. It stimulates contraction of the uterus and causes arterial vasoconstriction. Ergot alkaloids are avoided in clients with significant cardiovascular disease, peripheral disease, hypertension, eclampsia, or preeclampsia. These conditions are worsened by the vasoconstrictive effects of the ergot alkaloids. The nurse would check the client's blood pressure before administering the medication and would follow agency protocols regarding withholding of the medication. Options 1, 2, and 3 are items that are checked in the postpartum period, but they are unrelated to the use of this medication.

237.) A client who is on lithium carbonate (Lithobid) will be discharged at the end of the week. In formulating a discharge teaching plan, the nurse will instruct the client that it is most important to: 1. Avoid soy sauce, wine, and aged cheese. 2. Have the lithium level checked every week. 3. Take medication only as prescribed because it can become addicting. 4. Check with the psychiatrist before using any over-the-counter (OTC) medications or prescription medications.

4. Check with the psychiatrist before using any over-the-counter (OTC) medications or prescription medications. Rationale: Lithium is the medication of choice to treat manic-depressive illness. Many OTC medications interact with lithium, and the client is instructed to avoid OTC medications while taking lithium. Lithium is not addicting, and, although serum lithium levels need to be monitored, it is not necessary to check these levels every week. A tyramine-free diet is associated with monoamine oxidase inhibitors.

19.) Tamoxifen is prescribed for the client with metastatic breast carcinoma. The nurse understands that the primary action of this medication is to: 1. Increase DNA and RNA synthesis. 2. Promote the biosynthesis of nucleic acids. 3. Increase estrogen concentration and estrogen response. 4. Compete with estradiol for binding to estrogen in tissues containing high concentrations of receptors.

4. Compete with estradiol for binding to estrogen in tissues containing high concentrations of receptors. Rationale: Tamoxifen is an antineoplastic medication that competes with estradiol for binding to estrogen in tissues containing high concentrations of receptors. Tamoxifen is used to treat metastatic breast carcinoma in women and men. Tamoxifen is also effective in delaying the recurrence of cancer following mastectomy. Tamoxifen reduces DNA synthesis and estrogen response.

217.) A health care provider prescribes auranofin (Ridaura) for a client with rheumatoid arthritis. Which of the following would indicate to the nurse that the client is experiencing toxicity related to the medication? 1. Joint pain 2. Constipation 3. Ringing in the ears 4. Complaints of a metallic taste in the mouth

4. Complaints of a metallic taste in the mouth Rationale: Ridaura is the one gold preparation that is given orally rather than by injection. Gastrointestinal reactions including diarrhea, abdominal pain, nausea, and loss of appetite are common early in therapy, but these usually subside in the first 3 months of therapy. Early symptoms of toxicity include a rash, purple blotches, pruritus, mouth lesions, and a metallic taste in the mouth.

94.) The client with acquired immunodeficiency syndrome has begun therapy with zidovudine (Retrovir, Azidothymidine, AZT, ZDV). The nurse carefully monitors which of the following laboratory results during treatment with this medication? 1. Blood culture 2. Blood glucose level 3. Blood urea nitrogen 4. Complete blood count

4. Complete blood count Rationale: A common side effect of therapy with zidovudine is leukopenia and anemia. The nurse monitors the complete blood count results for these changes. Options 1, 2, and 3 are unrelated to the use of this medication.

124.) A client with chronic renal failure is receiving ferrous sulfate (Feosol). The nurse monitors the client for which common side effect associated with this medication? 1. Diarrhea 2. Weakness 3. Headache 4. Constipation

4. Constipation Rationale: Feosol is an iron supplement used to treat anemia. Constipation is a frequent and uncomfortable side effect associated with the administration of oral iron supplements. Stool softeners are often prescribed to prevent constipation. **Focus on the name of the medication. Recalling that oral iron can cause constipation will easily direct you to the correct option.**

155.) Mycophenolate mofetil (CellCept) is prescribed for a client as prophylaxis for organ rejection following an allogeneic renal transplant. Which of the following instructions does the nurse reinforce regarding administration of this medication? 1. Administer following meals. 2. Take the medication with a magnesium-type antacid. 3. Open the capsule and mix with food for administration. 4. Contact the health care provider (HCP) if a sore throat occurs.

4. Contact the health care provider (HCP) if a sore throat occurs. Rationale: Mycophenolate mofetil should be administered on an empty stomach. The capsules should not be opened or crushed. The client should contact the HCP if unusual bleeding or bruising, sore throat, mouth sores, abdominal pain, or fever occurs because these are adverse effects of the medication. Antacids containing magnesium and aluminum may decrease the absorption of the medication and therefore should not be taken with the medication. The medication may be given in combination with corticosteroids and cyclosporine. **neutropenia can occur with this medication**

157.) A client receiving nitrofurantoin (Macrodantin) calls the health care provider's office complaining of side effects related to the medication. Which side effect indicates the need to stop treatment with this medication? 1. Nausea 2. Diarrhea 3. Anorexia 4. Cough and chest pain

4. Cough and chest pain Rationale: Gastrointestinal (GI) effects are the most frequent adverse reactions to this medication and can be minimized by administering the medication with milk or meals. Pulmonary reactions, manifested as dyspnea, chest pain, chills, fever, cough, and the presence of alveolar infiltrates on the x-ray, would indicate the need to stop the treatment. These symptoms resolve in 2 to 4 days following discontinuation of this medication. **Eliminate options 1, 2, and 3 because they are similar GI-related side effects. Also, use the ABCs— airway, breathing, and circulation**

132.) The client with non-Hodgkin's lymphoma is receiving daunorubicin (DaunoXome). Which of the following would indicate to the nurse that the client is experiencing a toxic effect related to the medication? 1. Fever 2. Diarrhea 3. Complaints of nausea and vomiting 4. Crackles on auscultation of the lungs

4. Crackles on auscultation of the lungs Rationale: Cardiotoxicity noted by abnormal electrocardiographic findings or cardiomyopathy manifested as congestive heart failure is a toxic effect of daunorubicin. Bone marrow depression is also a toxic effect. Nausea and vomiting are frequent side effects associated with the medication that begins a few hours after administration and lasts 24 to 48 hours. Fever is a frequent side effect, and diarrhea can occur occasionally. The other options, however, are not toxic effects. **keep in mind that the question is asking about a toxic effect and think: ABCs—airway, breathing, and circulation**

117.) A nurse has given the client taking ethambutol (Myambutol) information about the medication. The nurse determines that the client understands the instructions if the client immediately reports: 1. Impaired sense of hearing 2. Distressing gastrointestinal side effects 3. Orange-red discoloration of body secretions 4. Difficulty discriminating the color red from green

4. Difficulty discriminating the color red from green Rationale: Ethambutol causes optic neuritis, which decreases visual acuity and the ability to discriminate between the colors red and green. This poses a potential safety hazard when driving a motor vehicle. The client is taught to report this symptom immediately. The client is also taught to take the medication with food if gastrointestinal upset occurs. Impaired hearing results from antitubercular therapy with streptomycin. Orange-red discoloration of secretions occurs with rifampin (Rifadin).

190.) A child is hospitalized with a diagnosis of lead poisoning. The nurse assisting in caring for the child would prepare to assist in administering which of the following medications? 1. Activated charcoal 2. Sodium bicarbonate 3. Syrup of ipecac syrup 4. Dimercaprol (BAL in Oil)

4. Dimercaprol (BAL in Oil) Rationale: Dimercaprol is a chelating agent that is administered to remove lead from the circulating blood and from some tissues and organs for excretion in the urine. Sodium bicarbonate may be used in salicylate poisoning. Syrup of ipecac is used in the hospital setting in poisonings to induce vomiting. Activated charcoal is used to decrease absorption in certain poisoning situations. Note that dimercaprol is prepared with peanut oil, and hence should be avoided by clients with known or suspected peanut allergy.

203.) A nurse is preparing to give the postcraniotomy client medication for incisional pain. The family asks the nurse why the client is receiving codeine sulfate and not "something stronger." In formulating a response, the nurse incorporates the understanding that codeine: 1. Is one of the strongest opioid analgesics available 2. Cannot lead to physical or psychological dependence 3. Does not cause gastrointestinal upset or constipation as do other opioids 4. Does not alter respirations or mask neurological signs as do other opioids

4. Does not alter respirations or mask neurological signs as do other opioids Rationale: Codeine sulfate is the opioid analgesic often used for clients after craniotomy. It is frequently combined with a nonopioid analgesic such as acetaminophen for added effect. It does not alter the respiratory rate or mask neurological signs as do other opioids. Side effects of codeine include gastrointestinal upset and constipation. The medication can lead to physical and psychological dependence with chronic use. It is not the strongest opioid analgesic available.

159.) A nurse is caring for a client receiving morphine sulfate subcutaneously for pain. Because morphine sulfate has been prescribed for this client, which nursing action would be included in the plan of care? 1. Encourage fluid intake. 2. Monitor the client's temperature. 3. Maintain the client in a supine position. 4. Encourage the client to cough and deep breathe.

4. Encourage the client to cough and deep breathe. Rationale: Morphine sulfate suppresses the cough reflex. Clients need to be encouraged to cough and deep breathe to prevent pneumonia. **ABCs—airway, breathing, and circulation**

223.) A client with a psychotic disorder is being treated with haloperidol (Haldol). Which of the following would indicate the presence of a toxic effect of this medication? 1. Nausea 2. Hypotension 3. Blurred vision 4. Excessive salivation

4. Excessive salivation Rationale: Toxic effects include extrapyramidal symptoms (EPS) noted as marked drowsiness and lethargy, excessive salivation, and a fixed stare. Akathisia, acute dystonias, and tardive dyskinesia are also signs of toxicity. Hypotension, nausea, and blurred vision are occasional side effects.

231.) A client admitted to the hospital gives the nurse a bottle of clomipramine (Anafranil). The nurse notes that the medication has not been taken by the client in 2 months. What behaviors observed in the client would validate noncompliance with this medication? 1. Complaints of hunger 2. Complaints of insomnia 3. A pulse rate less than 60 beats per minute 4. Frequent handwashing with hot, soapy water

4. Frequent handwashing with hot, soapy water Rationale: Clomipramine is commonly used in the treatment of obsessive-compulsive disorder. Handwashing is a common obsessive-compulsive behavior. Weight gain is a common side effect of this medication. Tachycardia and sedation are side effects. Insomnia may occur but is seldom a side effect.

236.) A client is being treated for depression with amitriptyline hydrochloride. During the initial phases of treatment, the most important nursing intervention is: 1. Prescribing the client a tyramine-free diet 2. Checking the client for anticholinergic effects 3. Monitoring blood levels frequently because there is a narrow range between therapeutic and toxic blood levels of this medication 4. Getting baseline postural blood pressures before administering the medication and each time the medication is administered

4. Getting baseline postural blood pressures before administering the medication and each time the medication is administered Rationale: Amitriptyline hydrochloride is a tricyclic antidepressant often used to treat depression. It causes orthostatic changes and can produce hypotension and tachycardia. This can be frightening to the client and dangerous because it can result in dizziness and client falls. The client must be instructed to move slowly from a lying to a sitting to a standing position to avoid injury if these effects are experienced. The client may also experience sedation, dry mouth, constipation, blurred vision, and other anticholinergic effects, but these are transient and will diminish with time.

196.) A client has been prescribed amikacin (Amikin). Which of the following priority baseline functions should be monitored? 1. Apical pulse 2. Liver function 3. Blood pressure 4. Hearing acuity

4. Hearing acuity Rationale: Amikacin (Amikin) is an antibiotic. This medication can cause ototoxicity and nephrotoxicity; therefore, hearing acuity tests and kidney function studies should be performed before the initiation of therapy. Apical pulse, liver function studies, and blood pressure are not specifically related to the use of this medication.

175.) A nurse notes that a client is receiving lamivudine (Epivir). The nurse determines that this medication has been prescribed to treat which of the following? 1. Pancreatitis 2. Pharyngitis 3. Tonic-clonic seizures 4. Human immunodeficiency virus (HIV) infection

4. Human immunodeficiency virus (HIV) infection Rationale: Lamivudine is a nucleoside reverse transcriptase inhibitor and antiviral medication. It slows HIV replication and reduces the progression of HIV infection. It also is used to treat chronic hepatitis B and is used for prophylaxis in health care workers at risk of acquiring HIV after occupational exposure to the virus. **Note the letters "-vir" in the trade name for this medication**

192.) A nurse is collecting medication information from a client, and the client states that she is taking garlic as an herbal supplement. The nurse understands that the client is most likely treating which of the following conditions? 1. Eczema 2. Insomnia 3. Migraines 4. Hyperlipidemia

4. Hyperlipidemia Rationale: Garlic is an herbal supplement that is used to treat hyperlipidemia and hypertension. An herbal supplement that may be used to treat eczema is evening primrose. Insomnia has been treated with both valerian root and chamomile. Migraines have been treated with feverfew.

73.) A client with myasthenia gravis is suspected of having cholinergic crisis. Which of the following indicate that this crisis exists? 1. Ataxia 2. Mouth sores 3. Hypotension 4. Hypertension

4. Hypertension Rationale: Cholinergic crisis occurs as a result of an overdose of medication. Indications of cholinergic crisis include gastrointestinal disturbances, nausea, vomiting, diarrhea, abdominal cramps, increased salivation and tearing, miosis, hypertension, sweating, and increased bronchial secretions.

212.) Mannitol (Osmitrol) is being administered to a client with increased intracranial pressure following a head injury. The nurse assisting in caring for the client knows that which of the following indicates the therapeutic action of this medication? 1. Prevents the filtration of sodium and water through the kidneys 2. Prevents the filtration of sodium and potassium through the kidneys 3. Decreases water loss by promoting the reabsorption of sodium and water in the loop of Henle 4. Induces diuresis by raising the osmotic pressure of glomerular filtrate, thereby inhibiting tubular reabsorption of water and solutes

4. Induces diuresis by raising the osmotic pressure of glomerular filtrate, thereby inhibiting tubular reabsorption of water and solutes Rationale: Mannitol is an osmotic diuretic that induces diuresis by raising the osmotic pressure of glomerular filtrate, thereby inhibiting tubular reabsorption of water and solutes. It is used to reduce intracranial pressure in the client with head trauma.

225.) A nursing student is assigned to care for a client with a diagnosis of schizophrenia. Haloperidol (Haldol) is prescribed for the client, and the nursing instructor asks the student to describe the action of the medication. Which statement by the nursing student indicates an understanding of the action of this medication? 1. It is a serotonin reuptake blocker. 2. It inhibits the breakdown of released acetylcholine. 3. It blocks the uptake of norepinephrine and serotonin. 4. It blocks the binding of dopamine to the postsynaptic dopamine receptors in the brain.

4. It blocks the binding of dopamine to the postsynaptic dopamine receptors in the brain. Rationale: Haloperidol acts by blocking the binding of dopamine to the postsynaptic dopamine receptors in the brain. Imipramine hydrochloride (Tofranil) blocks the reuptake of norepinephrine and serotonin. Donepezil hydrochloride (Aricept) inhibits the breakdown of released acetylcholine. Fluoxetine hydrochloride (Prozac) is a potent serotonin reuptake blocker.

119.) A client with diabetes mellitus who has been controlled with daily insulin has been placed on atenolol (Tenormin) for the control of angina pectoris. Because of the effects of atenolol, the nurse determines that which of the following is the most reliable indicator of hypoglycemia? 1. Sweating 2. Tachycardia 3. Nervousness 4. Low blood glucose level

4. Low blood glucose level Rationale: β-Adrenergic blocking agents, such as atenolol, inhibit the appearance of signs and symptoms of acute hypoglycemia, which would include nervousness, increased heart rate, and sweating. Therefore, the client receiving this medication should adhere to the therapeutic regimen and monitor blood glucose levels carefully. Option 4 is the most reliable indicator of hypoglycemia.

205.) A nurse is assisting in preparing to administer acetylcysteine (Mucomyst) to a client with an overdose of acetaminophen (Tylenol). The nurse prepares to administer the medication by: 1. Administering the medication subcutaneously in the deltoid muscle 2. Administering the medication by the intramuscular route in the gluteal muscle 3. Administering the medication by the intramuscular route, mixed in 10 mL of normal saline 4. Mixing the medication in a flavored ice drink and allowing the client to drink the medication through a straw

4. Mixing the medication in a flavored ice drink and allowing the client to drink the medication through a straw Rationale: Because acetylcysteine has a pervasive odor of rotten eggs, it must be disguised in a flavored ice drink. It is consumed preferably through a straw to minimize contact with the mouth. It is not administered by the intramuscular or subcutaneous route. **Knowing that the medication is a solution that is also used for nebulization treatments will assist you to select the option that indicates an oral route**

39.) The client with a gastric ulcer has a prescription for sucralfate (Carafate), 1 g by mouth four times daily. The nurse schedules the medication for which times? 1. With meals and at bedtime 2. Every 6 hours around the clock 3. One hour after meals and at bedtime 4. One hour before meals and at bedtime

4. One hour before meals and at bedtime Rationale: Sucralfate is a gastric protectant. The medication should be scheduled for administration 1 hour before meals and at bedtime. The medication is timed to allow it to form a protective coating over the ulcer before food intake stimulates gastric acid production and mechanical irritation. The other options are incorrect.

2.) Oral iron supplements are prescribed for a 6-year-old child with iron deficiency anemia. The nurse instructs the mother to administer the iron with which best food item? 1. Milk 2. Water 3. Apple juice 4. Orange juice

4. Orange juice Rationale: Vitamin C increases the absorption of iron by the body. The mother should be instructed to administer the medication with a citrus fruit or a juice that is high in vitamin C. Milk may affect absorption of the iron. Water will not assist in absorption. Orange juice contains a greater amount of vitamin C than apple juice.

15.) The client with small cell lung cancer is being treated with etoposide (VePesid). The nurse who is assisting in caring for the client during its administration understands that which side effect is specifically associated with this medication? 1. Alopecia 2. Chest pain 3. Pulmonary fibrosis 4. Orthostatic hypotension

4. Orthostatic hypotension Rationale: A side effect specific to etoposide is orthostatic hypotension. The client's blood pressure is monitored during the infusion. Hair loss occurs with nearly all the antineoplastic medications. Chest pain and pulmonary fibrosis are unrelated to this medication.

222.) A nurse has administered a dose of diazepam (Valium) to a client. The nurse would take which important action before leaving the client's room? 1. Giving the client a bedpan 2. Drawing the shades or blinds closed 3. Turning down the volume on the television 4. Per agency policy, putting up the side rails on the bed

4. Per agency policy, putting up the side rails on the bed Rationale: Diazepam is a sedative-hypnotic with anticonvulsant and skeletal muscle relaxant properties. The nurse should institute safety measures before leaving the client's room to ensure that the client does not injure herself or himself. The most frequent side effects of this medication are dizziness, drowsiness, and lethargy. For this reason, the nurse puts the side rails up on the bed before leaving the room to prevent falls. Options 1, 2, and 3 may be helpful measures that provide a comfortable, restful environment, but option 4 is the one that provides for the client's safety needs.

189.) Prostaglandin E1 is prescribed for a child with transposition of the great arteries. The mother of the child asks the nurse why the child needs the medication. The nurse tells the mother that the medication: 1. Prevents hypercyanotic (blue or tet) spells 2. Maintains an adequate hormone level 3. Maintains the position of the great arteries 4. Provides adequate oxygen saturation and maintains cardiac output

4. Provides adequate oxygen saturation and maintains cardiac output Rationale: A child with transposition of the great arteries may receive prostaglandin E1 temporarily to increase blood mixing if systemic and pulmonary mixing are inadequate to maintain adequate cardiac output. Options 1, 2, and 3 are incorrect. In addition, hypercyanotic spells occur in tetralogy of Fallot. **Use the ABCs—airway, breathing, and circulation—to answer the question. The correct option addresses circulation**

13.) The client with squamous cell carcinoma of the larynx is receiving bleomycin intravenously. The nurse caring for the client anticipates that which diagnostic study will be prescribed? 1. Echocardiography 2. Electrocardiography 3. Cervical radiography 4. Pulmonary function studies

4. Pulmonary function studies Rationale: Bleomycin is an antineoplastic medication (Chemotheraputic Agents) that can cause interstitial pneumonitis, which can progress to pulmonary fibrosis. Pulmonary function studies along with hematological, hepatic, and renal function tests need to be monitored. The nurse needs to monitor lung sounds for dyspnea and crackles, which indicate pulmonary toxicity. The medication needs to be discontinued immediately if pulmonary toxicity occurs. Options 1, 2, and 3 are unrelated to the specific use of this medication.

70.) Oxybutynin chloride (Ditropan XL) is prescribed for a client with neurogenic bladder. Which sign would indicate a possible toxic effect related to this medication? 1. Pallor 2. Drowsiness 3. Bradycardia 4. Restlessness

4. Restlessness Rationale: Toxicity (overdosage) of this medication produces central nervous system excitation, such as nervousness, restlessness, hallucinations, and irritability. Other signs of toxicity include hypotension or hypertension, confusion, tachycardia, flushed or red face, and signs of respiratory depression. Drowsiness is a frequent side effect of the medication but does not indicate overdosage.

44.) A client is receiving acetylcysteine (Mucomyst), 20% solution diluted in 0.9% normal saline by nebulizer. The nurse should have which item available for possible use after giving this medication? 1. Ambu bag 2. Intubation tray 3. Nasogastric tube 4. Suction equipment

4. Suction equipment Rationale: Acetylcysteine can be given orally or by nasogastric tube to treat acetaminophen overdose, or it may be given by inhalation for use as a mucolytic. The nurse administering this medication as a mucolytic should have suction equipment available in case the client cannot manage to clear the increased volume of liquefied secretions.

166.) Alendronate (Fosamax) is prescribed for a client with osteoporosis. The client taking this medication is instructed to: 1. Take the medication at bedtime. 2. Take the medication in the morning with breakfast. 3. Lie down for 30 minutes after taking the medication. 4. Take the medication with a full glass of water after rising in the morning.

4. Take the medication with a full glass of water after rising in the morning. Rationale: Precautions need to be taken with administration of alendronate to prevent gastrointestinal side effects (especially esophageal irritation) and to increase absorption of the medication. The medication needs to be taken with a full glass of water after rising in the morning. The client should not eat or drink anything for 30 minutes following administration and should not lie down after taking the medication.

67.) Phenazopyridine hydrochloride (Pyridium) is prescribed for a client for symptomatic relief of pain resulting from a lower urinary tract infection. The nurse reinforces to the client: 1. To take the medication at bedtime 2. To take the medication before meals 3. To discontinue the medication if a headache occurs 4. That a reddish orange discoloration of the urine may occur

4. That a reddish orange discoloration of the urine may occur Rationale: The nurse should instruct the client that a reddish-orange discoloration of urine may occur. The nurse also should instruct the client that this discoloration can stain fabric. The medication should be taken after meals to reduce the possibility of gastrointestinal upset. A headache is an occasional side effect of the medication and does not warrant discontinuation of the medication.

177.) A nurse is assisting in caring for a pregnant client who is receiving intravenous magnesium sulfate for the management of preeclampsia and notes that the client's deep tendon reflexes are absent. On the basis of this data, the nurse reports the finding and makes which determination? 1. The magnesium sulfate is effective. 2. The infusion rate needs to be increased. 3. The client is experiencing cerebral edema. 4. The client is experiencing magnesium toxicity.

4. The client is experiencing magnesium toxicity. Rationale: Magnesium toxicity can occur as a result of magnesium sulfate therapy. Signs of magnesium sulfate toxicity relate to the central nervous system depressant effects of the medication and include respiratory depression; loss of deep tendon reflexes; sudden decrease in fetal heart rate or maternal heart rate, or both; and sudden drop in blood pressure. Hyperreflexia indicates increased cerebral edema. An absence of reflexes indicates magnesium toxicity. The therapeutic serum level of magnesium for a client receiving magnesium sulfate ranges from 4 to 7.5 mEq/L (5 to 8 mg/dL).

232.) A client in the mental health unit is administered haloperidol (Haldol). The nurse would check which of the following to determine medication effectiveness? 1. The client's vital signs 2. The client's nutritional intake 3. The physical safety of other unit clients 4. The client's orientation and delusional status

4. The client's orientation and delusional status Rationale: Haloperidol is used to treat clients exhibiting psychotic features. Therefore, to determine medication effectiveness, the nurse would check the client's orientation and delusional status. Vital signs are routine and not specific to this situation. The physical safety of other clients is not a direct assessment of this client. Monitoring nutritional intake is not related to this situation.

99.) The client with acquired immunodeficiency syndrome and Pneumocystis jiroveci infection has been receiving pentamidine isethionate (Pentam 300). The client develops a temperature of 101° F. The nurse does further monitoring of the client, knowing that this sign would most likely indicate: 1. The dose of the medication is too low. 2. The client is experiencing toxic effects of the medication. 3. The client has developed inadequacy of thermoregulation. 4. The result of another infection caused by leukopenic effects of the medication.

4. The result of another infection caused by leukopenic effects of the medication. Rationale: Frequent side effects of this medication include leukopenia, thrombocytopenia, and anemia. The client should be monitored routinely for signs and symptoms of infection. Options 1, 2, and 3 are inaccurate interpretations.

121.) A client who is taking hydrochlorothiazide (HydroDIURIL, HCTZ) has been started on triamterene (Dyrenium) as well. The client asks the nurse why both medications are required. The nurse formulates a response, based on the understanding that: 1. Both are weak potassium-losing diuretics. 2. The combination of these medications prevents renal toxicity. 3. Hydrochlorothiazide is an expensive medication, so using a combination of diuretics is cost-effective. 4. Triamterene is a potassium-sparing diuretic, whereas hydrochlorothiazide is a potassium-losing diuretic.

4. Triamterene is a potassium-sparing diuretic, whereas hydrochlorothiazide is a potassium-losing diuretic. Rationale: Potassium-sparing diuretics include amiloride (Midamor), spironolactone (Aldactone), and triamterene (Dyrenium). They are weak diuretics that are used in combination with potassium-losing diuretics. This combination is useful when medication and dietary supplement of potassium is not appropriate. The use of two different diuretics does not prevent renal toxicity. Hydrochlorothiazide is an effective and inexpensive generic form of the thiazide classification of diuretics. **It is especially helpful to remember that hydrochlorothiazide is a potassium-losing diuretic and triamterene is a potassium-sparing diuretic**

160.) Meperidine hydrochloride (Demerol) is prescribed for the client with pain. Which of the following would the nurse monitor for as a side effect of this medication? 1. Diarrhea 2. Bradycardia 3. Hypertension 4. Urinary retention

4. Urinary retention Rationale: Meperidine hydrochloride (Demerol) is an opioid analgesic. Side effects of this medication include respiratory depression, orthostatic hypotension, tachycardia, drowsiness and mental clouding, constipation, and urinary retention.

140.) The client has a new prescription for metoclopramide (Reglan). On review of the chart, the nurse identifies that this medication can be safely administered with which condition? 1. Intestinal obstruction 2. Peptic ulcer with melena 3. Diverticulitis with perforation 4. Vomiting following cancer chemotherapy

4. Vomiting following cancer chemotherapy Rationale: Metoclopramide is a gastrointestinal (GI) stimulant and antiemetic. Because it is a GI stimulant, it is contraindicated with GI obstruction, hemorrhage, or perforation. It is used in the treatment of emesis after surgery, chemotherapy, and radiation.

104.) Disulfiram (Antabuse) is prescribed for a client who is seen in the psychiatric health care clinic. The nurse is collecting data on the client and is providing instructions regarding the use of this medication. Which is most important for the nurse to determine before administration of this medication? 1. A history of hyperthyroidism 2. A history of diabetes insipidus 3. When the last full meal was consumed 4. When the last alcoholic drink was consumed

4. When the last alcoholic drink was consumed Rationale: Disulfiram is used as an adjunct treatment for selected clients with chronic alcoholism who want to remain in a state of enforced sobriety. Clients must abstain from alcohol intake for at least 12 hours before the initial dose of the medication is administered. The most important data are to determine when the last alcoholic drink was consumed. The medication is used with caution in clients with diabetes mellitus, hypothyroidism, epilepsy, cerebral damage, nephritis, and hepatic disease. It is also contraindicated in severe heart disease, psychosis, or hypersensitivity related to the medication.

180.) A health care provider (HCP) writes a prescription for digoxin (Lanoxin), 0.25 mg daily. The nurse teaches the client about the medication and tells the client that it is important to: 1. Count the radial and carotid pulses every morning. 2. Check the blood pressure every morning and evening. 3. Stop taking the medication if the pulse is higher than 100 beats per minute. 4. Withhold the medication and call the HCP if the pulse is less than 60 beats per minute.

4. Withhold the medication and call the HCP if the pulse is less than 60 beats per minute. Rationale: An important component of taking this medication is monitoring the pulse rate; however, it is not necessary for the client to take both the radial and carotid pulses. It is not necessary for the client to check the blood pressure every morning and evening because the medication does not directly affect blood pressure. It is most important for the client to know the guidelines related to withholding the medication and calling the HCP. The client should not stop taking a medication.

Diaphragm must stay in place for at least ___ hrs after intercourse. It must be refitted if you gain or lose ___ lbs.

6hrs and 10lbs

Birth weight by age: doubles, triples

6mths= doubles in weight 1year old= triples in weight

what cranial nerve can cause crooked smile

7

* TIDAL VOLUME is

7 - 10ml / kg

Tidal volume?

7 - 10ml / kg

hyponatremia

<135, s/s: headache, muscle twitching, convulsions, diarrhea, lethargy, apprehension

Which of the following clients would qualify for hospice care?

A client with late-stage acquired immunodeficiency syndrome (AIDS)

Hypothyroidism

A hypothyroid state resulting from a hyposecretion of thyroid hormone. The condition occurs in adulthood.

More IVP info?

A laxative is given the night before in order to better visualize the organs.

Diabetic ketoacidosis

A life threatening complication of diabetes mellitus that develops when a severe insulin deficiency occurs. Hyperglycemia progresses to ketoacidosis over a period of several hours to several days. Acidosis occurs in clients with type 1 diabetes mellitus, persons with undiagnosed diabetes, and persons who stop prescribed treatment for diabetes.

Addisonian crisis

A life threatening disorder caused by adrenal hormone insufficiency. Crisis is precipitated by infection, trauma, stress, or surgery. Death can occur from shock, vascular collapse, or hyperkalemia.

Potassium and acid base balance?

A little trick regarding potassium: ALKALOSIS: K is LOW Acidosis is just the opposite: K is High The vital sign you should check first with high potassium is pulse (due to dysrhythmias).

Cushing's disease

A metabolic disorder characterized by abnormally increased secretion (endogenous) of cortisol, caused by increased amounts of adrenocorticotropic hormone (ACTH) secreted by the pituitary gland.

Cushing's syndrome

A metabolic disorder resulting from the chronic and excessive production of cortisol by the adrenal cortex or by the administration of glucocorticoids in large doses for several weeks or longer (exogenous or iatrogenic)

Type 1 diabetes mellitus

A nearly absolute deficiency of insulin; if insulin is not given, fats are metabolized for energy, resulting in ketonemia (acidosis).

Expected lab value changes with lipitor:

A normal Bilirubin is (0.2-1.2 mg/dl). A slight increase in Bilirubin is expected when taking lipitor. A liver function test will be ordered, and cholesterol q 3-6 months. LDL level should be below 130mg/dl

coagulation study

A patient is adm to the same day surg unit for a liver biopsy. To test for ___ a partial thromboplastin time, prothrombin time and platelet count would all be ordered.

Pt with edema and walking?

A patient with liver cirrhosis and edema may ambulate, then sit with legs elevated to try to mobilize the edema.

Western blot test?

A positive Western blot in a child <18 months (presence of HIV antibodies) indicates only that the mother is infected. Two or more positive p24 antigen tests will confirm HIV in kids <18 months. The p24 can be used at any age.

TB test confirmation?

A positive ppd confirms infection, not just exposure. A sputum test will confirm active disease.

Somogyi phenomenon

A rebound phenomenon that occurs in clients with type 1 diabetes mellitus. Normal or elevated blood glucose levels are present at bedtime; hypoglycemia occurs at about 2 to 3 am. Counterregulatory hormones, produced to prevent further hypoglycemia, result in hyperglycemia (evident in the prebreakfast blood glucose level). Treatment includes decreasing the evening (predinner or bedtime) dose of intermediate acting insulin or increasing the bedtime snack.

The nurse is assigned to care for an elderly client who is confused and repeatedly attempts to climb out of bed. The nurse asks the client to lie quietly and leaves her unsupervised to take a quick break. While the nurse is away, the client falls out of bed. She sustains no injuries from the fall. Initially, the nurse should treat this occurrence as:

A risk management incident.

Sickle Cell Anemia:Preventing a sickle Cell Crisis

A sickle cell crisis happens when sickled red blood cells block small blood vessels that carry blood to your bones. This causes pain that can begin suddenly and last several hours to several days.

Trousseau's sign

A sign of hypocalcemia. Carpal spasm can be elicited by compressing the brachial artery with a blood pressure cuff for 3 minutes.

Chvostek's sign

A spasm of the facial muscles elicited by tapping the facial nerve just anterior to the ear. The sign is noted in hypocalcemia.

A client's vital signs at the beginning of the shift are as follows: oral temperature 99.3°F (37°C), heart rate 82 beats per minute, respiratory rate 14 breaths per minute, and blood pressure 118/76. Four hours later the client's oral temperature is 102.2°F (39°C). Based on the temperature change, the nurse should anticipate the client's heart rate would be how many beats per minute? Why?

ANS: 111 BPM. ∆ 1 °F = 10 BPM 102.2 °F - 99.3 °F = 2.9 °F * (10 BPM / 1 °F) = ∆ 29 BPM + 82 BPM = 111 BPM

Which assessment finding by the nurse would prohibit application of a heating pad?

Active bleeding Rationale: Heat application increases blood flow; therefore, it's contraindicated in active bleeding. For this same reason, however, applying heat to a reddened abscess, edematous lower leg, or wound with purulent drainage promotes healing

Shortly after being admitted to the coronary care unit with an acute myocardial infarction (MI), a client reports midsternal chest pain radiating down the left arm. The nurse notices that the client is restless and slightly diaphoretic and measures a temperature of 99.6° F (37.6° C), a heart rate of 102 beats/minute; regular, slightly labored respirations at 26 breaths/minute; and a blood pressure of 150/90 mm Hg. Which nursing diagnosis takes highest priority?

Acute pain

What to do for addisons/cushings?

Addison's disease (need to "add" hormone) Cushing's syndrome (have extra "cushion" of hormones)

Addison's Disease vs Cushing's Disease

Addison's disease is characterized by the hyposecretion of adrenal cortex hormones (glucocorticoids and mineralcorticoids), whereas Cushing's disease is characterized by a hypersecretion of glucocorticoids.

mineral corticoids are give in

Addison's disease.

Sources of nutrition: Sodium-Restricted Diet

Aim for ~600mg Na/meal and 200mgNa/Bedtime Snack Sodium may need to be restricted in the diet for a variety of reasons including congestive heart failure, high blood pressure, and diseases associated with fluid imbalance like kidney and liver diseases

Internal radiation ( unsealed source)

All body fluids contaminated, private room , nurse wear dosimeter, limit visitors and time spend in room, rotate staff

Thyroid storm

An acute, potentially fatal exacerbation of hyperthyroidism that may result from manipulation of the thyroid gland during surgery, severe infection, or stress.

After a client receives an I.M. injection, he complains of burning pain in the injection site. Which nursing action would be the best to take at this time?

Apply a warm compress to dilate the blood vessels.

Latex allergies

Assess for allergies to bananas, apricots, cherries, grapes, kiwis, passion fruit, avocados, chestnuts, tomatoes, peaches

Latex allergies?

Assess for allergies to bananas, apricots, cherries, grapes, kiwis, passion fruit,avocados, chestnuts, tomatoes, peaches

The nurse is preparing to remove a previously applied topical medication from a client. The rationale for removing previously applied topical medications before applying new medications is to:

Avoid administering more than the prescribed dose.

After knee replacement surgery, a client is being discharged with acetaminophen and codeine tablets, 30 mg, for pain. During the client's discharge preparations, the nurse should include which instruction?

Avoid driving a car while taking the medication.

Sources of potassium?

Bananas, potatoes, citrus fruits

A 20-year-old mother of a premature neonate smoked cigarettes during her pregnancy. Her son is a client in a neonatal intensive care unit and has a diagnosis of acute respiratory distress syndrome. Because the mother is Roman Catholic, which nursing intervention would be most appropriate for the nurse to discuss with her?

Baptism of the neonate

Common S/S of emphysema?

Barrel chest

Common S/S of GERD?

Barrets Esophagus (erosion of the lower portion of the esophageal mucosa)

Common S/S of orbital fracture?

Battles Sign and Racoon's eyes

What is bleeding considered in ADPIE?

Bleeding is part of the 'circulation' assessment of the ABCD's in an emergent situation. Therefore, if airway and breathing are accounted for, a compound fracture requires assessment before Glasgow coma scale and a neuro check (D=disability, or neuro check)

Fat Embolism:

Blood tinged sputum (r/t inflammation), inc ESR, respiratory alkalosis (not acidosis r/t tachypnea), hypocalcemia,increased serum lipids, "snow storm" effect on CXR.

S/S of a fat embolism?

Blood tinged sputum (r/t inflammation), inc ESR, respiratory alkalosis (not acidosis r/t tachypnea), hypocalcemia,increased serum lipids, "snow storm" effect on CXR.

Common S/S of Hydrocephalosis?

Bossing Sign (prominent forehead)

An adult client, who happens to be the nurse's neighbor, is diagnosed with acquired immunodeficiency syndrome. He's assigned to the nurse's unit. The nurse tells the client's parents about the diagnosis; after all, they know their son is both the nurse's neighbor and friend. Several weeks later the nurse receives a letter from the client's attorney stating that the nurse has infringed on the client's basic legal rights and has committed an intentional tort. Which intentional tort has this nurse committed?

Breach of confidentiality Rationale: A nurse shouldn't disclose confidential information about a client to a third party who has no legal right to know; doing so is a breach of confidentiality. Defamation of character is injuring someone's reputation through false and malicious statements. Assault and battery occurs when the nurse forces a client to submit to treatment against the client's will. A nurse commits fraud when she misleads a client to conceal a mistake she made during treatment.

Which of the following outcome criteria would be most appropriate for the client with a nursing diagnosis of Ineffective airway clearance?

Breath sounds clear on auscultation

Cardiopulmonary arrest

Breathless, pulse-less, unconscious

Hypo-parathyroid?

CATS - convulsions, arrhythmias, tetany, spasms, stridor (decreased calcium), high Ca, low phosphorus diet

CONTACT DERMATITIS

CAUSED BY A HYPERSENSITIVITY RESPONSE TO EXTERNAL/CHEMICAL IRRITANTS

Guided imagery is great for?

CHRONIC pain

COPD and Pneumonia?

COPD is chronic, pneumonia is acute. Emphysema and bronchitis are both COPD. in COPD patients the baroreceptors that detect the CO2 level are destroyed. Therefore, O2 level must be low because high O2 concentration blows the patient's stimulus for breathing.

Why would somone who is allergic to latex be allergic to food too?

CROSS REACTION! may be allergic to some foods, as well. When this happens, your body responds with the same allergic symptoms that you would have if you were exposed to latex. Cross reactions differ from one person to another. Someone may have a reaction to all the foods noted to cause cross reaction while another may have no reaction at all. Likewise, if you are allergic to any of these foods, you may also be allergic to latex: •apples, bananas, kiwi, peaches, plums, figs, grapes, melons, papaya, passion fruit, cherries, nectarines, pears, pineapple and strawberries; •carrots, celery, raw potatoes, avocados and tomatoes; •chestnuts and hazelnuts; •wheat and rye.

CVA?

CVA (cerebrovascular accident) is with dead brain tissue.

Cerebrovascular Accident; caring for a client who has dysphagia

CVA-opp side is affected. (Ex. Right sided CVA, mean left is affected, so would eat on right side of mouth) *client needs thickened liquids, sitting upright, eat soft or semi-soft foods, have tv off so won't distract and aspirate

dilantin level (10-20).

Can cause gingival hyperplasia

No ______ when having an occult blood test

Cantalope ---- b/c it has vitamin C which causes a false positive

Detached Retina: Care

Care Includes: Area of detachment should be in the dependent position.

Shock: Care

Care Includes: Bed-rest w/ extremities elevated 20 degrees, knees straight, head slightly elevated (modified trendelenberg)

After Total Hip Replacement: Care

Care Includes: Don't sleep on the operated side; don't flex hip for more than 45 - 60 degrees, don't elevate HOB more than 45 degrees. Also maintain hip *ABDUCTION* by separating thighs with pillows.

After A Supratentorial Surgery (Incision Behind Hairline): Care

Care Includes: Elevate HOB 30-45 degrees

Above the Knee Amputation: Care

Care Includes: Elevate for the first 24 hours on a pillow, position prone daily to provide for hip extension.

Head Injury: Care

Care Includes: Elevate the HOB 30 degrees to decrease ICP.

Buck's Traction (Skin Traction): Care

Care Includes: Elevate the foot of the bed for counter-traction.

Below Knee Amputation: Care

Care Includes: Foot of the bed elevated for the first 24 hours, position prone daily to provide for hip extension.

Prolapsed Cord: Care

Care Includes: Knee to Chest position OR Trendelenburg

Heat Stroke: Care

Care Includes: Lie *FLAT* w/ legs *Elevated*

During Internal Radiation: Care

Care Includes: On bedrest while implant is in place.

Lumbar Puncture: Care After Procedure

Care Includes: Place the client in the supine position for 4 to 12 hours as prescribed.

After Thyroidectomy: Care

Care Includes: Position *LOW* or *semi - fowlers*, support the head, neck and shoulders.

Infant w/ Cleft Lip: Care

Care Includes: Position on the back or in an infant seat to prevent trauma to the suture line. While feeding, hold in the upright position.

After an Myringotomy: Care

Care Includes: Position patient on the side of the *affected* ear after surgery. (This allows drainage of secretions)

Tube Feeding w/ DECREASED LOC: Care

Care Includes: Position pt on *right side* (this promotes emptying of the stomach) with the HOB *elevated* (to prevent aspiration)

After Cataract Surgery: Care

Care Includes: The patient will sleep on the *unaffected* side with a night shield for 1 - 4 weeks.

Peritoneal Dialysis w/ Inadequate outflow: Care

Care Includes: Turn patient from side to side *BEFORE* checking for kinks in the tubing (According to Kaplan)

During Epidural Puncture: Care

Care Includes: position the patient *SIDE LYING*

Infant w/ Spina Bifidia: Care

Care includes: Position *prone* (on abdomen) so the sac does not rupture.

To assess the effectiveness of cardiac compressions during adult cardiopulmonary resuscitation (CPR), the nurse should palpate which pulse site?

Carotid

What instructions should a client be given before undergoing a paracentesis?

Client must empty bladder prior to procedure.

ACROMEGALY -

Coarse facial feature

Common S/S Acromegaly?

Coarse facial features

A client complains of abdominal discomfort and nausea while receiving tube feedings. Which intervention is most appropriate for this problem?

Decrease the rate of feedings.

Kidneys and ears?

Did you know there is an association between low-set ears and renal anomalies? Now you know what to look for if down's isn't there to choose. (just to expand on it a little, the kidneys and ears develop around the same time in utero. Hence, they're shaped similarly. Which is why when doing an assessment of a neonate, if the nurse notices low set or asymmetrical ears, there is good reason to investigate renal functioning. Knowing that the kidneys and ears are similar shapes helped me remember this).

Implementation

Diet high in Ca, and protein and vit d, estrogen replacement, weight bearing exercise , fosamax and safety to prevent fractures

Psuedomembrane in DIptheria?

Diptheria is an upper respiratory tract infection. It is characterized by sore throat, low fever and the psuedomembrane! Which is just a capsule around the tonsils that you can see with your penlight... It just looks like a whiteish yellow capsule around the tonisils

Gross things to remember about nurses with herpes!!!!

Disseminated Herpes Zoster is AIRBORNE PRECAUTIONS, as to Localized Herpes Zoster is CONTACT PRECAUTIONS. A nurse with a localized herpes zoster CAN care for patients as long as the patients are NOT immunosuppressed and the lesions must be covered!

After billroth II surgery, the client develops dumping syndrome, what does the nurse not want the client to do?

Do not have client sit up for at least 30 min after meals *Sitting upright will produce or promote dumping syndrome. Client needs to lie down after meals.

How do you treat a small bowel obstruction?

Don't fall for 'reestablishing a normal bowel pattern' as a priority with small bowel obstruction. Because the patient can't take in oral fluids 'maintaining fluid balance' comes first.

Pertussis precautions?

Droplet

Dumping syndrome?

Dumping syndrome: increase fat and protein, small frequent meals, lie down after meal to decrease peristalsis, wait 1 hr after meals to drink.

A client receives meperidine (Demerol) 50 mg I.M. for relief of surgical pain. Thirty minutes later, the nurse asks the client if the pain is relieved. Which step of the nursing process is the nurse using?

Evaluation

FIBRIN HYALIN

Expiratory Grunt

FIBRIN HYALIN -

Expiratory Grunt

Cmmon S/S Fibrin Hyalin?

Expiratory grunt-- Causes Infant respiratory distress!

Late Decels?

For late decels, turn the mother to her left side, to allow more blood flow to the placenta.

Side rail rules?

Four up can be considered a form of restraint. Even in LTC facility when a client is a fall risk, keep lower rails down, and one side of bed against the wall, lowest position, wheels locked.

Cute way to remember glascow coma?

GLASGOW COMA SCALE. EYES, VERBAL,MOTOR! It is similar to measuring dating skills...max 15 points -one can do it if below 8 you are in Coma. So, to start dating you gotta open your EYES first, if you albe to do that spontaneously and use them correctly to SEE whom you dating you earn 4. But if she has to scream on you to make you open them it is only 3....and 1 you dont care to open even if she tries to hurt you. if you get good EYE contact (4 points) then move to VERBAL. talk to her/ him! if you can do that You are really ORIENTED in situation she/he uncontiously gives you 4 points! if you like her try not to be CONFUSED (3), and of cause do not use INAPPROPRIATE WORDS (3), she will not like it)), try not to RESPOND WITH INCOMPREHENSIBLE SOUNDS (2), if you do not like herjust show no VERBAL RESPONSE(1) Since you've got EYE and VERBAL contact you can MOVE now using your Motor Response Points. THis is VERY important since Good moves give you 6!

Cushing's Disease and Syndrome Assessment

Generalized muscle weakness and wasting, moon face, buffalo hump, truncal obesity with thin extremities, hirsutism, hyperglycemia, hypernatremia, hypokalemia, hypocalcemia, hypertension, reddish-purple striae on the abdomen and upper thighs.

In a weekly parenting class, the nurse teaches parents ways to foster healthy self-concepts in their children. Which method is most important?

Giving positive feedback

A client receiving beta-adrenergic blockers may have alterations in which of the following laboratory values?

Glucose

Besides sodium, water also follows?

Glucose

Hyperthyroidism assessment

Goiter, palpitations, tachycardia, Afib, protruding eyeballs (exophthalmos), hypertension, heat intolernace, diaphoresis, weight loss, smooth skin and hair, tremors of the hands, mood swings.

DUCHENNE'S MUSCULAR DYSTROPHY -

Gowers' sign (use of hands to push one's self from the floor

APGAR: G

Grimace - cough/grimace/no response

Peptic ulcers caused by

H. pylori are treated with Flagyl, Prilosec and Biaxin. This treatment kills bacteria and stops production of stomach acid, but does not heal ulcer.

RISK FOR INFECTION

HIGHEST PRIORITY FOR AN ELDERLY CLIENT DX WITH HERPES ZOSTER IS?

Ventilator Alarms?

HOLD High alarm- Obstruction due to incr. secretions, kink, pt. coughs, gag or bites Low press alarm- Disconnection or leak in ventilatior or in pt. airway cuff, pt. stops spontaneous breathing

*Thyroid storm is *Myxedema coma is

HOT (hyperthermia) COLD (hypothermia)

Apresoline(hydralazine)

HTN or CHF, Report flu-like symptoms, rise slowly from sitting/lying position; take with meals.

Common S/S of Increased ICP?

HYPERTENSION, BRADYpnea, BRADYcardia (cushings triad)!

Common S/S of Shock?

HYPERtension TACHYpnea and TACHYcardia

0. INFECTIOUS MONONUCLEOSIS -

Hallmark: sore throat, cervical lymph adenopathy, fever

INFECTIOUS MONONUCLEOSIS

Hallmark: sore throat, cervical lymph adenopathy, fever

Common S/S of Infectious Mononucleosis?

Hallmark= Sore throat, cervical lymph adenopathy, fever

LVN/LPN cant?

Handle any blood

After intentionally taking an overdose of hydrocodone (Vicodin), a client is admitted to the emergency department. Activated charcoal is prescribed. Before administering the drug, the nurse should ensure that the client:

Has audible bowel sounds

Lupus

Have no baby until remission

Rules of nine for burns

Head and Neck= 9% Each upper ext= 9% Each lower ext= 18% 10Front trunk= 18% Back trunk= 18% Genitalia= 1%

Rule of nines?

Head and Neck= 9% Each upper ext= 9% Each lower ext= 18% Front trunk= 18% Back trunk= 18% Genitalia= 1%

Hepatitis

Hepatitis: -ends in a VOWEL, comes from the BOWEL (Hep A) Hepatitis B=Blood and Bodily fluids Hepatitis C is just like B

Administration of an Enema

Here, position the patient in *left side-lying* (Sims) w/ Knee Flexed.

Dengue hemorrhagic fever?

Hermans sign= flushes and redness of skin with lighter color at the center of the rash.... It is a disease that comes from tropical and subtropical areas that are spread by misquotos

Carbon dioxide narcosis?

High potassium is expected (hydrogen floods the cell forcing potassium out). Causes increased intracranial pressure.

A client has lymphedema in both arms and the nurse must measure blood pressure using a thigh cuff. In reference to the client's baseline arm blood pressure, the nurse should expect the thigh to have a:

Higher systolic blood pressure reading. Rationale: Systolic readings in the thigh may be 10 to 40 mm Hg higher than in the arm. Diastolic readings are the same in the arm and thigh.

chicken pox

Highly contagious viral infection, Symptoms are a body rash and fever. Also known as varicella.

How do the following impact blood pressure? A. Blood pressure cuff too narrow B. Blood pressure cuff too wide C. Assessing immediately after smoking D. Assessing immediately after eating E. Assessing when the client is in mild-to-moderate pain F. Assessing when the client experiences severe pain G. Assessing immediately after exercise

How do the following impact blood pressure? A. Blood pressure cuff too narrow: False Increase B. Blood pressure cuff too wide: False Decrease C. Assessing immediately after smoking: Increase D. Assessing immediately after eating: Increase E. Assessing when the client is in mild-to-moderate pain: Increase F. Assessing when the client experiences severe pain: Increase. Eventually chronic pain modulates to decrease. G. Assessing immediately after exercise: Immediately upon stopping it is increased, but within 5 minutes decreases.

hyperparathyroidism

Hypercalcaemia is characterized by high calcium in the blood. Symptoms include fatigue, polydispia, confusion, anorexia, depression, polyuria, nausea, proximal myopathy, constipation, nephrolithiasis, pacreatitis and peptic ulcer disease. It can be caused by ___ or malignancy among others (granulomatous disease, drugs, renal failue paget's disease, thyrotoxicosis, immobilisation, FHH, Addisons disease,etc.)

Dangerous thing to get during pregnancy?

I kept forgetting which was dangerous when you're pregnant; regular measles (rubeola), or German measles (rubella), so remember: -never get pregnant with a German (rubella)

A nurse is caring for a client with a history of GI bleeding, sickle cell disease, and a platelet count of 22,000/μl. The client is dehydrated and receiving dextrose 5% in half-normal saline solution at 150 ml/hour. He reports severe bone pain and is scheduled to receive a dose of morphine sulfate. In administering the medication, the nurse should avoid which route?

I.M.

ICP and Shock have?

ICP AND SHOCK HAVE OPPOSITE V/S ICP-increased BP, decreased pulse, decreased resp. SHOCK- decreased BP, increased pulse, increased resp.

*Cushings triad r/t

ICP in BRAIN (htn, bradycard, irr. resp)

HYPOCHLOREMIA

IN A PT WITH 2ND-3RD DEGREE BURN, THE NURSE SHOULD MONITOR FOR FLUID IMBALANCE AND WHICH ELECTROLYTE IMBALANCE?

IVF

ISOTONIC: 0.9%NACL,LR, D5W HYPOTONIC: D2.5W, 1/2NACL, 0.33%NACL, tap water HYPERTONIC: D10W, 3% NACL, 5%NACL, D5LR, D51/2NACL, D5NACL, TPN

Never give K+ in

IV push.

A client is hospitalized with Pneumocystis carinii pneumonia. The nurse notes that the client has had no visitors, is withdrawn, avoids eye contact, and refuses to take part in conversation. In a loud and angry voice the client demands that the nurse leave the room. The nurse formulates a nursing diagnosis of Social isolation. Based on this diagnosis, what's an appropriate goal for this client?

Identifying one way to increase social interaction

Hearing the baby in OB?

If the baby is a posterior presentation, the sounds are heard at the sides. If the baby is anterior, the sounds are heard closer to midline, between teh umbilicus and where you would listen to a posterior presentation. If the baby is breech, the sounds are high up in the fundus near the umbilicus. If the baby is vertex, they are a little bit above the symphysis pubis.

Yeast infection in a babys mouth?

If you can remove the white patches from the mouth of a baby it is just formula. If you can't, its candidiasis.

A preggo in a minus station?

If your laboring mom's water breaks and she is any minus station you better know there is a risk of prolapsed cord.

Tenkhoff cath?

It's ok to have abdominal craps, blood tinged outflow and leaking around site if the Peritoneal Dialysis cath was placed in the last 1-2 wks. Cloudy outflow NEVER NORMAL.

Common S/S of DKA

Kussmauls breathing (deep rapid RR)

LITHIUM

L-level of therapeutic affect is 0.5-1.5 I-indicate mania T-toxic level is 2-3 - N/V, diarrhea, tremors H-hyrdrate 2-3L of water/day I-increased UO and dry mouth U-uh oh; give Mannitol and Diamox if toxic s/s are present M-maintain Na intake of 2-3g/day

Strile dressing belongs to

LPN not CNA

Hct

M - 42-52%, W - 37-47% blood test

SOLAR RADIATION

MAJOR EITIOLOGICAL FACTOR IN BASAL CELL CARCINOMA

* Amphogel and Renegal take with

MEALS

Ask for anaphylactic rxn to eggs or neomycin before giving

MMR

High priority in Addisons?

Managing stress in a patient is paramount, because if the adrenal glands are stressed further it could result in Addisonian crisis. While we're on Addison's, remember blood pressure is the most important assessment parameter, as it causes severe hypotension.

Aucoustic neuroma

Nerve8 impact, deafness, dizziness. Craniatomy performed

Traction rule?

Never release traction unless you have an order from an MD to do so

PEMPHIGUS VULGARIS

Nikolsky's sign (separation of epidermis caused by rubbing of the skin)

PEMPHIGUS VULGARIS -

Nikolsky's sign (separation of epidermis caused by rubbing of the skin

Common S/S of Pemphigus Vulgaris?

Nilosky's sign (seperation of epidermis caused by rubbibng of the skin)

Kids pain relief in NCLEX land?

No aspirin with kids b/c it is associated with Reye's Syndrome, and also no nsaids such as ibuprofen. Give Tylenol.

A physician writes the following order for a client: "Prednisone 5 mg P.O. daily for 3 days." When the order is transcribed on the Kardex, the nurse who transcribes the order neglects to place the limitation of 3 days on the prescription. On the 4th day after the order was instituted, a nurse administers prednisone 5 mg by mouth. During an audit of the chart, the error is identified. The person most responsible for the error is the:

Nurse who administered the erroneous dose.

What is a major risk factor for diabetes mellitus?

Obesity

Rule of thumb for obsessions/distractions?

Obsession is to thought. Compulsion is to action.

While monitoring an I.V. infusion, the nurse notices that the ground on the infusion pump's plug is missing. What should the nurse do first?

Obtain another pump from central supply for the infusion.

An Irish immigrant is 1 day postoperative following extensive abdominal surgery. The physician has ordered 100 mg of meperidine (Demerol) every 4 hours as needed for pain. The nurse notices that the client hasn't asked for pain medication since surgery. When the nurse assesses the client, he's stoic and asks to be left alone. The nurse should:

Offer him his pain medication. Rationale: The Irish culture's usual reaction to pain is inexpressive and stoic. Irish people usually don't vocalize that they have pain and will try to hide the pain from family. The nurse should offer the pain medication so that comfort can be extended to the client.

Cranial Nerves: *I am sorry if this vulgar for some, but hey, it sticks Sensory=S Motor=M Both=B

Oh (Olfactory I) Some Oh (Optic II) Say Oh (Oculomotor III) Marry To (Trochlear IV) Money Touch (Trigeminal V) But And (Abducens VI) My Feel (Facial VII) Brother A (Auditory VIII) Says Girls (Glossopharyngeal IX) Big Vagina (Vagus X) Breast And (Accessory XI) Matter Hymen (Hypoglassal XII) More

A certified nursing assistant (CNA) is caring for a client with Clostridium difficile diarrhea and asks the charge nurse, "How can I keep from catching this from the client?" The nurse reminds the CNA to wash her hands and to ensure the client is placed:

On contact isolation. Rationale: C. difficile can be transmitted from person to person by hands or waste containers such as a bedpan. When in direct contact with the client, the nurse should practice contact isolation, which includes wearing gloves and a gown. Protective isolation is used to protect a client who is immunocompromised, which isn't evident in this case. Neutropenic precautions are for clients with an absolute neutrophil count of 1,000/μl or less; this isn't evident in this case. A negative-pressure room is used when the organism is spread by the airborne route, which isn't true of C. difficile diarrhea.

Kernigs sign?

One of the physically demonstrable symptoms of meningitis Severe stiffness of the hamstrings causes an inability to straighten the leg when the hip is flexed to 90 degrees.

Brudzinski's sign?

One of the physically demonstrable symptoms of meningitis. Severe neck stiffness causes a patient's hips and knees to flex when the neck is flexed ( The kid knees will fly up and he lift his bum off the table cause it hurts his neck)

MG and Guillian Barre?

Opposites!

Which of the following assessments would be most supportive of the nursing diagnosis Impaired skin integrity related to purulent wound drainage?

Oral temperature of 101° F (38.3° C)

Monamine Oxidase Inhibitors: Adverse Effects of Phenelzine (Nardil)

Orthostatic hypotension, dizziness, HA, fatigue, tremors, weakness, dry mouth, weight gain, constipation

G tube and J tubes are usually?

Other than initially to test tolerance, given as continuous feedings.

Common S/S of Basilar Fracture?

Ottorhea

Ovaries and Testes

Ovaries produce estrogen and progesterone. Testes control the development of the secondary sex characteristics, and produce testosterone.

Best time to take Growth Hormone ____, Steroids ___, Diuretics ___, Aricept ___

PM. AM AM AM

More info on TB testing a positive result?

PPD is positive if area of induration is: >5 mm in an immunocompromised patient >10 mm in a normal patient >15 mm in a patient who lives in an area where TB is very rare.

RESPIRATORY

PSORIASIS CAN BE BROUGHT ON BY _____ INFECTION

Hightest priority for RA?

Pain is usually the highest priority

The nurse is caring for a client receiving patient-controlled analgesia (PCA) for pain management. Which statement about PCA is true?

Pain relief is initiated by the client as needed.

Common S/S of glaucoma?

Painful vision loss, tunnel/gun barrel/ halo vision (peripheral vision loss)

GLAUCOMA

Painfull vision loss, tunnel/gun barrel/halo vision (Peripheral Vision Loss)

GLAUCOMA -

Painfull vision loss, tunnel/gun barrel/halo vision (Peripheral Vision Loss)

CATARACT -

Painless vision loss, Opacity of the lens, blurring of vision

Common S/S of Hodgkins Disease/Lymphoma?

Painless, progressive englargement of spleen and lymph tissues, and Reedstenberg cells!

A client hasn't voided since before surgery, which took place 8 hours ago. When assessing the client, the nurse will:

Palpate the bladder above the symphysis pubis.

cogentin

Parkinson & extrapyramidal effects of other drug

artane

Parkinson . ....sedative effects also

sinemet

Parkinson .....sweat, saliva, urine may turn reddish brown occasionally ..... causes drowsiness

A client on I.V. heparin should have which of the following laboratory values monitored closely to determine whether the therapeutic range is maintained?

Partial thromboplastin time (PTT) Rationale: PTT is the laboratory value that gives specific information regarding the effectiveness of heparin therapy. The other tests are inappropriate.

When you see coffee brown emesis think?>

Peptic ulcer

Likely cause of cardiac arrest in child?

Prolonged hypoxemia is a likely cause of cardiac arrest in a child.

Pituitary gland

Promotes growth of body tissue, influences water absorption by the kidney, and controls sexual development and function.

rH negative women with rH positive baby with previous any

RHOGAM is needed

* Non dairy sources of calcium include

RHUBARB, SARDINES, COLLARD GREENS

New admition, NG , teaching, critical judgement belong to

RN

Nondairy sources of calcium?

RUBARB, SARDINES, COLLARD GREENS

Dance Sign?

RUQ mass (intussusception) with RLQ empty space (movement of cecum out of normal position).

182.) A client with angina pectoris is experiencing chest pain that radiates down the left arm. The nurse administers a sublingual nitroglycerin tablet to the client. The client's pain is unrelieved, and the nurse determines that the client needs another nitroglycerin tablet. Which of the following vital signs is most important for the nurse to check before administering the medication? 1. Temperature 2. Respirations 3. Blood pressure 4. Radial pulse rate

Rationale: Nitroglycerin acts directly on the smooth muscle of the blood vessels, causing relaxation and dilation. As a result, hypotension can occur. The nurse would check the client's blood pressure before administering the second nitroglycerin tablet. Although the respirations and apical pulse may be checked, these vital signs are not affected as a result of this medication. The temperature also is not associated with the administration of this medication.

134.) A nurse reinforces instructions to a client who is taking levothyroxine (Synthroid). The nurse tells the client to take the medication: 1. With food 2. At lunchtime 3. On an empty stomach 4. At bedtime with a snack

Rationale: Oral doses of levothyroxine (Synthroid) should be taken on an empty stomach to enhance absorption. Dosing should be done in the morning before breakfast. **Note that options 1, 2, and 4 are comparable or alike in that these options address administering the medication with food.**

206.) A client is receiving baclofen (Lioresal) for muscle spasms caused by a spinal cord injury. The nurse monitors the client, knowing that which of the following is a side effect of this medication? 1. Muscle pain 2. Hypertension 3. Slurred speech 4. Photosensitivity

Rationale: Side effects of baclofen include drowsiness, dizziness, weakness, and nausea. Occasional side effects include headache, paresthesia of the hands and feet, constipation or diarrhea, anorexia, hypotension, confusion, and nasal congestion. Paradoxical central nervous system excitement and restlessness can occur, along with slurred speech, tremor, dry mouth, nocturia, and impotence. **Option 3 is most closely associated with a neurological disorder**

A female client is readmitted to the facility with a warm, tender, reddened area on her right calf. Which of the following contributing factors would the nurse recognize as most important?

Recent pelvic surgery

Common S/S of Ulcerative Colitis?

Recurrent bloody diarrhea

Rifampin

Red orange tears and urine, also contraceptives don't work as well

When assessing a client with cellulitis of the right leg, which of the following would the nurse expect to find?

Red, swollen skin with inflammation spreading to surrounding tissues

In depth- Color codes??

Red- Immediate: Injuries are life threatening but survivable with minimal intervention. Ex: hemothrax, tension pneumothorax, unstable chest and abdominal wounds, INCOMPLETE amputations, OPEN fx's of long bones, and 2nd/3rd degree burn with 15%-40% of total body surface, etc. Yellow- Delayed: Injuries are significant and require medical care, but can wait hrs without threat to life or limb. Ex: Stable abd wounds without evidence of hemorrhage, fx requiring open reduction, debridement, external fixation, most eye and CNS injuries, etc. Green- Minimal: Injuries are minor and tx can be delayed to hrs or days . Individuals in this group should be moved away from the main triage area. Ex: upper extremity fx, minor burns, sprains, sm. lacerations, behavior disorders. Black- Expectant: Injuries are extensive and chances of survival are unlikely. Seperate but dont abandoned, comfort measures if possible. Ex: Unresponsive, spinal cord injuries, woulds with anatomical organs, 2nd/3rd degree burn with 60% of body surface area , seizures, profound shock with multipe injuries, no pulse, b.p, pupils fixed or dilated.

emergency red, yellow, green, black

Red- Immediate: Injuries are life threatening but survivable with minimal intervention. Ex: hemothrax, tension pneumothorax, unstable chest and abdominal wounds, INCOMPLETE amputations, OPEN fx's of long bones, and 2nd/3rd degree burn with 15%-40% of total body surface, etc. Yellow- Delayed: Injuries are significant and require medical care, but can wait hrs without threat to life or limb. Ex: Stable abd wounds without evidence of hemorrhage, fx requiring open reduction, debridement, external fixation, most eye and CNS injuries, etc. Green- Minimal: Injuries are minor and tx can be delayed to hrs or days . Individuals in this group should be moved away from the main triage area. Ex: upper extremity fx, minor burns, sprains, sm. lacerations, behavior disorders. Black- Expectant: Injuries are extensive and chances of survival are unlikely. Seperate but dont abandoned, comfort measures if possible. Ex: Unresponsive, spinal cord injuries, woulds with anatomical organs, 2nd/3rd degree burn with 60% of body surface area , seizures, profound shock with multipe injuries, no pulse, b.p, pupils fixed or dilated.

Color codes?

Red--unstable, ie, occluded airway, actively bleeding, see first Yellow---stable, can wait up to an hour for treatment, ie burns, see second Green---stable, can wait even longer to be seen, "walking wounded" Black--unstable clients that will probably not make it, need comfort measures DOA---dead on arrival

Emergency Triage

Red--unstable, ie, occluded airway, actively bleeding, see first Yellow---stable, can wait up to an hour for treatment, ie burns, see second Green---stable, can wait even longer to be seen, "walking wounded" Black--unstable clients that will probably not make it, need comfort measures DOA---dead on arrival

When assessing a client's incision 1 day after surgery, the nurse expects to see which of the following as signs of a local inflammatory response?

Redness and warmth

Negative Feedback loop

Regulates hormone secretion by the hypothalamus and pituitary gland. Increased amounts of target gland hormones in the bloodstream decrease secretion of the same hormone and other hormones that stimulate its release.

Adrenal gland

Regulates sodium and electrolyte balance; affects carbohydrate, fat, and protein metabolism; influences the development of sexual characteristics; and sustains the flight or fight response.

Heart Defects?

Remember for cyanotic -3T's( Tof, Truncys arteriosus, Transposition of the great vessels). Prevent blood from going to heart. If problem does not fix or cannot be corrected surgically, CHF will occur following by death.

The nurse has just removed an I.V. catheter from a client's arm because fluid has infiltrated the arm. The physician orders warm compresses for the area. Based on the principles of heat and cold application, the nurse would:

Remove the warm compresses after 20 minutes for at least 15 minutes.

Renal impairment labs?

Renal impairment: serum creatinine elevated and urine clearance decreased

Common S/S of Tetany?

Risus Sardonicus

Common S/S of pneumonia?

Rusty sputum

Woman in Labor w/ NON-Reassuring FHR: S&S and Care

S&S Include: late decelerations, decreased variability, fetal bradycardia, etc. Care: turn on *LEFT SIDE* (also give O2, stop pitocin and increase fluids)

Labs in DKA>?

Serum acetone and serum ketones rise. As you treat the acidosis and dehydration expect the potassium to drop rapidly, so be ready, with potassium replacement. Fluids are the most important intervention, so get fluids going first. With HHNS there is no ketosis, and no acidosis. Potassium is low in HHNS (d/t diuresis).

Following a tonsillectomy, a client returns to the medical-surgical unit. The client is lethargic and reports having a sore throat. Which position would be most therapeutic for this client?

Side-lying

GLASGOW Scale: How to remember

So, to start dating you gotta open your EYES first, if you albe to do that spontaneously and use them correctly to SEE whom you dating you earn 4. But if she has to scream on you to make you open them it is only 3.... and 1 you dont care to open even if she tries to hurt you if you get good EYE contact (4 points) then move to ....... = VERBAL talk to her/ him! if you can do that You are really ORIENTED in the situation she/he gets you 4 points! if you like her try not to be CONFUSED (3), and of course do not use INAPPROPRIATE WORDS (3), she will not like it)), try not to RESPOND WITH INCOMPREHENSIBLE SOUNDS (2), if you do not like her just show no VERBAL RESPONSE(1) Since you've got EYE and VERBAL contact you can MOVE now using your Motor Response Points. THis is VERY important since Good moves give you 6

3 heart defects. Remember for cyanotic -3T's

Tertology of Fallot, Truncys arteriosus, Transposition of the great vessels Prevent blood from going to heart. If problem does not fix or cannot be corrected surgically, CHF will occur following by death.

Transesophageal Fistula (TEF) - esophagus doesn't fully develop (this is a surgical emergency

The 3 C's of TEF in the newborn: 1) Choking 2) Coughing 3) Cyanosis

Diabetes insipidus

The hyposecretion of antidiuretic hormone from the posterior pituitary gland, resulting in failure of tubular reabsorption of water in the kidneys and diuresis.

autosomal recessive

Trait is recessive: phenotype can skip a generation, affected individual can have unaffected parents, affects both male and female

Insulin treatments

Used to treat type 1 and 2 diabetes mellitus when diet, weight control therapy, and oral hypoglycemic agents have failed to maintain satisfactory blood glucose levels.

Common S/S Meniere's Disease?

Vertigo, Tinnitus

LENTIGO

WHAT TYPE OF LESION MAY DEVELOP INTO TRUE MELANOMA OVER TIME?

NG TUBE INSERTION

WHAT WOULD THE NURSE PREPARE TO DO FOR THE CLIENT WHO IS 6HR POST BURN AND HAS ABSENT BOWEL SOUNDS AND ABD DISTENTION?

CANDIDA

WHITE MILKY PLAQUE ON TONGUE THAT DEVELOPS AS A RESULT OF OVERGROWTH OF BACTERIA AFTER ATBS

Give NSAIDS, Corticosteroids, drugs for Bipolar, Cephalosporins, and Sulfanomides

WITH food.

The nurse is providing care for a client who underwent mitral valve replacement. The best example of a measurable client outcome goal is to:

Walk from his room to the end of the hall and back before discharge.

Milk for kids?

Watch out for questions suggesting a child drinks more than 3-4 cups of milk each day. (Milks good, right?) Too much milk reduces intake of other essential nutrients, especially iron. Watch for anemia with milk-aholics. And don't let that mother put anything but water in that kid's bottle during naps/over-night. Juice or milk will rott that kids teeth right out of his head.

Weight naso-intestinal tubes

Weighted NI (Naso intestinal tubes) must float from stomach to intestine. Don't tape the tube right away after placement, may leave coiled next to pt on HOB. Position patient on RIGHT to facilitate movement through pylorus

Traction in kids?

What traction is used in a school-age kid with a femur or tibial fracture with extensive skin damage? Ninety, ninety. Huh? I never heard of it either. The name refers to the angles of the joints. A pin is placed in the distal part of the broken bone, and the lower extremity is in a boot cast. The rest is the normal pulleys and ropes you're used to visualizing with balanced suspension. While we're talking about traction, a kid's hinder should clear the bed when in Bryant's traction (also used for femurs and congenial hip for young kids).

Common S/S of asthma

Wheezing on EXPIRATION

When are oral meds prescribed for type 2 diabetics?

When diet and weight control therapy have failed to maintain satisfactory blood glucose levels.

MORE info on DKA? ugh!

While treating DKA, bringing the glucose down too far and too fast can result in increased intracranial pressure d/t water being pulled into the CSF. Polyuria is common with the hypercalcemia caused by hyperparathyroidism.

How do you teach someone to reduce back aches?

With low back aches, bend knees to relieve William's position

A child, age 3, is brought to the emergency department in respiratory distress caused by acute epiglottiditis. Which clinical manifestations should the nurse expect to assess? a. severe sore throat, drooling, and inspiratory strider b. low grade fever, stridor, and a barking cough c. pulmonary congestion, a productive cough, and a fever d. sore throat, a fever, and general malaise

a - a child with acute epiglottiditis appears acutely ill and clinical manifestations may include drooping (because of difficulty swallowing), severe sore throat, hoarseness, a high temperature, and severe inspiratory stridor. A low grade fever, stridor, and barking cough that worsens at night are suggestive of croup. Pulmonary congestion, productive cough, and fever along with nasal flaring, retractions, chest pain, dyspnea, decreased breath sounds, and crackles indicate pneumococcal pneumonia. A sore throat, fever, and general malaise point to viral pharyngitis.

A nurse is caring for a client with lower back pain who is scheduled for myelography using metrizamide (a water-soluble contrast dye). After the test, the nurse should place the client in which position? a. head of the bed elevated 45 degrees b. prone c. supine with feet raised d. supine with the head lower than the trunk

a - after a myelogram, positioning depends on the dye injected. When a water-soluble dye such as metrizamide in injected, the head of the bed is elevated to a 45-degree angle to slow the upward dispersion of the dye. The prone and supine positions are contraindicated when a water-soluble contrast dye is used. The client should be positioned supine with the head lower than the trunk after an air-contrast study.

when caring for a client after a closed renal biopsy, the nurse should: a. maintain the client on strict bed rest in a supine position for 6 hours b. insert an indwelling catheter to monitor urine output c. apply a sandbag to the biopsy site to prevent bleeding d. administer IV opioid medications to promote comfort

a - after a renal biopsy, the client is maintained on strict bed rest in a supine position for at least 6 hours to prevent bleeding. If no bleeding occurs, the client typically resumes general activity after 24 hours. Urine output is monitored, but an indwelling catheter is not typically inserted. A pressure dressing is applied over the site, but a sandbag is not necessary. Opioids to control pain would not be anticipated; local discomfort at a biopsy site can be controlled with analgesics.

the comatose victim of the car accident is to have a gastric lavage. Which of the following positions would be most appropriate for the client during this procedure? a. lateral b. supine c. trendelenburg's d. lithotomy

a - an unconscious client is best positioned in a lateral or semiprone position because these positions allow the jaw and tongue to fall forward, facilitate drainage of secretions, and prevent aspiration. Positioning the client supine carries a major risk of airway obstruction from the tongue, vomit, or nasopharyngeal secretions. Trendeleburg's position, with the head lower than the heart, decreases effective lung volume and increases the risk of cerebral edema. The lithotomy position has no purpose in this situation.

The nurse is assessing a client at her postpartum checkup 6 weeks after a vaginal delivery. The mother is bottle feeding her baby. Which client finding indicates a problem at this time? a. firm fundus at the symphysis b. white, thick vaginal discharge c. striae that are silver in color d. soft breasts without milk

a - by 4 to 6 weeks postpartum, the fundus should be deep in the pelvis and the size of a non-pregnant uterus. Subinvolution, caused by infection or retained placental fragments, is a problem associated with a uterus that is larger than expected at this time. Normal expectations include a white, thick vaginal discharge, striae that are beginning to fade to silver, and breasts that are soft without evidence of milk production (in a bottle feeding mother).

A client with type 1 diabetes must undergo bowel resection in the morning. How should the nurse proceed while caring for him on the morning of surgery? a. administer half of the client's typical morning insulin dose as ordered b. administer an oral antidiabeteic agent as ordered c. administer an I.V. insulin infusion as ordered d. administer the client's normal daily dose of insulin as ordered

a - if the nurse administers the client's normal daily dose of insulin while he's on nothing-by-mouth status before surgery, he'll experience hypoglycemia. Therefore, the nurse should administer half the daily insulin dose as ordered. Oral antidiabetic agents aren't effective for type 1 diabetes I.V. insulin infusions aren't necessary to manage blood glucose levels in clients undergoing routine surgery.

The best indicator that the client has learned how to give an insulin self-injection correctly is when the client can: a. perform the procedure safely and correctly b. critique the nurse's performance of the procedure c. explain all of the steps of the procedure correctly d. correctly answer a post-test about the procedure

a - the nurse should judge that learning has occurred from evidence of a change I the client's behavior. A client who performs a procedure safely and correctly demonstrates that he has acquired a skill. Evaluation of this skill acquisition requires performance of that skill by the client with observation by the nurse. The client must also demonstrate cognitive understanding, as shown by the ability to critique the nurse's performance. Explaining the steps demonstrates of knowledge at the cognitive level only. A post-test does not indicate the degree to which the client has learned a psychomotor skill.

Change in color is ?

a LATE sign! Always~!

diabetes mellitus

a chronic disorder of glucose intolerance and impaired carbohydrate, protein, and lipid metabolism caused by a deficiency of insulin or resistance to the action of insulin. A deficiency of effective insulin results in hyperglycemia.

Which of the following statements would provide the best guide for activity during the rehabilitation period for a client who has been treated for retinal detachment? a) Activity is resumed gradually, and the client can resume her usual activities in 5 to 6 weeks. b) Activity level is determined by the client's tolerance; she can be as active as she wishes. c) Activity level will be restricted for several months, so she should plan on being sedentary. d) Activity level can return to normal and may include regular aerobic exercises.

a) CORRECT ANSWER Activity is resumed gradually, and the client can resume her usual activities in 5 to 6 weeks. Reason: The scarring of the retinal tear needs time to heal completely. Therefore, resumption of activity should be gradual; the client may resume her usual activities in 5 to 6 weeks. Successful healing should allow the client to return to her previous level of functioning.

A client is prescribed metaproterenol (Alupent) via a metered-dose inhaler, two puffs every 4 hours. The nurse instructs the client to report adverse effects. Which of the following are potential adverse effects of metaproterenol? a) Irregular heartbeat. b) Constipation. c) Pedal edema. d) Decreased pulse rate.

a) CORRECT ANSWER Irregular heartbeat. Reason: Irregular heartbeats should be reported promptly to the care provider. Metaproterenol (Alupent) may cause irregular heartbeat, tachycardia, or anginal pain because of its adrenergic effect on beta-adrenergic receptors in the heart. It is not recommended for use in clients with known cardiac disorders. Metaproterenol does not cause constipation, pedal edema, or bradycardia.

Which of the following should the nurse use to determine achievement of the expected outcome for an infant with severe diarrhea and a nursing diagnosis of Deficient fluid volume related to passage of profuse amounts of watery diarrhea? a) Moist mucous membranes. b) Passage of a soft, formed stool. c) Absence of diarrhea for a 4-hour period. d) Ability to tolerate intravenous fluids well.

a) CORRECT ANSWER Moist mucous membranes. Reason: The outcome of moist mucous membranes indicates adequate hydration and fluid balance, showing that the problem of fluid volume deficit has been corrected. Although a normal bowel movement, ability to tolerate intravenous fluids, and an increasing time interval between bowel movements are all positive signs, they do not specifically address the problem of deficient fluid volume.

Prochlorperazine (Compazine) is prescribed postoperatively. The nurse should evaluate the drug's therapeutic effect when the client expresses relief from which of the following? a) Nausea. b) Dizziness. c) Abdominal spasms. d) Abdominal distention.

a) CORRECT ANSWER Nausea. Reason: Prochlorperazine is administered postoperatively to control nausea and vomiting. Prochlorperazine is also used in psychotherapy because of its effects on mood and behavior. It is not used to treat dizziness, abdominal spasms, or abdominal distention.

Obsession is to_______ as Compulsion is to _______

a. thought b. action

Hep A precautions?

acute stage gown and gloves are required. In the convalescent stage it is no longer contagious.

Glucose Tolerance test

aids in the diagnosis of diabetes mellitus. 2 hours after injection or ingestion of glucose, a level higher than 200 mg/dL confirms the diagnosis of diabetes mellitus.

hepatitis B

an acute (sometimes fatal) form of viral hepatitis caused by a DNA virus that tends to persist in the blood serum and is transmitted by sexual contact or by transfusion or by ingestion of contaminated blood or other bodily fluids

With flecainide (Tambocor)

an antiarrythmic, limit fluids and sodium intake, because sodium increases water retention which could lead to heart failure.

Meds for MG

anticholisterase, corticosteroids, immunosuppressants

vitamin K

antidote for coumadin toxicitiy

protamine sulfate

antidote for heparin toxicity

Client Rights: Refusing Treatment

arguement among family, or doubt on healthcare team, treatment must be given until case is resolved in court. -Only time a person does not have right to make decision to refuse treatment is when greater public interest would be in danger

Guillain Barre Syndrome

ascending muscle weakness

Guillian Barre?

ascending paralysis. Keep eye on respiratory system.

Pyelogram

assess allergies

Pyelogram-

assess allergies

fluid volume excess

assess by daily weight

MRI

assess for claustrophobia, remove metal jewelry, ask if the client has metal implant

ICP

assess for decreased level of consciousness

client who is immediate post gastroscopy period, what would be included in the plan of care

assess gag reflex prior to administration of fluids, due to anestetic that would been sprayed in throat prior

A client returns from an endoscopic procedure during which he was sedated. Before offering the client food, it's most important for the nurse to:

assess his gag reflex.

pres school

associatative play

Theophylline

asthma or COPD..therap drug level: 10-20

When is Rhogam given and how?

at 28 weeks, 72 hours post partum, IM. Only given to Rh NEGATIVE mother.

IV push should go over..

at least 2 min

After staying several hours with her 9-year-old daughter who is admitted to the hospital with an asthma attack, the mother leaves to attend to her other children. The child exhibits continued signs and symptoms of respiratory distress. Which of the following findings should lead the nurse to believe the child is experiencing anxiety? a) Not able to get comfortable. b) Frequent requests for someone to stay in the room. c) Inability to remember her exact address. d) Verbalization of a feeling of tightness in her chest.

b - A 9-year-old child should be able to tolerate being alone. Frequently asking for someone to be in the room indicates a degree of psychological distress that, at this age, suggests anxiety. The inability to get comfortable is more characteristic of a child in pain. Inability to answer questions correctly may reflect a state of anoxia or a lack of knowledge. Tightness in the chest occurs as a result of bronchial spasms.

A pregnant client in her third trimester is started on chlorpromazine (Thorazine) 25 mg four times daily. Which of the following instructions is most important for the nurse to include in the client's teaching plan? a) "Don't drive because there's a possibility of seizures occurring." b) "Avoid going out in the sun without a sunscreen with a sun protection factor of 25." c) "Stop the medication immediately if constipation occurs." d) "Tell your doctor if you experience an increase in blood pressure."

b - Chlorpromazine is a low-potency antipsychotic that is likely to cause sun-sensitive skin. Therefore the client needs instructions about using sunscreen with a sun protection factor of 25 or higher. Typically, chlorpromazine is not associated with an increased risk of seizures. Although constipation is a common adverse effect of this drug, it can be managed with diet, fluids, and exercise. The drug does not need to be discontinued. Chlorpromazine is associated with postural hypotension, not hypertension. Additionally, if postural hypotension occurs, safety measures, such as changing positions slowly and dangling the feet before arising, not stopping the drug, are instituted.

A 7 year old with a history of tonic-clonic seizures has been actively seizing for 10 minutes. The child weighs 22 kg and currently has an intravenous (IV) line of D5 1/2 NS + 20 meq KCL/L running at 60 ml/hr. Vital signs are a temperature of 38 degrees C, heart rate of 120, respiratory rate of 28, and oxygen saturation of 92%. Using the SBAR (Situation-Background-Assessment-Recommendation) technique for communication, the nurse calls the primary healthcare provider with a recommendation for: a) Rectal diazepam (Diastat). b) IV lorazepam (Ativan). c) Rectal acetaminophen (Tylenol). d) IV fosphenytoin.

b - IV ativan is the benzodiazepine of choice for treating prolonged seizure activity. IV benzodiazepines potentiate the action of the gamma-aminobutyric acid (GABA) neurotransmitter, stopping seizure activity. If an IV line is not available, rectal Diastat is the benzodiazepine of choice. The child does have a low-grade fever; however, this is likely caused by the excessive motor activity. The primary goal for the child is to stop the seizure in order to reduce neurologic damage. Benzodiazepines are used for the initial treatment of prolonged seizures. Once the seizure has ended, a loading dose of fosphenytoin or phenobarbital is given.

A client with bipolar disorder, manic phase, just sat down to watch television in the lounge. As the nurse approaches the lounge area, the client states, "The sun is shining. Where is my son? I love Lucy. Let's play ball." The client is displaying: a) Concreteness. b) Flight of ideas. c) Depersonalization. d) Use of neologisms.

b - The client is demonstrating flight of ideas, or the rapid, unconnected, and often illogical progression from one topic to another. Concreteness involves interpreting another person's words literally. Depersonalization refers to feelings of strangeness concerning the environment or the self. A neologism is a word made up by a client.

A client with Rh isoimmunization gives birth to a neonate with an enlarged heart and severe, generalized edema. The neonate is immediately transferred to the neonatal intensive care unit. Which nursing diagnosis is most appropriate for the client? a. ineffective parenting related to the neonate's transfer to the intensive care unit b. impaired parenting related to the neonate's transfer to the intensive care unit c. deficient fluid volume related to severe edema d. fear related to removal and loss of the neonate by statute

b - because the neonate is severely ill and needs to be placed in the neonatal intensive care unit, the client may have a nursing diagnosis of 'impaired parenting related to the neonate's transfer to the neonatal intensive care unit.' (another pertinent nursing diagnosis may be 'compromised family coping related to lack the opportunity for bonding.) Rh isoimmunization isn't a socially unacceptable infection. This condition causes an excess fluid volume (not deficient) related to cardiac problems. Rh isoimmunization doesn't lead to loss of the neonate by statute.

The major goal of therapy in crisis intervention is to: a. withdraw from the stress b. resolve the immediate problem c. decrease anxiety d. provide documentation of events

b - during a period of crisis, the major goal is to resolve the immediate problem, with hopes of getting the individual to the level of functioning that existed before the crisis or to a higher level of functioning. Withdrawing from stress doesn't address the immediate problem and isn't therapeutic. The client's anxiety will decrease after the immediate problem is resolved. Providing support and safety are necessary interventions while working toward accomplishing the goal. Documentation is necessary for maintaining accurate records of treatment, but isn't a major goal.

Before discharge, which instruction should a nurse give to a client receiving digoxin (Lanoxin)? a. "take an extra dose of digoxin if you miss one dose." b. "call the physician if your heart rate is above 90 beats/minute c. "call the physician if your pulse drops below 80 beats/minute." d. "take digoxin with meals."

b - the nurse should instruct the client to notify the physician if his heart rate is greater than 90 beats/ minute because cardiac arrhythmias may occur with digoxin toxicity. To prevent toxicity, the nurse should instruct the client never to take an extra dose of digoxin if he misses a dose. The nurse should show the client how to take his pulse and tell him to call the physician if his pulse rate drops below 60 beats/minute - not 80 beats/minute, which is a normal pulse rate and doesn't warrant action. The client shouldn't take digoxin with meals; doing so slows the absorption rate.

The nurse observes that the right eye of an unconscious client does not close completely. Which nursing intervention is most appropriate a. have the client wear eyeglasses at all time b. lightly tape the eyelid shut c. instill artificial tears once every shift d. clean the eyelid with a washcloth every shift

b - when the blink reflex is absent or the eyes do not close completely, the cornea may become dry and irritated. Corneal abrasion can occur. Taping the eye closed will prevent injury. Having the client wears eyeglasses or cleaning the eyelid will not protect the cornea from dryness or irritation. Artificial tears instilled once per shift are not frequent enough for preventing dryness.

A client with an incomplete small-bowel obstruction is to be treated with a Cantor tube. Which of the following measures would most likely be included in the client's care once the Cantor tube has passed into the duodenum? a) Maintain bed rest with bathroom privileges. b) Advance the tube 2 to 4 inches at specified times. c) Avoid frequent mouth care. d) Provide ice chips for the client to suck.

b) CORRECT ANSWER Advance the tube 2 to 4 inches at specified times. Reason: Once the intestinal tube has passed into the duodenum, it is usually advanced as ordered 2 to 4 inches every 30 to 60 minutes. This, along with gravity and peristalsis, enables passage of the tube forward. The client is encouraged to walk, which also facilitates tube progression. A client with an intestinal tube needs frequent mouth care to stimulate saliva secretion, to maintain a healthy oral cavity, and to promote comfort regardless of where the tube is placed in the intestine. Ice chips are contraindicated because hypotonic fluid will draw extra fluid into an already distended bowel.

A client who took an overdose of Tylenol in a suicide attempt is transferred overnight to the psychiatric inpatient unit from the intensive care unit. The night shift nurse called the primary health care provider on call to obtain initial prescriptions. The primary health care provider prescribes the typical routine medications for clients on this unit: Milk of Magnesia, Maalox and Tylenol as needed. Prior to implementing the prescriptions, the nurse should? a) Ask the primary health care provider about holding all the client's PM prescriptions. b) Question the primary health care provider about the Tylenol prescription. c) Request a prescription for a medication to relieve agitation. d) Suggest the primary health care provider write a prescription for intravenous fluids.

b) CORRECT ANSWER Question the primary health care provider about the Tylenol prescription. Reason: The nurse should question the Tylenol order because the client overdosed on Tylenol, and that analgesic would be contraindicated as putting further stress on the liver. There is no need to hold the PM Milk of Magnesia or Maalox. There is no indication that the client is agitated or needs medication for agitation. There is little likelihood that the client needs an IV after being transferred out of an intensive care unit, as the client will be able to take oral fluids.

A nurse assesses a client's respiratory status. Which observation indicates that the client is having difficulty breathing? a) Diaphragmatic breathing b) Use of accessory muscles c) Pursed-lip breathing d) Controlled breathing

b) CORRECT ANSWER Use of accessory muscles Reason: The use of accessory muscles for respiration indicates the client is having difficulty breathing. Diaphragmatic and pursed-lip breathing are two controlled breathing techniques that help the client conserve energy.

Dystocia?

baby cannot make it down to canal

what is dystocia?

baby cannot make it down to canal

what food to avoid for pheochromocytoma

banana ,vanilla, chocolate, caffeine

Croup

bark like cough, syspnea, cyanosis, inspiratory stridor

Post cooperatively Cardiac Cathetarization

bed rest for 6-8 hours

Detached Retina

bed-rest, do not encourage to cough, eye patch worn at nigh

Shock?

bedrest with extremities elevated 20 degrees, knees straight, head slightly elevated (modified Trendelenburg)

perform amniocentesis @____ wks and ck for ______.

before 20 weeks gestation to check for cardiac and pulmonary abnormalities

Amniocentesis is performed? and why?

before 20 weeks gestation to check for cardiac and pulmonary abnormalities.

A nurse, a licensed practical nurse (LPN), and a nursing assistant are caring for a group of clients. The nurse asks the nursing assistant to check the pulse oximetry level of a client who underwent laminectomy. The nursing assistant reports that the pulse oximetry reading is 89%. The client Kardex contains an order for oxygen application at 2 L/min should the pulse oximetry level fall below 92%. The nurse is currently assessing a postoperative client who just returned from the postanesthesia unit. How should the nurse proceed? a. immediately go the client's room and assess vital signs, administer oxygen at 2 L/minute, and notify the physician. b. ask the nursing assistant to notify the physician of the low pulse oximetry level c. ask the LPN to obtain vital signs and administer oxygen at 2 L/min to the client who underwent laminectomy d. complete the assessment of the new client before attending to the client who underwent laminectomy

c - because it's important to get more information about the client with a decreased pulse oximetry level, the nurse should ask the LPN to obtain vital signs and administer oxygen as ordered. The nurse must attend to the newly admitted client without delaying treatment to the client who is already in her care. The nurse can effectively do this by delegating tasks to an appropriate health team member such as an LPN. The nurse doesn't need to immediately attend to the client with a decreased pulse oximetry level; she may wait until she completes the assessment of the newly admitted client. The physician doesn't need to be notified at this time because an order for oxygen administration is already on record.

The nurse should instruct the family of a child with newly diagnosed hyperthyroidism to: a. keep their home warmer than usual b. encourage plenty of outdoor activities c. promote interactions with one friend instead of groups d. limit bathing to prevent skin irritation

c - children with hyperthyroidism experience emotional labiality that may strain interpersonal relationships. Focusing on one friend's is easier than adapting to group dynamics until the child's condition improves. Because of their high metabolic rate, children with hyperthyroidism complain of being too warm. Bright sunshine may be irritating because of disease-related ophthalmopathy. Sweating is common and bathing should be encouraged.

A worried mother confides in the nurse that she wants to change physicians because her infant is not getting better. The best response by the nurse is which of the following? a) "This doctor has been on our staff for 20 years." b) "I know you are worried, but the doctor has an excellent reputation." c) "You always have an option to change. Tell me about your concerns." d) "I take my own children to this doctor."

c) CORRECT ANSWER "You always have an option to change. Tell me about your concerns." Reason: Asking the mother to talk about her concerns acknowledges the mother's rights and encourages open discussion. The other responses negate the parent's concerns.

A client in the triage area who is at 19 weeks' gestation states that she has not felt her baby move in the past week and no fetal heart tones are found. While evaluating this client, the nurse identifies her as being at the highest risk for developing which problem? a) Abruptio placentae. b) Placenta previa. c) Disseminated intravascular coagulation. d) Threatened abortion.

c) CORRECT ANSWER Disseminated intravascular coagulation. Reason: A fetus that has died and is retained in utero places the mother at risk for disseminated intravascular coagulation (DIC) because the clotting factors within the maternal system are consumed when the nonviable fetus is retained. The longer the fetus is retained in utero, the greater the risk of DIC. This client has no risk factors, history, or signs and symptoms that put her at risk for either abruptio placentae or placenta previa, such as sharp pain and "woody," firm consistency of the abdomen (abruption) or painless bright red vaginal bleeding (previa). There is no evidence that she is threatening to abort as she has no complaints of cramping or vaginal bleeding.

A client who has a history of Crohn's disease is admitted to the hospital with fever, diarrhea, cramping, abdominal pain, and weight loss. The nurse should monitor the client for: a) Hyperalbuminemia. b) Thrombocytopenia. c) Hypokalemia. d) Hypercalcemia.

c) CORRECT ANSWER Hypokalemia. Reason: Hypokalemia is the most expected laboratory finding owing to the diarrhea. Hypoalbuminemia can also occur in Crohn's disease; however, the client's potassium level is of greater importance at this time because a low potassium level can cause cardiac arrest. Anemia is an expected development, but thrombocytopenia is not. Calcium levels are not affected.

The nurse is caring for a client with asthma. The nurse should conduct a focused assessment to detect which of the following? a) Increased forced expiratory volume. b) Normal breath sounds. c) Inspiratory and expiratory wheezing. d) Morning headaches.

c) CORRECT ANSWER Inspiratory and expiratory wheezing. Reason: The hallmark signs of asthma are chest tightness, audible wheezing, and coughing. Inspiratory and expiratory wheezing is the result of bronchoconstriction. Even between exacerbations, there may be some soft wheezing, so a finding of normal breath sounds would be expected in the absence of asthma. The expected finding is decreased forced expiratory volume [forced expiratory flow (FEF) is the flow (or speed) of air coming out of the lung during the middle portion of a forced expiration] due to bronchial constriction. Morning headaches are found with more advanced cases of COPD and signal nocturnal hypercapnia or hypoxemia.

The nurse walks into a client's room to administer the 9:00 a.m. medications and notices that the client is in an awkward position in bed. What is the nurse's first action? a) Ask the client his name. b) Check the client's name band. c) Straighten the client's pillow behind his back. d) Give the client his medications.

c) CORRECT ANSWER Straighten the client's pillow behind his back. Reason: The nurse should first help the client into a position of comfort even though the primary purpose for entering the room was to administer medication. After attending to the client's basic care needs, the nurse can proceed with the proper identification of the client, such as asking the client his name and checking his armband, so that the medication can be administered.

Which of the following interventions would be most appropriate for the nurse to recommend to a client to decrease discomfort from hemorrhoids? a) Decrease fiber in the diet. b) Take laxatives to promote bowel movements. c) Use warm sitz baths. d) Decrease physical activity.

c) CORRECT ANSWER Use warm sitz baths. Reason: Use of warm sitz baths can help relieve the rectal discomfort of hemorrhoids. Fiber in the diet should be increased to promote regular bowel movements. Laxatives are irritating and should be avoided. Decreasing physical activity will not decrease discomfort.

A client has just been diagnosed with early glaucoma. During a teaching session, the nurse should: a) provide instructions on eye patching. b) assess the client's visual acuity. c) demonstrate eyedrop instillation. d) teach about intraocular lens cleaning.

c) CORRECT ANSWER demonstrate eyedrop instillation. Reason: Eyedrop instillation is a critical component of self-care for a client with glaucoma. After demonstrating eyedrop instillation to the client and family, the nurse should verify their ability to perform this measure properly. An eye patch isn't necessary unless the client has undergone surgery. Visual acuity assessment isn't necessary before discharge. Intraocular lenses aren't implanted in clients with glaucoma.

When developing a care plan for a client with a do-not-resuscitate (DNR) order, a nurse should: a) withhold food and fluids. b) discontinue pain medications. c) ensure access to spiritual care providers upon the client's request. d) always make the DNR client the last in prioritization of clients.

c) CORRECT ANSWER ensure access to spiritual care providers upon the client's request. Reason: Ensuring access to spiritual care, if requested by the client, is an appropriate nursing action. A nurse should continue to administer appropriate doses of pain medication as needed to promote the client's comfort. A health care provider may not withhold food and fluids unless the client has a living will that specifies this action. A DNR order does not mean that the client does not require nursing care.

Implementation for hypoparathyroidism

calcium ad iv, vit d, low phosphate and increase ca in diet

Calan (verapamil)

calcium channel blocker: tx of HTN, angina...assess for constipation

NG tube rules?

can be irrigated with cola, and should be taught to family when a client is going home with an NG tube.

external catheters

can be removed during the day

4 year olds?

can not interpret TIME. Need to explain time in relationship to a known COMMON EVENT (eg: "Mom will be back after supper").

Sickle cell anemia

check for sing of infection, check for area for pain and swelling. Encourage fluids, provide analgesics( PCA pump)

ACID ash diet?

cheese, corn, cranberries, plums, prunes, meat, poultry, pastry, bread

MI

chest pain radiation to arm, jaw, neck, indigestion, low grade fever,

Where are chest tubes placed?

chest tubes are placed in the pleural space.

glascow scale of 5

coma

Ostemilitis and phemonia

common with untreated sickle cell anemia

The nurse accidentally administers 40 mg of propranolol (Inderal) to a client instead of 10 mg. Although the client exhibits no adverse reactions to the larger dose, the nurse should:

complete an incident report.

hypocalcaemia

complication of leukemia, s/s anorexia,n/v kidney stones,bone pain. Give isotonic saline, lasix,pamidronate and phosphates as perscribed

Herniated disk diagnostic test

computerized tomograpy, MRI, surgical procedure, laminectomy, use log rolls

Bence Jones protein in urine?

confirms multiple myeloma

s/s hypoglycemia?

confusion, clammy skin, increase pulse

Platelet transfusions

consists of special pump

Hepatitis A precautions

contact precautions

Parathyroid glands

control calcium and phosphorus metabolism; produce parathyroid hormone.

A client with burns on his groin has developed blisters. As the client is bathing, a few blisters break. The best action for the nurse to take is to: a) remove the raised skin because the blister has already broken. b) wash the area with soap and water to disinfect it. c) apply a weakened alcohol solution to clean the area. d) clean the area with normal saline solution and cover it with a protective dressing.

d - The nurse should clean the area with a mild solution such as normal saline, and then cover it with a protective dressing. Soap and water and alcohol are too harsh. The body's first line of defense broke when the blisters opened; removing the skin exposes a larger area to the risk of infection.

a nurse is caring for a client who required chest tube insertion for a pneumothorax. To assess for pneumothorax resolution, the nurse can anticipate that the client will require: a. monitoring of arterial oxygen saturation (SaO2) b. arterial blood gas (ABG) studies c. chest auscultation d. chest x ray

d - chest x ray confirms diagnosis by revealing air or fluid in the pleural space. SaO2 values may initially decrease with a pneumothorax but typically return to normal within 24 hours. ABG studies may show hypoxemia, possibly with respiratory acidosis and hypercapnia but these are not necessarily related to a pneumothorax. Chest auscultation will determine overall lung status, but it's difficult to determine if the best has re-expanded sufficiently.

The client with a hearing aid does not seem to be able to hear the nurse. The nurse should do which of the following? a. contact the client's audiologist b. cleanse the hearing aid ear mold in normal saline c. irrigate the ear canal d. check the hearing aid's placement

d - inadequate amplification can occur when a hearing aid is not place properly. The certified audiologist is licensed to dispense hearing aids. The ear mold is the only part of the hearing aid that may be wash frequently; it should be washed daily with soap and water. Irrigation of the ear canal is done to remove impacted cerumen or a foreign body

A nurse is caring for a client diagnosed with a cerebral aneurysm who reports a severe headache. Which action should the nurse perform? a) Sit with the client for a few minutes. b) Administer an analgesic. c) Inform the nurse manager. d) Call the physician immediately.

d) CORRECT ANSWER Call the physician immediately. Reason: The nurse should notify the physician immediately because the headache may be an indication that the aneurysm is leaking. Sitting with the client is appropriate but only after the physician has been notified of the change in the client's condition. The physician will decide whether or not administration of an analgesic is indicated. Informing the nurse manager isn't necessary.

During the first feeding, the nurse observes that the neonate becomes cyanotic after gagging on mucus. Which of the following should the nurse do first? a) Start mouth-to-mouth resuscitation. b) Contact the neonatal resuscitation team. c) Raise the neonate's head and pat the back gently. d) Clear the neonate's airway with suction or gravity.

d) CORRECT ANSWER Clear the neonate's airway with suction or gravity. Reason: If a neonate gags on mucus and becomes cyanotic during the first feeding, the airway is most likely closed. The nurse should clear the airway by gravity (by lowering the infant's head) or suction. Starting mouth-to-mouth resuscitation is not indicated unless the neonate remains cyanotic and lowering his head or suctioning doesn't clear his airway. Contacting the neonatal resuscitation team is not warranted unless the infant remains cyanotic even after measures to clear the airway. Raising the neonate's head and patting the back are not appropriate actions for removing mucus. Doing so allows the mucus to remain lodged causing further breathing difficulties.

A nurse is caring for a client with a diagnosis of Impaired gas exchange. Based upon this nursing diagnosis, which outcome is most appropriate? a) The client maintains a reduced cough effort to lessen fatigue. b) The client restricts fluid intake to prevent overhydration. c) The client reduces daily activities to a minimum. d) The client has normal breath sounds in all lung fields.

d) CORRECT ANSWER The client has normal breath sounds in all lung fields. Reason: If the interventions are effective, the client's breath sounds should return to normal. The client should be able to cough effectively and should be encouraged to increase activity, as tolerated. Fluids should help thin secretions, so fluid intake should be encouraged.

When assessing an elderly client, the nurse expects to find various aging-related physiologic changes. These changes include: a) increased coronary artery blood flow. b) decreased posterior thoracic curve. c) decreased peripheral resistance. d) delayed gastric emptying.

d) CORRECT ANSWER delayed gastric emptying. Reason: Aging-related physiologic changes include delayed gastric emptying, decreased coronary artery blood flow, an increased posterior thoracic curve, and increased peripheral resistance.

A client with chronic undifferentiated schizophrenia is admitted to the psychiatric unit of a local hospital. During the next several days, the client is seen laughing, yelling, and talking to himself. This behavior is characteristic of: a) delusion. b) looseness of association. c) illusion. d) hallucination.

d) CORRECT ANSWER hallucination. Reason: Auditory hallucination, in which one hears voices when no external stimuli exist, is common in schizophrenic clients. Such behaviors as laughing, yelling, and talking to oneself suggest such a hallucination. Delusions, also common in schizophrenia, are false beliefs or ideas that arise without external stimuli. Clients with schizophrenia may exhibit looseness of association, a pattern of thinking and communicating in which ideas aren't clearly linked to one another. Illusion is a less severe perceptual disturbance in which the client misinterprets actual external stimuli. Illusions are rarely associated with schizophrenia.

Myasthenia gravis?

decrease in receptor sites for acetylcholine. Since smallest concentration of ACTH receptors are in cranial nerves, expect fatigue and weakness in eye, mastication, pharyngeal muscles. Tensilon test given if muscle is tense in myasthenia gravis.

hypormagnesiamia

decrease respiration and decrease bladder, absent DTR

Atropine used to

decrease secretions

leading cause of skin breakdown ?

decreased tissue perfusion.

ovarian function

decreases then stops 45-50 years old

Iron

deferoxamine

Myasthenia Gravis

deficiency of acetylchoine, muscular weakness, dyphagia, respiratory distress

somatic neuropathy

diminished perception of vibration, pain, and temperature; hypersensitivity to light touch; occasionally severe "burning" pain

kayexalate

do not drink with orange juice

Steroid and digitalis

do not work together because of low k cause digitalis toxicity

Cast care

don't cover until dry, handle with palms of hands, don't rest on hard surfaces, keep above level of heart, check pulses, color , sensation. Complication of cast is numbness, tingling, no pulses, bluish color of the skin

After Total Hip Replacement?

don't sleep on operated side, don't flex hip more than 45- 60 degrees, don't elevate HOB more than 45 degrees. Maintain hip abduction by separating thighs with pillows.

Risperdal, antipsychotic,

doses over 6mg can cause tarditive dyskinesia, first line antipsychotic in children

What to do in a sucking stab wound?

dress the wound and tape it on three sides which allows air to escape. Do not use an occlusive dressing, which could convert the wound from open pneumo to closed one, and a tension pneumothorax is worse situation. After that get your chest tube tray, labs, iv.

sterile field items

drop items into sterile field

Meningitis is what precaution

droplet

implementation for morning sickness

dry crackers on arising, small frequent meals

Anticholinergic effects--assessment

dry mouth==can't spit urinary retention=can't **** constipated =can't **** blurred vision=can't see

Carafate

duodenal ulcers..coats the ulcer...so take before meals.

Stomas?

dusky means poor blood supply, protruding means prolapsed, sharp pain + rigidity means peritonitis, mucus in ileal conduit is expected.

s/s embolism

dyspenic, diaphoretic, restless, coughing, chest pain. place patient one left side and lower head of bed.

Hiatal Hernia

dyspepsia, regurgitaion, hearburn, do not lie after food, small frequesnt meals, elevate head when sleeping

Hematoma (lab signs)

elevated temp and decreased hematocrit

Defibriallation

emergency treatment, star CPR first,

Amniocentesis is done on

empty bladder. at 16 weeks detect genetic disorder, at 30 weeks determine lung maturity. US needed to determine position of placenta and fetus

Wilms tumor?

encapsulated above the kidneys causing flank pain.

Beta Blocker

ends in "lol"

ACE inhibitor

ends in "pril"

Hepatitis?

ends in a VOWEL, comes from the BOWEL (Hep A) Hepatitis B=Blood and Bodily fluids Hepatitis C is just like B

Prozac

energizing antidepressant,

The nurse should encourage a client with a wound to consume foods high in vitamin C because this vitamin:

enhances protein synthesis.

In an emergency

ensure patient safety before assessing

Lumbar puncture-

fetal postion. post- nuero assess q15-30 until stable, flat2-3hr, encourage fluids, oral anlgesics for headache, observe dressing

Stress test

fetal response to stress, performed after 28 weeks, position the client semi Fowler or side lying. positive lte decel, potential risk is C section, negative no late decel

Hydroxyurea

for sickle cell, report GI symptoms immediately, could be sign of toxicity

Hydroxyurea,

for sickle cell, report GI symptoms immediately, could be sign of toxicity

Botox?

for strabismus. Patch the GOOD eye so that the weaker eye can get stronger. Found a cool link about its use in peds pt with strabismus. I had to look it up cuz I heard it was important *ah hem ah hem*

Trendelenberg's test?

for varicose veins. If they fill proximally = varicosity.

Alzemer's disease

forgetfulness, short attention span, agitation, night wandering

Cystitis

forse fluid to 3000 ml, acidify urine, antibiotics, teach to void following intercourse, teach proper cleaning after defecation

_____ year old kids cannot interpret TIME. Need to explain time in relationship to a known COMMON EVENT (eg: "Mom will be back after supper").

four

Anemia

frequent rest, high protein, high iron, high vitamin, protect form infection

8-12

games and hobbies

Pheochromocytoma?

hypersecretion of epi/norepi, persistent HTN, increased HR, hyperglycemia, diaphoresis, tremor, pounding HA; avoid stress, frequent bating and rest breaks, avoid cold and stimulating foods, surgery to remove tumor

Thyroid storm?

increased temp, pulse and HTN

Hypernatremia?

increased temp, weakness, disorientation/delusions, hypotension, tachycardia; hypotonic solution

Theophylline increases the risk of ________and decreases the effects of_____ and _____

increases digoxin toxicity and decrease lithium and Dilantin

The nurse must assess skin turgor of an elderly client. When evaluating skin turgor, the nurse should remember that:

inelastic skin turgor is a normal part of aging.

hepatitis A

infection inflammation of the liver caused by HAV, usually transmitted orally through fecal contamination of food or water

crohns

inflam. anywhere in bowel give low fiber tx: antidiarrheal ABX: sulfonamides steroids

Diverticulitis?

inflammation of the diverticulum in the colon pain is around LL quadrant.

Common S/S of LTB?

inspiratory stridor.. LTB = croup!!!!

What is a bad sign in asthma?

intercostal retractions

edema in the

interstitial space not in the cardiovascular space

Tardive Dyskinesia

irreversible - involuntary movements of the tongue, face and extremities, may happen after prolonged use of antipsychotics

Bentyl

irritable bowel....assess for anticholinergic side effects.

S/S of hypoglycemia

irritable, confusion, tremors, blurred vision, skin cool clammy, tachycardia

Kidney Glucose threshold

is 180

Med of choice for CHF (congestive heart failure)

is Ace inhibitor.

Med of choice for anaphylactic shock

is Epinephrine

Med of choice for Status Epilepticus

is Valium.

-Munchausen Syndrome

is a psychiatric disorder that causes an individual to self-inflict injury or illness or to fabricate symptoms of physical or mental illness, in order to receive medical care or hospitalization. In a variation of the disorder, Munchausen by proxy (MSBP), an individual, typically a mother, intentionally causes or fabricates illness in a child or other person under her care

Munchhausen Syndrome?

is a psychiatric disorder that causes an individual to self-inflict injury or illness or to fabricate symptoms of physical or mental illness, in order to receive medical care or hospitalization. In a variation of the disorder, Munchausen by proxy (MSBP), an individual, typically a mother, intentionally causes or fabricates illness in a child or other person under her care.

methadone

is an opioid analgesic used to detoxify/treat pain in narcotic addicts.

Med of choice for Asystole (no heart beat)

is atropine

Dig and Lasix

is getting enough potassium, because low potassium potentiates Dig and can cause dysrrhythmias.

SIADH

is increase in ADH, increase in fluid retention= tachycardia, output decrease, specific gravity increase, kidneys cannot excrete, sodium decrease

Med of choice for bipolar

is lithium.

Mucomyst

is the antedote to tylenol and is administered orally

Adenosine

is the treatment of choice for paroxysmal atrial tachycardia.

Cast care

isometric exercise, check for odors, don not put anything into the cast

Med that can't be infused Intra osseously?

isoproterenol, a beta agonist.

Cystic Fibrosis: Reinforcing Teaching about pancrelipase (pancrease)

its a combination of lipase, amylase, and protease, which are enzymes needed for digestion of fats, carbohydrates and proteins. these are normally produced in pancreas, but people with cystic fibrosis do not excrete them. Pancrelipase products are made from enzymes taken from pigs. -take before meals, and not if allergic to pork, never crush or chew or store in fridge

Too much of K give what

kayexalate

burn patient

keep clean, high risk for infection.

Hepatitis A infant

keep in private room, Increase carbs and protein, low fat diet.

Aids patient develop purple brown spots.

keep skin clean and prevent infection

Buck's traction=

knee immobility

Bucks traction =

knee immobility

Pt position for Prolapsed Cord?

knee-chest position or Trendelenburg

patient with multiple sclerosis

know they have decrease pain perception

MEASLES -

koplik's spots

MEASLES

koplik's spots.

ammonia

lactulose

childhood >6yrs l

learn to delay need gratification

preadolescence 9-12 yrs

learns to relate to friends of of opposite sex

External radiotherapy

leave marking on skin, avoid use of cream and lotion, check for redness, cracking , wear cotton clothing, if you are pregnant you can't care for this patient.

What if a toddler says no to medication?

leave the room and come back in five minutes, because to a toddler it is another episode. Next time, don't ask.

The nurse is administering I.M. injections to an older client. The nurse should remember that an older client has:

less subcutaneous tissue and muscle mass than a younger client. Rationale: When administering I.M. injections, the nurse should remember that an older client has less subcutaneous tissue and muscle mass than a younger client.

report to the doctor

less than 3 movement per hour

Addison's disease Assessment

lethargy, fatigue, GI disturbances, weigh loss, mentrual changes, impotence in men, hypoglycemia, hyponatremia, hyperkalemia, hypercalcemia, postural hypotension, and hyperpigmentation of skin (bronzed) with primary disease.

Hypothyroidism assessment

lethargy, fatigue, weakness, intolerance to cold, weight gain, dry skin, bradycardia, constipation, myxedema, forgetfulness, menstrual disturbances, cardiac enlargement/CHF

s/s hyperglycemia?

lethargy, hot dry skin, rapid respirations

Oncovin (vincristine)

leukemia..given IV ONLY

ICP

lie the person 30 -45, avoid neck flex-ion, head rotation, prevent Valsalva, restrict fluid

autonomic dysreflexia

life threatening inhibited sympathetic response of nervous system to a noxious stimulus- patients with spinal cord injuries at T-7 or above is usually caused by a full bladder.

Common S/S of PTB?

low-grade afternoon fever.

PTB (TB)

low-grade afternoon fever.

Meningitis

lumbar puncture occur, stiff neck,

Number 1 reason for SIADH is

lung cancer, give Na because of a lot off ware caused dilution of sodium

pH regulation 2 organs

lungs & kidney

PDA

machine like murmur.

PDA -

machine like murmur.

Post subtotal thyriodectomy for treatment of hyperthyroidism.

may need calcium gluconate to treat tetany from damaged parathyroid gland. have tracheotomy setup, and suction equipment available.

autonomic dysreflexia

may occur oct spinal cord injury from 1-6 weeks after injury characterized by diaphoresis, nauseam headache, HTN, sit them up

give carafate (GI med) before

meals to coat stomach

Iatragenic

means it was caused by treatment, procedure, or medication.

CVP

measure cardiac output normal is 3 -12, less than 3 means hypovolemia, more than 12 means hypervolemia

First sign of cystic fibrosis?

meconium ileus at birth. Baby is inconsolable, do not eat, not passing meconium.

Pitocin

med used for uterine stimulation

* Ethambutol

messes with your Eye Apply eye drop to conjunctival sac and after wards apply pressure to nasolacrimal duct / inner canthus

Ethambutol

messes with your Eyes

Postoperative Nursing: Recognizing development of deep vein thrombosis

monitor for soreness or pain when standing or walking, calf tenderness, and skin warmth. *use compression stockings or foot pump *give low dose of warfarin

Oral Hpoglycemics: reinforcing teaching for a client taking Glyburide

monitor when taking aspirin, IBU, tums, & cinnamon *watch for heartburn, hypoglycemia, nausea, contraindicated for clients with sulfa allergy, type I diabetes, DKA

CUSHING'S SYNDROME

moon face appearance and buffalo hump

CUSHING'S SYNDROME

moon face appearance and buffalo hump.

Lymes mostly found in

mostly in Conneticuts

Rh?

mothers receive rhogam to protect next baby.

Akathisia

motor restlessness, need to keep going, tx with antiparkinsons meds, can be mistaken for agitation.

Hypercalemia?

muscle weakness, lack of coordination, abdominal pain, confusion, absent tendon reflexes, sedative effect on CNS

HypoKalemia?

muscle weaknesses, dysrhythmias, increase K (raisins, bananas, apricots, oranges, beans, potatoes, carrots, celery)

Birth control- Diaphram?

must stay in place 6 hours after intercourse. They are also fitted so must be refitted if you lose or gain a significant amount of weight.

Multiple sclerosis

myelin sheat destruction, disruption in nerve impulse conduction.

Acetaminophen Antidote

n-Acetylcysteine.

Addisonian Crisis?

n/v, confusion, abdominal pain, extreme weakness, hypoglycemia, dehydration, decreased BP

activase

name a thrombolytic agent for DVT, pulmonary embolism

hypomagnesium

neuromuscular irritability

Cephalhematoma= collection of blood from ruptured blood vessels. Lump _______ the suture line . Swelling reabsorbs within _____.

never crosses and reabsorbs 2 to 3 weeks

Kids with RSV?>

no contact lenses or pregnant nurses in rooms where ribavirin is beingmadministered by hoot, tent, etc.

Ulcerative Colitis

no fat or pus in stool, bloody diarrhea

Nuetropenic patients?

no live vaccines, no fresh fruits, no flowers should be used for neutropenic patients.

Gout Diet

no red meat or shellfish

EEG

no sleep the night before, meals not withheld, no stimulants for 24hr before, tranquilizer/stimulant meds

More info on EEG?

no sleep the night before, meals not withheld, no stimulants for 24hr before, tranquilizer/stimulant meds held 24-48hr before, may be asked to hyperventilate 3-4min and watch a bright flashing light.

Orange tag in pysch

non emergent Psych

RBC

norm 4.5-6.0 for males

4.0-5.5

norm range for RBC females

8-10

norm range for calcium

3.5-5.0

norm range for potassium

4,000 - 11,000

norm range of WBC

0.7-1.5

norm range of creatinine

Na

norma range of 135-145 electrolyte

Hgb

normal range for men 14-18, women 12-16

70-110

normal range of glucose

Peritoneal Dialysis normal findings within the 1st ___ and abnormal findings.

oIt's ok to have abdominal craps, blood tinged outflow and leaking around site if the Peritoneal Dialysis cath (tenkhoff) was placed in the last 1-2 wks. Cloudy outflow NEVER NORMAL.

Allen's test

occlude both ulnar and radial artery until hand blanches then release ulnar. If the hand pinks up, ulnar artery is good and you can carry on with ABG/radial stick as planned. ABGS must be put on ice and whisked to the lab.

Allen's test?

occlude both ulnar and radial artery until hand blanches then release ulnar. If the hand pinks up, ulnar artery is good and you can carry on with ABG/radial stick as planned. ABGS must be put on ice and whisked to the lab.

Gastric ulcer pain

occurs 30 minutes to 90 minutes after eating, not at night, and doesn't go away with food

Neostigmine/Atropine (anticholinergic) to reverse effect

of pancuronium.

Murphy's sign

pain with palpation of gall bladder area seen with cholecystitis

Murphy's sign?

pain with palpation of gall bladder area seen with cholecystitis

osteomylitis

pain, swelling, redness, infection caused by Staphylococcus, confirmed by x ray

While a nurse is teaching a stress management class, a client who has been reading literature on stress states, "The book I'm reading keeps mentioning 'endorphins.' What are endorphins?" The most accurate response from the nurse would be that endorphins are:

pain-blocking chemicals produced by the body. Rationale: Endorphins are narcotic-like substances that lock into the narcotic receptors at nerve endings in the brain and spinal cord and block the transmission of pain signals

Priapism:

painful erection lasting longer than 6 hrs.

placentae previa

painless bleeding

BLADDER CA -

painless hematuriA

BLADDER CA

painless hematuria

Common S/S of Bladder Cancer?

painless hematuria...

HODGEKIN'S DSE/LYMPHOMA

painless, progressive enlargement of spleen & lymph tissues, Reedstenberg Cells

HODGEKIN'S DSE/LYMPHOMA -

painless, progressive enlargement of spleen & lymph tissues, Reedstenberg Cells

find jaundice on asians where?

palate

with chronic pancreatitis

pancreatic enzymes are given with meals.

toddler

paralele play

Placement of a wheelchair?

parallel to the bed on the side of weakness

When you see Coffee-brown emesis, think

peptic ulcer

Diff between placenta previa and placenta abrupto?

placenta previa = there is no pain, there is bleeding. Placenta abruption = pain, but no bleeding.

Following a fall from a horse during rodeo practice, an 18-year-old client is seen in the emergency department. He has a large, dirty laceration on his leg. The wound is vigorously cleaned, closed, and dressed. In the past, the client has received the full immunization regimen for tetanus toxoid. The nurse asks the client about his tetanus immunization history and he says, "I had my last shot when I was 11 years old." The nurse should:

plan on administering a dose of tetanus vaccine.

. DM

polyuria, polydypsia, polyphagia

s/s of cerebral palsy

poor muscle control due to birth injuries and/or decrease oxygen to brain tissues.

Cerebral palsy?

poor muscle control due to birth injuries and/or decrease oxygen to brain tissues

Infant with Spina Bifida?

position prone (on abdomen) so that sac does not rupture

Group-A strep?

precedes rheumatic fever. Chorea is part of this sickness (grimacing, sudden body movements, etc.) and it embarrasses kids. They have joint pain. Watch for elevated antistreptolysin O to be elevated. Penicillin!

Haldol

preferred anti-psychotic in elderly, but high risk extrapyramidal side effects (dystonia, tarditive dyskinesia, tightening of jaw, stiff neck, swollen tongue, later on swollen airway), monitor for early signs of reaction and give IM Benadryl

prolamped umbilical cord

premature rupture of membrane, presenting part not engaged, fetal distress, protruding cord, call for help

client is diagnosed with Graves disease, why does client need to take propanolol (Inderal)

propanolol blocks the cardiovascular symptoms of graves disease

radiation for prostate cancer

proper skin care includes COLD PACKS

Pancreatitis patients

put them in fetal position, NPO, gut rest, prepare antecubital site for PICC cuz they'll probably be getting TPN/Lipids

Normal Physiological changes of pregnancy:Pyrosis

pyrosis=heartburn, experienced in mid to late part of pregnancy

Phenergan an antiemetic used to

reduce nausea

BPH (benign prostatic hyperplasia)

reduced size & force of urine

Common S/S of BPH?

reduced size and force of urine

CHOLERA -

rice watery stool.

best position after bottle feeding.

ride side

Cor Pulmonae?

right sided heart failure caused by right ventricular failure (so pick edema, jvd, if it is a choice.

Col pulmonale

right sided heart failure noted with peripheral edema and congestion of venous system

TETANUS

risus sardonicus.

TETANUS -

risus sardonicus.

. TYPHOID -

rose spots in abdomen

TYPHOID

rose spots in abdomen.

PNEUMONIA

rusty sputum

PNEUMONIA -

rusty sputum

Meningitis: Expected findings: (Brainspine Infection)

s/s fever, chills, confusion, nausea, vomiting, sensitivity to light, often SEVERE HEADACHE, stiff neck, may have infection in blood, nose, or ear. *monitor for DEHYDRATION, maintain dim lighting and assist with ADL'S

Multiple sclerosis

s/s include urinary retnetion, decreases sensitivity, late cognitive changes secondary to progressive demylination of spinal cord

Meningocelle

sac like lesion, cover the lesion with sterile dressing, abdomen , semi prone, surgical repair 24-48

INTUSSUSCEPTION

sausage shaped mass, Dance Sign (empty portion of RLQ

INTUSSUSCEPTION

sausage shaped mass, Dance Sign (empty portion of RLQ)

kwell

scabies and lice...(scabies)apply lotion once and leave on for 8-12 hours...(lice) use the shampoo and leave on for 4 minutes with hair uncovered then rinse with warm water and comb with a fine tooth comb

Navane

schizophrenia..assess for EPS

increased pulse

secondary to deoxygenation of tissues

Eclampsia is a ?

seizure

Paracentesis-

semi fowlers or upright on edge of bed, empty bladder. Post- v.s., report elevated temp, observe for signs of hypovolemia.

Paracentesis?

semi fowlers or upright on edge of bed, empty bladder. Post- v.s., report elevated temp, observe for signs of hypovolemia.

Paracentesis

semi fowlers or upright on edge of bed, empty bladder. Post- v.s., report elevated temp, observe for signs of hypovolemia

Post Thyroidectomy?

semi-Fowler's, prevent ncek flexion/hyperextension, trach at bedside

Low crit/hemoglobin?

should be evaluated for signs of bleeding, such as dark stools.

If you THINK a patient has new HTN?

should have BP assessed in both arms

Change of shift report

should not contain rumors, gossips, opinions, value judgements

Ketoacidosis type 1

sick or infection. BS 300, 800, skin is warm, fruity odor, Kussmals respiration( metabolic acidosis)

Do not give demerol to pts. with

sickle cell crisis.

closed method wound care is

silvadene

Post Bronchoscopy procedure

sit of lie the patient on his side, NPO until gag reflex, can experience sore throat

Home care for coup

steamy shower, sudden exposure to cold, cool , humidified air

MALARIA

stepladder like fever with chills.

Peritonitis can be mask with

steroids

Dobutamine

stimulates heart after heart attack or during surgery.

BOTOX for

strabismus.

KAWASAKI SYNDROME -

strawberry tongue

KAWASAKI SYNDROME

strawberry tongue.

>200

the Dr. is concerned if the cholesterol is at this level

>150

the Dr. wants to start a diet and exercise program if triglycerides are this level

Test 4 hypersensitivity before

the administration of asparginase.

Greeks?

they put an amulet or any other use of protective charms around their baby's neck to avoid "evil eye" or envy of others

Hyperthyroidism?

think of MICHAEL JACKSON in THRILLER! SKINNY, NERVOUS, BULDGING EYES, Up all night, heart beating fast

scoliosis s/s?

thoracic area becomes noticeable distorted.

Insomnia is a side effect of________. Saunders confirms it. Makes sense though!

thyroid hormones Increased met. rate, your body is "too busy to sleep" as opposed to the folks with hypothyroidism who may report somnolence (dec. met rate, body is slow and sleepy). Ok some more facts.

ASA overdose

tinnitus and GI distress

Paget's Disease s/s

tinnitus, bone pain, enlargement of bone, thick bones.

Paget's disease?

tinnitus, bone pain, enlargement of bone, thick bones.

Sulfonylurea antidiabetics and alcohol

to prevent a serious reaction, inform the pt. taking a sulfonylurea to avoid consuming alcohol. If alcohol is consumed, a disulfiram (Antabuse) reaction can occur.

Schilling test

to test pernicious anemia, if positive test patient should take vit B12 for life

Babinski sign?

toes curl= GREAT Toes fan = BAD

Decorticate is Decerebate is

toward the 'cord'. the other way (out)

lithium-depression

toxic level of 2.0 meq/L for this medication and what is it used for

Torch

toxoplasmosis cats, other( syphylis, hep b, hel A and Aids), Rubella, Cytomegolovirus, Herpes simples

Hospital

tracheostomy set at bedside, crouplette or mist tent, oxygen with humidification IV fluids, Tylenol, monitor for hypoxia

Diazepam is a commonly used

tranquilizer given to reduce anxiety

EEG

tranquilizer or stimulant meds withhold 25-48( caffeine, cigar), kept awake night before test, may be asked to hyperventilate

Hepatitis B

transmitted through parenteral drug use and sexual contact . Determine exposure before implementing

HypoMg?

tremors, tetany, seizures, dyrshythmias, depression, confusion, dysphagia; dig toxicity

Charcots sign?

triad for multiple sclerosis 1. Nystagmus 2. Intention tremor 3. Scanning or staccato speech

Peritoneal Dialysis when outflow is inadequate?

turn pt from side to side BEFORE checking for kinks in tubing (according to Kaplan)

What to do in the case of an Air/Pulmonary Embolism?

turn pt to left side and lower the head of the bed.

Trigeminal nerve

twitching of the nerve on the face, use warm water, warm food

Sickle cell crisis?

two interventions to prioritize: fluids and pain relief.

hepatic coma

tx: (increase ammonia, decrease LOC) neomycin sulfate lactulose enema decrease protein

+GBS

tx: ABX:Pcn

Alcoholism

tx: Anxiolytics q 2hrs: Benzodiazepines: diazepam, lorazepam, alprazolam, chlordiazepoxide, flurazepam (S/E= sedate, constipate, increase risk of seizure, relax you) Non-Benzo: Buspirone

stomach Ca

tx: B12 shots chemo: 5-fu, doxorubicin, mitomycin, platinol-aq radiation

CHF

tx: BNP <100 HF..OVER <1000 WAY SICK!! DIGOXIN (0.5-2.0) lasix, HCTZ, bumex, dyazide, aldactone AceInhibitor Beta blocker

hyper MG

tx: Calcium gluconate

Mania

tx: Haldol, thorazine, zyprexia, risperdal, eskalith -S/E= dystonia(tonic contracture of mouth/torso), akathisia (restless), pseudoparkinsonism (pin rolling, mask face, drooling) anticonvulants:tegretol, depakote, neurontin, lamictal, trileptal (check liver function)

DKA

tx: IVF isotonic-then when B/S down to 300 give D5W and K+

Pancreatitis

tx: NPO morphine, fentanyl, PCA, dilaudid, toradol (NSAID) steroids anticholinergics: cogentin, lonox protonix zantac, pepcis antacids insulin

pulmonary embolism

tx: O2, ventilato, heparin, coumadin, lovenox

OCD

tx: SSRI: luvox, celexa, prozac, effexor (2-6 weeks to work, S/E=HA, sweating, blurred vision, wgt loss) TCAS:amitriptyline, nortripyline, imipramine (risk of over dosing, S/E=anticolenergic=dry mouth, blurred vision, U. retension, dizzy, sedation)

cirrhosis

tx: antacids, vitamins, diuretics drecrease protein, decrease NA

peptic ulcers

tx: antacids-liquid prilosec, plevacid, protonix, nexium zantac, pepcid, axid ABX: blaxin, amoxil, flagyl

HYPER thyroid

tx: antithyroids: propacil, PTU, tapazole....tapered off iodine compunds: SSKI, lugols (stains teeth) beta blockers: propanolol radioactive iodine

CVA

tx: aspirin plavix reopro iv integrilin

LOC problems

tx: barbiturate induced coma decreases cerebral metabolism

preeclampsia

tx: bedrest, mag sulfate diastolic>100=apresoline

hyper K

tx: ca gluconate/ insulin, kayexalate

imcompetent cervix

tx: cerclage 14-18 wks

MI

tx: check CKMB, LDH, Troponin (0.00-0.10) give lidocaine, amiodaron

Prostate Ca

tx: check PSA want <4 bladder spasma: B & O supp., ditropan colace radiation/chemo hormone therapy: estrogen, lupron

Breast Ca

tx: chemo: Taxol, doxorubicin estrogen blocking agents: noladex, tamoxifen estrogen synthesis inhibitors: lupron, zoladex radiation

Uterine Ca

tx: chemo: doxorubian, cisplatin radiation estrogen inhibitors: depro-provera, nolvadex, tamoxifen

hydatidiform mole

tx: cxr, check HCG level weekly, 2-4wk, 1-2 weeks for 6months-1yr

nephrotic syndrome

tx: diuretics prednisone decrease NA, increase protein dialysis

Cushings

tx: do 24 urine check cortisol level adrenalectomy, quiet room increase K, DECREASE NA, increase protein, increase CA

hypo MG

tx: eat magnesium spinach, green beans, cucumber, celery, seeds

addisons disease

tx: give juice/NA+ wgt daily florinef (aldosterone)=mineral corticoid

HYPERparathyroidism

tx: increase CA decreased phosporus

Pulmonary edema

tx: lasix, bumex nitroglycerine, digoxin, morphine, nesiritide, primacor, doButamine

ectopic pregnancy

tx: methotrexate

Angina

tx: nitroglycerine beta blockers: propanolol, lopressor, atenolol, coreg Ca channel lockers: nefedipine, verapamil, amlodipine, dilatiazem asprin

hyper CA

tx: phospho soda fleets enema steroids phosphorus, vitamin D, calcitonin

bleeding esophageal varices

tx: sandostatin sengstaken blakemore tube mg sulfate neomycin

Burns

tx: silvadene-apply direct, can decr. WBC, rash sulfamylon-stings silver nitrate-keep drsg wet, eletrolyte problems betadine-strings, stains, allergy

HYPOparathyroidism

tx:decreased CA increase phosporus IV ca Renagel, foslo, as-cal

Sengstaken blakemore tube ?

used for tx of esophageal varices, keep scissors at bedside.

Consideration when administered iron

used straw for iron no to harm teeth, prefer IV iron than IM because of pain, give iron with juice

Thyroid stimulating hormone blood test

used to differentiate the diagnosis of primary hypothyroidism. Elevated values indicate primary hypothyroidism. Decreased values indicate hyperthyroidism or secondary hypothyroidism.

Furosemide (lasix)

used to manage edema associated with heart failure & hepatic or renal disease to control hypertension

INH

used to treat and prevent TB, do not give with dilantin, can cause phenytonin toxicity, monitor LFT's, give B6 along with, hypotension will occur initially, then resolve

INH

used to treat and prevent TB, do not give with dilantin, can cause phenytonin toxicity, monitor LFT's, give B6 along with, hypotension will occur initially, then resolv

Greenstick fracture?

usually seen in kids bone breaks on one side and bends on the other

How does hyperglycemia affect a Diabetics HbA1c?

usually the cause of an increase in the HbA1c value.

low BP, low Urine output, low resps, and patella relfex absent

what are s/s of mag sulfate toxicity

HTN, edema, and proteinuria

what are s/s of preeclampsia

coffe ground emesis, cola colored urine and tarry stools

what are some elimination symptoms of hemophilia

night sweats, productive cough, fatigue weight loss,

what are some signs of TB

TB skin test, chest xray, sputum test

what are the three diagnostic test for TB

joint pain, infections and dehydration

what are three common signs of sickle cell anemia

left sided heart failure

what condition has s/s of blood tinged sputum, orthopnea, cough, cyanosis and proxysmal nocturnal dyspnea

right sided heart failure

what condition has s/s of fatigue >VP, distended jugular, ascites and dependent edema

placenta previa is painless bleeding

what differentiates placenta previa and abruptio placenta

potassium

what electrolyte is affected with burns to the skin

anemia, NV, anoexia, alopecia, avoid pregnanacy

what four s/s of anticancer meds

cirrhosis

what happens if ammonia levels are high

pancreatitis

what happens if amylase levels are high

dehydrated

what if specific gravity is high what is pt.

dilantin - for siezures

what is 20 mcg/ml and used for siezures or as an anti-eptileptic.

calcium gluconate

what is antidote for mag sulfate toxicity

between 4cm and 8 cm dilation -active first stage

what is cervix dilation to be safe to give meds.

after delivery of placenta and continues for one to four hours after delivery

what is fourth stage of labor

CBC, Hct, Hgb levels

what is monitored if pt. is on thrombolytic agents

never add meds to bloods

what is never to be added to blood in transfusion

7-15

what is normal range of BUN

3.5-5 meq/L

what is normal range of K+

anemia

what is pt. if s/s of pale cold low Hct, and low Hgb and sore tongue

complete dilation to delivery of fetusq

what is second stage of labor

onset of contractions to complete dilation

what is the duration of the first stage of labor

2-3

what is the norm range for INR

30-70 ml/hr

what is the normal rate ml/hr of urination

2.0 ng/ml

what is the toxic level of digoxin

after fetus delivery to placenta delivery

what is third stage of labor

20 mcg/ml

what is toxic level of theophylline

bed rest, > protein, and BP meds

what is trx for preeclampsia

avoid ASA

what medication is avoided in sickle cell anemia

1.5-2.5 x the normal range of PT or PTT

what multiplier for coumadin or heparin to be therapeutic

no bp or injections on affected side

what nursing procedures are avoided to affected side of mastectomy

silvadene

what ointment is used to prevent infections to burn victims

arms above the head

what position should a mastectomy pat. avoid with her arms

stop pitocin drip

what should nurse do to pt. with pitocin drip when pt. shows HTN, tetanic contractions, arrhythmia, NV.

aneurysm, CVA, HTN, anticoags, internal bleeding

what symptoms contraindicate thrombolytic agents

positive sputum test

what test determines active TB

hydration, oxygen and pain relief

what three treatments for sickle cell anemia

hematuria, nosebleeds, gum bleeds, bruising

what to look for when pt. is using anticoagulants, liver dx, platelets <150K, hemophilia, and thromoblytic meds

digoxin - increase cardiac output

what toxic level is 2.0ng/ml and what is it used for

theophylline - COPD

what toxic level is 20 mcg/ml and what is it used for

protein and calories

what two nutrients are essential for burn victims

no solid foods

what type of diet may a patient with radium implants have

mitral

what u hear at 5th intercostal mid-clavicular and PMI (maximum impulse)

tricuspid valve

what u hear on left sternum 4th intercostal

ASTHMA -

wheezing on expiration.

DKA?

when body is breaking down fat instead of sugar for energy. Fats leave ketones (acids) that cause pH to decrease. 71. DKA is rare in diabetes mellitus type II because there is enough insulin to prevent breakdown of fats.

first 15 min

when is a blood transfusion to be checked for allergic reaction

right 2nd intercostal sternum

where find aortic heart sounds

tylenol or acetaminaphen

which is more addicitive NSAIDS or acetaminophen

PTT

which levels are affected by heparin

coumadin

which med is causing the PT time levels

Heart problems?

with R side cardiac cath=look for valve problems with L side in adults look for coronary complications

Osteoarthrithis

with age and obesity joint pain and swelling, limited movement, joint stiffness after the est

-Pancreatic enzymes are taken

with each meal! Not before, not after, but WITH each meal.

Tagamet

with food (H2; messes with elderly ppl be careful ! Interacts with alot of things)

Nurses responsibility

witness signature and ensure that consent is attached to the chart

Adrenal medulla

works as part of the sympathetic nervous system and produces epinephrine and norepinephrine.

Valium for seizures?

yes in emergency

do you taper prednisone?

yes to avoid acute adrenal insufficiency .

ABG drawin?

you need to put the blood in a heparinized tube, make sure there are no bubbles, put on ice immediately after drawing, with a lable indicating if the pt was on room air or how many liters of O2. Remember to preform the Allen's Test prior to doing an ABG to check for sufficient blood flow

When drawing an ABG,

you need to put the blood in a heparinized tube, make sure there are no bubbles, put on ice immediately after drawing, with a lable indicating if the pt was on room air or how many liters of O2. Remember to preform the Allen's Test prior to doing an ABG to check for sufficient blood flow

Glomerulonephritis considerations

you should consider blood pressure to be your most important assessment parameter. Dietary restrictions you can expect include fluids, protein, sodium, and potassium.

HHNS Vs DKA

· Diabetic ketoacidosis (DKA) is a condition of DKA=insulin deficiency resulting in acidemia due to altered metabolism. · Hyperosmolar hyperglycemic nonketotic syndrome (HHNS) is primarily a disorder of dehydration due to hyperglycemia

non stress test

évaluantes réponse of the fetal heart rate to the stress of fetal movement

Diabetes Insipidus?

(decreased ADH): excessive urine output and thirst, dehydration, weakness, administer Pitressin

If mixing antipsychotics

(ie Haldol, Throazine, Prolixin) with fluids, med is incompatible with caffine and apple juice

After Infratentorial Surgery?

(incision at nape of neck)--> position pt flat and lateral on either side.

SIADH?

(increased ADH): change in LOC, decreased deep tendon reflexes, tachycardia, n/v/a, HA; administer Declomycin, diuretics

What to do in the case of- A woman in labor with Un-Reassuring FHR?

(late decels, decreased variability, fetal bradycardia, etc) --> turn on left side (and give O2, stop Pitocin, increase IV fluids)

To prevent dumping syndrome?

(post-operative ulcer/stomach surgeries) --> eat in reclining position, lie down after meals for 20-30 minutes (also restrict fluids during meals, low CHO and fiber diet, small frequent meals)

Buck's Traction?

(skin traction) --> elevate foot of bed for counter-traction

Magnesium sulfate

(used to halt preterm labor) is contraindicated if deep tendon reflexes are ineffective. If patient experiences seizure during magnesium adm. Get the baby out stat (emergency).

A patient with a history of mitral valve replacement, hypertension, and type 2 diabetes mellitus undergoes emergency surgery to remove an embolus in her right leg. Which factor contraindicates the use of epidural analgesia in this patient? 1) Anticoagulant therapy 2) Diabetes mellitus 3) Hypertension 4) Embolectomy

*1) Anticoagulant therapy*

A patient with severe hemorrhoids is incontinent of liquid stool. Which of the following interventions is contraindicated? 1) Apply an indwelling fecal drainage device. 2) Apply an external fecal collection device. 3) Place an incontinence garment on the patient. 4) Place a waterproof pad under the patient's buttocks.

*1) Apply an indwelling fecal drainage device.* An indwelling fecal drainage device is contraindicated for children; for more than 30 consecutive days of use; and for patients who have severe hemorrhoids, recent bowel, rectal, or anal surgery or injury; rectal or anal tumors; or stricture or stenosis. External devices are not typically used for patients who are ambulatory, agitated, or active in bed because the device may be dislodged, causing skin breakdown. External devices cannot be used effectively when the patient has Impaired Skin Integrity because they will not seal tightly. Absorbent products are not contraindicated for this patient unless Impaired Skin Integrity occurs. Even with absorbent products or an external collection device, the nurse should place a waterproof pad under the patient to protect the bed linens.

At last measurement, the client's vital signs were as follows: oral temperature 98°F (36.7°C), heart rate 76, respiratory rate 16, and blood pressure (BP) 118/60. Four hours later, the vital signs are as follows: oral temperature 103.2°F (38.5°C), heart rate 76 beats/minute, respiratory rate 14 breaths/minute, and blood pressure 120/66. Which should the nurse's first intervention be at this time? 1) Ask the client if he has had a warm drink in the last 30 minutes. 2) Notify the primary care provider of the client's temperature. 3) Ask the client if he is feeling chilled. 4) Take the temperature by a different route.

*1) Ask the client if he has had a warm drink in the last 30 minutes.*

In a small rural hospital they work with a wide variety of clients. Of this afternoon client's admitted, the nurse acknowledges the client with the highest susceptibility to infection is the individual with: 1) Burns 2) Diabetes 3) Pulmonary emphysema 4) Peripheral vascular disease

*1) Burns*

The nurse assesses the following changes in a client's vital signs. Which client situation should be reported to the primary care provider? 1) Decreased blood pressure (BP) after standing up 2) Decreased temperature after a period of diaphoresis 3) Increased heart rate after walking down the hall 4) Increased respiratory rate when the heart rate increases

*1) Decreased blood pressure (BP) after standing up* Orthostatic Hypotension

In preventing and controlling the transmission of infections, the single most important technique is: 1) Hand hygiene 2) The use of disposable gloves 3) The use of isolation precautions 4) Sterilization of equipment

*1) Hand hygiene*

The nurse measures the urine output of a patient who requires a bedpan to void. Which action should the nurse take first? Put on gloves and: 1) Have the patient void directly into the bedpan. 2) Pour the urine into a graduated container. 3) Read the volume with the bedpan on a flat surface at eye level. 4) Observe color and clarity of the urine in the bedpan.

*1) Have the patient void directly into the bedpan.* First, the nurse should put on gloves and have the patient void directly into the bedpan. Next, she should pour the urine into a graduated container, place the measuring device on a flat surface, and read the amount at eye level. She should observe the urine for color, clarity, and odor. Then, if no specimen is required, she should discard the urine in the toilet and clean the container and bedpan. Finally, she should record the amount of urine voided on the patient's intake and output record.

A patient tells you that she has trouble falling asleep at night, even though she is very tired. A review of symptoms reveals no physical problems and she takes no medication. She has recently quit smoking, is trying to eat healthier foods, and has started a moderate-intensity exercise program. Her sleep history reveals no changes in bedtime routine, stress level, or environment. Based on this information, the most appropriate nursing diagnosis would be Disturbed Sleep Pattern related to: 1) Increased exercise. 2) Nicotine withdrawal. 3) Caffeine intake. 4) Environmental changes.

*1) Increased exercise.*

The surgeon orders hourly urine output measurement for a patient after abdominal surgery. The patient's urine output has been greater than 60 ml/hour for the past 2 hours. Suddenly the patient's urine output drops to almost nothing. What should the nurse do first? 1) Irrigate the catheter with 30 ml of sterile solution. 2) Replace the patient's indwelling urinary catheter. 3) Infuse 500 ml of normal saline solution IV over 1 hour. 4) Notify the surgeon immediately.

*1) Irrigate the catheter with 30 ml of sterile solution.* If the patient's urinary output suddenly ceases, the nurse should irrigate the urinary catheter to assess whether the catheter is blocked. If no blockage is detected, the nurse should notify the surgeon. The surgeon may request that the catheter be changed if irrigation does not help or if the tubing is not kinked. However, the nurse should not change a catheter in the immediate postoperative period without consulting with the surgeon. The surgeon may prescribe an IV fluid bolus if the patient is suspected to have a deficient fluid volume.

What is typically the most reliable indicator of pain? 1) Patient's self-report 2) Past medical history 3) Description by caregiver(s) 4) Behavioral cues

*1) Patient's self-report*

How should the nurse dispose of the breakfast tray of a patient who requires airborne isolation? 1) Place the tray in a specially marked trash can inside the patient's room. 2) Place the tray in a special isolation bag held by a second healthcare worker at the patient's door. 3) Return the tray with a note to dietary services so it can be cleaned and reused for the next meal. 4) Carry the tray to an isolation trash receptacle located in the dirty utility room and dispose of it there.

*1) Place the tray in a specially marked trash can inside the patient's room.*

A nurse is teaching a group of mothers about first aid. Should poison come in contact with their child's clothing and skin, which action should the nurse instruct the mothers to take first? 1) Remove the contaminated clothing immediately. 2) Flood the contaminated area with lukewarm water. 3) Wash the contaminated area with soap and water and rinse. 4) Call the nearest poison control center immediately.

*1) Remove the contaminated clothing immediately.* Remove contaminated clothing immediately - then wash with water - irrigate it and contact poison control.

Physiological changes associated with aging place the older adult especially at risk for which nursing diagnosis? 1) Risk for Falls 2) Risk for Ineffective Airway Clearance (choking) 3) Risk for Poisoning 4) Risk for Suffocation (drowning)

*1) Risk for Falls* Risk for Falls due to loss of muscle strength and joint mobility

Which of the following factors has the greatest positive effect on sleep quality? 1) Sleeping hours in synchrony with the person's circadian rhythm 2) Sleeping in a quiet environment 3) Spending additional time in stage IV of the sleep cycle 4) Napping on and off during the daytime

*1) Sleeping hours in synchrony with the person's circadian rhythm*

Which of the following nursing activities is of highest priority for maintaining medical asepsis? 1) Washing hands 2) Donning gloves 3) Applying sterile drapes 4) Wearing a gown

*1) Washing hands*

When should the nurse assess pain? 1) Whenever a full set of vital signs is taken 2) During the admission interview 3) Every 4 hours for the first 2 days after surgery 4) Only when the patient complains of pain

*1) Whenever a full set of vital signs is taken*

The patient takes anticoagulants. Which instruction is most important for the nurse to include on the patient's care plan? "Teach the patient to: 1) use an electric razor for shaving." 2) apply skin moisturizer." 3) use less soap when bathing." 4) floss teeth daily."

*1) use an electric razor for shaving."* The nurse should instruct the patient prescribed an anticoagulant to use an electric razor instead of a double-edge razor for shaving to prevent the risk of excess bleeding. Older adults should be encouraged to use skin moisturizers and use less soap while bathing to combat excess drying of the skin that occurs as a result of aging. However, even if this patient is an older adult, a risk for bleeding takes priority over a risk for dry skin. Everyone should be encouraged to floss their teeth daily; however, some patients with severe bleeding risk may be told not to floss.

Which of the following statements by the nurse reflects a need for immediate follow-up regarding the physical effects of chronic pain on body function? 1. "His pulse and blood pressure are within his normal baseline limits, so I'm sure the pain medication is working." 2. "Please take his pulse and blood pressure, and let me know if they are elevated above his normal baselines." 3. "If his pulse and blood pressure are above his normal baseline, let me know, and I will medicate him for pain." 4. "Unmanaged pain usually manifests itself in both an elevated pulse and blood pressure."

*1. "His pulse and blood pressure are within his normal baseline limits, so I'm sure the pain medication is working."* Except in cases of severe traumatic pain, which sends a person into shock, most people reach a level of adaptation in which physical signs return to normal. Thus clients in pain will not always have changes in their vital signs. Changes in vital signs are more often indicative of problems other than pain. Although the remaining options recognize the phenomena, they are not assuming that no elevation of vital signs means the absence of pain.

A client who is 2 days' postoperative reports feeling "constipated" to the nurse. The client has good bowel sounds in all four quadrants and has tolerated liquids well. Her pain is being controlled with an opioid analgesic. Which of the following interventions should the nurse try initially? 1. "Let me get you some apple juice." 2. "Ambulating may get your bowels moving." 3. "I'll see about getting a different pain medication." 4. "Your health care provider might prescribe an enema if I call."

*1. "Let me get you some apple juice."*

A 74-year-old client has been having sleeping difficulties. To have a better idea of the client's problem, the nurse should respond: 1. "What do you do just before going to bed?" 2. "Let's make sure that your bedroom is completely darkened at night." 3. "Why don't you try napping more during the daytime?" 4. "Do you eat a small snack before going to bed?"

*1. "What do you do just before going to bed?"* To assess the client's sleeping problem, the nurse should inquire about predisposing factors, such as by asking "What do you do just before going to bed?" Assessment is aimed at understanding the characteristics of any sleep problem and the client's usual sleep habits so that ways for promoting sleep can be incorporated into nursing care. Older adults sleep best in softly lit rooms. Napping more during the daytime is often not the best solution. The nurse should first assess the client's sleeping problem. The client does not always have to eat something before going to bed.

The nurse is discussing a middle-age adult male client's report of nocturia. The client has diabetes that is managed with diet and exercise as well as hypertension that is currently well-controlled with medication. The nurse should include which of the following as possible causes for his frequent urination at night? (Select all that apply.) 1. An enlarged prostate gland 2. Poorly controlled blood glucose 3. Drinking a cup of tea before bed 4. Possible side effect of his medication 5. Taking his diuretic too close to bedtime 6. Consuming too many liquids during the day

*1. An enlarged prostate gland* *2. Poorly controlled blood glucose* *3. Drinking a cup of tea before bed* *5. Taking his diuretic too close to bedtime*

Which of the following symptomatology is reflective of a lower urinary tract infection? (Select all that apply.) 1. Chills and fever 2. Nausea and vomiting 3. Frequency or urgency 4. Cloudy or blood-tinged urine 5. Pelvic tenderness or flank pain 6. Burning or pain when voiding

*1. Chills and fever* *2. Nausea and vomiting* *3. Frequency or urgency* *4. Cloudy or blood-tinged urine* *6. Burning or pain when voiding*

Which of the following statements should the nurse use to instruct the nursing assistant caring for a client with an indwelling urinary catheter? 1. Empty the drainage bag at least every 8 hours. 2. Clean up the length of the catheter to the perineum. 3. Use clean technique to obtain a specimen for culture and sensitivity. 4. Place the drainage bag on the client's lap while transporting the client to testing.

*1. Empty the drainage bag at least every 8 hours.*

When a client's husband questions how a patient-controlled analgesia (PCA) pump works, the nurse explains that the client: 1. Has control over the frequency of the intravenous (IV) analgesia 2. Can choose the dosage of the drug received 3. May request the type of medication received 4. Controls the route for administering the medication

*1. Has control over the frequency of the intravenous (IV) analgesia* With a PCA system the client controls medication delivery. The PCA system is designed to deliver no more than a specified number of doses. The client does not choose the dosage. The health care provider prescribes the type of medication to be used. The advantage for the client is that he or she may self-administer opioids with minimal risk for overdose. The client does not control the route for administration. Systemic PCA typically involves IV drug administration but can also be given subcutaneously.

Communication involves both active listening and body language working together. The nurse actively listens to the client and: 1. Sits facing the client 2. Keeps the arms and legs crossed 3. Leans back in the chair away from the client 4. Avoids eye contact as much as is physically possible

*1. Sits facing the client* Active listening means to be attentive to what the client is saying both verbally and nonverbally. A nonverbal skill to facilitate attentive listening is to sit facing the client. This posture gives the message that the nurse is there to listen and is interested in what the client is saying. For active listening, the arms and legs should be uncrossed. This posture suggests that the nurse is "open" to what the client says. For active listening, the nurse should lean toward the client. This posture conveys that the nurse is involved and interested in the interaction. For active listening, the nurse should establish and maintain intermittent eye contact. This conveys the nurse's involvement in and willingness to listen to what the client is saying.

When discussing the benefits of physical activity and exercise with a client, the nurse identifies which of the following as a positive outcome to the client? (Select all that apply.) 1. Stress management 2. Enhanced cardiac output 3. Improved bone integrity 4. Facilitation of weight control 5. Increased cognitive function 6. Increased musculoskeletal flexibility

*1. Stress management* *2. Enhanced cardiac output* *3. Improved bone integrity* *4. Facilitation of weight control* *6. Increased musculoskeletal flexibility* Regular physical activity and exercise enhances functioning of all body systems, including cardiopulmonary functioning (endurance), musculoskeletal fitness (flexibility and bone integrity), weight control and maintenance (body image), and psychological well-being. Effects on cognitive function are not consistent.

Which of the following clients presents with an increased risk for urinary incontinence? (Select all that apply.) 1. The 74-year-old diagnosed with parkinsonism 5 years ago 2. The 25-year-old with Crohn's disease diagnosed 4 years ago 3. The 62-year-old Alzheimer's disease client diagnosed 8 years ago 4. The 34-year-old mother of two diagnosed with multiple sclerosis 8 years ago 5. The 73-year-old diagnosed with benign prostatic hyperplasia (BPH) 6 years ago 6. The 69-year-old client diagnosed with type 2 diabetes 9 years ago

*1. The 74-year-old diagnosed with parkinsonism 5 years ago* *3. The 62-year-old Alzheimer's disease client diagnosed 8 years ago* *4. The 34-year-old mother of two diagnosed with multiple sclerosis 8 years ago* *5. The 73-year-old diagnosed with benign prostatic hyperplasia (BPH) 6 years ago* *6. The 69-year-old client diagnosed with type 2 diabetes 9 years ago*

A timed urine specimen collection is ordered. The test will need to be restarted if which of the following occurs? 1. The client voids in the toilet. 2. The urine specimen is kept cold . 3. The first voided urine is discarded. 4. The preservative is placed in the collection container.

*1. The client voids in the toilet.*

A client with a fractured left femur has been using crutches for the past 4 weeks. The physician tells the client to begin putting a little weight on the left foot when walking. Which of the following gaits should the client be taught to use? 1. Two-point 2. Three-point 3. Four-point 4. Swing-through

*1. Two-point* The two-point gait requires at least partial weight bearing on each foot. The client moves a crutch at the same time as the opposing leg, so that the crutch movements are similar to arm motion during normal walking. In a three-point gait, weight is borne on both crutches and then on the uninvolved leg. The four-point gait gives stability to the client but requires weight bearing on both legs. Each leg is moved alternately with each opposing crutch so that three points of support are on the floor at all times. This client is only supposed to use partial weight bearing, so this gait would not be appropriate. Paraplegics who wear weight-supporting braces on their legs use the swing-through gait. It would not be appropriate for this client.

The nurse instructs the client that before the fecal occult blood test (FOBT) she may eat: 1. Whole wheat bread 2. A lean, T-bone steak 3. Veal 4. Salmon

*1. Whole wheat bread*

The nurse must administer an enema to an adult patient with constipation. Which of the following would be a safe and effective distance for the nurse to insert the tubing into the patient's rectum? Choose all that apply. 1) 2 inches 2) 3 inches 3) 4 inches 4) 5 inches

*2) 3 inches* *3) 4 inches* When administering an enema, the nurse should insert the tubing about 3 to 4 inches into the patient's rectum. Two inches would not be effective because it would not place the fluid high enough in the rectum. Five inches is too much.

An older adult receiving hospice care has dementia as a result of metastasis to the brain. His bone cancer has progressed to an advanced stage. Why might the client fail to request pain medication as needed? The client: 1) Experiences less pain than in earlier stages of cancer. 2) Cannot communicate the character of his pain effectively. 3) Recalls pain at a later time than when it occurs. 4) Relies on caregiver to provide pain relief without asking.

*2) Cannot communicate the character of his pain effectively.*

The healthcare team suspects that a patient has an intestinal infection. Which action should the nurse take to help confirm the diagnosis? 1) Prepare the patient for an abdominal flat plate. 2) Collect a stool specimen that contains 20 to 30 ml of liquid stool. 3) Administer a laxative to prepare the patient for a colonoscopy. 4) Test the patient's stool using a fecal occult test.

*2) Collect a stool specimen that contains 20 to 30 ml of liquid stool.* To confirm the diagnosis of an infection, the nurse should collect a liquid stool specimen that contains 20 to 30 ml of liquid stool. An abdominal flat plate and a fecal occult blood test cannot confirm the diagnosis. Colonoscopy is not necessary to obtain a specimen to confirm the diagnosis.

When changing a diaper, the nurse observes that a 2-day-old infant has had a green black, tarry stool. What should the nurse do? 1) Notify the physician. 2) Do nothing; this is normal. 3) Give the baby sterile water until the mother's milk comes in. 4) Apply a skin barrier cream to the buttocks to prevent irritation.

*2) Do nothing; this is normal.* During the first few days of life, a term newborn passes green black, tarry stools known as meconium. Stools transition to a yellow green color over the next few days. After that, the appearance of stools depends upon the feedings the newborn receives. Sterile water does nothing to alter this progression. Meconium stools are more irritating to the buttocks than other stools because they are so sticky and the skin usually must be rubbed to cleanse it.

Which one of the following clients would probably have a higher than normal respiratory rate? A client who has: 1) Had surgery and is receiving a narcotic analgesic. 2) Had surgery and lost a unit of blood intraoperatively. 3) Lived at a high altitude and then moved to sea level. 4) Been exposed to the cold and is now hypothermic.

*2) Had surgery and lost a unit of blood intraoperatively.* Hypovolemia / shock. BP decreases, respiratory rate increases

After undergoing dural puncture while receiving epidural pain medication, a patient complains of a headache. Which action can help alleviate the patient's pain? 1) Encourage the client to ambulate to promote flow of spinal fluid. 2) Offer caffeinated beverages to constrict blood vessels in his head. 3) Encourage coughing and deep breathing to increase CSF pressure. 4) Restrict oral fluid intake to prevent excess spinal pressure.

*2) Offer caffeinated beverages to constrict blood vessels in his head.*

While performing a physical assessment, the student nurse tells her instructor that she cannot palpate her patient's bladder. Which statement by the instructor is best? "You should: 1) Try to palpate it again; it takes practice but you will locate it." 2) Palpate the patient's bladder only when it is distended by urine." 3) Document this abnormal finding on the patient's chart." 4) Immediately notify the nurse assigned to your patient."

*2) Palpate the patient's bladder only when it is distended by urine."* The bladder is not palpable unless it is distended by urine. It is not difficult to palpate the bladder when distended. The nurse should document her finding, but it is not an abnormal finding. It is not necessary to notify the nurse assigned to the patient.

A client has severe right-sided weakness and is unable to complete bathing and grooming independently. Based on this observation, the nurse identifies a nursing diagnosis of: 1) Powerlessness 2) Self-care deficit 3) Tissue integrity impairment 4) Knowledge deficit of hygiene practices

*2) Self-care deficit* The client who is unable to complete bathing and grooming independently has a nursing diagnosis of self-care deficit. Being unable to complete bathing and grooming are not defining characteristics for the nursing diagnosis of powerlessness. Being unable to complete bathing and grooming are not defining characteristics for the nursing diagnosis of tissue integrity impairment. There is no indication this client has a knowledge deficit of hygiene practices.

A patient with diarrhea is incontinent of liquid stool. The nurse documents that he now has excoriated skin on his buttocks. Which finding by the nurse led to this documentation? 1) Skin was softened from prolonged exposure to moisture. 2) Superficial layers of skin were absent. 3) Epidermal layer of skin was rubbed away. 4) Lesion caused by tissue compression was present.

*2) Superficial layers of skin were absent.* Excoriation is a loss of the superficial layers of the skin caused by the digestive enzymes in feces. Maceration is the softening of skin from exposure to moisture. Abrasion, a rubbing away of the epidermal layer of the skin, especially over bony areas, is often caused by friction or searing forces that occur when a patient moves in bed. Pressure ulcers are lesions caused by tissue compression and inadequate perfusion that are a result of immobility.

In which situation would using standard precautions be adequate? (Select all that apply.) 1) While interviewing a client with a productive cough 2) While helping a client to perform his own hygiene care 3) While aiding a client to ambulate after surgery 4) While inserting a peripheral intravenous catheter

*2) While helping a client to perform his own hygiene care* *3) While aiding a client to ambulate after surgery* *4) While inserting a peripheral intravenous catheter*

Which of the following statements made by a nurse discussing the effect of an antibiotic on the gastrointestinal system reflects the best understanding of the possible occurrence of diarrhea? 1. "The GI tract naturally rids itself of bacterial toxins by increasing peristalsis, and that causes diarrhea." 2. "The antibiotic is responsible for killing off the GI tract's normal bacterial, and diarrhea is the result." 3. "For some, antibiotics irritate the mucous lining of the intestines, causing decreased absorption and diarrhea." 4. "When you are taking an antibiotic, your body is fighting off an infection, and peristalsis is faster and so diarrhea occurs."

*2. "The antibiotic is responsible for killing off the GI tract's normal bacterial, and diarrhea is the result."*

The nurse and a client are discussing possible behaviors that might be interfering with the client's ability to fall asleep. Which of the following assessment questions is most likely to identify possible problems with the client's sleep routine that possibly are contributing to the difficulty? 1. "When do you usually retire for the night?" 2. "What do you do to help yourself fall asleep?" 3. "How much time does it usually take for you to fall asleep?" 4. "Have you changed anything about your presleep ritual lately?"

*2. "What do you do to help yourself fall asleep?"* As people try to fall asleep, they close their eyes and assume relaxed positions. Stimuli to the RAS decline. If the room is dark and quiet, activation of the RAS further declines. At some point the BSR takes over, causing sleep. If the client engages in activities such as reading or watching television as a means of falling asleep, this could be causing the problem. Although the other questions are not inappropriate, they are not as directed toward the cause of the problem.

Which of the following statements made by an older adult reflects the best understanding of the role of fiber regarding bowel patterns? 1. "The more fiber I eat, the fewer problems I have with my bowels." 2. "Whole grain cereal and toast for breakfast keeps my bowels moving regularly." 3. "My wife makes whole grain muffins; they are really good and good for me too." 4. "I use to have trouble with constipation until I started taking a fiber supplement."

*2. "Whole grain cereal and toast for breakfast keeps my bowels moving regularly."*

The nurse recognizes that the leading cause of death for the otherwise healthy 1-year-old is: 1. Physical abuse 2. Accidental injury 3. Contagious diseases 4. Stranger abduction

*2. Accidental injury* Injuries are the leading cause of death in children older than 1 year of age and cause more deaths and disabilities than do all diseases combined.

Which of the following symptoms should the nurse assess with a client who is deprived of sleep? 1. Elevated blood pressure and confusion 2. Confusion and irritability 3. Inappropriateness and rapid respirations 4. Decreased temperature and talkativeness

*2. Confusion and irritability* Psychological symptoms of sleep deprivation include confusion and irritability. Elevated blood pressure is not a symptom of sleep deprivation. Rapid respirations are not a symptom of sleep deprivation. There may be a decreased ability of reasoning and judgment that could lead to inappropriateness. Decreased temperature is not a symptom of sleep deprivation. The client with sleep deprivation is often withdrawn, not talkative.

The nurse is caring for a hospitalized client who normally works the night shift at his job. The client states, "I don't know what is wrong with me. I have been napping all day and can't seem to think clearly." The nurse's best response is 1) "You are sleep deprived, but that will resolve in a few days." 2) "You are experiencing hypersomnia, so it will be important for you to walk in the hall more often." 3) "There has been a disruption in your circadian rhythm. What can I do to help you sleep better at night?" 4) "I will notify the doctor and ask him to prescribe a hypnotic medication to help you sleep."

*3) "There has been a disruption in your circadian rhythm. What can I do to help you sleep better at night?"* The data suggests that the patient is used to being awake at night and sleeping during the day. The hospital routine has disrupted this normal pattern.

A client who has been hospitalized for an infection states, "The nursing assistant told me my vital signs are all within normal limits; that means I'm cured." The nurse's best response would be: 1) "Your vital signs confirm that your infection is resolved; how do you feel?" 2) "I'll let your health care provider know so you can be discharged." 3) "Your vital signs are stable, but there are other things to assess." 4) "We still need to keep monitoring your temperature for a while."

*3) "Your vital signs are stable, but there are other things to assess."*

A nurse is teaching wellness to a women's group. The nurse should explain the importance of consuming at least how much fluid to promote healthy bowel function (assume these are 8-ounce glasses)? 1) 2 to 4 glasses a day 2) 4 to 6 glasses a day 3) 6 to 8 glasses a day 4) 8 to 10 glasses a day

*3) 6 to 8 glasses a day* A minimum of 6 to 8 glasses of fluid should be consumed each day to promote healthy bowel function.

A client is admitted to the emergency department with complaints of nausea and vomiting. Arterial blood gas studies show metabolic alkalosis. The nurse understands that alkalosis is a:

pH greater than 7.45.

Ulcers

pain after meal, do not drink milk, food not too cold or hot, reduce stress. Give Tagamet will meal, NG tube. will receive vit B12 for the rest of the life. avoid dumping syndrome

Demerol is for

pain control

McBurney's Point

pain in RLQ indicative of appendicitis

Mcburney's point?

pain in RLQ indicative of appendicitis-- RLQ - appendicitis, watch for peritonitis

Appendicitis

pain is in RL quadrant with rebound tenderness.

osteoporosis

reduce in one mass, decrease in height, low back pain, kyphosis

BPH -

reduced size & force of urine

CABG?

the great saphenous vein is taken it is turned inside out due to the valves that are inside.

Wilm's tumor is usually encapsulated above the ________ and causing _______.

the kidneys and flank pain

For a lumbar puncture,

pt is positioned in lateral recumbent fetal position, keep pt flat for 2-3 hrs afterwards, sterile dressing, frequent neuro assessments

After Cataract surgery?

pt will sleep on unaffected side with a night shield for 1-4 weeks.

PCWP

pulm capillary wedge pressure is 8-13. Readings of 18-20 are considered high.

Common S/S of appendicitis

rebound tenderness

APPENDICITIS

rebound tenderness.

If patients have hallucinations ______ them. In delusions________ them.

redirect distract

Emphysema?

the stimulus to breathe is low PO2, not increased PCO2 like the rest of us, so don't slam them with oxygen. Encourage pursed-lip breathing which promotes CO2 elimination, encourage up to 3000mL/day fluids, high-fowlers and leaning forward.

After Gtube placement?

the stomach contents are drained by gravity for 24 hours before it can be used for feedings.

Thorazine, haldol (antipsychotic) can lead

to EPS (extrapyramidal side effects)

vit b6 with TB drugs

to avoid peripheral neuropathy

Give neostigmine

to clients with Myesthenia Gravis about 45 min. before eating, so it will help with chewing and swallowing.

increase formula or breast feeding to excrete bilirubin in stool

to lower baby bilirubin levels - bilirubin light or fiber optic blanket monitor skin temp and do this

reason for vitamin K IM to be given to newborn at birth

to prevent hemorrhagic disorders

What treats tet spells?

treated with morphine.

Extravasion

tx: sodium thiosulphate dimethyl sulphoxide dextrazoxane (totect)

HYPOthyroid

tx: synthroid proloid cytomel...coranry heart disease..put on heart monitor

premature labor

tx: tocolytics: mg sulfate, terbutaline

kidney stone

tx: toradol antiemetics: phenergan, zofran, dilaudid

Erbs point - left third intercostal of sternum

where can u find S1,S2

left 2nd intercostal sternum

where do u listen for pulominc valve

ASTHMA

wheezing on expiration.

27%

whole left leg and front right leg burn is

cardiac output decreases with

dysrythmias. Dopamine increases BP.

Cytovene,

e, used for retinitis caused by cytomegalovirus, pt will need regular eye exams, report dizziness, confusion, or seizures immediately

ventricular gallop

earliest sign of heart failure

Cullens sign?

ecchymosis in umbilical area, seen with pancreatitis

Cullen's sign

echimosis around umbilicus means bleeding

Venous VV and Arterial AA?

eleVate Veins; dAngle Arteries for better perfusion

Cardioversion

elective procedure, informed consent, give vallium

Head Injury?

elevate HOB 30 degrees to decrease intracranial pressure

procedure to insert and salem sump NG tube

elevate bed 60-90% to facilitate swallowing and move of tube through GT tract.

Above the knee amputation?

elevate for first 24 hours on pillow, position prone daily to provide for hip extension.

Pneumonia?

fever and chills are usually present. For the elderly confusion is often present.

Glomelonephritis

fever chills, hematuria, protenuria, edema, hypertension, occur after strep. Increase calorie and decrease protein

thromboembolism s/s

fever, dyspnea, chest pain, cough, signs of shock

For an EEG test?

hold meds for 24-48 hrs prior, no caffine or cigarettes for 24 hrs prior, pt can eat, pt must stay awake at night before exam, pt may be asked to hyperventilate and watch a bright flashing light, after EEG, assess pt for seizures, pt's will be at increased risk

0-1

holding objects

To remember blood sugar?

hot and dry-sugar high (hyperglycemia) cold and clammy-need some candy (hypoglycemia)

is increased with exercise

how can you increase HDL levels

48-72 hours

how long after getting the TB skin test does it need to be checked

decsends one fingerbreath per day below umbilicis and 6 fingers below after 2 weeks

how long does it take for the top of the uterus to descend after delivery

18-24 months until 0 symptoms

how long is TB pt. medications to be taken for

2 weeks and negetive sputum test

how long should TB pt. be in isolation

50-200 J

how many Joules for cardioversion

Cushings?

hyperNa, hypoK, hyperglycemia, prone to infection, muscle wasting, weakness, edema, HTN, hirsutism, moonface/buffalo hump

Grave's disease/hyperthyroidism

hyperactivity, sensitivity to heat, weight loss, tachycardia, diarhea, expphthalumis( bulging eyes), nervousness, jittery, irradiation, surgery, light cloth

Parathyroid gland disorders

hyperparathyroidism and hypoparathyroidism

Thyroid storm

hyperpyrexia( high temp), confusion, agitation, increase in pulse

Laparoscopy

general diet, Foley in , co2 introduced, the patient go home same day

Eclampsia is characterizied by

generalized seizures

how to deal with attention seeking patient?

give unsolicited attention when the patient is exhibiting acceptable behaviors, it rewards non-attention seeking behavior

MMR SHot?

given SQ not IM.

TPN (total parenteral nutrition)

given in subclavian line.

Diamox

glaucoma, high altitude sickness...dont take if allergic to sulfa drugs

Timolol (Timoptic)

gluacoma

Nonketotic type 2

glucose 800, ketosis do not occur, hypo tension, dry mucous membrane,seizures

isometric exercise

good for clients who have arthritis or joint disease

Cancer

good mouth care, calorie, high protein diet, small frequent meal, frequent feeding, pain relief, discuss body image, antiemics

Glaucoma intraocular pressure is

greater than the normal (22 mm Hg), give miotics to constrict (pilocarpine) NO ATROPINE.

Stranger anxiety is greatest , Separation anxiety peaks in____

greatest 7 - 9 months peaks in toddlerhood

Danger signs of pregnancy

gush of fluid or bleeding from vagina, regular uterine contractions, severe headache, visual disturbances, abdominal pain, persistent vomiting,fever, chills, sweling face and fingers

Magnesium sulfate (used to ________ is contraindicated_________ are ineffective. If patient experiences seizure during magnesium adm. It's an (emergency) and you must _______.

halt preterm labor) if deep tendon reflexes get the baby out

percodan

has ASA.

hypertonic iv

have a higher osmolality than blood. when they are used, water moves by osmosis from the cells into the ECF, Like TPN

Chest pain ?

heartburn that radiates to jaw

Omphalocele?

herniation of abdominal contents- dress with loose saline dressing covered with plastic wrap, and keep eye on temp. Kid can lose heat quickly.

hypermagnesemia

high DTR

diet for patient with burn

high calorie, high carbs and high protein

Take Vermox with

high fat diet (increases absorption)

What diet for crons or ul colitis

high protein, high calorie, low fat and low fiber

dilantin use in elderly

high risk for toxicity

Gout

high uric acid, joint deterioration, low purine diet, No fish or organ meats

Glycosylated hemoglobin A1c

indicates how well blood glucose levels have been controlled for the prior 3 to 4 months. Goal for clients with diabetes mellitus is 7% or lower. For clients w/o diabetes, the normal range is 4-6%.

Ballance sign

indicates rupture of sleep seen of left side

Implementation

increase in fluid intake, Iv lasix, prevent fracture, monitor potassium levels

hypovolemia (lab signs)

increased hemtocrit and hemoglobin

The first s/s of ards?

increased respirations. Later comes dyspnea, retractions, air hunger, cyanosis.

US is done on

full bladder

What to give for person with hypoglycemia

skin milk, hard candies, 4 ounces of juice

Cushing's Disease

(Remember: *UP, UP, UP, DOWN, UP*) - HYPERnatremia, HYPERtension, INCREASED blood volume, HYPOkalemia, HYPERglycemia

Tension Pneumothorax?

Trachea shifts to the opposite side

Best way to warm a newborn?

skin to skin contact on mom with a blanket

Decrease protein

=Decreased BUN levels

Thyroid gland

controls the rate of body metabolism and growth and produces thyroxine (T4), triiodothyronin (T3), and thyrocalcitonin.

How to itch under a cast?

cool air via blow dryer, ice pack for 10- 15 minutes. NEVER use qtip or anything to scratch area

school age

cooperative play

Pain in the LLQ indicative of?

diverticulitis , low residue, no seeds, nuts, peas

Cardinal signs of ARDS?

hypoxemia (low oxygen level in tissues).

chronic renal failure and dehydration

if BUN level is 16 what is pt.

overhydrated

if BUN level is 6 what is pt.

by auscultation you can hear the fetus

18-20 weeks

1-7

imitation

Serizure

Protect from injury, do not restrain, do not insert anything, monitor duration, pattern, low cars high fat diet cause sugar increases the risk of seizure

spinal shock occurs

immediately after spinal injury

Dig rule for kids?

if HR is <100 do not give dig to children.

over hydrated

if Hct is 35% in a man what is pt.

dehydrated

if Hct is 55% in a man what is the pt. experiencing

anemic

if Hgb is low the pt. is this

dehydration

if Na is >145 the pt. is this

pt. has an infection

if WBC is 12,000 what is patient

risk for infection

if WBC is 3,000 what is the patient at risk for

normal

if a nonstress test of OB pt. has a reactiv increased HR in response to fetal activity this is to be.

CRF

if creatinine is also high or above 1.5 what is pt.

Every new admission needs?

if he has an advance directive, and if not you will explain it, and he will have the option to sign or not.

Rule of thumb for assisting pysch patients?

if patients have hallucinations redirect them. In delusions distract them.

no surgery

if pt. on anticoags what is medical procedures are avoided

gout

if uric acid is high what is pt.

diabetes insipidus

if urine specific gravity is low what is pt.

alcoholic patient

if vital signs are steadily increasing think delirium tremens.

Autonomic Dysreflexia/ Hyperreflexia?

(S&S: pounding headache, profuse sweating, nasal congestion, goose flesh, bradycardia, hypertension) --> place client in sitting position (elevate HOB) first before any other implementation.

If kid has cold, can still give

immunizations

Aminoglycocide

(__Mycin ; except erythromycine) Adverse Effects are bean shaped - Nephrotoxic to Kidneys and Ototoxic to Ears

at risk for developing here zoster

immunocompromised pt

Mechanical Ventilation

listen to breath sounds they should be equal and bilateral, Assess need for suctioning, provide oral hygiene, care for patient first than equipment, alternated way of communication

2.0 meq/L

lithium has a toxic level of?

Tylenol poisioining ?

liver failure possible for about 4 days. Close observation required during this time-frame, as well as tx with Mucomyst.

Common sites for metastatsis?

liver, brain, lung, bone, and lymph

edoctrin

loop diuretic, potassium wasting. increase potassium intake

Spinal cord injury

loss of sensation below injury, postural hypotantion hemoregulation problems, log roll the patient

What diet for ulcers

low fat, low carbs

Spinal bifida

low folic acid in mom

Thyroidectomy( post op)

low or semi Fowler, support head, observe for complication, laryngeal nerve injury, thyroid storm, tetany, hemmorhage

After Thyroidectomy?

low or semi-Fowler's, support head, neck and shoulders.

Low Residue diet?

low residue diet means low fiber

Minere

low sodium diet, person has severe vertigo

Electrolyte Imbalances: Expected findings

low- irrigular heartbeat, confusion, B/P changes, nervous system or bone disorders high- weakness or twitching of muscles, numbness, fatigue, irregular heartbeat, B/P changes

if pt is physically assualtive,

restore the clients self control, safety

Labs for congenital heart disease?

result in hypoxia which the body attempts to compensate for (influx of immature rbc's)? Labs supporting this would show increased hematocrit, hemoglobin, and rbc count.

GTT for preggos? (glucose tolerance test)

result of 140 or higher needs further evaluation.

Tet spells?

results from a transient increase in resistance to blood flow to the lungs with increased preferential flow of desaturated blood to the body. Tet spells are characterized by a sudden, marked increase in cyanosis followed by syncope, and may result in hypoxic brain injury and death. Older children will often squat during a tet spell, which increases systemic vascular resistance and allows for a temporary reversal of the shunt.

Narcan

reverses respiratory depression. , -antidote to opioids overdose

CHOLERA

rice watery stool.

Zoloft

s/e agitation, sleep disturb, and dry mouth

Clozapine

s/e agranulocytosis, tachycardia and siezures

Dilantin

s/e rash (stop med), gingival hyperplasia (good hygiene) toxicity-->poor gait + coordination, slurred speech, nausea, lethargy, and diplopia.

First sign of pe?

sudden chest pain, followed by dyspnea and tachypnea.

Crackles most likely are?

suggest pneumonia, which is likely to be accompanied by hypoxia, which would manifest itself as mental confusion, etc.

open method of wound care

sulfamylon

lumbar puncture for a child

need restraints

Age 4=5 year shots?

needs DPT/MMR/OPV

disoriented patient..

needs assessment

transplant pt room

needs isolation

Amynoglycosides (like Vancomycin) cause ( 2 things )

nephrotoxicity & ototoxicity

TEF?

Transesophageal Fistula- esophagus doesn't fully develop (this is a surgical emergency) The 3 C's of TEF in the newborn: 1) Choking 2) Coughing 3) Cyanosis

A client asks about the medication he's receiving. The nurse's reply is based on the knowledge that:

the client has a right to know the medication he's getting and its adverse effects.

post op client of pneumonectomy, what adverse finding indicates acute pulmonary edema?

Frothy sputum, dyspnea, cough, crakles, and possibly cynosis

RETINAL DETACHMENT -

Visual Floaters, flashes of light, curtain vision

Presbyopia

Visual changes with aging

Common S/S of retinal detachment?

Visual floaters, flashes of light, curtain vision

The nurse is caring for a young adult with hepatitis A. The client is crying and saying that she hates the way she looks with yellow skin. Which of the following would be the nurse's best response?

"I know you're upset; your skin will return to its normal color as you get well."

EPIGLOTITIS -

- 3Ds' Drooling, Dysphonia, Dysphagia

PTB s/s

- low-grade afternoon fever

CPR for adult breath rate

12per min, mouth to mouth

haldol

monitor blood pressure every 30 mins. ensure patient safety.

hypercalcaemia

A 24 yr old is adm to the ER for confusion. This pt has a hx of myeloma dx, constipation, intense abdominal pain, and polyuria. Which of the following would you suspect?

Dawn phenomenon

A nocturnal release of growth hormone, which may cause blood glucose level elevations before breakfast in the client with diabetes mellitus. Treatment includes administering an evening dose of intermediate acting insulin at 10 pm.

air hunger

A patient admitted to the hospital w/mycardial infarcation develops severe pulmonary edema. Which of the following should the nurse expect?

PANCREATITIS -

Cullen's sign (ecchymosis of umbilicus); (+) Grey turners spots

Peptic ulcers caused by H. pylori are treated with 3 meds :

Flagyl, Prilosec and Biaxin This treatment kills bacteria and stops production of stomach acid, but does not heal ulcer.

Common S/S Hepatic Encephalopathy ?

Flapping tremors

TB health risk?

If a TB patient is unable/unwilling to comply with tx they may need supervision (direct observation). TB is a public health risk.

A client on prolonged bed rest has developed a pressure ulcer. The wound shows no signs of healing even though the client has received skin care and has been turned every 2 hours. Which factor is most likely responsible for the failure to heal?

Inadequate protein intake

SNS

Increase in BP, HR and RR (dilated bronchioled), dilated pupils (blurred vision), Decreased GUT (urniary retention), GIT (constipation), Constricted blood vessels and Dry mouth.

Order of assessment?

Inspection, Palpation, Percussion and Ausculation. EXCEPT with abdomen cuz you don't wanna mess with the bowels and their sounds so you Inspect, Auscultate, Percuss then Palpate (same with kids, I suppose since you wanna go from least invasive to most invasive sine they will cry BLOOD MURDER ! Gotta love them kids !)

Order of assessment:

Inspection, Palpation, Percussion and Ausculation. EXCEPT with abdomen cuz you don't wanna mess with the bowels and their sounds so you Inspect, Auscultate, Percuss then Palpate (same with kids, I suppose since you wanna go from least invasive to most invasive sine they will cry BLOOD MURDER ! Gotta love them kids !)

Tonometry

Intramuscular pressure for glaucoma

Common S/S of measles?

Kopliks spots

Common S/S of Cushings syndrome?

Moon face appearance and buffalo hump

OB secret?

NEVER check the monitor or a machine as a first action. Always assess the patient first; for exmaple listen to the fetal heart tones with a stethoscope in NCLEX land. Sometimes it's hard to tell who to check on first, the mother or the baby; it's usually easy to tell the right answer if the mother or baby involves a machine. If you're not sure who to check first, and one of the choices involves the machine, that's the wrong answer.

Can you take ASA during pregnancy ?

NO

Immobilizing Interventions:Findings to Report with a client in traction

Provide care to client in traction (check the weights are hanging freely, observe skin for irritation and site of skeletal traction insertion for signs of infection; use aseptic technique when cleaning the site of insertion)

HIV precautions?

Standard

Common S/S of Malaria?

Stepladder like fever with chills

Halo?

Remember safety first, have a screwdriver nearby.

APAGR: R

Respirations - strong cry/weak cry/absent

Chief concern in CF

Respiratory problems!

A client in her first postpartum month has developed mastitis secondary to breast-feeding. Her nurse, a mother who developed and recovered from mastitis after her third child, says, "I remember the discomfort I had and how quickly it resolved when I began getting treatment." The therapeutic communication being used by the nurse is:

Self-disclosure. Rationale: Using self-disclosure as a therapeutic communication technique encourages an open and authentic relationship between the nurse and her client. Self-disclosure involves the nurse revealing personal information. Clarification involves the nurse asking the client for more information. Reflection is reviewing the client's ideas. Restating is the nurse's repetition of the client's main message.

The nurse is caring for a 40-year-old woman who is a Black Muslim. Which of the following choices is a concept on which Black Muslims focus?

Self-esteem

Adrenal cortex

Synthesizes glucocorticoids and mineralocorticoids and secretes small amounts of sex hormones.

SLE?

Systemic Lupus Ethramoutus.... (Lupus)

T3 and T4 resin uptake test

T3 and T4 regulate thyroid stimulating hormone. T3 is elevated in hyperthyroidism and may be decreased in hypothyroidism. T4 is elevated in hyperthyroidism and decreased in hypothyroidism.

POTASSIUM HYDROXIDE PREP

THE FASTEST AND MOST EFFECTIVE METHOD TO DX FUNGAL INFECTION IS USING...

RISK FOR INFECTION

THE PRIORITY NURSING CONCERN FOR CLIENTS WITH DISORDERS SUCH AS BURNS, PSORIASIS, AND SHINGLES IS?

18-30 seconds

THERAPEUTIC range of coumadin found on PT

45-75 seconds

THERAPEUTIC range of heparin found on PTT

Radioactive iodine?

The key word here is flush. Flush substance out of body w/3-4 liters/day for 2 days, and flush the toilet twice after using for 2 days. Limit contact w/patient to 30 minutes/day. No pregnant visitors/nurses, and no kids.

renal disorders: Managing Elevated Phosphorus levels

The kidneys help regulate the level of phosphorus in your blood. If your kidney function is impaired, eventually you'll likely have elevated phosphorus levels (hyperphosphatemia). In turn, the elevated phosphorus decreases the level of calcium in your blood, which can lead to bone disease.

When should blood glucose be checked when exercising?

The pt. should be taught to monitor their blood glucose before, during, and after exercise. If the blood glucose is higher than 250 mg/dL and urinary ketones are present, the pt. should not exercise until the glucose level is closer to normal and ketones are absent.

Hypophysectomy

The removal of the pituitary gland.

Anytime you see fluid retention.

Think heart problems first

Physiological jaundice occurs _______

after 24 hours.

Librium

alcohol w/d...dont take alchol with this...very bad nausea and vomiting can occur.

Flu shot always ask?

allergic to eggs???

Pancreatic enzymes

are taken with each meal! Not before, not after, but WITH each meal.

Detached Retina?

area of detachment should be in the dependent position-- dependent meaning supported by something

CT scan ck for

assess allergies

208.) A client with myasthenia gravis verbalizes complaints of feeling much weaker than normal. The health care provider plans to implement a diagnostic test to determine if the client is experiencing a myasthenic crisis and administers edrophonium (Enlon). Which of the following would indicate that the client is experiencing a myasthenic crisis? 1. Increasing weakness 2. No change in the condition 3. An increase in muscle spasms 4. A temporary improvement in the condition

auto-define "A client with myasthen..." Rationale: Edrophonium (Enlon) is administered to determine whether the client is reacting to an overdose of a medication (cholinergic crisis) or to an increasing severity of the disease (myasthenic crisis). When the edrophonium (Enlon) injection is given and the condition improves temporarily, the client is in myasthenic crisis. This is known as a positive test. Increasing weakness would occur in cholinergic crisis. Options 2 and 3 would not occur in either crisis.

pap smear teaching

avoid douching

Why would a pt with leukemia have epistaxis?

b/c of low platelets

EMPHYSEMA

barrel chest.

If kid has a cold?

can still give immunizations

side lateral

drainage of oral secretion

Neostigmine/Atropine (anticholinergic) to reverse

effect of pancuronium.

A 74-year-old client has three grown children who each have families of their own. The client is retired and looks back on his life with satisfaction. According to Erickson, the nurse assesses that the client is in a stage of:

generativity.

Implementation for Non ketotic type 2 diabetes

give 0.9 NACL. IV Regular insulin

Implementation for Ketoacidosis

give 0.9 or 0.45 NACL, K electrolytes, EKG check, check on patient every 15 minutes

Liver biopsy

give K before the test, hold the breath for 5-10 sec, the patinet should be on the right side, check for bleeding

hypoK

give K+, aldactone

Cystic fibrosis?

give diet low fat, high sodium, fat soluble vitamins ADEK. Aerosal bronchodilators, mucolytics and pancreatic enzymes.

Autonomic hyperflexia/ dysreflexia

headache, diaphoretic, piloerection( goose bumps) bradycardia, hypertestion, bladder problem, sitting position or choice

LTB

inspiratory stridor

LEPROSY

lioning face.

Heart Failure: Understanding medication Self Administration

monitor serum potassium level, DECREASED potassium levels increases digoxin toxicity. *Cardiotonic drugs affect the intracellular calcium levels in the heart= increased cardiac output, renal blood flow, and urine formation. Neg side affect is bradycardia.. MONITOR apical pulse

When on nitroprusside

monitor thiocynate (cyanide). Normal value should be 1, >1 is heading toward toxicity

PTU & tapazole

prevention of thyroid storm

Common S/S of pernicious anemia

red beefy tongue

PERNICIOUS ANEMIA

red beefy tongue.

Koplick's spots

red spots with blue center characteristic of PRODROMAL stage of Measles. Usually in mouth.

Koplick's spots?

red spots with blue center characteristic of PRODROMAL stage of Measles.Usually in mouth.

fixed and dilated pupils?

represents at neurological emergency

Cephalhematoma (caput succinidanium)?

resolves on its own in a few days. This is the type of edema that crosses the suture lines.

Myxedema/ hypothroidism?

slowed physical and mental function, sensitivity to cold, dry skin and hair

Best way to tube feed or feed kids?

small frequent is better than large

Uremic Fetor?

smell urine on the breath

Ativan is the treatment of choice for

status epilepticus

20-36 weeks

umbilicus

Risperdal

antipsychotic, doses over 6mg can cause tarditive dyskinesia, first line antipsychotic in children

S/s of Air/Pulmonary Embolism

chest pain, difficulty breathing, tachycardia, pale/cyanotic, sense of impending doom

Hyperparathyroidism

muslce weakness, cardiac dystitmia, renal calculi, pathological fracture

Dilantin

seizures. thera drug level: 10-20

which hand hold cane?

strong side

A client says to the nurse, "I know that I'm going to die." Which of the following responses by the nurse would be best?

"Why do you think you're going to die?"

BOTOX

(Botulin Toxin) can be used with strabismus also to relax vocal cords in spasmodic dysphonia

A client is admitted with acute chest pain. When obtaining the health history, which question will be most helpful for the nurse to ask?

"What were you doing when the pain started?"

Upon entering a client's room, the client frowns and states, "I've had my light on for 20 minutes. It's about time you got here. I'm sick of this place and the staff." The nurse's best response would be:

"You seem upset this morning."

Decorticate is toward the

'cord'. Decerebrit is the other way (out)

When giving Kayexalate we need to worry about dehydration because

( K ha ineverse relationship with Na)

myxedema

(Hypothyroidism) Deficiency of thyroid hormone; causes non-pitting edema or myxedema. Puffy, edematous face, coarse facial features, dry skin, and dry, coarse hair and eyebrows..decrease temp. DECREASE in all body functions.

To Prevent Dumping Syndrome: Care

(Post-Operatvie Ulcer/Stomach Surgeries) Care Includes: Eat in the reclining position, lie down after meals for 20-30 minutes. Also *restrict fluids during meals, LOW CHO and fiber Diet*, and small frequent meals.

Addison's Disease

(Remember: *DOWN, DOWN, DOWN, UP, DOWN*) - HYPOnatremia, HYPOtension, DECREASED blood volume, HYPERkalemia, and HYPOglycemia

After lumbar puncture?

(and also oil-based Myelogram)--> pt lies in flat supine (to prevent headache and leaking of CSF)

Long term use of amphogel

(binds to phosphates, increases Ca, robs the bones...leads to increased Ca resortion from bones and WEAK BONES)

Bethamethasone?

(celestone)=surfactant. Med for lung expansion.

After Supratentorial Surgery?

(incision behind hairline) --> elevate HOB 30-45 degrees

Temp conversion?

* To convert Centigrade to F... C × 9, ÷ 5 and + 32 * To convert Fahrenheit to C.... F - 32, × 5 ÷ 9

Nurses need to implement appropriate body mechanics in order to prevent injury to themselves and their clients. Which principle of body mechanics should the nurse incorporate into client care? 1. Flex the knees and keep the feet wide apart. 2. Assume a position far enough away from the client. 3. Twist the body in the direction of movement. 4. Use the strong back muscles for lifting or moving.

*1. Flex the knees and keep the feet wide apart.* The correct answer is to flex the knees and keep the feet wide apart. This will create a wide base of support, providing greater stability for the nurse and reducing the risk of back injury. The nurse should be positioned close to the client and use the arms and legs. Dividing balanced activity between arms and legs reduces the risk of back injury. Facing the direction of movement prevents abnormal twisting of the spine, also reducing the risk of back injury.

PSORIASIS

CHARACTERIZED BY PRESENCE OF SILVERY PLAQUES

Suicide :Theraputic Response:

*1st response identify clients psychosocial assessment (an accurate assessment so that the client's psychological and emotional needs are identified) *gain trust and permission from client, use a welcoming environment, make yourself completely tuned into the client *assess events leading up to suicidal thoughts *establish a human to human connection, non judgemental

The nurse instructs a woman about providing a clean catch urine specimen. Which of the following statements indicates that the patient correctly understands the procedure? 1) "I will be sure to urinate into the 'hat' you placed on the toilet seat." 2) "I will cleanse my genital area from front to back before I collect the specimen midstream." 3) "I will need to lie still while you put in a urinary catheter to obtain the specimen." 4) "I will collect my urine each time I urinate for the next 24 hours."

*2) "I will cleanse my genital area from front to back before I collect the specimen midstream."* To obtain a clean catch urine specimen, the nurse should instruct the patient to cleanse the genital area from front to back and collect the specimen midstream. This follows the principle of going from "clean" to "dirty." The nurse should have the ambulatory patient void into a "hat" (container for collecting the urine of an ambulatory patient) when monitoring urinary output, but not when obtaining a clean catch urine specimen. A urinary catheter is required for a sterile urine specimen, not a clean catch specimen. A 24-hour urine collection may be necessary to evaluate some disorders but a clean catch specimen is a one-time collection.

Which is the most commonly reported "incident" in hospitals? 1) Equipment malfunction 2) Patient falls 3) Laboratory specimen errors 4) Treatment delays

*2) Patient falls* Patient falls, usually in an attempt to go to the bathroom

The nurse knows that which of the following habits may interfere with a client's sleep? 1. Listening to classical music 2. Finishing office work 3. Reading novels 4. Drinking warm milk

*2. Finishing office work* At home a client should not try to finish office work or resolve family problems before bedtime. Noise should be kept to a minimum. Soft music may be used to mask noise if necessary. Reading a light novel, watching an enjoyable television program, or listening to music helps a person to relax. Relaxation exercises can be useful at bedtime. A dairy product snack such as warm milk or cocoa that contains L-tryptophan may be helpful in promoting sleep.

While undergoing a soapsuds enema, the client complains of abdominal cramping. The nurse should: 1. Immediately stop the infusion 2. Lower the height of the enema container 3. Advance the enema tubing 2 to 3 inches 4. Clamp the tubing

*2. Lower the height of the enema container*

The nurse is working with a client who has left-sided weakness. After instruction, the nurse observes the client ambulate in order to evaluate the use of the cane. Which action indicates that the client knows how to use the cane properly? 1. The client keeps the cane on the left side. 2. Two points of support are kept on the floor at all times. 3. There is a slight lean to the right when the client is walking. 4. After advancing the cane, the client moves the right leg forward.

*2. Two points of support are kept on the floor at all times.* Two points of support, such as both feet or one foot and the cane, should be on the floor at all times. The cane should be kept on the stronger side, the client's right side. The client should keep his or her body upright and midline. Leaning can cause the client to lose his or her balance and fall. After advancing the cane, the client should move the weaker leg, the client's left leg, forward to the cane.

The client needs to use crutches at home, and will have to manage going up and down a short flight of stairs. The nurse evaluates the use of an appropriate technique if the client: 1. Uses a banister or wall for support when descending 2. Uses one crutch for support while going up and down 3. Advances the crutches first to ascend the stairs 4. Advances the affected leg after moving the crutches to descend the stairs

*4. Advances the affected leg after moving the crutches to descend the stairs* To descend stairs, the crutches are placed on the stairs and the client moves the affected leg, then the unaffected leg to the stairs with the crutches. The client should continue to use the crutches for support, not the banister or wall. The client should continue to use both crutches for support when going up or down stairs. When ascending stairs, the client moves the unaffected leg up the stair, then the crutches and affected leg.

The unit manager is evaluating the care of a new nursing staff member. Which of the following is an appropriate technique for the nurse to implement in order to obtain a clean-voided urine specimen? 1. Apply sterile gloves for the procedure. 2. Restrict fluids before the specimen collection. 3. Place the specimen in a clean urinalysis container. 4. Collect the specimen after the initial stream of urine has passed.

*4. Collect the specimen after the initial stream of urine has passed.*

Which of the following information provided by the client's bed partner is most associated with sleep apnea? 1. Restlessness 2. Talking during sleep 3. Somnambulism 4. Excessive snoring

*4. Excessive snoring* Partners of clients with sleep apnea often complain that the client's snoring disturbs their sleep. Restlessness is not most associated with sleep apnea. Sleep talking is associated with sleep-wake transition disorders; somnambulism is associated with parasomnias (specifically, arousal disorders and sleep-wake transition disorders).

An ambulatory client is admitted to the extended care facility with a diagnosis of Alzheimer's disease. In using a falls assessment tool, the nurse knows that the greatest indicator of risk is: 1. Confusion 2. Impaired judgment 3. Sensory deficits 4. History of falls

*4. History of falls* According to the falls assessment tool, the greatest indicator of risk is a history of falls. According to the falls assessment tool, the second leading risk factor for falls is confusion. According to the falls assessment tool, impaired judgment is the fourth leading risk factor for falls. According to the falls assessment tool, sensory deficit is the fifth leading risk factor for falls.

The nurse determines that the nursing diagnosis stress urinary incontinence related to decreased pelvic muscle tone is the most appropriate for an oriented adult female client. A therapeutic nursing intervention based on this diagnosis is to: 1. Apply adult diapers 2. Catheterize the client 3. Administer Urecholine 4. Teach Kegel exercises

*4. Teach Kegel exercises*

Diabetic Ketoacidosis Causes (severe bloodsugar elevation)

*occurs when not enough circulating insulin *body forced to breakdown body fats for energy=build up of ketones. This bulild up of ketones turnes blood acidotic= coma/death *s/s - fatigue, frequent urination, increased thirst, fruity breath, nausea, vomiting, rapid breathing, muscle stiffness

To maintain proper posture, it is important to a. sleep on the softest mattress possible b. avoid arching shoulders forward when sitting c. keep your knees locked when standing upright d. keep your stomach muscles relaxed to prevent back spasms

*b. avoid arching shoulders forward when sitting* Arching shoulders forward when sitting alters the curvature of the spine and contributes to poor body alignment.

The term "Kussmaul" refers to a high-pitched, harsh, crowing inspiratory sound that occurs due to partial obstruction of the larynx. a. true b. false

*b. false* The term for this sound of respiratory distress is "stridor."

When developing a care plan for an older adult the nurse should consider which challenges faced by clients in this age group?

Adjusting to retirement, deaths of family members, and decreased physical strength

hyperpituitarism leads to:

Conditions such as Acromegaly and Cushing's Disease.

cognitive Disorders: Assisting a client who has dementia

*orient client to familiar or pleasurable activities *precautions for safety measures, wandering, eliminate hazards *monitor I&O, weight regularly *nonverbal communication increases, verbal decreases *communication should include social and historical events from earlier yrs of client's life, using repetition

Digoxin Toxicity

*DECREASED POTASSIUM levels increases digoxin toxicity. * s/s = nausea, vomiting, anorexia, bradycardia, depression, yellow halos in visual field, hyperkalemia, photophobia, diplopia

DO NOT delegate what you can...EAT (Acronym)

*E*valuate, *A*ssess, *T*each

Liver biopsy

Adm vit k , npo morning of exam 6hr, give sedative, Teach pt that he will be asked to hold breath for 5-10sec, supine postion, lateral with upper arms elevated. Post- postion on right side, frequent v.s., report severe ab pain stat, no heavy lifting 1wk.

Sources of Nutrition: Expected findings in a school age child with vitamin C deficiency:

*Edema, bronchial infections, fatigue, weakness, swollen/bleeding gums, nosebleeds, spontaneous pinpoint hemorrhages can be observed *stops healing of bone fractures, anemia

The nurse is assessing the confused client. In trying to determine the client's level of pain, the nurse should a. be aware that confused clients don't feel as much pain due to their confusion b. observe the client carefully for changes in behavior or vital signs c. ask the client's family how much pain the client normally has d. use only pain scales that feature numbers or "faces" the client can point to

*b. observe the client carefully for changes in behavior or vital signs* The nurse should observe the confused client for nonverbal cues to pain.

Immobilizing Interventions: prioritizing care for a client with a long arm cast

*use sling for arm/cast support, *keep arm raised above heart for first 48hrs *raise arm over head throughout day to prevent shoulder stiffness *monitor skin color and warmth of skin, keep dry and move fingers frenquently

Four nurses are inserting catheters in their clients. Which nurse's statement, related to this intervention, is incorrect? I am inserting this catheter to a. empty your bladder prior to your procedure b. treat your problem of leaking urine c. obtain a sterile urine specimen d. measure the amount of urine left after you emptied your bladder

*b. treat your problem of leaking urine* Insertion of a urinary catheter is not a "treatment" for incontinence. "Never event" by CMS - CAUTI

The nurse is assisting the client in caring for her ostomy. The client states, "Oh, this is so disgusting. I'll never be able to touch this thing." The nurse's best response is a. "I'm sure you will get used to taking care of it eventually." b. "Yes, it is pretty messy, so I'll take care of it for you today." c. "It sounds like you are really upset." d. "You sound very angry. Should I call the chaplain for you?"

*c. "It sounds like you are really upset."* This statement reflects the principles of therapeutic communication.

Which diagnostic test/exam would best measure a client's level of hypoxemia? a. chest x-ray b. pulse oximeter reading c. ABG d. peak expiratory flow rate

*c. ABG* The term "hypoxemia" means low blood oxygen level. Arterial blood gas sampling is the most direct way in which the level of oxygen in the blood can be measured.

Bipolar disorder: understanding the disease:

*Mania stage= irrational behavior, racing thoughts, acting extremely irresponsible, or participating in dangerous activities *flucuates between mania stage and depressed stage. *starts between ages 15-24, therapy revolves around solving the relationship between thoughts, behaviors, and emotions

The client has been on a low-protein diet. This will most likely affect which pharmacokinetic process? a. Absorption b. Excretion c. Distribution d. Metabolism

*c. Distribution* A low-protein diet may lead to an inadequate level of plasma proteins, which will affect availability of "free" drug.

Liver biopsy?

Adm vit k , npo morning of exam 6hr, give sedative, Teach pt that he will be asked to hold breath for 5-10sec, supine postion, lateral with upper arms elevated. Post- postion on right side, frequent v.s., report severe ab pain stat, no heavy lifting 1wk.

Menieres's disease?

Admin diuretics to decrease endolymph in the cochlea, restrict Na, lay on affected ear when in bed. Triad: 1)Vertigo 2)Tinnitus 3)N/V

AIRBORN Transmission Precautions: MTV

*M* = Measles *T* = Tuberculosis *V* = Varicella (Chicken Pox)/Herpes Zoster (Shingles) Remember: Private room w/ negative pressure w/ 6 - 12 air exchange/hr AND MASK. *N95* mask for TB.

CONTACT Precautions: MRS WEE

*M* = Multidrug Resistant Organism *R* = Respiratory Infection *S* = Skin Infections *W* = Wound Infections *E* = Enteric Infection (Clostridium Difficile) *E* = Eye Infections (Conjunctivitis)

NO VITAMIN C with

Allopurinol

WHITEHEADS

CLOSED COMEDONES ARE WHAT TYPE OF SKIN ERUPTION?

G tube feeding is done by

CNA

For which sleep disorder would the nurse most likely need to include safety measures in the client's plan of care? a. Snoring b. Enuresis c. Narcolepsy d. Hypersomnia

*c. Narcolepsy* Narcolepsy can occur suddenly during the daytime hours when a person is involved in any type of activity. This could put the person at risk for harm depending on the activity in which he is engaged.

Laparoscopy

CO2 used to enhances visual, general anesthesia, foley. Post- walk patient to decrease CO2 build up used for procedure

Laparoscopy-

CO2 used to enhances visual, general anesthesia, foley. Post- walk patient to decrease CO2 build up used for procedure.

The nurse is completing a head-to-toe assessment on her client at the beginning of the shift for the hospital unit. This would be considered a/an a. Focused assessment b. Initial assessment c. Ongoing assessment d. Special needs assessment

*c. Ongoing assessment* This type of assessment can be completed at any time after the initial assessment. Gathering data at the beginning of a shift will enable the nurse to more effectively evaluate how to proceed with the plan of care for the shift.

Laparoscopy?

CO2 used to enhances visual, general anesthesia, foley. Post- walk patient to decrease CO2 build up used for procedure.

PSORIASIS

COAL TAR SHAMPOO IS PRESCRIBED FOR WHAT TYPE OF SKIN CONDITION?

Of the following factors, which would put a client at greatest risk for impaired skin integrity? a. the medication digoxin b. moisture c. decreased sensation d. dehydration

*c. decreased sensation* Decreased sensation would greatly increase the risk for injury with a tear or break in the skin. This could lead to a delay in seeking treatment due to lack of awareness.

A client is prepared to receive elective cardioversion to treat atrial fibrillation

*Remove all oxygen from client to prevent fire/combustion

DROPLET Precautions: SPIDERMAN (Some letters are repeated consecutively)

*S* = Sepsis *S* = Scarlet Fever *S* = Streptococcal Pharyngitis *P* = Paravirus B19 *P* = Pneumonia *P* = Pertussis *I* = Influenza *D* = Diptheria (Pharyngeal) *E* = Epiglottitis *R* = Rubella *M* = Mumps *M* = Meningitis *M* = Mycoplasma or Meningeal Pneumonia *AN* = Adenovirus Remember: Private room or cohort and MASK

Of the following interventions for the client who is immobile, the nurse will give priority to a. encouraging a diet high in fiber and extra fluids b. administering the PRN medication for sleep c. having the client use his incentive spirometer q2hrs d. massaging the client's legs every hour

*c. having the client use his incentive spirometer q2hrs* Use of the incentive spirometer helps to prevent atelectasis, which improves oxygenation - a priority need.

Contact transmitted diseases acronym?

CONTACT PRECAUTION MRS.WEE M - multidrug resistant organism R - respiratory infection S - skin infections * W - wound infxn E - enteric infxn - clostridium difficile E - eye infxn - conjunctivitis

It is most important for the nurse to understand the various ways in which pain is classified a. so that he can document the client's pain using accurate terms b. so that he can be clear in his communication with the physician c. so that he can develop an effective pain management plan d. so that he can educate the client thoroughly

*c. so that he can develop an effective pain management plan* ANS: C Different modalities are used in the treatment/ management of pain and are often based on how the pain is classified (e.g., acute vs. chronic).

The primary provider has written a medication prescription. The nurse is having difficulty deciphering what has been written. The best strategy to clarify the information is a. Ask the patient what medication the provider prescribed. b. Call the pharmacist and ask her to read the prescription. c. Ask the nurse who knows the provider's handwriting to read the prescription. d. Call the provider and ask him to clarify the prescription.

*d. Call the provider and ask him to clarify the prescription.* All other answers increase the risk of a medication error.

There is a 24-hour urine collection in process for a client. The NAP inadvertently empties one specimen into the toilet instead of the collection "hat." The nurse should a. Continue with the collection of urine until the 24-hour time period is finished. b. Make a note to the lab to inform them that one specimen was missed during the collection. c. Begin filling a new collection container and take both containers to the lab at the end of the collection period. d. Dispose of the urine already collected and begin an entirely new 24-hour collection.

*d. Dispose of the urine already collected and begin an entirely new 24-hour collection.* Once one specimen is "missed" during a 24-hour urine collection, the results of the lab test will be inaccurate and the collection must be restarted.

Substance Abuse: Use of Support Groups

*active participation= better chance of maintaining abstinence, first step is formal treatment, than mutual support groups

SKIN Infections: V CHIPS

*V* = Varicella *C* = Cutaneous Diptheria *H* = Herepes Zoster *I* = Impetigo *P* = Pediculosis *S* = Scabies

Iron Deficiency Anemia: Foods that aid in Iron Absorption:

*VITAMIN C aids in absorption- take at meal time with iron enriched foods. (Vitamin C = citrus fruits, strawberries, green leafy veges, bell peppers, califlower) *AVOID calcium supplements, black or green tea, or coffee *+++ oysters, prune juice, walnuts, chickpeas, beans, peanuts, lentils, peas+++

Identify the true statement about devices used when assisting clients to ambulate. a. The client should stand a foot back from the back legs of a walker. b. A cane should be used by the client to support the weakest side of the body. c. A transfer belt should be placed around the client's chest for maximum " lift." d. Each crutch-walking "gait" begins with the client in the tripod position.

*d. Each crutch-walking "gait" begins with the client in the The tripod position is the basic crutch standing position from which the client then moves forward.

Use of the statements "Tell me more about..." or "I see" encourage clients to continue talking and expressing themselves. This is called: a. Summarizing b. Open-ended questions c. Focusing d. Encouraging elaboration

*d. Encouraging elaboration*

The client calls the nurse to the room and states, "Look, my incision is popping open where they did my hip surgery!" The nurse notes that the wound edges have separated 1 cm at the center and there is straw-colored fluid leaking from one end. The nurse's best action is to a. Notify the surgeon STAT. b. Place a clean, sterile 4 x 4 over the incision and monitor the drainage. c. Wrap an ace bandage firmly around the area and have the client maintain bedrest. d. Immediately cover the wound with sterile towels soaked in normal saline and call the surgeon.

*b. Place a clean, sterile 4 x 4 over the incision and monitor the drainage.* A 1 cm separation of wound edges only in the center of a surgical incision on the hip is too small to truly be termed dehiscence. Even if there were a large separation, there are no "internal viscera" to protrude.

In caring for a client with a tracheostomy, the nurse would give priority to the nursing diagnosis of a. Risk for ineffective airway clearance b. Anxiety related to suctioning c. Social isolation related to altered body image d. Impaired tissue integrity

*a. Risk for ineffective airway clearance* While other diagnoses may be applicable, maintaining a patent airway by tending to excessive secretions is a priority.

The client has a draining abdominal wound that has become infected. In caring for the client, the nurse will implement a. contact precautions b. droplet precautions c. no precautions d. airborne precautions

*a. contact precautions* Contact precautions are used when "contact" with the infected drainage could lead to transmission of the infection.

The female client states to the nurse, "I'm so distressed. It seems like every time I laugh hard, I wet myself." The nurse knows that this condition is known as a. stress incontinence b. urge incontinence c. functional incontinence d. unconscious incontinence

*a. stress incontinence* Stress incontinence results from increased pressure within the abdominal cavity.

Mr. Zenobia's chronic cancer pain has recently increased, and he asks the home health nurse what can be done. In relationship to his long-acting morphine, which of the following is an appropriate response by the nurse? a. "If you take more morphine, it will not change your pain relief." b. "I'll call the physician and ask for an increased dose." c. "The amount you are taking now is all I can give you." d. "I'm worried if we increase your dose that you will stop breathing."

*b. "I'll call the physician and ask for an increased dose."* There is no ceiling on the analgesic effect of opioid narcotics. Patients develop a tolerance to the effects, which often necessitates an increase in the dose.

When gathering admission assessment data the nurse obtains a weight of 200 pounds. The client states, "I've never weighed that much!" The nurse should a. Explain to the client how weight gain occurs b. Check the calibration and re-weigh the client c. Document the weight as 200 pounds d. Instruct the UAP to re-weigh the client in 2 hours

*b. Check the calibration and re-weigh the client* It is important to FIRST validate data when there is a mismatch between what the client states as history and the data obtained. Validating data often includes ensuring that equipment is functioning properly first.

The nurse is teaching the client about his upcoming procedure and the client is very stressed. It would be most important for the nurse to a. Use humor first to decrease the client's stress level b. Determine if the teaching should take place at a different time c. Introduce himself as the RN to give credibility to his message d. Speak to the client when family members are there so they can teach the client

*b. Determine if the teaching should take place at a different time* Clients who are stressed may be unable to listen fully and will not receive/understand the intended message.

When administering a drug via a parenteral routes, the drug would be absorbed fastest if given per the IM route. a. True b. False

*b. False* Absorption refers to the "movement" of the drug from the site of administration into the blood stream. Therefore, the intravenous, parenteral route leads to "instant" absorption.

Light sleep and slowing brain and body processes are associated with which stage of NREM sleep? a. I b. II c. III d. IV

*b. II* These are characteristics of a person in Stage II of NREM sleep.

Which of the following actions violates a principle that is key to proper hand washing at the bedside? a. Washing your hands for 1 minute b. Shaking your hands dry over the sink c. Using warm, not very hot water d. Using the soap provided by the agency

*b. Shaking your hands dry over the sink* Shaking your hands will not completely remove the excess moisture, allowing for the reacquisition of bacteria on the area.

During the communication process, "decoding" is a. The selection of words by the sender b. The interpretation of the message by the receiver c. The method by which the message is given d. The way in which feedback is interpreted

*b. The interpretation of the message by the receiver*

Mrs. Addie is 70 years old. While the nurse is gathering admission assessment data, the patient states, "I've taken a tablespoon of Milk of Magnesia every day for 3 years." Which nursing diagnosis is most appropriate for the nurse to use in her plan of care? a. Diarrhea b. Constipation c. Risk for Ineffective Therapeutic Regimen d. Perceived Constipation

*d. Perceived Constipation* Daily laxative use by the patient might suggest that she perceives she is constipated, and the nurse would gather further assessment data related to the client's bowel pattern. There is not enough data to infer actual constipation.

The nurse knows that the results of a fecal occult blood test can be inaccurate if a. the client has had an excessive intake of red meat b. the female client is menstruating c. the client takes high doses of vitamin C d. all of the above

*d. all of the above*

Nursing Care of Newborn: Safety precautions

*main focus- monitor VS, keep baby warm -baby's respirations may not stabilize for about 2hrs after birth, at this point its abnormal for respiratory rate to be greater than 60 breaths per min. -leave kelly clamp on till after plastic umbilical cord clamp has been applied -remove secretions from mouth& nose with gloved finger or sm. syringe

Diabetes Mellitus: Reinforce teaching to a school age child:

*most common type I insulin dependent *a multidose basal-bolus insulin program works well for most= glougine at HS, paired with aspart insulin and using insulin algorithms as a guide for insulin food-activity decisions 4-5x a day. *frequent BG monitors

Reinforcing Teaching for External Radiation Therapy:

*obtain inform consent *inform client not to scrub areas of body marked with ink, so treatment can be applied to same area *high protein, high carb, low-residue diet *avoid heat pads, skin creams, keep skin dry, wear loose clothing *administer antidiarrheal medication (for pelvic radiation)

postpartum client has developed thrombophlebitis, what should nurse do?

*place patient in trendelenburgs position. * To prevent thrombophlebitis, get client ambulating early!!

Fractures: Preventing complications while in skeletal Traction:

*provide pin care, keep crusting around pins to a minimum to prevent infection and unnecessary pulling of skin.( Osteomyelitis most serious complication associated) *Keep hydrated, follow plan to prevent constipation *ROM to unaffected extremities, breathing excercises

Cataracts: Postoperative discharge teaching

*wear eye shield at all times for at least 1-2 weeks *wash eyelid with cotton pad and warm water, sleep on unoperative side *do not put head below waist 2-3 days, do not get constipated * some mucos and tears is normal, some irritation and scratchyness is normal

EEG,

, hold meds for 24-48 hrs prior, no caffine or cigarettes for 24 hrs prior, pt can eat, pt must stay awake night before exam, pt may be asked to hyperventilate and watch a bright flashing light, after EEG, assess pt for seizures, pt's will be at increased risk

Maslow for a guy who lost his house in a fire?

- A guy loses his house in a fire. Priority is using community resources to find shelter, before assisting with feelings about the tremendous loss. (Maslow).

Autonomic Dysreflexia/Hyperflexia: S&S and Care

- Acutely, this is a reaction of the autonomic (involuntary) nervous system to overstimulation (due to lesions or injury of the spinal cord) S&S Include: Pounding headache, profuse sweating, nasal congestion, goose flesh, bradycardia, *Hypertension* Care Includes: Place the client in *Sitting Position* (elevate HOB) FIRST before any other implementation.

Liver biopsy

- Adm vit k , npo morning of exam 6hr, give sedative, Teach pt that he will be asked to hold breath for 5-10sec, supine postion, lateral with upper arms elevated. Post- postion on right side, frequent v.s., report severe ab pain stat, no heavy lifting 1wk.

MENINGITIS -

- Kernig's sign (leg flex then leg pain on extension), Brudzinski sign (neck flex = lower leg flex).

Myelogram

- Npo 4-6hr, allergy hx, phenothiazines, cns depressants, and stimulants withheld 48hr prior, table will be moved to various postions during test. Post- neuro q2-4, water soluble HOB up, oil soluble HOB down, oral analgesics for h/a, encourage po fluids, assess for distended bladder, inspect site

Rifampin

- Red orange tears and urine, also contraceptives don't work as well

CHICKEN POX

- Vesicular Rash (central to distal) dew drop on rose petal

EMPHYSEMA

- barrel chest

Hypertension: Adverse Effects of Captopril (Capoten)

- captopril is an ACE inhibitor used for the treatment of hypertension -cough being the most common ADR(due to elevated levels of bradykinin) -rash and taste disturbances (metallic or loss of taste) -short half-life necessitates 2-3 times daily dosing -postural hypotension ,Hyperkalemia

Cullen's sign

- ecchymosis in umbilical area, seen with pancreatitis

TETANY -

- hypocalcemia (+) Trousseau's sign/carpopedal spasm; Chvostek sign (facial spasm).

sickle cell crisis

- intense pain because blood vessels can become blocked or the defective red blood cells can damage organs in the body. HYDRATE

Tardive Dyskinesia

- irreversible - involuntary movements of the tongue, face and extremities, may happen after prolonged use of antipsychotics

Head Injury: Providing Care for a client with Increased Intracranial Pressure

- monitor v/s closely, accurately and periodically -maintain airway, if client is not intubated place client on side to decrease possibility of airway occulusion -watch for coughing when suctioning=can create seizure activity -administer medications(corticosteroids, dilantin, antibiotics) _Restrict fluids, and elevate bed to 30 degrees -maintain normal body temperature

Akathisia

- motor restlessness, need to keep going, tx with antiparkinsons meds, can be mistaken for agitation

APPENDICITIS

- rebound tenderness

PERNICIOUS ANEMIA

- red beefy tongue.

LIVER CIRRHOSIS

- spider like varices.

MALARIA

- stepladder like fever with chills

Shilling Test

- test for pernicious anemia/ how well one absorbs Vit b12

Greek heritage ( after baby is born)

- they put an amulet or any other use of protective charms around their baby's neck to avoid "evil eye" or envy of others

Hemovac

- used after mastectomy, empty when full or q8hr, remove plug, empty contents, place on flat surface, cleanse opening and plug with alcohol sponge, compress evacuator completely to remove air, release plug, check system for operation.

cerebrovascular accident: priority Interventions

-A CT scan will determine if the client is having a stroke(hemorrhagic or ischemic accident) -if a stroke was determined to be hemorrhagic, rt-PA is contraindicated -The time of onset of a stroke to t-PA administration is critical. Administration within 3 hours -Controlling the blood pressure is critical, why= intracerebral hemorrhage is the major adverse effect of thrombolytic therapy. - crucial to monitor pupil size & pupillary response to indicate changes around the cranial nerves

Care of the older Adult Client: Temperature changes

-Diminished ability to sense temp changes may occur with aging, check water w/ wrist before immursing older adults in it. -older adults don't bathe as frequent due to diminished perspiration -normal temp for elderly 36 deg.c (normal temp of young adult would be possibly a fever to an elderly adult

Decorticate and Decerebrate?

Decorticate positioning in response to pain = Cortex involvement. Decerebrate in response to pain = Cerebellar, brain stem involvement

Assessment & management of Newborn complications: Hyperbilirubinemia:

-Jaundice (yellow discoloration of skin, whites of the eyes) first in the face. s/s- hard to wake up infant, shrill cry, irritable, arches neck or body backwards -treatment= phototherapy & encourage feeding

Methylzanthines: Adverse Effects of Theophylline (Theo-Dur)

-Methylxanthines are known to stimulate peptic acid secretion -caution in patients with active peptic ulcerdisease. -cautiously in patients with impaired renal function -Severe Potential Hazard= Seizures, Head Injury, Cerebral Vascular Disorder,Gastroesophageal Reflux Disease, -Theophylline is removed by hemodialysis, so don't give to dialysis pt -Toxicity is most likely to occur when levels exceed 20 mcg/mL.

NMS?

-NMS is like S&M; -you get hot (hyperpyrexia) -stiff (increased muscle tone) -sweaty (diaphoresis) -BP, pulse, and respirations go up & -you start to drool

Diabetic Ketoacidosis: Discharge Teaching

-Signs and symptoms may include fast, deep breathing, increased urination, fruity-smelling breath, or weakness and confusion. The high sugar level, ketones, and acid can cause you to become very sick. High sugar levels also can lead to dehydration (loss of body fluids and salts)

Leukemia:providing oral Hygiene

-The mouth is especially affected because its cells are replaced at a high rate. Chemotherapy and radiation therapy affect the ability of cells to divide - which creates big problems for the normal tissue repair process in the mouth. -Oral mucositis may appear from 4 to 7 days after the initiation of chemotherapy -Fungal (candida) infections of the oral mucosa -spontaneous oral bleeding when the platelet count goes below 20,000 -hypomature enamel.

Cataract

Decrease ICP, check for sudden pain means hemorrhage, sleep on unaffected side

mood stabilizers: Reinforcing Teaching about Taking Lithium Carbonate (Eskalith)

-antimania drug used to treat schizoaffective disorder -increase risk of seizure -amount of salt in body influences the effects of medicine, this can increase salt loss from body -drink lots of fluids, avoid caffeine, don't get overheated -take after meals, No to low salt diet -#1 side affect= blurred vision confusion

Ergonomic Principles: Reinforcing Teaching about the use of a Standartized Walker

-check for correct height. Have client stand inside of walker with arms at their sides. Walker handles should be even with client's wrists -used with client's that have unilateral or bilateral weakness or an inability to bear weight on one leg -it requires arm strength and balance

Abdomen: Auscultating Bowel sounds

-dilates with gas and fluid, the bowel sounds become high pitched -Vascular bruits are the audible manifestation of turbulent blood flow( organ moves against the serosal surface), swishing sounds -rubs are the result of inflamed peritoneal surfaces grating on each other during respiration.(usually rare, but found norm over the liver or spleen) -absence of bowel sounds indicators of intra-abdominal infection

Newborn Assessment:Head

-general apperance, facial bruising is caused by birth trauma and should be noted. -norm shape- flattened over forhead, raises to a point -norm circumference- between 33& 38 cm -Fontanelle size- the posterior fontanelle can be already closed at birth= NOT ABNORMAL, should feel soft yet spongy -sunken fontanelle= dehydration -over riding suture lines is a normal finding -widely spaced suture lines=RED FLAG

Hemodialysis and Peritoneal Dialysis: Monitoring for complications

-hemo= monitor for hemmorraging, hepatitis, air embolism, sepis, muscle cramps. - peritoneal= monitor for fever, abdl tenderness, n&v, peritonitis (cloudy outflow,bleeding) -Also with peritoneal, increase fiber to prevent constipation, and weigh before and after

newborn abnormal cries at birth

-high pitch cry= abnormal -to stimulate a cry- rub soles of feet. Trendelenburg or side laying position

vascular Access: Treating IV infiltration

-infusing fluids stopped immediately and the cannula removed. -elevating the edematous limb or at least making sure the site is non-dependent and no pressure is applied -Pain control measures may be appropriate

Meeting the nutritional needs of newborns: Formula feeding

-once removed from fridge, needs to be used or thrown with in 2hrs. -once feeding occurs, any remaining formula should be tossed 1hr after start of feeding. -Do not put in fridge or heat up to reuse

Care of newborn: Purpose of prophylactic eye care:

-purpose fights against gonorrhea conjunctivitis or opthalmia neonatorum, which can be passed through the vaginal canal through delivery. -Its applied over lower lids of both eyes, than manipulate eyelids to spread medication over eyes -silver nitrate or erythromycin (most popular) are used

FIVE INTERVENTIONS FOR PSYCH PATIENTS

-safety -setting limits -establish trusting relationship -meds -least restrictive methods/environment

Nutrition for infants: introducing solid foods

-should start at about 6months,some start as early as 4months

Kawasaki disease causes?

a heart problem, but what specifically? Coronary artery aneurysms d/t the inflammation of blood vessels.

Tracheostomy Care

-tie new trache tie before removing old tie to prevent accidental dislodgment -use precut guaze -soak iiner cannula in antiseptic soak with hydrogen peroxide, rinse well

Fetal alcohol sydrome?

-upturned nose -flat nasal bridge -thin upper lip -SGA

Fetal alcohol syndrome

-upturned nose -flat nasal bridge -thin upper lip -SGA

Incentive Spirometry steps?

1) Sit upright 2) Exhale 3) Insert mouthpiece 4) Inhale for 3 seconds, and then HOLD for 10 seconds

Blood transfusion- sign of allergies in order:

1)Flank pain 2)Frequent swallowing 3)Rashes 4)Fever 5)Chills

Thrombocyopenia- bleeding precautions

1)Soft bristled toothbrush 2)No insertion of anything! (c/i suppositories, douche)

142.) A health care provider has written a prescription for ranitidine (Zantac), once daily. The nurse should schedule the medication for which of the following times? 1. At bedtime 2. After lunch 3. With supper 4. Before breakfast

1. At bedtime Rationale: A single daily dose of ranitidine is usually scheduled to be given at bedtime. This allows for a prolonged effect, and the greatest protection of the gastric mucosa. **recall that ranitidine suppresses secretions of gastric acids**

72.) Cinoxacin (Cinobac), a urinary antiseptic, is prescribed for the client. The nurse reviews the client's medical record and should contact the health care provider (HCP) regarding which documented finding to verify the prescription? Refer to chart. 1. Renal insufficiency 2. Chest x-ray: normal 3. Blood glucose, 102 mg/dL 4. Folic acid (vitamin B6) 0.5 mg, orally daily

1. Renal insufficiency Rationale: Cinoxacin should be administered with caution in clients with renal impairment. The dosage should be reduced, and failure to do so could result in accumulation of cinoxacin to toxic levels. Therefore the nurse would verify the prescription if the client had a documented history of renal insufficiency. The laboratory and diagnostic test results are normal findings. Folic acid (vitamin B6) may be prescribed for a client with renal insufficiency to prevent anemia.

149.) A client taking fexofenadine (Allegra) is scheduled for allergy skin testing and tells the nurse in the health care provider's office that a dose was taken this morning. The nurse determines that: 1. The client should reschedule the appointment. 2. A lower dose of allergen will need to be injected. 3. A higher dose of allergen will need to be injected. 4. The client should have the skin test read a day later than usual.

1. The client should reschedule the appointment. Rationale: Fexofenadine is an antihistamine, which provides relief of symptoms caused by allergy. Antihistamines should be discontinued for at least 3 days (72 hours) before allergy skin testing to avoid false-negative readings. This client should have the appointment rescheduled for 3 days after discontinuing the medication.

Priortizing care for a client experiencing anaphylatic shock

1. if faintness- lie pt. down, if difficult breathing or swelling in throat, keep pt sitting up. 2. Ensure pt does not choke or vomit, so place client in recovery position. 3.get help promptly 4.shot of adrenaline into muscle side of thigh 5.Intravenous fluids given 6. 6-12 hrs observation

normal magnesium range

1.5-2.5mEq/L

6.) The burn client is receiving treatments of topical mafenide acetate (Sulfamylon) to the site of injury. The nurse monitors the client, knowing that which of the following indicates that a systemic effect has occurred? 1.Hyperventilation 2.Elevated blood pressure 3.Local pain at the burn site 4.Local rash at the burn site

1.Hyperventilation Rationale: Mafenide acetate is a carbonic anhydrase inhibitor and can suppress renal excretion of acid, thereby causing acidosis. Clients receiving this treatment should be monitored for signs of an acid-base imbalance (hyperventilation). If this occurs, the medication should be discontinued for 1 to 2 days. Options 3 and 4 describe local rather than systemic effects. An elevated blood pressure may be expected from the pain that occurs with a burn injury.

By dopler you can hear the fetus by

12 weeks

normal Sodium range

136-145mEq/L

A client undergoes hypersensitivity testing with the intradermal technique. When the nurse administers the allergen, what should the angle of the needle be?

15 degrees

Stroke brain attack

15- 30 elevate the head, Eschimic stroke give thrombolitics, orient the patient, ROM, good skin care, feed on unaffected side, avoid mild because milk increase risk of aspiration cause milk high consistency fluid

Platelets normal levels

150,000-450,000.

Kidney Glucose threshold is

180

Burn Degrees?

1st Degree - Red and Painful 2nd Degree - Blisters 3rd Degree - No Pain because of blocked and burned nerves

What is nageles Rule to determine expected due date?

1st day of last period, than subtract 3 months, and than add 7 days, than add 1 year

176.) A nurse notes that a client is taking lansoprazole (Prevacid). On data collection, the nurse asks which question to determine medication effectiveness? 1. "Has your appetite increased?" 2. "Are you experiencing any heartburn?" 3. "Do you have any problems with vision?" 4. "Do you experience any leg pain when walking?"

2. "Are you experiencing any heartburn?" Rationale: Lansoprazole is a gastric acid pump inhibitor used to treat gastric and duodenal ulcers, erosive esophagitis, and hypersecretory conditions. It also is used to treat gastroesophageal reflux disease (GERD). It is not used to treat visual problems, problems with appetite, or leg pain. **NOTE: "-zole" refers to gastric acid pump inhibitors**

172.) A nurse provides dietary instructions to a client who will be taking warfarin sodium (Coumadin). The nurse tells the client to avoid which food item? 1. Grapes 2. Spinach 3. Watermelon 4. Cottage cheese

2. Spinach Rationale: Warfarin sodium is an anticoagulant. Anticoagulant medications act by antagonizing the action of vitamin K, which is needed for clotting. When a client is taking an anticoagulant, foods high in vitamin K often are omitted from the diet. Vitamin K-rich foods include green, leafy vegetables, fish, liver, coffee, and tea.

74.) A client with myasthenia gravis is receiving pyridostigmine (Mestinon). The nurse monitors for signs and symptoms of cholinergic crisis caused by overdose of the medication. The nurse checks the medication supply to ensure that which medication is available for administration if a cholinergic crisis occurs? 1. Vitamin K 2. Atropine sulfate 3. Protamine sulfate 4. Acetylcysteine (Mucomyst)

2. Atropine sulfate Rationale: The antidote for cholinergic crisis is atropine sulfate. Vitamin K is the antidote for warfarin (Coumadin). Protamine sulfate is the antidote for heparin, and acetylcysteine (Mucomyst) is the antidote for acetaminophen (Tylenol).

9.) The nurse is applying a topical corticosteroid to a client with eczema. The nurse would monitor for the potential for increased systemic absorption of the medication if the medication were being applied to which of the following body areas? 1. Back 2. Axilla 3. Soles of the feet 4. Palms of the hands

2. Axilla Rationale: Topical corticosteroids can be absorbed into the systemic circulation. Absorption is higher from regions where the skin is especially permeable (scalp, axilla, face, eyelids, neck, perineum, genitalia), and lower from regions in which permeability is poor (back, palms, soles).

108.) A nurse is performing a follow-up teaching session with a client discharged 1 month ago who is taking fluoxetine (Prozac). What information would be important for the nurse to gather regarding the adverse effects related to the medication? 1. Cardiovascular symptoms 2. Gastrointestinal dysfunctions 3. Problems with mouth dryness 4. Problems with excessive sweating

2. Gastrointestinal dysfunctions Rationale: The most common adverse effects related to fluoxetine include central nervous system (CNS) and gastrointestinal (GI) system dysfunction. This medication affects the GI system by causing nausea and vomiting, cramping, and diarrhea. Options 1, 3, and 4 are not adverse effects of this medication.

136.) A nurse performs an admission assessment on a client who visits a health care clinic for the first time. The client tells the nurse that propylthiouracil (PTU) is taken daily. The nurse continues to collect data from the client, suspecting that the client has a history of: 1. Myxedema 2. Graves' disease 3. Addison's disease 4. Cushing's syndrome

2. Graves' disease Rationale: PTU inhibits thyroid hormone synthesis and is used to treat hyperthyroidism, or Graves' disease. Myxedema indicates hypothyroidism. Cushing's syndrome and Addison's disease are disorders related to adrenal function.

65.) A nurse is reinforcing discharge instructions to a client receiving sulfisoxazole. Which of the following should be included in the list of instructions? 1. Restrict fluid intake. 2. Maintain a high fluid intake. 3. If the urine turns dark brown, call the health care provider (HCP) immediately. 4. Decrease the dosage when symptoms are improving to prevent an allergic response.

2. Maintain a high fluid intake. Rationale: Each dose of sulfisoxazole should be administered with a full glass of water, and the client should maintain a high fluid intake. The medication is more soluble in alkaline urine. The client should not be instructed to taper or discontinue the dose. Some forms of sulfisoxazole cause urine to turn dark brown or red. This does not indicate the need to notify the HCP.

168.) Colcrys (colchicine) is prescribed for a client with a diagnosis of gout. The nurse reviews the client's medical history in the health record, knowing that the medication would be contraindicated in which disorder? 1. Myxedema 2. Renal failure 3. Hypothyroidism 4. Diabetes mellitus

2. Renal failure Rationale: Colchicine is contraindicated in clients with severe gastrointestinal, renal, hepatic or cardiac disorders, or with blood dyscrasias. Clients with impaired renal function may exhibit myopathy and neuropathy manifested as generalized weakness. This medication should be used with caution in clients with impaired hepatic function, older clients, and debilitated clients. **Note that options 1, 3, and 4 are all endocrine-related disorders: Myxedema=Hypothyroidism**

126.) A nurse is caring for a client who has been prescribed furosemide (Lasix) and is monitoring for adverse effects associated with this medication. Which of the following should the nurse recognize as a potential adverse effect Select all that apply. 1. Nausea 2. Tinnitus 3. Hypotension 4. Hypokalemia 5. Photosensitivity 6. Increased urinary frequency

2. Tinnitus 3. Hypotension 4. Hypokalemia Rationale: Furosemide is a loop diuretic; therefore, an expected effect is increased urinary frequency. Nausea is a frequent side effect, not an adverse effect. Photosensitivity is an occasional side effect. Adverse effects include tinnitus (ototoxicity), hypotension, and hypokalemia and occur as a result of sudden volume depletion.

55.) A client who is receiving digoxin (Lanoxin) daily has a serum potassium level of 3.0 mEq/L and is complaining of anorexia. A health care provider prescribes a digoxin level to rule out digoxin toxicity. A nurse checks the results, knowing that which of the following is the therapeutic serum level (range) for digoxin? 1. 3 to 5 ng/mL 2. 0.5 to 2 ng/mL 3. 1.2 to 2.8 ng/mL 4. 3.5 to 5.5 ng/mL

2.) 0.5 to 2 ng/mL Rationale: Therapeutic levels for digoxin range from 0.5 to 2 ng/mL. Therefore, options 1, 3, and 4 are incorrect.

The physician prescribes the following preoperative medications to a client for I.M. administration: meperidine (Demerol), 50 mg; hydroxyzine (Vistaril), 25 mg; and glycopyrrolate (Robinul), 0.3 mg. The medications are dispensed as follows: meperidine, 100 mg/ml; hydroxyzine, 100 mg/2 ml; and glycopyrrolate, 0.2 mg/ml. How many milliliters, in total, should the nurse administer?

2.5 ml

The client is to receive an I.V. infusion of 3,000 ml of dextrose and normal saline solution over 24 hours. The nurse observes that the rate is 150 ml/hour. If the solution runs continuously at this rate, the infusion will be completed in:

20 hours.

pathological jaundice= occurs before _______and the last ____ days.

24hrs and 7 days

profound mental retardation

25 and less

severe retardation

25-40

Rhogam : given at _____ and ____ postpartum, medication route ___ Only given to _________ mother Also if indirect Coomb's test is positive, don't need to give _____cuz she has antibody only give if negative coombs

28 weeks and 72 hrs IM Rh NEGATIVE Rhogam

Common S/S of epiglottitis?

3 D'S --Drooling, Dysphonia, Dysphagia

106.) Fluoxetine (Prozac) is prescribed for the client. The nurse reinforces instructions to the client regarding the administration of the medication. Which statement by the client indicates an understanding about administration of the medication? 1. "I should take the medication with my evening meal." 2. "I should take the medication at noon with an antacid." 3. "I should take the medication in the morning when I first arise." 4. "I should take the medication right before bedtime with a snack."

3. "I should take the medication in the morning when I first arise." Rationale: Fluoxetine hydrochloride is administered in the early morning without consideration to meals. **Eliminate options 1, 2, and 4 because they are comparable or alike and indicate taking the medication with an antacid or food.**

197.) Collagenase (Santyl) is prescribed for a client with a severe burn to the hand. The nurse provides instructions to the client regarding the use of the medication. Which statement by the client indicates an accurate understanding of the use of this medication? 1. "I will apply the ointment once a day and leave it open to the air." 2. "I will apply the ointment twice a day and leave it open to the air." 3. "I will apply the ointment once a day and cover it with a sterile dressing." 4. "I will apply the ointment at bedtime and in the morning and cover it with a sterile dressing."

3. "I will apply the ointment once a day and cover it with a sterile dressing." Rationale: Collagenase is used to promote debridement of dermal lesions and severe burns. It is usually applied once daily and covered with a sterile dressing.

Autonomic Dysreflexia?

Autonomic dysreflexia: potentially life threatening emergency - elevate head of bed to 90 degree - loosen constrictive clothing - assess for bladder distention and bowel impaction (triger) - Administer antihypertensive meds (may cause stroke, MI, seisure )

1) A nurse is caring for a client with hyperparathyroidism and notes that the client's serum calcium level is 13 mg/dL. Which medication should the nurse prepare to administer as prescribed to the client? 1. Calcium chloride 2. Calcium gluconate 3. Calcitonin (Miacalcin) 4. Large doses of vitamin D

3. Calcitonin (Miacalcin) Rationale: The normal serum calcium level is 8.6 to 10.0 mg/dL. This client is experiencing hypercalcemia. Calcium gluconate and calcium chloride are medications used for the treatment of tetany, which occurs as a result of acute hypocalcemia. In hypercalcemia, large doses of vitamin D need to be avoided. Calcitonin, a thyroid hormone, decreases the plasma calcium level by inhibiting bone resorption and lowering the serum calcium concentration.

After A Infratentorial Surgery (Incision at the nape of the Neck): Care

Care Includes: Position the patient *FLAT* and lateral on either side.

130.) The nurse is analyzing the laboratory results of a client with leukemia who has received a regimen of chemotherapy. Which laboratory value would the nurse specifically note as a result of the massive cell destruction that occurred from the chemotherapy? 1. Anemia 2. Decreased platelets 3. Increased uric acid level 4. Decreased leukocyte count

3. Increased uric acid level Rationale: Hyperuricemia is especially common following treatment for leukemias and lymphomas because chemotherapy results in a massive cell kill. Although options 1, 2, and 4 also may be noted, an increased uric acid level is related specifically to cell destruction.

210.) Dantrolene (Dantrium) is prescribed for a client with a spinal cord injury for discomfort resulting from spasticity. The nurse tells the client about the importance of follow-up and the need for which blood study? 1. Creatinine level 2. Sedimentation rate 3. Liver function studies 4. White blood cell count

3. Liver function studies Rationale: Dantrolene can cause liver damage, and the nurse should monitor liver function studies. Baseline liver function studies are done before therapy starts, and regular liver function studies are performed throughout therapy. Dantrolene is discontinued if no relief of spasticity is achieved in 6 weeks.

59.) A client is diagnosed with an acute myocardial infarction and is receiving tissue plasminogen activator, alteplase (Activase, tPA). Which action is a priority nursing intervention? 1. Monitor for renal failure. 2. Monitor psychosocial status. 3. Monitor for signs of bleeding. 4. Have heparin sodium available.

3. Monitor for signs of bleeding. Rationale: Tissue plasminogen activator is a thrombolytic. Hemorrhage is a complication of any type of thrombolytic medication. The client is monitored for bleeding. Monitoring for renal failure and monitoring the client's psychosocial status are important but are not the most critical interventions. Heparin is given after thrombolytic therapy, but the question is not asking about follow-up medications.

After a Lumbar Puncture (also an Oil Based Myelogram): Care

Care Includes: patient lies *FLAT/SUPINE* (This is done to prevent headache and leaking of CSF)

58.) Isosorbide mononitrate (Imdur) is prescribed for a client with angina pectoris. The client tells the nurse that the medication is causing a chronic headache. The nurse appropriately suggests that the client: 1. Cut the dose in half. 2. Discontinue the medication. 3. Take the medication with food. 4. Contact the health care provider (HCP).

3. Take the medication with food. Rationale: Isosorbide mononitrate is an antianginal medication. Headache is a frequent side effect of isosorbide mononitrate and usually disappears during continued therapy. If a headache occurs during therapy, the client should be instructed to take the medication with food or meals. It is not necessary to contact the HCP unless the headaches persist with therapy. It is not appropriate to instruct the client to discontinue therapy or adjust the dosages.

103.) A nurse is caring for a hospitalized client who has been taking clozapine (Clozaril) for the treatment of a schizophrenic disorder. Which laboratory study prescribed for the client will the nurse specifically review to monitor for an adverse effect associated with the use of this medication? 1. Platelet count 2. Cholesterol level 3. White blood cell count 4. Blood urea nitrogen level

3. White blood cell count Rationale: Hematological reactions can occur in the client taking clozapine and include agranulocytosis and mild leukopenia. The white blood cell count should be checked before initiating treatment and should be monitored closely during the use of this medication. The client should also be monitored for signs indicating agranulocytosis, which may include sore throat, malaise, and fever. Options 1, 2, and 4 are unrelated to this medication.

From the ass? From the Mouth? Metobolic

From the a** (diarrhea)= metabolic acidosis From the mouth (vomitus)=metabolic alkalosis

The nurse assesses a client who is complaining of frequent episodes of epistaxis. The nurse knows the client has:

nosebleeds

albumin

3.5-5.0, best indicator of long term nutritional status

normal range for potassium

3.5-5.3 mEq/L

Which of the following groups of clients is at an increased risk for developing a wound infection?

Clients who are undernourished

EPIGLOTITIS

3Ds' Drooling, Dysphonia, Dysphagia

4.) The camp nurse asks the children preparing to swim in the lake if they have applied sunscreen. The nurse reminds the children that chemical sunscreens are most effective when applied: 1. Immediately before swimming 2. 15 minutes before exposure to the sun 3. Immediately before exposure to the sun 4. At least 30 minutes before exposure to the sun

4. At least 30 minutes before exposure to the sun Rationale: Sunscreens are most effective when applied at least 30 minutes before exposure to the sun so that they can penetrate the skin. All sunscreens should be reapplied after swimming or sweating.

76.) Carbidopa-levodopa (Sinemet) is prescribed for a client with Parkinson's disease, and the nurse monitors the client for adverse reactions to the medication. Which of the following indicates that the client is experiencing an adverse reaction? 1. Pruritus 2. Tachycardia 3. Hypertension 4. Impaired voluntary movements

4. Impaired voluntary movements Rationale: Dyskinesia and impaired voluntary movement may occur with high levodopa dosages. Nausea, anorexia, dizziness, orthostatic hypotension, bradycardia, and akinesia (the temporary muscle weakness that lasts 1 minute to 1 hour, also known as the "on-off phenomenon") are frequent side effects of the medication.

Common S/S Duchennes Muscular Dystrophy?

Gower's sign (use of the hands to push ones self from the floor_) Looks EXACTLY like the Pilates Downward dog!!!!!!!

200.) A client is seen in the clinic for complaints of skin itchiness that has been persistent over the past several weeks. Following data collection, it has been determined that the client has scabies. Lindane is prescribed, and the nurse is asked to provide instructions to the client regarding the use of the medication. The nurse tells the client to: 1. Apply a thick layer of cream to the entire body. 2. Apply the cream as prescribed for 2 days in a row. 3. Apply to the entire body and scalp, excluding the face. 4. Leave the cream on for 8 to 12 hours and then remove by washing.

4. Leave the cream on for 8 to 12 hours and then remove by washing. Rationale: Lindane is applied in a thin layer to the entire body below the head. No more than 30 g (1 oz) should be used. The medication is removed by washing 8 to 12 hours later. Usually, only one application is required.

36.) The client has a PRN prescription for ondansetron (Zofran). For which condition should this medication be administered to the postoperative client? 1. Paralytic ileus 2. Incisional pain 3. Urinary retention 4. Nausea and vomiting

4. Nausea and vomiting Rationale: Ondansetron is an antiemetic used to treat postoperative nausea and vomiting, as well as nausea and vomiting associated with chemotherapy. The other options are incorrect.

143.) A client has just taken a dose of trimethobenzamide (Tigan). The nurse plans to monitor this client for relief of: 1. Heartburn 2. Constipation 3. Abdominal pain 4. Nausea and vomiting

4. Nausea and vomiting Rationale: Trimethobenzamide is an antiemetic agent used in the treatment of nausea and vomiting. The other options are incorrect.

17.) The client with ovarian cancer is being treated with vincristine (Oncovin). The nurse monitors the client, knowing that which of the following indicates a side effect specific to this medication? 1. Diarrhea 2. Hair loss 3. Chest pain 4. Numbness and tingling in the fingers and toes

4. Numbness and tingling in the fingers and toes Rationale: A side effect specific to vincristine is peripheral neuropathy, which occurs in almost every client. Peripheral neuropathy can be manifested as numbness and tingling in the fingers and toes. Depression of the Achilles tendon reflex may be the first clinical sign indicating peripheral neuropathy. Constipation rather than diarrhea is most likely to occur with this medication, although diarrhea may occur occasionally. Hair loss occurs with nearly all the antineoplastic medications. Chest pain is unrelated to this medication.

184.) A nurse preparing a client for surgery reviews the client's medication record. The client is to be nothing per mouth (NPO) after midnight. Which of the following medications, if noted on the client's record, should the nurse question? 1. Cyclobenzaprine (Flexeril) 2. Alendronate (Fosamax) 3. Allopurinol (Zyloprim) 4. Prednisone

4. Prednisone Rationale: Prednisone is a corticosteroid that can cause adrenal atrophy, which reduces the body's ability to withstand stress. Before and during surgery, dosages may be temporarily increased. Cyclobenzaprine is a skeletal muscle relaxant. Alendronate is a bone-resorption inhibitor. Allopurinol is an antigout medication.

144.) A client is taking docusate sodium (Colace). The nurse monitors which of the following to determine whether the client is having a therapeutic effect from this medication? 1. Abdominal pain 2. Reduction in steatorrhea 3. Hematest-negative stools 4. Regular bowel movements

4. Regular bowel movements Rationale: Docusate sodium is a stool softener that promotes the absorption of water into the stool, producing a softer consistency of stool. The intended effect is relief or prevention of constipation. The medication does not relieve abdominal pain, stop gastrointestinal (GI) bleeding, or decrease the amount of fat in the stools.

66.) Trimethoprim-sulfamethoxazole (TMP-SMZ) is prescribed for a client. A nurse should instruct the client to report which symptom if it developed during the course of this medication therapy? 1. Nausea 2. Diarrhea 3. Headache 4. Sore throat

4. Sore throat Rationale: Clients taking trimethoprim-sulfamethoxazole (TMP-SMZ) should be informed about early signs of blood disorders that can occur from this medication. These include sore throat, fever, and pallor, and the client should be instructed to notify the health care provider if these symptoms occur. The other options do not require health care provider notification.

Best time to take meds

Growth Hormone PM, Steroids AM, Diuretics AM, Aricept AM.

normal calcium range

4.5-5.5 mEq/l

185.) Which of the following herbal therapies would be prescribed for its use as an antispasmodic? Select all that apply. 1.Aloe 2.Kava 3.Ginger 4.Chamomile 5.Peppermint oil

4.Chamomile 5.Peppermint oil Rationale: Chamomile has a mild sedative effect and acts as an antispasmodic and anti-inflammatory. Peppermint oil acts as an antispasmodic and is used for irritable bowel syndrome. Topical aloe promotes wound healing. Aloe taken orally acts as a laxative. Kava has an anxiolytic, sedative, and analgesic effect. Ginger is effective in relieving nausea.

TEF

4Cs' Coughing, Choking, Cyanosis, Continous Drooling

TEF -

4Cs' Coughing, Choking, Cyanosis, Continous Drooling

APETM

5 areas of listening to heart

Huntington's Chorea

50% genetic, autosomal dominant disorder S/S: chorea --> writhing, twisting, movements of face, limbs and body -gait deteriorates to no ambulation -no cure, just palliative care

Huntington's Chorea?

50% genetic, autosomal dominant disorder S/S: chorea --> writhing, twisting, movements of face, limbs and body -gait deteriorates to no ambulation -no cure, just palliative care

mild mental ratardation

55-70

A client is scheduled for an excretory urography at 10 a.m. An order states to insert a saline lock I.V. device at 9:30 a.m. The client requests a local anesthetic for the I.V. procedure and the physician orders lidocaine and prilocaine cream (EMLA cream). The nurse should apply the cream at

7:30 a.m.

AST lab

8-20, assess Liver damage

Regurgitation pressure in trachea

80,120

ICP (intracranial pressure) should be _____. and measure ________.

< 2 and measure head circumference.

AMMONIA DERMATITIS

A BED RIDDEN INCONTINENT PATIENT COMPLAINING OF PRURITIS AND EXCESSIVE DRYNESS IN LOWER BACK MAY BE INDICATIVE OF?

How to detect female cancers using Breast Self Examinations

A BSE should not be conducted around the same time of menstrual cycle. *lay flat on back, using opposite hand to breast with arm raised, use 3 middle fingers in circular motion.

ventricular tachycardia

A RAPID HEART RHYTHM IN WHICH THE ELECTRICAL IMPULSE BEGINS IN THE VENTRICLE (INSTEAD OF THE ATRIUM), WHICH MAY RESULT IN INADEQUATE BLOOD FLOW AND EVENTUALLY DETERIORATE INTO CARDIAC ARREST. administer lidocaine

addison's disease

A condition caused by having low levels of Corticoids / Corticosteroids (steroid hormones produces by the Adrenal Cortex). Causes weakness, weight loss, increases pigmentation of skin

Addison's disease

A condition caused by having low levels of Corticoids / Corticosteroids (steroid hormones produces by the Adrenal Cortex). Causes weakness, weight loss, increases pigmentation of skin. Will need steroid replacement forever.

Hyperthyroidism

A condition that occurs as a result of excessive thyroid hormone secretion.

spina bifida

A congenital defect of the spine in which part of the spinal cord and its meninges are exposed through a gap in the backbone. It often causes permanent paralysis of the lower limbs, and sometimes a mental handicap.

varicella

A contagious viral disease characterized by fever, headache, and a crop of red spots that become mscules, papules, vesicles, and crusts; also called chickenpox

myasthenia gravis

A disease in which acetylcholine receptors on muscle cells are destroyed so that muscles can no longer respond to the acetylcholine signal to contract. Symptoms include muscular weakness and progressively more common bouts of fatigue. The disease's cause is unknown but is more common in females than in males; it usually strikes between the ages of 20 and 50. ptosis, diplopia

respiratory distress

A form of ___ characterized by gasping, labored breathing, or dyspnea, SOB, , increased PaCo2, decreased PaO2

cystic fibrosis

A genetic disorder that occurs in people with two copies (BOTH PARENTS) of a certain recessive allele; characterized by an excessive secretion of mucus and consequent vulnerability to infection; fatal if untreated.

CUSHING'S SYNDROME

A group of signs and symptoms associated with hypersecretion of the glucocorticoids by the adrenal cortex. Prone to CHF secondary to Increased sodium

PKU

A human metabolic disease caused by a mutation in a gene coding for a phenylalanine processing enzyme (phenylalanine hydroxylase), which leads to accumulation of phenylalanine and mental retardation if not treated; inherited as an autosomal recessive phenotype.

Myxedema coma

A rare but serious disorder that results from persistantly low thyroid production. Coma can be precipitated by acute illness, rapid withdrawal of thyroid medication, anesthesia and surgery, hypothermia, and the use of sedatives and opioid analgesics.

What is considered adequate urinary output for perfusion of kidney, heart, and brain?

A rate of 30ml/hr is considered adequate.

Type 2 diabetes mellitus

A relative lack of insulin or resistance to the action of insulin; usually, insulin is sufficient to stabilize fat and protein metabolism but not carbohydrate metabolism.

delirium tremens

A severe reaction that can be part of alcohol withdrawl, characterized by sweating, trembling, anxiety, and hallucinations

A. Placenta previa B.Placenta abruption WHo has pain? who has bleeding?

A. there is no pain, there is bleeding B. pain, but no bleeding

Apgar Scoring?

A= appearance (color all pink, pink and blue, blue [pale]) P= pulse (>100, < 100, absent) G= grimace (cough, grimace, no response) A= activity (flexed, flaccid, limp) R= respirations (strong cry, weak cry, absent)

**Acid Ash diet - ** Alk Ash diet-

ACID- cheese, corn, cranberries, plums, prunes, meat, poultry, pastry, bread ALK-milk, veggies, rhubarb, salmon

D.I.

ADH/Vasopressin (pitressin) (DDAVP)

Ritalin

ADHD..assess for heart related side effects report immediately. ..child may need a drug holiday b/c it stunts growth

Lumbar Puncture?

AFTER the procedure, the client should be placed in the supine position for 4 to 12 hrs as prescribed. (Saunders 3rd ed p. 229)

Disseminated Herpes Zoster is __________ as to Localized Herpes Zoster is _____. A nurse with a localized herpes zoster CAN care for patients as long as the patients_______ immunosuppressed and the lesions must be covered.

AIRBORNE PRECAUTIONS CONTACT PRECAUTIONS are NOT

ITCHING

ANTIHISTAMINES ARE USEFUL TO HELP RELIEVE...?

A client with terminal breast cancer is being cared for by a longtime friend who is a physician. The client has identified her twin sister as the agent in her durable power of attorney. The client loses decision-making capacity, and the twin sister says to the nurse, "There will be a different physician caring for my sister now. I've dismissed her friend." In response, the nurse should:

Abide by the wishes of the sister who is the durable power of attorney agent.

galactorrhea

Abnormal production and secretion of milk from the beasts or any white discharge from the nipple. can be caused by Haldo.

Anorexia sucks because?

Absence of menstruation leads to osteoporosis in the anorexic.

Hypothalamus

Activates, controls, and integrates the peripheral autonomic nervous system, endocrine processes, and many somatic functions, such as body temperature, sleep, and appetite.

APGAR: A (2nd A)

Activity - flexed/flaccid/limp

Adrenal gland disorders

Adrenal cortex: Addison's disease, Primary hyperaldosteronism (Conn's syndrome), Cushing's disease, and Cushing's syndrome. Adrenal Medulla: Pheochromocytoma

Pancreatitis prioritys?

After pain relief, cough and deep breathe is important because of fluid pushing up in the diaphragm.

Risk factors for Endocrine Disorders

Age, Heredity, Congenital factors, Trauma, Environmental factors, Consequence of other disorders.

For NG tube

Always check placement with X rays, gastric content should be less than 4, if residual is more than 100 stop feeding

Room air is?

Ambient air contains 21 % O2

Assesing heart sounds

American People Eat Too Much

Nitrazine paper?

Amniotic fluid is alkaline, and turns paper blue. Urine and normal vaginal discharge are acidic, and turn it pink.

Meconium stained protocol?

Amniotic fluid yellow with particles = meconium stained`

ALS?

Amyotrophic lateral sclerosis a condition in which there is a degeneration of motor neurons in both the upper & lower motor neuron systems.

What to do if your patients chest tube accidently getes removed?

An occulsive dressing is used

barium enema

An x-ray exam using an opaque contrast medium to examine the lower GI tract

Hep B vaccine always ask?

Anaphylactic reaction to baker's yeast is contraindication for Hep B vaccine.

Grey Turners sign?

Purple bruises around the belly button... Pancreatitis!

Pituitary Gland Disorders

Anterior Pituitary: hyperpituitarism and hypopituitarism Posterior Pituitary: Diabetes insipidus and Syndrome of inappropriate antidiuretic hormone (SIADH)

Biperiden(akineton)

Anti-Parkinson Drugs - Anticholinergics

carafate

Anti-ulcer. Give this drug 60 minutes before meals and at HS. Is a liquid and is given to coat the esophagus, protecting it from erosion by gastric acid.

valium

Antianxiety Agent, monitor for restlessness and tachycardia

ketocanazole

Antifungal, brand name Nizoral, treat candidisis

gemfibrozil

Antihyperlipidemic, montior hepatic function

Stelazine

Antipsychotics

Following transsphenoidal hypophysectomy, monitor for:

Any postnasal drip or nasal drainage, which might indicate leakage of cerebrospinal fluid (check nasal drainage for glucose).

Apgar scores/scoring?

Apgar measures HR,RR,Muscle tone, Reflexes,Skin color each 0-2 point. 8-10 OK. 0-3 RESUSCITATE.

APGAR: A (1st A)

Appearance - color pink/pink and blue/blue [pale]

Hyperthryroidism

Increased metabolic rate, anxiety, and extreme nervousness

Koplik's spots

Are small blue-white spots visible on the oral mucosa and are characteristic of measles infection.

Common S/S of guillian Barre Syndrome

Ascending muscle paralysis.. dont confuse with MG

Before giving MMR>?

Ask for anaphylactic rxn to eggs or neomycin before

A client hasn't voided since before surgery, which took place 8 hours ago. Which action should the nurse do first?

Assess the client for bladder fullness.

When preparing a client with a draining vertical incision for ambulation, where should the nurse apply the thickest portion of a dressing?

At the base of the wound Rationale: When a client is ambulating, gravity causes the drainage to flow downward. Covering the base of the wound with extra dressing will contain the drainage. Applying the thickest portion of the dressing at the top, in the middle, or over the total wound won't contain the drainage.

Neuromuscular Blocking Agents: Effects of Atropine

Ataxia, coma, confusion, dizziness, HA, headache, insomnia, tachycardia, hypotension, pulmonary edema, delayed gastric emptying

Huntington's

Autonomic dominant disease, Aspiration and muscle degeneration problem. Genetic testig is very important

Air/Pulmonary Embolism: S&S and Care

S&S Include: chest pain, difficulty breathing, tachycardia, pale/cyanotic, sense of impending doom. Care: turn patient to the *LEFT SIDE* and *LOWER* the head of the bed

A multigavid client in labor at 38 weeks' gestation has been diagnosed with Rh sensitization and probably fetal hydrops and anemia. When the nurse observes the fetal heart rate pattern on the monitor, which of the following patterns is most likely? a. early deceleration pattern b. sinusoidal pattern c. variable deceleration pattern d. late deceleration pattern

B - The fetal heart rate of a multipara diagnosed with Rh sensitization and probably fetal hydrops and anemia will most likely demonstrate a sinusoidal pattern that resembles a sine wave. It has been hypothesized that this pattern reflects an absence of autonomic nervous control over the fetal heart rate resulting from severe hypoxia. This client will most likely requires a cesarean delivery to improve the fetal outcome. Early decelerations are associated with cord compression; and late deceleration are associated with poor placental perfusion

The nurse is making an occupied bed. Arrange the following steps in the order the nurse should perform them. A. Position the patient laterally near the side rail farthest from you (that side rail is up); roll the soiled linens under him. B. Lower the side rail on the side of the bed you are working on. C. Raise the side rail on the side of the bed you are working on. D. After placing clean linens and tucking them under the soiled linens, roll the patient over the "hump" and position him facing you on the near side of the bed.

B. Lower the side rail on the side of the bed you are working on. A. Position the patient laterally near the side rail farthest from you (that side rail is up); roll the soiled linens under him. D. After placing clean linens and tucking them under the soiled linens, roll the patient over the "hump" and position him facing you on the near side of the bed. C. Raise the side rail on the side of the bed you are working on. First lower the side rail on your side of the bed. This allows you to maintain good body mechanics while positioning the patient (in step 1). Position patient laterally near far side rail, and roll soiled linens under him. Then place clean linens on the side nearest you, and tuck them under soiled linens. Next, roll the patient over the "hump," and position him on his other side, facing you. Do this before raising the near side rail so you do not have to reach across the side rail to help the patient roll and turn to his other side.

When the nurse walks into the patient's room, she notices fire coming from the patient's trash can. Rank the following actions in the order they should be performed by the nurse. A. Activate the fire alarm. B. Move the patient out of the room. C. Close all doors and windows. D. Put out the fire using the proper extinguisher.

B. Move the patient out of the room. A. Activate the fire alarm. C. Close all doors and windows. D. Put out the fire using the proper extinguisher. R.A.C.E. - rescue, alarm, contain, and exstinquish or evacuate

Common S/S of Lyme's disease

Bull's eye rash

Common S/S of Cystitis?

Burning on urination

Common S/S of SLE?

Butterfly rashes

hypertension =

B/P above 140/90

Somogyi phenomenon

B/S DOWN @ 2-3AM-THEN UP high in morning tx: snack @ bedtime, NPH, Lente

Dawn Phenomenon

B/S high @ 5-8 am tx: NPH, Lente @10pm b4 bed

EXCESSIVE DRYNESS

BENZOYL PEROXIDE MAY CAUSE...

Cushings ulcers r/t

BRAIN injury

DM during illness

BS increases, even if they don't eat, check BS every 3, 4 houres

Hypoglycemia

BS less than 50

Most accurate way to test kids for medication accuracy?

BSA is considered the most accurate method for medication dosing with kids. (I though it was weight, but apparently not)

The nurse has been teaching a client about a high-protein diet. The teaching is successful if the client identifies which of the following meals as high in protein?

Baked beans, hamburger, and milk

Behavior/Developmental-Peds

Behavior motivated by need to avoid anxiety and satisfy needs 1. Infancy 0-18 months others will satisy needs 2. childhood >6yrs learn to delay need gratification 3. juvenile 6-9 years learn to relate to peers 4. preadolescence 9-12 yrs learns to relate to friends of of opposite sex 5. early adolescence12-14yrs:learn independence and how to relate to opposite sex 6. late adolecence 14-21yrs: develop intimate relationship with person of opposite sex is this not about communication?....

Peds weight ?

Birth weight doubles by 6 month and triple by 1 year of age.

During a meal, a client with hepatitis B dislodges her I.V. line and bleeds on the surface of the over-the-bed table. It would be most appropriate for the nurse to instruct a housekeeper to clean the table with:

Bleach.

A client has a nursing diagnosis of Risk for injury related to adverse effects of potassium-wasting diuretics. What's the correctly written client outcome for this nursing diagnosis?

By discharge, the client correctly identifies three potassium-rich foods. Rationale: A client outcome must be measurable, concise, realistic for the client, and obtainable through nursing management. For each client outcome, the nurse should include only one client behavior, should express that behavior in terms of client expectations, and should indicate a time frame. Knowing the importance of consuming potassium-rich foods and knowing which foods are high in potassium aren't measurable. Understanding all complications isn't measurable or specific to the nursing diagnosis listed.

A 40-year-old executive who was unexpectedly laid off from work 2 days earlier complains of fatigue and an inability to cope. He admits drinking excessively over the previous 48 hours. This behavior is an example of: a. alcoholism b. manic episode c. situational crisis d. depression

C - A situational crisis results from a specific event in the life of a person who is overhwlemed by the situation and reacts emotional. Fatigue, insomnia, and inability to make decisions are common signs and symtpoms. The situations crisis may precipitate behavior that causes a criss (alcohol or drug abuse). There isn't enough information to label this client an alcoholic. A manic episodes is characterized by euphoria and labile effect. Symptoms of depression are usually present for 2 or more weeks.

COAL (cane walking):

C - cane O - opposite A - affected L - leg

DM

polyuria, polydypsia, polyphagia

Kussmaul respirations

Deep, rapid breathing; usually the result of an accumulation of certain acids when insulin is not available in the body. signs of Hyperglycemia.may give skim milk

Hypocalemia?

CATS - convulsions, arrhythmias, tetany, spasms and stridor

Pain medication for Pancreantitis

With this, give *DEMEROL* not Morphine Sulfate.

Halo

CSF celebral fluid, can explain to you if it contains sugar in it

Definitive diagnosis for abd. aortic aneurysm (AAA) -

CT scan

RETINO BLASTOMA -

Cat's eye reflex (grayish discoloration of pupils)

Common S.S of Retino Blastoma?

Cat's eye reflex (grayish discoloration of the pupil)

measles infection

Characterized by Koplik's spots (blue-white spots visible on the oral mucosa) and a body rash that typically begins on the face and travels downward.

trigeminal neuralgia

Characterized by severe lightning-like facial pain due to an inflammation of the fifth cranial nerve. These sudden, intense, brief attacks of sharp pain affect the cheek, lips, and gums only on the side of the face innervated by the affected nerve. eat soft warm food

Common S/S of MS>?

Charcot's Triad (IAN)

MS

Charcot's Triad (IAN)

The nurse is about to administer a medication to a client with whom the nurse is unfamiliar. To verify the client's identity, the nurse should:

Check the client's identification bracelet.

Bryant's traction=

Children <3 yr, <35 lbs with femur fx

Common S/S of hypocalcemia?

Chvostek and Trosseaus sign! Also hypomag!

Chvostek and Trosseaus sign

Chvostek= Tapping on the face above the cheek bone will cause spams! Trosseaus= using a BP cuff and inflating it, and it will cause spasms of the wrist

Cause of Ascites?

portal hypotension + albuminemia

Complication of chest tubes

Constant bubling in water seal means air leak, tube becomes disconnected from drainage means cuff of contaminated tip, inserts sterile one and reinsert, tubes becomes disconnected put in sterile water.

Rheumatoid arthritis

Contracture deformities, nodules, fever and rash

Infant with Cleft lip?

position on back or in infant seat to prevent trauma to suture line. While feeding, hold in upright position.

Which action by the nurse might be a barrier to obtaining complete and reliable information from an interview with the client? a. Noting that the client's body language indicates that he or she is fatigued b. Maintaining eye contact with the client if it is not culturally inappropriate to do so c. Carefully guiding the conversation so that important topics are discussed d. Asking the client directly, "Why are you not taking your insulin?"

Correct answer: D During the client interview avoid asking "why" questions. Not only do they suggest disapproval, but also many clients do not know "why" they do or do not comply with therapy.

Acute media Ottitis

position on side of affected ear to promote drainage, tubes will fall of in 6 month

Cushing ulcers and cushings triad?

Cushings ulcers r/t BRAIN injury Cushings triad r/t ICP in BRAIN (htn, bradycard, irr. resp)

Anthrax

Cutaneous skin contact lesion on skin inhalation resp failure, hemorrhage, no person to person contact

Cyanotic CH abnormalities

Cyanosis, clubbing of the fingers, seizure, difficult eating

After appendectomy?

position on the RIGHT side with legs flexed

Asthmas and wheezers?

Coughing w/o other s/s is suggestive of asthma. Speaking of asthma, watch out if your wheezer stops wheezing. It could mean he is worsening.

Vit K is to Protamine Sulfate is to Ca Glu is to Mucomyst is to Amicar is to

Coumadin Heparin MgSO4 TPA

Hyperkalemia can be treated with the administration of 50% dextrose and insulin. The 50% dextrose

Counteracts the effects of insulin. Rationale: The 50% dextrose is given to counteract the effects of insulin. Insulin drives potassium into the cell, thereby lowering serum potassium levels. The dextrose doesn't directly cause the potassium excretion or any movement of potassium.

ANGINA

Crushing stubbing pain relieved by NTG

ANGINA -

Crushing stubbing pain relieved by NTG

MI

Crushing stubbing pain which radiates to left shoulder, neck, arms, unrelieved by NTG

PANCREATITIS

Cullen's sign (ecchymosis of umbilicus); (+) Grey turners spots.

Common S/S of pancreatitis?

Cullens sign ( ecchymosis of umbilicus) (+) Grey Turner spots

What is obtained before starting any iv antibiotic?

Cultures

The nurse is taking the health history of an 85-year-old client. Which information will be most useful to the nurse for planning care?

Current health promotion activities

Prednisone toxicity

Cushing ....buffalo hump, moon face, high blood sugar, HTN

While assessing the fundus of a multiparous client on the first postpartum day, the nurse performs hand washing and dons clean gloves. Which of the following should the nurse do next? a. place the non-dominant hand above the symphysis pubis and the dominant hand at the umbilicus b. ask the client to assume a side-lying position with the knees flexed c. perform massage vigorously at the level of the umbilicus if the fundus feels boggy d. place the client on a bedpan in case the uterine palpation stimulates the client to void

D - The nurse should place the non-dominant hand above the symphysis pubis and the dominant hand at the umbilicus to palpate the fundus. This prevents the uterine inversion and trauma, which can be very painful to the client. The nurse should ask the client to assume a supine, not side-lying, position with the knees flexed. The fundus can be palpated in this position and the perineal pads can be evaluated for lochia amounts. The fundus should be massaged gently if the fundus feels boggy. Vigorous massaging may fatigue the uterus and cause it to become firm and then boggy again. The nurse should ask the client to void before fundal evaluation. A full bladder can cause discomfort to the client, the uterus to be deviated to one side, and postpartum hemorrhage.

Enema positioning?

position pt in left side-lying (Sim's) with knee flexed

Increased perfusion for VEINS and ARTERIES

Ele*V*ate *V*eins (*Think V,V*) and D*A*ngle *A*rteries. (*Think A,A*)

When administering an enema, list the following steps in the order in which they should be performed. Label the steps from 1 to 6, with 1 being the first step to perform. A. Document the results of the procedure. B. Assess the patient for cramping. C. Insert the tubing about 3 to 4 inches into the rectum. D. Lubricate the tip of the enema tubing generously. E. Raise the container to the correct height and instill the solution at a slow rate. F. Encourage the patient to hold the solution for 3 to 15 minutes, depending on the type of enema.

D. Lubricate the tip of the enema tubing generously. C. Insert the tubing about 3 to 4 inches into the rectum. E. Raise the container to the correct height and instill the solution at a slow rate. B. Assess the patient for cramping. F. Encourage the patient to hold the solution for 3 to 15 minutes, depending on the type of enema. A. Document the results of the procedure. You must lubricate the tip before inserting the tubing. You would then begin instilling the solution before assessing for cramping that the instillation might produce. Only after the solution is instilled would you ask the patient to hold the solution. The last action is to document the results of the procedure, after the procedure is finished.

Risus Sardonicus?

a highly characteristic, abnormal, sustained spasm of the facial muscles that appears to produce grinning--- From Tetany! Sad and scary looking!

Intracavitary radiation for cancer of the cervix, what is # 1 risk

DEHYDRATION

Delegation Rule of Thumb?

DO NOT delegate what you can EAT! E - evaluate A - assess T - teach

Droplet transmitted diseases acronym?

DROPLET think of SPIDERMAN! S - sepsis S - scarlet fever S - streptococcal pharyngitis P - parvovirus B19 P - pneumonia P - pertussis I - influenza D - diptheria (pharyngeal) E - epiglottitis R - rubella M - mumps M - meningitis M - mycoplasma or meningeal pneumonia An - Adenovirus

A client is receiving an I.V. infusion of dextrose 5% in water and lactated Ringer's solution at 125 ml/hour to treat a fluid volume deficit. Which of these signs indicates a need for additional I.V. fluids?

Dark amber urine

The physician has ordered a wet-to-dry dressing for an infected pressure ulcer. The nurse knows that the primary reason for this treatment is to accomplish which action?

Debriding the wound

What's the most appropriate nursing diagnosis for the client with acute pancreatitis?

Deficient fluid volume

How to Dx a AAA?

Definitive diagnosis for abd. aortic aneurysm (AAA) --> CT scan

Common S/S of MG?

Descending muscle weakness

Developmental milestones?

Developmental 2-3 months: turns head side to side 4-5 months: grasps, switch & roll 6-7 months: sit at 6 and waves bye-bye 8-9 months: stands straight at eight 10-11 months: belly to butt (phrase has 10 letters) 12-13 months: twelve and up, drink from a cup

A client is receiving 125 ml/hour of continuous I.V. fluid therapy. The nurse examines the venipuncture site and finds it red and swollen. Which of the following interventions would the nurse perform first?

Discontinue the infusion.

What could cause bronchopulmonary dysplasia?

Dysplasia means abnormality or alteration. Mechanical ventilation can cause it. Premature newborns with immature lungs are ventilated and over time it damages the lungs. Other causes could be infection, pneumonia, or other conditions that cause inflammation or scarring.

burn to face

EMERGENCY

** SIGNS of a Fractured hip

EXTERNAL ROTATION, SHORTENING, ADDUCTION

Signs of a hip fracture?

EXTERNAL ROTATION, SHORTENING, ADDUCTION

Tuberculosis: Recognizing signs and symptoms

Early sign= slight cough with expectoration of mucus, fever, night sweats Late sign= bloody productive cough, chest pain, dyspnea, weight loss

Hyperglycemia

Elevated blood glucose level greater than 250 mg/dL.

Using Maslow's hierarchy of human needs, the nurse assigns highest priority to which client need?

Elimination

A client is treated in the emergency department for a severe reaction to a bee sting. Which drug should the nurse instruct the client to carry in the future to prevent an anaphylactic reaction from bee stings?

Epinephrine Rationale: Epinephrine is the drug of choice for anaphylactic reactions that occur at home because after proper instruction, the client or a family member can administer the drug I.M. or S.C. to counteract the hypotensive effects of histamine. All of the other options are inappropriate.

A client who has recently begun menopause expresses dejection about having no children despite having been pregnant three times. In planning the client's care, it would be most therapeutic if the nurse focuses on which need?

Esteem

A client is to be discharged from an acute care facility after treatment of right leg thrombophlebitis. The nurse notes that the client's leg is pain-free, without redness or edema. The nurse's actions reflect which step in the nursing process?

Evaluation

Hyperglycemic hyperosmolar nonketotic syndrome

Extreme hyperglycemia without acidosis. A complication of type 2 diabetes mellitus, which may result in dehydration or vascular collapse but does not include the acidosis component of diabetic ketoacidosis. Onset is usually slow, taking from hours to days.

TOPICAL RETINOIDS

FIRST LINE THERAPY FOR ACNE CONSISTS OF....

SQ TISSUE

FULL THICKNESS BURNS INVOLVES DESTRUCTION DOWN TO THE.....?

Which client characteristic would be an example of noncompliance?

Failure to progress

What can also cause an s3 heart sound?

Fluid volume overload caused by IVC fluids infusing too quickly (or whatever reason) and CHF

Chemotherapy Agents: Interactions with Tamoxifen (Nolvadex)

Fluoxetine=Don't use warfarin, halperidol, benadryl, bupropion, sertaline, carbamazepine

The nurse must apply an elastic bandage to a client's ankle and calf. She should apply the bandage beginning at the client's:

Foot Rationale: An elastic bandage should be applied from the distal area to the proximal area. This method promotes venous return. In this case, the nurse should begin applying the bandage at the client's foot. Beginning at the ankle, lower thigh, or knee doesn't promote venous return.

Blood typing?

For blood types: "O" is the universal donor (remember "o" in donor) "AB" is the universal receipient

For cord compression in OB?

For cord compression, place the mother in the TRENDELENBERG position because this removes pressure of the presenting part off the cord. (If her head is down, the baby is no longer being pulled out of hte body by gravity) If the cord is prolapsed, cover it with sterile saline gauze to prevent drying of the cord and to minimize infection.

amphojel

Gerd & kidney stones, watch out for constipation

Anaphylactic reaction to baker's yeast is contraindication for

Hep B vaccine.

Disease precautions?

Hepatitis A is contact precautions Tetanus, Hepatitis B, HIV are STANDARD precautions

Hodgkins disease?

Hodgkin's disease= cancer of lymph is very curable in early stage.

DVT

Homan's Sign

Before Epidural?

Hydration is a big priority!

Hyper reflexive? Absent reflexsive?

Hyper reflexes (upper motor neuron issue "your reflexes are over the top") Absent reflexes (lower motor neuron issue)

Other S/S of MS?

Hyperactive deep tendon reflexes, vision changes, fatigue and spasticity

Addison's disease

Hyposecretion of adrenal cortex hormones (glucocorticoids and mineralocorticoids) from the adrenal gland, resulting In deficiency of the corticosteroid hormones. The condition is fatal if left untreated.

Penis Problems?

Hypospadias: abnormality in which urethral meatus is located on the ventral (back) surface of the penis anywhere from the corona to the perineum (remember hypo, low (for lower side or under side) Epispadias: opening of the urethra on the dorsal (front) surface of the penis Priapism: painful erection lasting longer than 6 hrs.

Major risks of epidural?

Hypotension and bradypnea/bradycardia are major risks and emergencies.

What will alter the accuracy of o2 sats?

Hypotension and vasoconstricting meds

If your patient starts seeing bugs?

If your normally lucid patient starts seeing bugs you better check his respiratory status first. The first sign of hypoxia is restlessness, followed by agitation, and things go downhill from there all the way to delirium, hallucinations, and coma. So check the o2 stat, and get abg's if possible.

CPR in a five year old?

In a five-year old breathe once for every 5 compressions doing cpr.

A 46-year-old woman who recently immigrated to the United States is hospitalized on a respiratory isolation unit for multiple drug-resistant pulmonary tuberculosis. What's the appropriate way that the nurse can meet the client's love and belonging needs?

Include her family in her care.

Before administering a medication through a nasogastric (NG) tube, the nurse should do which of the following first?

Inject 10 cc of air into the NG tube and aspirate.

Side effects of thyroid hormones?

Insomnia. Saunders confirms it. Makes sense though!Increased met. rate, your body is "too busy to sleep" as opposed to the folks with hypothyroidism who may report somnolence (dec. met rate, body is slow and sleepy).

The nurse observes a client for signs of distress. Observation is which method of physical examination?

Inspection

Heroin withdrawl in a neonate?

Irritable, and poor sucking

Facts about hemophilia?

It is X-linked. Mother passes the disease to her son

Phenalalanine?

It is an amino acid that is BAD for people with PKU. It is found in food only and is also found in aspartame

How do children less than one breathe?

It is essential to maintain nasal patency with children < 1 yr. because they are obligatory nasal breathers.

GLASGOW COMA SCALE. EYES, VERBAL,MOTOR!

It is similar to measuring dating skills... Max 15 points -one can do it Below 8 you are in Coma.

Prior to liver biopsy?

Its important to be aware of the lab result for prothrombin time

MMR and Varicella?

Just know the MMR and Varicella immunizations come later (15 months).

Dystocia?

Just means a difficult or abnormal delivery or birth... example.. shoulder dystocia

No PEE, No ??

K+; DO NOT give Potassium without adequate urine output.

A client who is dehydrated has urinary incontinence and excoriation in the perineal area. Which of the following actions would be a priority?

Keeping the perineal area clean and dry

MENINGITIS

Kernig's sign (leg flex then leg pain on extension), Brudzinski sign (neck flex = lower leg flex).

Common S/S of Meningitis?

Kernigs Sign (leg flex then leg pain on extension) Brudinzski sign (neck flex= lower leg flex)

Kerning sing

Knee can't staighten during meningitis

DKA

Kussmauls breathing (Deep Rapid RR)

DKA -

Kussmauls breathing (Deep Rapid RR)

Lumbar puncture

Lateral recumbent position, neuro check, position flat 4-12 hours, encourage fluid

intenal radiation ( sealed source)

Lead container, long handed forceps save all dressing, bed lined, urine and feces are not radiative, don't stand close or in line with source . Use dosimeter, limit 30 minutes contact per 8 hrs shift

For PVD remember DAVE

Legs are Dependent forArterial & for Venous Elevated) more to come.

Potassium lab importance?

Let's say every answer in front of you is an abnormal value. If potassium is there you can bet it is a problem they want you to identify, because values outside of normal can be life threatening. Normal potassium is 3.5-5.0. Even a bun of 50 doesn't override a potassium of 3.0 in a renal patient in priority.

Highest priortiy in status elipticus?

Level of consciousness is the most important assessment parameter

Dilantin therapeutic level and s/e

Level: 10-20 S/e: gingival hyperplasia

Alcohol withdraw

Librium.

How to position heat stroke patient?

Lie flat with legs elevated

Common S/S of leprosy?

Lioning face

developmental dyplasia

Look for folds and creases that are longer and deeper on affected side, limited abduction, limb with decrease and no internal rotation

Hypoglycemia

Low blood glucose level (lower than 70 mg/dL) that results from too much insulin, not enough food, or excess activity.

More labs suggestive of renal failure?

Low magnesium and high creatinine signal

Are narcotics same as surgery in cesareans?

Lower strength narcotics than surgeries are used.

MRSA and VRSA precautions?

MRSA - Contact precaution ONLY VRSA - Contact AND airborne precaution (Private room, door closed, negative pressure)

HyperKalemia?

MURDER - muscle weakness, urine (oliguria/anuria), respiratory depression, decreased cardiac contractility, ECG changes, reflexes

Common s/s of PDA

Machine like murmur

Macro and microvascular complications from diabetes mellitus.

Macrovascular = coronary artery disease, cardiomyopathy, hypertension, cerebrovascular disease, and peripheral vascular disease. Microvascular = retinopathy, nephropathy, and neuropathy.

Head injury

Maintain head at 30 degree, give mannitol and lasix, never lay them flat. Seizure precaution

Cane teaching

cane on good side

When patient is in distress?

Med administration is rarely a good choice

Carafad

Medication for HH give 1 hour before food or 2 hours after food

Otitis media can lead to

Meningitis

While examining a client's leg, the nurse notes an open ulceration with visible granulation tissue in the wound. Until a wound specialist can be contacted, which of the following types of dressings is most appropriate for the nurse to apply?

Moist sterile saline gauze

Sources of nutrition:managing potassium level

Most of the extra potassium eaten in the diet is removed by the kidneys -healthy kidneys take in at least 4.7 grams of potassium each day. -kidney disease (not on dialysis) take in 2 to 4 grams of potassium per day -SAFE range of blood potassium levels is 3.5-5.0 mEq/L.

Airborne Transmitted diseases acronym?

My - Measles Chicken - Chicken Pox/Varicella Hez - Herpez Zoster/Shingles TB

The difference between Myasthenia Gravis, Myastenia Crisis, and Cholinergic Crisis?

Myasthenia Gravis: worsens with exercise and improves with rest. Myasthenia Crisis: a positive reaction to Tensilon--will improve symptoms Cholinergic Crisis: caused by excessive medication-stop med-giving Tensilon will make it worse

Blood tests for MI

Myoglobin, CK and Troponin

Myelogram

NPO 4-6 hrs, CNS depressents, phnergan withhold. dye is for this procedure

Cardiac Caricaturization

NPO 8, 12 hours, signed permit, empty bladder, check pulse, explain may experience of feeling palpitation and desire to cough with dye injected

ECT

NPO after midnight, patient will be under anesthesia.

Barium swallow

NPO after midnight, tell stool will be light colored

Indocin

NSAID, take with food

Normal Hemoglobin?

Neonates 18-27 3 mos 10.6-16.5 3 yrs 9.4-15.5 10 yrs 10.7-15.5

Jews?

No meat and milk together

Suctioning is good--except

No nasotracheal suctioning with head injury or skull fracture.

PKU ?

No phenylalanine with a kid positive for PKU (no meat no dairy no aspartme

What diet for devirticulitis

No seeds in food, high in fiber diet

Nonfat milk?

Nonfat milk reduces reflux by increasing lower esophageal sphincter pressure

s3 heart sound is normal? not?

Normal in CHF from the squishin'.. NOT normal in an MI patient

Myelogram

Npo 4-6hr, allergy hx, phenothiazines, cns depressants, and stimulants withheld 48hr prior, table will be moved to various postions during test. Post- neuro q2-4, water soluble HOB up, oil soluble HOB down, oral analgesics for h/a, encourage po fluids, assess for distended bladder, inspect site.

Myelogram?

Npo 4-6hr, allergy hx, phenothiazines, cns depressants, and stimulants withheld 48hr prior, table will be moved to various postions during test. Post- neuro q2-4, water soluble HOB up, oil soluble HOB down, oral analgesics for h/a, encourage po fluids, assess for distended bladder, inspect site.

BLACKHEADS

OPEN COMEDONES ARE WHAT TYPE OF SKIN ERUPTION?

Eyes?

OU- both eyes OS- left eye OD- right eye ( dominent Right eye- just a tip to remember)

Common S/S of Cataract?

PainLESS vision loss, opacity of lens, blurring of the vision

Glaucoma

Person's peripheral vision decreases, halos, headache, eye pain, Avoid mydriatics( Atropine)

rubella infection

Petechiae that occur on the soft palate are characteristic of ___.

When preparing to administer drugs to a client, the nurse reviews information about which branch of pharmacology that deals with the absorption, distribution, metabolism, and excretion of a drug?

Pharmacokinetics Rationale: Pharmacokinetics deals with drug absorption, distribution, metabolism, and excretion in a living organism. Pharmacodynamics deals with drug action, pharmacotherapeutics deals with drug uses and intended effects, and pharmacognosy deals with the study of natural drug sources.

PARKINSON'S

Pill-rolling tremors

PARKINSON'S -

Pill-rolling tremors

Common S/S of parkinsons?

Pilling rolling tremors

How to put on traction?

Place the apparatus first then place the weight

The nurse identifies the needs of a client with potential health problems during which step of the nursing process?

Planning

*Complications of Mechanical Ventilation:

Pneumothorax, Ulcers

Complications of mechanical ventilation?

Pneumothorax, Ulcers

Post spleenectomy?

Pneumovax 23 gets administered post splenectomy to prevent pneumococcal sepsis.

Diabetes Myelitis

Polyuria, Polydispsia, Polyphagia, Weight change

After Myringotomy?

Position on the side of the AFFECTED EAR after surgery to allow drainage of secretions

Peds positioning for GERD?

Position prone w hob elevated with gerd. In almost every other case, though, you better lay that kid on his back (Back To Sleep - SIDS).

_______ fontanelles closes before _______ fontanelles.

Posterior before anterior

Preload and Afterload?

Preload affects amount of blood that goes to the R ventricle. Afterload is the resistance the blood has to overcome when leaving the heart. calcium channel blockers affect the afterload

Protocol for Airborne Transmission?

Private Room - negative pressure with 6-12 air exchanges/hr Mask, N95 for TB

Protocol for Droplet Precautions?

Private Room or cohort mask

Gout Meds

Probenecid (Benemid), Colchicine, Allopurinol (Zyloprim)

If a blood pressure cuff is too small for a client, blood pressure readings taken with such a cuff may do which of the following?

Produce a false-high measurement Rationale: Using an undersized blood pressure cuff produces falsely elevated blood pressures because the cuff can't record brachial artery measurements unless it's excessively inflated. The sciatic nerve would be damaged by hyperinflation of the blood pressure cuff because the sciatic nerve is located in the lower extremity of the body

Which action would be contraindicated for a client who develops a temperature of 102° F (38.9° C)?

Providing a low-calorie diet

Common S/S of Diptheria?

Psuedo membrane formation

pulmonary edema

Pts w/ ___ experience air hunger, anxiety, and agitation.

When instilling eardrops?

Pull pinna down and back for kids < 3 yrs. when instilling eardrops.

APGAR: P

Pulse - >100/<100/Absent

Brachial Pulse?

Pulse area cpr on infant

PTB means?

Pumonary tuberculosis

blood transfusion reaction

S/S: Itching, hives, fever, chills, facial swelling, wheezing, throat swelling, NAUSEA, VOMITTING, LOWER BACK PAIN, HEMATURIA.===STOP IV , OBTAIN URINE SPECIMEN

Meningeal irriatation>?

S/s nuchal rigidity, positive Brudzinski + Kernig signs and PHOTOPHOBIA too!

ventricular gallop is

S3, occurs after S2 and is caused by a premature rush of blood into a ventricle that is stiff or dilated as a result of heart failure and hypertension.

Hypernatremia? SALT?

SALT! Skin flushed Agitation Low grade fever Thirst

Hemodyalysis

SC catheter or AV fistula, AV check for thrill ad bruit, don't use extremity with HD access for bp. Monitor for hemmorhage

EMOLLIENTS AND MOISTURIZERS

SELF CARE TEACHING FOR A PT WITH PSORIASIS WOULD INCLUDE APPLICATION OF...?

The MMR vaccine is given SQ, IM, or IV

SQ

A school-age child is diagnosed with influenza, and the nurse is teaching the parents about caring for the child. Which statement explains why the child should not receive aspirin or aspirin-containing medications to relieve achiness and fever?

Salicylates have been linked with Reye's syndrome in children.

CYSTIC FIBROSIS

Salty skin

CYSTIC FIBROSIS -

Salty skin

Common S/S of cystic fibrosis?

Salty skin

Common S/S of Intusseption?

Sausage shaped mass , Dance sign (empty portion of RLQ)

Instructions when giving a client nitro

Store meds in a dark container *place tablet under tongue *burning sensation indicates tablet is fresh *position supine with elevated legs to manage hypotension

Parkisons?

RAT: rigidity, akinesia (loss of muscle mvt), tremors. Treat with levodopa.

INTRAVASCULAR DEHYDRATION

RATIONALE BEHIND ADM IV FLUIDS TO A BURNT PATIENT IS...

COPD patients?

REMEMBER: 2LNC or less (hypoxic NOT hypercapnic drive), Pa02 of 60ish and Sa02 90% is normal for them b/c they are chronic CO2 retainers. ...

TOURNIQUET EFFECT

REMOVING JEWELRY FROM AN AFFECTED BURN SITE HELPS IN PREVENTING...

Questions about a halo?

Remember safety first, have a screwdriver nearby.

Common S/S of Thypohiod

Rose spots on abdomen

What disease leads to cardiac valve malfunctions?

Rhematic fever

Rh- mothers receive _____ to protect next baby.

Rhogam

Common S/S of Cholera?

Rice watery stool

A client who received general anesthesia returns from surgery. Postoperatively, which nursing diagnosis takes highest priority for this client?

Risk for aspiration related to anesthesia

While caring for a client who is immobile, the nurse documents the following information in the client's chart: "Turn client from side to back every two hours." "Skin intact; no redness noted." "Client up in chair three times today." "Improved skin turgor noted." Which nursing diagnosis accurately reflects this information?

Risk for impaired skin integrity related to immobility

Medications for Rheumatoid Arthritis: Findings to report while taking prednisone (Deltasone)

Risk for osteoporosis, Insomnia, vertigo, adrenal suppression, delayed wound healing

group A Streptococcus

Scarlet fever is a ___ bacteria infection.

When to test urine for ketones and glucose?

Second voided urine most accurate

Willam's position?

Semi Fowlers with knees flexed (inc. knee gatch) to relieve lower back pain.

* William's position

Semi Fowlers with knees flexed (inc. knee gatch) to relieve lower back pain.

Awesome remembering for cranial nerves?

Sensory=S Motor=M Both=B Oh (Olfactory I) Some Oh (Optic II) Say Oh (Oculomotor III) Marry To (Trochlear IV) Money Touch (Trigeminal V) But And (Abducens VI) My Feel (Facial VII) Brother A (Auditory VIII) Says Girls (Glossopharyngeal IX) Big Vagina (Vagus X) Bras And (Accessory XI) Matter Hymen (Hypoglassal XII) More

Besides meds and congenital problems .. what can lead to decreased preload ?

Sepsis and anaphylaxis (along with the obvious hemorrhaging) reduce circulating volume by way of increased capillary permeability, which leads to reduced preload (volume in the left ventricle at the end of diastole). This is a toughie...think about it.

Position during epidural puncture?

Side lying

Which change is demonstrated when a nurse helps a young mother adjust to the birth of her child?

Situational

Dunlap traction=

Skeletal or skin

A 49-year-old client with acute respiratory distress watches everything the staff does and demands full explanations for all procedures and medications. Which of the following actions would best indicate that the client has achieved an increased level of psychological comfort?

Sleeping undisturbed for 3 hours Rationale: Sleeping undisturbed for a period of time would indicate that the client feels more relaxed, comfortable, and trusting and is less anxious. Decreasing eye contact, asking to see family, and joking may also indicate that the client is more relaxed. However, these also could be diversions.

Common S/S of Liver cirrhoisis?

Spider like varices- Varices can be in stomach, esophagus or the skin! They are just spider/varicose veins! Ithcy on the skin

Tetanus, Hepatitis B, HIV precautions

Standard Precautions

VRSA?

Staph infection that is resistant to Vancomycin treatement! Worse than MRSA

Common S/S of Kawasaki syndrome?

Strawberry tongue

Thyroidectomy

Surgical removal of the thyroid gland to treat persistant hyperthyroidism or thyroid tumors.

TIA?

TIA (transient ischemic attack) mini stroke with no dead brain tissue

Thoracentesis?

Take v.s., shave area around needle insertion, position patient with arms on pillow on over bed table or lying on side, no more than 1000cc at a one time. Post- listen for bilateral breath sounds, v.s., check leakage, sterile dressing.

Thoracentesis prep

Take v.s., shave area around needle insertion, position patient with arms on pillow on over bed table or lying on side, no more than 1000cc at a one time. Post- listen for bilateral breath sounds, v.s., check leakage, sterile dressing.

A parish nurse conducts socialization groups for elderly people, health screening clinics, and home visits. If the nurse prioritizes her programs according to Maslow's hierarchy of basic human needs, on which effort would she work first?

Teaching a class on scheduling and organizing accurate medication usage to prevent underdosing or overdosing

Guthrie test?

Tests for PKU, baby should have eaten source of protein first

Hypoparathyroidism

Tetany, musculat irritablity, positive chvocted sign, positive trausseau sing

No Cantalope?

Thank you, I finally realize why a person shouldn't have cantaloupe before a occult stool test, because cantaloupe is high in vit c and vit c causes a false + for occult blood. Now I just need to figure out why they can't have fish.

Cold stress in a newborn?

The biggest concern is respiratory distress.

alveoli

The collection of fluid in the ___, particularly dangerous because it impedes gas exchange. Common causes of pulmonary edema are increased pulmonary blood pressure or infection of the respiratory system.

why use a protein restricted diet for advanced liver disease?

The liver can not rid the body of ammonia that is made by the breakdown of protein. A protein restricted diet will therefore decrease ammonia production.

Myxedema

The most severe form of hypothyroidism characterized by swelling of the hands, face, feet, and periorbital tissues. At this stage, the disease may lead to coma and death.

Which statement accurately characterizes the Z-track method for I.M. injections?

The needle remains in place for 10 seconds after injection.

Adrenalectomy

The surgical removal of an adrenal gland. Lifelong replacement of glucocorticoids and mineralocorticoids is necessary with a bilateral adrenalectomy. Temporary replacement may be necessary for up to 2 years for a unilateral adrenalectomy.

In a client who had major surgery 5 days ago, which of the following assessment findings would be the best indication of a wound infection?

Thick, yellow wound drainage

Tetraology of Fallot?

Think DROP(child drops to floor or squats) or POSH Defect, septal Right Ventricular hypertrophy Overriding aorts Pulmonary stenosis

FHR patterns in ob?

Think VEAL CHOP V C E H A O L P V = variable decels; C = cord compression caused E = early decels; H = head compression caused A = accels; O = okay, not a problem! L = late decels = placental insufficiency, can't fill

Thyroid storm and myxedema?

Thyroid storm is HOT (hyperthermia) Myxedema coma is COLD (hypothermia)

Nursing care for a client includes removing elastic stockings once per day. What's the rationale for this intervention?

To permit veins in the legs to fill with blood

Signs observed in hypocalemia?

Trousseau and Tchovoski signs observed in hypocalcemia

An occupational health nurse has a client who recently became blind from an industrial accident and has been receiving therapy to help him work through the loss and grief process. Which behavior by the client would most suggest that the therapy is having its intended effect?

Turning to his family, friends, and caregivers for support

METHOTREXATE

USED IN SEVERE PSORIASIS

Cryptoorchidism?

Undescended testis is a known risk factor for testicular cancer later in life. Start teaching boys testicular self exam around 12, because most cases occur during adolescence.

To prevent dumping syndrome?

Unusual positional tip - Low-fowlers recommended during meals to prevent dumping syndrome. Limit fluids while eating.

Where should placenta be?

Upper part of the uterus

Use of cold and hot?

Use cold for acute pain (sprained ankle) and heat for chronic pain (arthritis)

Vertical C section?

Used in plus sized women or risky or emergency c sections. less chance of harm for the baby

WART

VIRUS INDUCED EPIDERMAL TUMOR THAT MAY REAPPEAR

CHICKEN POX

Vesicular Rash (central to distal) dew drop on rose petal

Common S/S of chicken pox?

Vesicular rash (central to distal) dew drop on rose petal

RETINAL DETACHMENT

Visual Floaters, flashes of light, curtain vision

Cystoscopy

Visualization of the bladder, bowel prep, force fluids, NPO, urine is pink

NO _____ with allopurinol

Vitamin C

Shift to the left means?

WBC shift to the left in a patient with pyelonephritis (neutrophils kick in to fight infection)

DECREASED SODIUM INCREASED POTASSIUM

WHAT ELECTROLYTE ABNORMALITIES WOULD THE NURSE EXPECT TO OCCUR WHILE WORKING WITH A PATIENT WITH PARTIAL AND FULL THICKNESS BURNS?

LIGHT SKINNED PPL WHO WORK INDOORS

WHAT GROUP OF PEOPLE ARE MOST AT RISK FOR DEV MALIGNANT MELANOMA?

Before a transesophageal echocardiogram, a client is given an oral topical anesthetic spray. Upon return from the procedure, the nurse observes that the client has no active gag reflex. In response, the nurse should:

Withhold food and fluids.

The nurse is to give a client a 325-mg aspirin suppository. The client has diarrhea and is in the bathroom. The best nursing approach at this time would be to:

Withhold the suppository and notify the client's physician.

Cardiopulmonary arrest implemnatian

Witness event defibrilate first, No witness shout for help, star cpr and then defibrillate. For child star compression first

Renal biopsy

X rays taken, position pron, hold breath during procedure, flat in bed for 41 hrs, pressure on site for 20 minutes

What can't you give to immunosupressed pts?

Yogurt - it has live cultures

Whats petaling?

You can petal the rough edges of a plaster cast with tape to avoid skin irritation.

scarlet fever

___ is caused by infection w/ group A Streptococcus bacteria, child will have a "strawberry tongue" which is a characteristic sign and the pharynx will be red and swollen.

stridor

___ is noisy breathing caused by laryngeal swelling or spasm and is not assoc w/pulmonary edema.

MS?

a chronic, progressive disease with demyelinating lesions in the CNS which affect the white matter of the brain and spinal cord. Motor S/S: limb weakness, paralysis, slow speech Sensory S/S: numbness, tingling, tinnitus Cerebral S/S: nystagmus, ataxia, dysphagia, dysarthria

The wife of a 67-year-old client who has been taking imipramine (Tofranil) for 3 days asks the nurse why her husband isn't better. The nurse should tell the wife: a) "It takes 2 to 4 weeks before the full therapeutic effects are experienced." b) "Your husband may need an increase in dosage." c) "A different antidepressant may be necessary." d) "It can take 6 weeks to see if the medication will help your husband."

a - Imipramine, a tricyclic antidepressant, typically requires 2 to 4 weeks of therapy before the full therapeutic effects are experienced. Because the client has been taking the drug for only 3 days, it is too soon to determine if the current dosage of imipramine is effective. It is also too soon to consider taking another antidepressant.

A client with a bleeding ulcer is vomiting bright red blood. The nurse should assess the client for which of the following indicators of early shock? a) Tachycardia. b) Dry, flushed skin. c) Increased urine output. d) Loss of consciousness.

a - In early shock, the body attempts to meet its perfusion needs through tachycardia, vasoconstriction, and fluid conservation. The skin becomes cool and clammy. Urine output in early shock may be normal or slightly decreased. The client may experience increased restlessness and anxiety from hypoxia, but loss of consciousness is a late sign of shock.

Total parenteral nutrition (TPN) is prescribed for a client who has recently had a significant small and large bowel resection and is currently not taking anything by mouth. The nurse should: a. administer TPN through a nasogastric or gastrostomy tube b. handle TPN using strict aseptic technique c. auscultate for bowel sounds prior to administering TPN d. designate a peripheral intravenous (IV) site for TPN administration

a - TPN is hypertonic, high-calorie, high-protein, intravenous (IV) fluid that should be provided to clients without functional gastrointestinal tract motility, to better meet their metabolic needs and to support optimal nutrition and healing. TPN is ordered once daily, based on the client's current electrolyte and fluid balance, and must be handled with strict aseptic technique (because of its high glucose content, it is a perfect medium for bacterial growth). Also, because of the high tonicity, TPN must be administered through a central venous access, not a peripheral IV line. There is no specific need to auscultate for bowel sounds to determine whether TPN can safely be administered

A nurse is caring for a client with poorly managed diabetes mellitus who has a serious foot ulcer. When she informs him that the physician has ordered a wound care nurse to examine his foot, the client asks why he should see anyone other than this nurse. He states, "It's no big deal. I'll keep it covered and put antibiotic ointment on it." What is the nurse's best response? a) "We're very concerned about your foot and we want to provide the best possible care for you." b) "This is a big deal and you need to recognize how serious it is." c) "This is the physician's recommendation. The wound care nurse will see you today." d) "You could lose your foot if you don't see the wound care nurse."

a - The client's response indicates that he's in denial and needs further insight and education about his condition. Letting the client know that the nurse has his best interests in mind helps him accept the wound-care nurse. Although telling the client that his condition is serious and that the wound care nurse will see him that day are true statements, they're much too direct and may increase client resistance. Telling the client he could lose his foot is inappropriate and isn't therapeutic communication.

A client is scheduled for surgery under general anesthesia. The night before surgery, the client tells the nurse, "I can't wait to have breakfast tomorrow." Based on this statement, which nursing diagnosis should be the nurse's priority? a) Deficient knowledge related to food restrictions associated with anesthesia b) Fear related to surgery c) Risk for impaired skin integrity related to upcoming surgery d) Ineffective coping related to the stress of surgery

a - The client's statement reveals a Deficient knowledge related to food restrictions associated with general anesthesia. Fear related to surgery, Risk for impaired skin integrity related to upcoming surgery, and Ineffective coping related to the stress of surgery may be applicable nursing diagnoses but they aren't related to the client's statement.

A nurse is documenting a variance that has occurred during the shift, and this report will be used for quality improvement to identify high-risk patterns and potentially initiate in-services programs. This is an example of which type of report? a. incident report b. nurse's shift report c. transfer report d. telemedicine report.

a - an incident report, also termed a variance report or occurrence report, is a tool healthcare agencies use to document anything out of the ordinary that results in or has the potential to result in harm to a client, employee, or visitor. These reports are used for quality improvement and not for disciplinary action. They are a means of identifying risks and high-risk patterns and initiating in-service programs to prevent further problems. A nurse's shift report is given by a primary nurse to the nurse replacing him or her by the charge nurse to the nurse who assumes responsibility for continuing client care. A transfer report is a summary of a client's condition and care when transferring clients from one unit or institution to another. A telemedicine report can link healthcare professional immediately and enable nurses to receive and give critical information about clients in a timely fashion.

Acid/ ASH diet?

a diet intended for health and may be prescribed by your physician to control illness or disease. Once your liver has metabolized the food you ingest, it leaves a mineral deposit, known as ash, which can be acidic or alkaline depending on its effect on the pH of your urine. Eating foods that create an acid ash may help to balance your system or create a hostile environment to inhibit bacterial growth. .... Acid/ ash diet is used to BALANCE the acid/alkaline ash in the body... Ash wiill affect the urinary system and even the nervous system. Imbalance will be diet modified

Myesthenia gravis is caused by

a disorder in the transmission of impulses from nerve to muscle cell.

Orthostatis is verfied by?

a drop in presssure with increasing heart rate... Orthostatis= orthostatic hypotension

Before going for Pulmonary Fuction Tests (PFT's),

a pt's bronchodilators will be with-held and they are not allowed to smoke for 4 hrs prior

Before a pft? (pulmonary function test)

a pt's bronchodilators will be with-held and they are not allowed to smoke for 4 hrs prior

STD= gonnorrhea?

a reportable disease

Which of the following client statements indicates that the client with hepatitis B understands discharge teaching? a) "I will not drink alcohol for at least 1 year." b) "I must avoid sexual intercourse." c) "I should be able to resume normal activity in a week or two. d) "Because hepatitis B is a chronic disease, I know I will always be jaundiced."

a) CORRECT ANSWER "I will not drink alcohol for at least 1 year." Reason: It is important that the client understand that alcohol should be avoided for at least 1 year after an episode of hepatitis. Sexual intercourse does not need to be avoided, but the client should be instructed to use condoms until the hepatitis B surface antigen measurement is negative. The client will need to restrict activity until liver function test results are normal; this will not occur within 1 to 2 weeks. Jaundice will subside as the client recovers; it is not a permanent condition.

A nurse preceptor is working with a student nurse who is administering medications. Which statement by the student indicates an understanding of the action of an antacid? a) "The action occurs in the stomach by increasing the pH of the stomach contents and decreasing pepsin activity." b) "The action occurs in the small intestine, where the drug coats the lining and prevents further ulceration." c) "The action occurs in the esophagus by increasing peristalsis and improving movement of food into the stomach." d) "The action occurs in the large intestine by increasing electrolyte absorption into the system that decreases pepsin absorption."

a) CORRECT ANSWER "The action occurs in the stomach by increasing the pH of the stomach contents and decreasing pepsin activity." Reason: The action of an antacid occurs in the stomach. The anions of an antacid combine with the acidic hydrogen cations secreted by the stomach to form water, thereby increasing the pH of the stomach contents. Increasing the pH and decreasing the pepsin activity provide symptomatic relief from peptic ulcer disease. Antacids don't work in the large or small intestine or in the esophagus.

A client is scheduled for an excretory urography at 10 a.m. An order directs the nurse to insert a saline lock I.V. device at 9:30 a.m.. The client requests a local anesthetic for the I.V. procedure and the physician orders lidocaine-prilocaine cream (EMLA cream). The nurse should apply the cream at: a) 7:30 a.m. b) 8:30 a.m. c) 9 a.m. d) 9:30 a.m.

a) CORRECT ANSWER 7:30 a.m. Reason: It takes up to 2 hours for lidocaine-prilocaine cream (EMLA cream) to anesthetize an insertion site. Therefore, if the insertion is scheduled for 9:30 a.m., EMLA cream should be applied at 7:30 a.m. The local anesthetic wouldn't be effective if the nurse administered it at the later times.

A nurse is caring for a client who has a tracheostomy and temperature of 103° F (39.4° C). Which intervention will most likely lower the client's arterial blood oxygen saturation? a) Endotracheal suctioning b) Encouragement of coughing c) Use of a cooling blanket d) Incentive spirometry

a) CORRECT ANSWER Endotracheal suctioning Reason: Endotracheal suctioning removes secretions as well as gases from the airway and lowers the arterial oxygen saturation (SaO2) level. Coughing and using an incentive spirometer improve oxygenation and should raise or maintain oxygen saturation. Because of superficial vasoconstriction, using a cooling blanket can lower peripheral oxygen saturation readings, but SaO2 levels wouldn't be affected.

The nurse is serving on the hospital ethics committee which is considering the ethics of a proposal for the nursing staff to search the room of a client diagnosed with substance abuse while he is off the unit and without his knowledge. Which of the following should be considered concerning the relationship of ethical and legal standards of behavior? a) Ethical standards are generally higher than those required by law. b) Ethical standards are equal to those required by law. c) Ethical standards bear no relationship to legal standards for behavior. d) Ethical standards are irrelevant when the health of a client is at risk.

a) CORRECT ANSWER Ethical standards are generally higher than those required by law. Reason: Some behavior that is legally allowed might not be considered ethically appropriate. Legal and ethical standards are often linked, such as in the commandment "Thou shalt not kill." Ethical standards are never irrelevant, though a client's safety or the safety of others may pose an ethical dilemma for health care personnel. Searching a client's room when they are not there is a violation of their privacy. Room searches can be done with a primary health care provider's order and generally are done with the client present.

Which nursing action is required before a client in labor receives epidural anesthesia? a) Give a fluid bolus of 500 ml. b) Check for maternal pupil dilation. c) Assess maternal reflexes. d) Assess maternal gait.

a) CORRECT ANSWER Give a fluid bolus of 500 ml. Reason: One of the major adverse effects of epidural administration is hypotension. Therefore, a 500-ml fluid bolus is usually administered to prevent hypotension in the client who wishes to receive an epidural for pain relief. Assessing maternal reflexes, pupil response, and gait isn't necessary.

A nurse takes informed consent from a client scheduled for abdominal surgery. Which of the following is the most appropriate principle behind informed consent? a) Protects the client's right to self-determination in health care decision making. b) Helps the client refuse treatment that he or she does not wish to undergo. c) Helps the client to make a living will regarding future health care required. d) Provides the client with in-depth knowledge about the treatment options available.

a) CORRECT ANSWER Protects the client's right to self-determination in health care decision making. Reason: Informed consent protects the client's right to self-determination in health care decision making. Informed consent helps the client to refuse a treatment that the client does not wish to undergo and helps the client to gain in-depth knowledge about the treatment options available, but the most important function is to encourage shared decision making. Informed consent does not help the client to make a living will.

A client has the following arterial blood gas values: pH, 7.30; PaO2, 89 mm Hg; PaCO2, 50 mm Hg; and HCO3-, 26 mEq/L. Based on these values, the nurse should suspect which condition? a) Respiratory acidosis b) Respiratory alkalosis c) Metabolic acidosis d) Metabolic alkalosis

a) CORRECT ANSWER Respiratory acidosis Reason: This client has a below-normal (acidic) blood pH value and an above-normal partial pressure of arterial carbon dioxide (PaCO2) value, indicating respiratory acidosis. In respiratory alkalosis, the pH value is above normal and the PaCO2 value is below normal. In metabolic acidosis, the pH and bicarbonate (HCO3-) values are below normal. In metabolic alkalosis, the pH and HCO3- values are above normal.

A 10-year-old child diagnosed with acute glomerulonephritis is admitted to the pediatric unit. The nurse should ensure that which action is a part of the child's care? a) Taking vital signs every 4 hours and obtaining daily weight b) Obtaining a blood sample for electrolyte analysis every morning c) Checking every urine specimen for protein and specific gravity d) Ensuring that the child has accurate intake and output and eats a high-protein diet

a) CORRECT ANSWER Taking vital signs every 4 hours and obtaining daily weight Reason: Because major complications — such as hypertensive encephalopathy, acute renal failure, and cardiac decompensation — can occur, monitoring vital signs (including blood pressure) is an important measure for a child with acute glomerulonephritis. Obtaining daily weight and monitoring intake and output also provide evidence of the child's fluid balance status. Sodium and water restrictions may be ordered depending on the severity of the edema and the extent of impaired renal function. Typically, protein intake remains normal for the child's age and is only increased if the child is losing large amounts of protein in the urine. Checking urine specimens for protein and specific gravity and daily monitoring of serum electrolyte levels may be done, but their frequency is determined by the child's status. These actions are less important nursing measures in this situation.

A nurse has received change-of-shift-report and is briefly reviewing the documentation about a client in the client's medical record. A recent entry reads, "Client was upset throughout the morning." How could the charting entry be best improved? a) The entry should include clearer descriptions of the client's mood and behavior. b) The entry should avoid mentioning cognitive or psychosocial issues. c) The entry should list the specific reasons that the client was upset. d) The entry should specify the subsequent interventions that were performed.

a) CORRECT ANSWER The entry should include clearer descriptions of the client's mood and behavior. Reason: Entries in the medical record should be precise, descriptive, and objective. An adjective such as "upset" is unclear and open to many interpretations. As such, the nurse should elaborate on this description so a reader has a clearer understanding of the client's state of mind. Stating the apparent reasons that the client was "upset" does not resolve the ambiguity of this descriptor. Cognitive and psychosocial issues are valid components of the medical record. Responses and interventions should normally follow assessment data but the data themselves must first be recorded accurately.

A nurse is facilitating mandated group therapy for clients who have sexually abused children. Children who are victims of sexual abuse are typically: a) from any segment of the population. b) of low socioeconomic background. c) strangers to the abuser. d) willing to engage in sexual acts with adults.

a) CORRECT ANSWER from any segment of the population. Reason: Victims of childhood sexual abuse come from all segments of the population and from all socioeconomic backgrounds. Most victims know their abuser. Children rarely willingly engage in sexual acts with adults because they don't have full decision-making capacities.

A client with chronic heart failure is receiving digoxin (Lanoxin), 0.25 mg by mouth daily, and furosemide (Lasix), 20 mg by mouth twice daily. The nurse instructs the client to notify the physician if nausea, vomiting, diarrhea, or abdominal cramps occur because these signs and symptoms may signal digoxin toxicity. Digoxin toxicity may also cause: a) visual disturbances. b) taste and smell alterations. c) dry mouth and urine retention. d) nocturia and sleep disturbances.

a) CORRECT ANSWER visual disturbances. Reason: Digoxin toxicity may cause visual disturbances (such as, flickering flashes of light, colored or halo vision, photophobia, blurring, diplopia, and scotomata), central nervous system abnormalities (such as headache, fatigue, lethargy, depression, irritability and, if profound, seizures, delusions, hallucinations, and memory loss), and cardiovascular abnormalities (abnormal heart rate and arrhythmias). Digoxin toxicity doesn't cause taste and smell alterations. Dry mouth and urine retention typically occur with anticholinergic agents, not inotropic agents such as digoxin. Nocturia and sleep disturbances are adverse effects of furosemide — especially if the client takes the second daily dose in the evening, which may cause diuresis at night.

Signs of Tetany

cardiac dysrythmias, carpopedal spasm, dysphagia, muscle cramps, numbness of face and extremities, positive Chvostek's sign, positive Trousseau's sign, photophobia, wheezing and dyspnea.

S/S of peritonitis

abdominal rigidity caused by blood inside the tissue, Decrease bowel sounds

Care for total hip replacement

abduction pillows, do not sleep on operated side, do not flex hip more than 45 60, do not elevate the head of the bed more than 45

Hypospadias

abnormality in which urethral meatus is located on the ventral (back) surface of the penis anywhere from the corona to the perineum (remember hypo, low (for lower side or under side)

SSRI's (antidepressants) take

about 3 weeks to work.

fundal height 12-14 weeks

above symphysis

is area is "blanched"

absence of normal red tones = tissue ischemia

Graves disease/ Hyperthyroidism?

accelerated physical and mental function; sensitivity to heat, fine/soft hair

Dopamine

acts as a vasoconstrictor increasing BP and renal perfusion (urine flow)

When using a bronchodilator inhaler inconjuction with a glucocorticoid inhaler; __________.

administer the bronchodilator first.

Test 4 hypersensitivity before

administration of asparginase.

The endocrine glands:

adrenal, hypothalamus, ovaries, pancreas, parathyroid, pituitary, testes, thyroid.

Non stress test is done

after 28 weeks record FHR, client should eat snacks, good result is 2 or more FHT, acceleration of 15 bpm lasting 15 sec to 20

After Endoscopy?

after endoscopy check gag reflex.

Antacids

after meals

Premarin

after menopause estrogen replacement

SARS (severe acute resp syndrome) precautions

airborne + contact (just like varicella)

The person who hyperventilates is most likely to experience respiratory_________.

alkalosis

Guan Baire/ immune disease

altered nerve conduction, paralisis, respiratory depression, Altered of autonomic function( hypotension, tachycardia, flashing,sweating, paralysis)

antibiotics

always check for allergies b4 administering...., Make sure culture and sensitivity has been done before adm. First dose of antibiotic.

What to check children for at age 12 months?

always check lead posioning levels

Compartment syndrome?

an emergency situation. Paresthesias and increased pain are classic symptoms. Neuromuscular damage is irreversible 4-6 hours after onset.

streptokinase

an enzyme produced by some strains of streptococcus that can liquefy blood clots by converting plasminogen to plasmin

Osteomyeltitis?

an infectious bone dz. Give blood cultures and antibiotics, then if necessary surgery to drain abscess.

INtal

an inhaler used to treat allergy induced asthma may cause bronchospasm, think... INto the nasthmatic lung

INtal

an inhaler used to treat allergy induced asthma may cause bronchospasm, think... INto the asthmatic lung

Diff between angina and MI?

angina (low oxygen to heart tissues) = no dead heart tissues. MI= dead heart tissue present.

A patient with vertical c- section will likely have?

another c section with any more kids

Fontanelles?

anterior closes by 18 months. Posterior 6 to 8 weeks.

Anterior fontanelle and Posterior fontanelle close @

anterior fontanelle closes by 18 months. Posterior 6 to 8 weeks

bactrim

antibiotic..dont take if allergic to sulfa drugs...diarrhea common side effect...drink plenty of flu

vistaril

anxiety & also itching ....watch for dry mouth,

Stranger Danger>?!?

anxiety is greatest 7 - 9 months, Separation anxiety peaks in toddlerhood

give prophylactic antibiotic therapy before

any invasive procedure.

Place_____ first then place the _____ when putting traction

apparatus then weight

RLQ

appendicitis, watch for peritonitis

Indocin

arthritis (osteo, rhematoid, gouty), bursitis, and tendonitis.

Kids with HIV?

avoid OPV and Varicella vaccinations (live), but give Pneumococcal and influenza. MMR is avoided only if the kid is severely immunocompromised. Parents should wear gloves for care, not kiss kids on the mouth, and not share eating utensils.

Implementation for sleep apnea

avoid alcohol, weight reduction, continou CPAP, BiPAP, surgery

asthma prevention

avoid allergens, dander, mold, pollen, dust

Implementation for pheochromocytoma

avoid emotional stress, encourage rest, avoid

Supine

avoid hip flex-ion

For the client with pheochromocytoma:

avoid stimuli that can precipitate a hypertensive crisis, such as increased abdominal pressure and vigorous abdominal palpation.

After surgery for an ileal conduit, the nurse should closely assess the client for the occurrence of which of the following complications related to pelvic surgery? a) Peritonitis. b) Thrombophlebitis. c) Ascites. d) Inguinal hernia.

b - After pelvic surgery, there is an increased chance of thrombophlebitis owing to the pelvic manipulation that can interfere with circulation and promote venous stasis. Peritonitis is a potential complication of any abdominal surgery, not just pelvic surgery. Ascites is most frequently an indication of liver disease. Inguinal hernia may be caused by an increase in intra-abdominal pressure or a congenital weakness of the abdominal wall; ventral hernia occurs at the site of a previous abdominal incision.

The nurse has administered aminophylline to a client with emphysema. The medication is effective when there is: a) Relief from spasms of the diaphragm. b) Relaxation of smooth muscles in the bronchioles. c) Efficient pulmonary circulation. d) Stimulation of the medullary respiratory center.

b - Aminophylline, a bronchodilator that relaxes smooth muscles in the bronchioles, is used in the treatment of emphysema to improve ventilation by dilating the bronchioles. Aminophylline does not have an effect on the diaphragm or the medullary respiratory center and does not promote pulmonary circulation.

A client complains of severe abdominal pain. To elicit as much information as possible about the pain, the nurse should ask: a) "Do you have the pain all the time?" b) "Can you describe the pain?" c) "Where does it hurt the most?" d) "Is the pain stabbing like a knife?"

b - Asking an open-ended question such as "Can you describe the pain?" encourages the client to describe any and all aspects of the pain in his own words. The other options are likely to elicit less information because they're more specific and would limit the client's response.

A 10-month-old child with recurrent otitis media is brought to the clinic for evaluation. To help determine the cause of the child's condition, the nurse should ask the parents: a) "Does water ever get into the baby's ears during shampooing?" b) "Do you give the baby a bottle to take to bed?" c) "Have you noticed a lot of wax in the baby's ears?" d) "Can the baby combine two words when speaking?"

b - In a young child, the eustachian tube is relatively short, wide, and horizontal, promoting drainage of secretions from the nasopharynx into the middle ear. Therefore, asking if the child takes a bottle to bed is appropriate because drinking while lying down may cause fluids to pool in the pharyngeal cavity, increasing the risk of otitis media. Asking if the parent noticed earwax, or cerumen, in the external ear canal is incorrect because wax doesn't promote the development of otitis media. During shampooing, water may become trapped in the external ear canal by large amounts of cerumen, possibly causing otitis external (external ear inflammation) as opposed to internal ear inflammation. Asking if the infant can combine two words is incorrect because a 10-month-old child isn't expected to do so.

The mother of a client with chronic undifferentiated schizophrenia calls the visiting nurse in the outpatient clinic to report that her daughter has not answered the phone in 10 days. "She was doing so well for months. I don't know what's wrong. I'm worried." Which of the following responses by the nurse is most appropriate? a) "Maybe she's just mad at you. Did you have an argument?" b) "She may have stopped taking her medications. I'll check on her." c) "Don't worry about this. It happens sometimes." d) "Go over to her apartment and see what's going on."

b - Noncompliance with medications is common in the client with chronic undifferentiated schizophrenia. The nurse has the responsibility to assess this situation. Asking the mother if they've argued or if the client is mad at the mother or telling the mother to go over to the apartment and see what's going on places the blame and responsibility on the mother and therefore is inappropriate. Telling the mother not to worry ignores the seriousness of the client's symptoms.

A client with inflammatory bowel disease is receiving total parenteral nutrition (TPN). The basic component of the client's TPN solution is most likely to be: a) An isotonic dextrose solution. b) A hypertonic dextrose solution. c) A hypotonic dextrose solution. d) A colloidal dextrose solution.

b - The TPN solution is usually a hypertonic dextrose solution. The greater the concentration of dextrose in solution, the greater the tonicity. Hypertonic dextrose solutions are used to meet the body's calorie demands in a volume of fluid that will not overload the cardiovascular system. An isotonic dextrose solution (e.g., 5% dextrose in water) or a hypotonic dextrose solution will not provide enough calories to meet metabolic needs. Colloids are plasma expanders and blood products and are not used in TPN.

rheumatic fever can lead to

cardiac valves malfunctions

The nurse is caring for several mother-baby couplets. In planning the care for each of the couplets, which mother would the nurse expect to have the most severe afterbirth pains? a) G 4, P 1 client who is breastfeeding her infant. b) G 3, P 3 client who is breastfeeding her infant. c) G 2, P 2 cesarean client who is bottle-feeding her infant. d) G 3, P 3 client who is bottle-feeding her infant.

b - The major reasons for afterbirth pains are breast-feeding, high parity, overdistended uterus during pregnancy, and a uterus filled with blood clots. Physiologically, afterbirth pains are caused by intermittent contraction and relaxation of the uterus. These contractions are stronger in multigravidas in order to maintain a contracted uterus. The release of oxytocin when breast-feeding also stimulates uterine contractions. There is no data to suggest any of these clients has had an overdistended uterus or currently has clots within the uterus. The G 3, P 3 client who is breast-feeding has the highest parity of the clients listed, which—in addition to breast-feeding—places her most at risk for afterbirth pains. The G 2, P 2 postcesarean client may have cramping but it should be less than the G 3, P 3 client. The G 3, P 3 client who is bottle-feeding would be at risk for afterbirth pains because she has delivered several children, but her choice to bottle-feed reduces her risk of pain.

A nurse is monitoring a client receiving tranylcypromine sulfate (Parnate). Which serious adverse reaction can occur with high dosages of this monoamine oxidase (MAO) inhibitor? a) Hypotensive episodes b) Hypertensive crisis c) Muscle flaccidity d) Hypoglycemia

b - The most serious adverse reaction associated with high doses of MAO inhibitors is hypertensive crisis, which can lead to death. Although not a crisis, orthostatic hypotension is also common and may lead to syncope with high doses. Muscle spasticity (not flaccidity) is associated with MAO inhibitor therapy. Hypoglycemia isn't an adverse reaction of MAO inhibitors.

Which scenario complies with Health Insurance Portability and Accountability Act of 1996 (HIPAA) regulations? a) Two nurses in the cafeteria are discussing a client's condition. b) The health care team is discussing a client's care during a formal care conference. c) A nurse checks the computer for the laboratory results of a neighbor who has been admitted to another floor. d) A nurse talks with her spouse about a client's condition.

b - To provide interdisciplinary continuity of care, nurses must share relevant information during client care conferences. Nurses discussing information in the cafeteria may be overheard; this indiscretion violates HIPAA regulations. Looking up laboratory results for a neighbor is a HIPAA violation, as is discussing a client's condition with one's spouse.

A nurse is instructing a client with bipolar disorder on proper use of lithium carbonate (Eskalith), the drug's adverse effects, and symptoms of lithium toxicity. Which client statement indicates that additional teaching is required? a. "I can still eat my favorite salty foods." b. "when my moods fluctuate, I'll increase my dose of lithium." c. "a good blood level of the drug means the drug concentration has stabilized." d. "eating too much watermelon will affect my lithium level."

b - a client who states that he'll increase his dose of lithium if his mood fluctuates requires additional teaching because increasing the dose of lithium without evaluating the client's laboratory values can cause serious health problems, such as lithium toxicity, overdose, and renal failure. Clients taking lithium don't need to limit their sodium intake. A low sodium diet causes lithium retention. A therapeutic lithium blood level indicates that the drug concentration has stabilized. The client demonstrates effective teaching by stating his lithium levels will be affected by foods that have a diuretic effect, such as watermelon, cantaloupe, grapefruit juice and cranberry juice.

A 10-month-old child has cold symptoms. The mother asks how she can clear infant's nose. Which of the following would be the nurse's best recommendation? a. use a cool air vaporizer with plain water b. use saline nose drops and then a bulb syringe c. blow into the child's mouth to clear the nose d. administer a nonprescription vasoconstrictive nose spray.

b - although a cool air vaporizer may be recommended to humidify the environment, using saline nose drops and then a bulb syringe before meals and at nap and bed times will allow the child to breathe more easily. Saline helps to loosen secretions and keep the mucous membranes moist. The bulb syringe then gently aids in removing the loosened secretions. Blowing into the child's mouth to clear the nose introduces more organisms to the child. A nonprescription vasoconstrictive nasal spray is not recommended for infants because if the spray in used for longer than 3 days a rebound effect with increased inflammation occurs.

a client comes to the outpatient department complaining of vaginal discharge, dysuria, and genital irritation. Suspecting a sexually transmitted disease (STD), the physician orders diagnostic testing of the vaginal discharge. Which STD must be reported to the public health department. a. bacterial vaginitis b. gonorrhea c. genital herpes d. human papillomavirus (HPV)

b - gonorrhea must be reported to the public health department. Bacterial vaginitis, genital herpes, and HPV aren't reportable diseases

A client has a herniated disk in the region of the third and fourth lumbar vertebrae. Which nursing assessment finding most supports this diagnosis? a. hypoactive bowel sounds b. severe lower back pain c. sensory deficits in one arm d. weakness and atrophy of the arm muscles

b - the most common finding in a client with a herniated lumbar disk is severe lower back pain, which radiates to the buttocks, legs, and feet - usually unilaterally. A herniated disk also may cause sensory and motor loss (such as foot drop) in the area innervated by the compressed spinal nerve root. During later stages, it may cause weakness and atrophy of leg muscles. The condition doesn't affect bowel sounds or the arms.

A home health nurse who sees a client with diverticulitis is evaluating teaching about dietary modifications necessary to prevent future episodes. Which statement by the client indicates effective teaching? a) "I'll increase my intake of protein during exacerbations." b) "I should increase my intake of fresh fruits and vegetables during remissions." c) "I'll snack on nuts, olives, and popcorn during flare-ups." d) "I'll incorporate foods rich in omega-3 fatty acids into my diet."

b) CORRECT ANSWER "I should increase my intake of fresh fruits and vegetables during remissions." Reason: A client with diverticulitis needs to modify fiber intake to effectively manage the disease. During episodes of diverticulitis, he should follow a low-fiber diet to help minimize bulk in the stools. A client with diverticulosis should follow a high-fiber diet. Clients with diverticular disease don't need to modify their intake of protein and omega-3 fatty acids.

The nurse has discussed sexuality issues during the prenatal period with a primigravida who is at 32 weeks' gestation. She has had one episode of preterm labor. The nurse determines that the client understands the instructions when she says: a) "I can resume sexual intercourse when the bleeding stops." b) "I should not get sexually aroused or have any nipple stimulation." c) "I can resume sexual intercourse in 1 to 2 weeks." d) "I should not have sexual intercourse until my next prenatal visit."

b) CORRECT ANSWER "I should not get sexually aroused or have any nipple stimulation." Reason: This client has already had one episode of preterm labor at 32 weeks' gestation. Sexual intercourse, arousal, and nipple stimulation may result in the release of oxytocin which can contribute to continued preterm labor and early delivery. The client should be advised to refrain from these activities until closer to term, which is 6 to 8 weeks later. Telling the client that intercourse is acceptable after the bleeding stops is incorrect and may lead to early delivery of a preterm neonate. The client should not have intercourse for at least 6 weeks because of the danger of inducing labor. There is no indication when the client's next prenatal visit is scheduled.

When planning care for a client with a head injury, which position should the nurse include in the care plan to enhance client outcomes? a) Trendelenburg's b) 30-degree head elevation c) Flat d) Side-lying

b) CORRECT ANSWER 30-degree head elevation Reason: For clients with increased intracranial pressure (ICP), the head of the bed should be elevated to 30 degrees to promote venous outflow. Trendelenburg's position is contraindicated because it can raise ICP. Flat or neutral positioning is indicated when elevating the head of the bed would increase the risk of neck injury or airway obstruction. A side-lying position isn't specifically a therapeutic treatment for increased ICP.

A woman who has recently immigrated from Africa who delivered a term neonate a short time ago requests that a "special bracelet" be placed on the baby's wrist. The nurse should: a) Tell the mother that the bracelet is not recommended for cleanliness reasons. b) Apply the bracelet on the neonate's wrist as the mother requests. c) Place the bracelet on the neonate, limiting its use to when the neonate is with the mother. d) Recommend that the mother wait until she is discharged to apply the bracelet.

b) CORRECT ANSWER Apply the bracelet on the neonate's wrist as the mother requests. Reason: The nurse should abide by the mother's request and place the bracelet on the neonate. In some cultures, amulets and other special objects are viewed as good luck symbols. By allowing the bracelet, the nurse demonstrates culturally sensitive care, promoting trust. The neonate can wear the bracelet while with the mother or in the nursery. The bracelet can be used while the neonate is being bathed, or if necessary and acceptable to the client removed and replaced afterward.

When measuring the fundal height of a primigravid client at 20 weeks' gestation, the nurse will locate the fundal height at which of the following points? a) Halfway between the client's symphysis pubis and umbilicus. b) At about the level of the client's umbilicus. c) Between the client's umbilicus and xiphoid process. d) Near the client's xiphoid process and compressing the diaphragm.

b) CORRECT ANSWER At about the level of the client's umbilicus. Reason: Measurement of the client's fundal height is a gross estimate of fetal gestational age. At 20 weeks' gestation, the fundal height should be at about the level of the client's umbilicus. The fundus typically is over the symphysis pubis at 12 weeks. A fundal height measurement between these two areas would suggest a fetus with a gestational age between 12 and 20 weeks. The fundal height increases approximately 1 cm/week after 20 weeks' gestation. The fundus typically reaches the xiphoid process at approximately 36 weeks' gestation. A fundal height between the umbilicus and the xiphoid process would suggest a fetus with a gestational age between 20 and 36 weeks. The fundus then commonly returns to about 4 cm below the xiphoid owing to lightening at 40 weeks. Additionally, pressure on the diaphragm occurs late in pregnancy. Therefore, a fundal height measurement near the xiphoid process with diaphragmatic compression suggests a fetus near the gestational age of 36 weeks or older.

A nurse is assessing the legs of a client who's 36 weeks pregnant. Which finding should the nurse expect? a) Absent pedal pulses b) Bilateral dependent edema c) Sluggish capillary refill d) Unilateral calf enlargement

b) CORRECT ANSWER Bilateral dependent edema Reason: As the uterus grows heavier during pregnancy, femoral venous pressure rises, leading to bilateral dependent edema. Factors interfering with venous return, such as sitting or standing for long periods, contribute to edema. Absence of pedal pulses and sluggish capillary refill signal inadequate circulation to the legs — an unexpected finding during pregnancy. Unilateral calf enlargement, also an abnormal finding, may indicate thrombosis.

A parent brings a 5-year-old child to a vaccination clinic to prepare for school entry. The nurse notes that the child has not had any vaccinations since 4 months of age. To determine the current evidence for best practices for scheduling missed vaccinations the nurse should: a) Ask the primary care provider. b) Check the website at the Center for Disease Control and Prevention (CDC). c) Read the vaccine manufacturer's insert. d) Contact the pharmacist.

b) CORRECT ANSWER Check the website at the Center for Disease Control and Prevention (CDC). Reason: The CDC is the federal body that is ultimately responsible for vaccination recommendations for adults and children. A division of the CDC, the Advisory Committee on Immunization Practices, reviews vaccination evidence and updates recommendation on a yearly basis. The CDC publishes current vaccination catch-up schedules that are readily available on their website. The lack of vaccinations is a strong indicator that the child probably does not have a primary care provider. If consulted, the pharmacist would most likely have to review the CDC guidelines that are equally available to the nurse. Reading the manufacturer's inserts for multiple vaccines would be time consuming and synthesis of the information could possibly lead to errors.

A 10-year-old with glomerulonephritis reports a headache and blurred vision. The nurse should immediately: a) Put the client to bed. b) Obtain the child's blood pressure. c) Notify the physician. d) Administer acetaminophen (Tylenol).

b) CORRECT ANSWER Obtain the child's blood pressure. Reason: Hypertension occurs with acute glomerulonephritis. The symptoms of headache and blurred vision may indicate an elevated blood pressure. Hypertension in acute glomerulonephritis occurs due to the inability of the kidneys to remove fluid and sodium; the fluid is reabsorbed, causing fluid volume excess. The nurse must verify that these symptoms are due to hypertension. Calling the physician before confirming the cause of the symptoms would not assist the physician in his treatment. Putting the client to bed may help treat an elevated blood pressure, but first the nurse must establish that high blood pressure is the cause of the symptoms. Administering Tylenol for high blood pressure is not recommended.

A primigravid client gives birth to a full-term girl. When teaching the client and her partner how to change their neonate's diaper, the nurse should instruct them to: a) fold a cloth diaper so that a double thickness covers the front. b) clean and dry the neonate's perineal area from front to back. c) place a disposable diaper over a cloth diaper to provide extra protection. d) position the neonate so that urine will fall to the back of the diaper.

b) CORRECT ANSWER clean and dry the neonate's perineal area from front to back. Reason: When changing a female neonate's diaper, the caregiver should clean the perineal area from front to back to prevent infection and then dry the area thoroughly to minimize skin breakdown. For a male, the caregiver should clean and dry under and around the scrotum. Because of anatomic factors, a female's diaper should have the double thickness toward the back. The diaper, not the neonate, should be positioned properly. Placing a disposable diaper over a cloth diaper isn't necessary. The direction of urine flow can't be ensured.

30 seconds

check the normal time for PTT is

diameter is greater than 6 mm.

besides asymmetry, border irregularity, and color variation what is also significant for a melanoma

Who produces insulin?

beta cells of pancreas produce insulin

Hirschsprungs?

bile is lower obstruction, no bile is upper obstruction; ribbon like stools.

After intentionally taking an overdose of amitriptyline (Elavil), a client is admitted to the emergency department. The nurse knows that the activated charcoal given to the client will:

bind with the ingested drug. Rationale: Activated charcoal binds with the drug so that it isn't absorbed. It isn't given to promote vomiting or stimulate bowel motility, and it doesn't neutralize the drug.

Long term use of amphogel

binds to phosphates, increases Ca, robs the bones...leads to increased Ca resortion from bones and WEAK BONES)

Racoon eyes

black eyes because of head injury

Turne's sign

bleeding sign seen on the back

Ca

blood clotting, skeletal muscle contraction, regulate parathyroid and vit D

blood sugar post delivery

blood sugar will decrease secondary to sudden decrease in insulin requirements.

Battle's sign

bluish discoloration behind ears because of head trauma

Amiodorone is effective in

both ventricular and atrial complications.

Hypervolemia?

bounding pulse, SOB, dyspnea, rares/crackles, peripheral edema, HTN, urine specific gravity <1.010; Semi-Fowler's

* IVP requires ______ so they can visualize the bladder better

bowel prep

IVP requires?

bowel prep so they can visualize the bladder better

What happens when phenylalanine increases?

brain problems occur

when phenylalanine increases, __________ occurs.

brain problems occur.

Main hypersensitivity for antiplatelet drugs?

broncospasm (anaphylaxis)

ADDISON'S DISEASE

bronze like skin pigmentation

Common S/S of addisions?

bronze like skin pigmentation

ADDISON'S DISEASE

bronze like skin pigmentation.

implementation for constipation and hemorrhoid

bulk food, fiber, stool softeners, increase fluid intake

36%

burns to head, left arm, and torso

SLE -

butterfly rashes

SLE

butterfly rashes.

funds rises until 36 month of gestation

by 1 cm

A nurse is conducting an initial assessment on a client with possible tuberculosis. Which assessment finding indicates a risk factor for tuberculosis? a) The client sees his physician for a check-up yearly. b) The client has never traveled outside of the country. c) The client had a liver transplant 2 years ago. d) The client works in a health care insurance office.

c - A history of immunocompromised status, such as that which occurs with liver transplantation, places the client at a higher risk for contracting tuberculosis. Other risk factors include inadequate health care, traveling to countries with high rates of tuberculosis (such as southeastern Asia, Africa, and Latin America), being a health care worker who performs procedures in which exposure to respiratory secretions is likely, and being institutionalized.

A client with pneumonia has a temperature of 102.6F (39.2C), is diaphoretic, and has a productive cough. The nurse should include which of the following measures in the plan of care? a. position changes every 4 hours b. nasotracheal suctioning to clear secretions c. frequent linen changes d. frequent offering of a bedpan

c - frequent linen changes are appropriate for this client because of the diaphoresis. Diaphoresis produces general discomfort. The client should be kept dry to promote comfort. Position changes need to be done every 2 hours. Nasotracheal suctioning is not indicated with the client's productive cough. Frequent offering of a bedpan is not indicated by the data provided in this scenario.

4 options for cancer?

chemo, radiation, surgery, allow to die with dignity.

A nurse is helping a physician insert a subclavian central line. After the physician has gained access to the subclavian vein, he connects a 10-ml syringe to the catheter and withdraws a sample of blood. He then disconnects the syringe from the port. Suddenly, the client becomes confused, disoriented, and pale. The nurse suspects an air embolus. She should: a) place the client in a supine position and prepare to perform cardiopulmonary resuscitation. b) place the client in high-Fowler's position and administer supplemental oxygen. c) turn the client on his left side and place the bed in Trendelenburg's position. d) position the client in the shock position with his legs elevated.

c - A nurse who suspects an air embolism should place the client on his left side and in Trendelenburg's position. Doing so allows the air to collect in the right atrium rather than enter the pulmonary system. The supine position, high-Fowler's position, and the shock position are therapeutic for other situations but not for air embolism.

When obtaining the vital signs of a client with multiple traumatic injuries, a nurse detects bradycardia, bradypnea, and systolic hypertension. The nurse must notify the physician immediately because these findings may reflect which complication? a) Shock b) Encephalitis c) Increased intracranial pressure (ICP) d) Status epilepticus

c - When ICP increases, Cushing's triad may develop, which involves decreased heart and respiratory rates and increased systolic blood pressure. Shock typically causes tachycardia, tachypnea, and hypotension. In encephalitis, the temperature rises and the heart and respiratory rates may increase from the effects of fever on the metabolic rate. (If the client doesn't maintain adequate hydration, hypotension may occur.) Status epilepticus causes unceasing seizures, not changes in vital signs.

A client has an episiotomy to widen her birth canal. Birth extends the incision into the anal sphincter. This complication is called: a. a first-degree laceration b. a second-degree laceration c. a third-degree laceration d. a fourth-degree laceration

c - birth may extend an episiotomy incision to the anal sphincter (a third degree laceration) or the anal canal (a fourth degree laceration). A first degree laceration involves the fourchette, perineal skin, and vaginal mucous membranes. A second degree laceration extends to the fasciae and muscle of the perineal body.

The neonoate of a client with type 1 diabetes is at high risk for hypoglycemia. An initial sign the nurse should recognize as indicating hypoglycemia in a neonate is: a. peripheral acrocyanosis b. bradycardia c. lethargy d. jaundice

c - lethargy in the neonate may be seen with hypoglycemia because of a glucose in the nerve cells. Peripheral acrocyanosis is normal in the neonate because of immature capillary function. Tachycardia - not bradycardia - is seen with hypoglycemia. Jaundice isn't a sign of hypoglycemia.

The physician ordered IV naloxone (Narcan) to reverse the respiratory depression from morphine administration. After administration of the naloxone the nurse should: a. check respirations in 5 minutes because naxolone is immediately effective in relieving respiratory depression b. check respirations in 30 minutes because the effects of morphine will have worn off by then c monitor respirations frequently for 4 to 6 hours because the client may need repeated doses of naloxone d. monitor respirations each time the client receives morphine sulfate 10 mg IM

c - the nurse should monitor the client's respirations closely for 4 to 6 hours because naloxone has a shorter duration of action than opioids. The client may need repeated doses of naloxone to prevent or treat a recurrence of the respiratory depression. Naloxone is usually effective in a few minutes; however, its effects last only 1 to 2 hours and ongoing monitoring of the client's respiratory rate will be necessary. The client's dosage of morphine will be decreased or a new drug will be ordered to prevent another instance of respiratory depression.

After discussing asthma as a chronic condition, which of the following statements by the father of a child with asthma best reflects the family's positive adjustment to this aspect of the child's disease? a) "We try to keep him happy at all costs; otherwise, he has an asthma attack." b) "We keep our child away from other children to help cut down on infections." c) "Although our child's disease is serious, we try not to let it be the focus of our family." d) "I'm afraid that when my child gets older, he won't be able to care for himself like I do."

c) CORRECT ANSWER "Although our child's disease is serious, we try not to let it be the focus of our family." Reason: Positive adjustment to a chronic condition requires placing the child's illness in its proper perspective. Children with asthma need to be treated as normally as possible within the scope of the limitations imposed by the illness. They also need to learn how to manage exacerbations and then resume as normal a life as possible. Trying to keep the child happy at all costs is inappropriate and can lead to the child's never learning how to accept responsibility for behavior and get along with others. Although minimizing the child's risk for exposure to infections is important, the child needs to be with his or her peers to ensure appropriate growth and development. Children with a chronic illness need to be involved in their care so that they can learn to manage it. Some parents tend to overprotect their child with a chronic illness. This overprotectiveness may cause a child to have an exaggerated feeling of importance or later, as an adolescent, to rebel against the overprotectiveness and the parents.

The nurse meets with the client and his wife to discuss depression and the client's medication. Which of the following comments by the wife would indicate that the nurse's teaching about disease process and medications has been effective? a) "His depression is almost cured." b) "He's intelligent and won't need to depend on a pill much longer." c) "It's important for him to take his medication so that the depression will not return or get worse." d) "It's important to watch for physical dependency on Zoloft."

c) CORRECT ANSWER "It's important for him to take his medication so that the depression will not return or get worse." Reason: Improved balance of neurotransmitters is achieved with medication. Clients with endogenous depression must take antidepressants to prevent a return or worsening of depressive symptoms. Depression is a chronic disease characterized by periods of remission; however, it is not cured. Depression is not dependent on the client's intelligence to will the illness away. Zoloft is not physically addictive.

Which of the following laboratory findings are expected when a client has diverticulitis? a) Elevated red blood cell count. b) Decreased platelet count. c) Elevated white blood cell count. d) Elevated serum blood urea nitrogen concentration.

c) CORRECT ANSWER Elevated white blood cell count. Reason: Because of the inflammatory nature of diverticulitis, the nurse would anticipate an elevated white blood cell count. The remaining laboratory findings are not associated with diverticulitis. Elevated red blood cell counts occur in clients with polycythemia vera or fluid volume deficit. Decreased platelet counts can occur as a result of aplastic anemias or malignant blood disorders, as an adverse effect of some drugs, and as a result of some heritable conditions. Elevated serum blood urea nitrogen concentration is usually associated with renal conditions.

A nurse is developing a nursing diagnosis for a client. Which information should she include? a) Actions to achieve goals b) Expected outcomes c) Factors influencing the client's problem d) Nursing history

c) CORRECT ANSWER Factors influencing the client's problem Reason: A nursing diagnosis is a written statement describing a client's actual or potential health problem. It includes a specified diagnostic label, factors that influence the client's problem, and any signs or symptoms that help define the diagnostic label. Actions to achieve goals are nursing interventions. Expected outcomes are measurable behavioral goals that the nurse develops during the evaluation step of the nursing process. The nurse obtains a nursing history during the assessment step of the nursing process.

A client who has been diagnosed with gastroesophageal reflux disease (GERD) complains of heartburn. To decrease the heartburn, the nurse should instruct the client to eliminate which of the following items from the diet? a) Lean beef. b) Air-popped popcorn. c) Hot chocolate. d) Raw vegetables.

c) CORRECT ANSWER Hot chocolate. Reason: With GERD, eating substances that decrease lower esophageal sphincter pressure causes heartburn. A decrease in the lower esophageal sphincter pressure allows gastric contents to reflux into the lower end of the esophagus. Foods that can cause a decrease in esophageal sphincter pressure include fatty foods, chocolate, caffeinated beverages, peppermint, and alcohol. A diet high in protein and low in fat is recommended for clients with GERD. Lean beef, popcorn, and raw vegetables would be acceptable.

Which of the following is a priority during the first 24 hours of hospitalization for a comatose client with suspected drug overdose? a) Educate regarding drug abuse. b) Minimize pain. c) Maintain intact skin. d) Increase caloric intake.

c) CORRECT ANSWER Maintain intact skin. Reason: Maintaining intact skin is a priority for the unconscious client. Unconscious clients need to be turned every hour to prevent complications of immobility, which include pressure ulcers and stasis pneumonia. The unconscious client cannot be educated at this time. Pain is not a concern. During the first 24 hours, the unconscious client will mostly likely be on nothing-by-mouth status.

A nurse is performing a psychosocial assessment on a 14-year-old adolescent. Which emotional response is typical during early adolescence? a) Frequent anger b) Cooperativeness c) Moodiness d) Combativeness

c) CORRECT ANSWER Moodiness Reason: Moodiness may occur often during early adolescence. Frequent anger and combativeness are more typical of middle adolescence. Cooperativeness typically occurs during late adolescence.

An anxious young adult is brought to the interviewing room of a crisis shelter, sobbing and saying that she thinks she is pregnant but does not know what to do. Which of the following nursing interventions is most appropriate at this time? a) Ask the client about the type of things that she had thought of doing. b) Give the client some ideas about what to expect to happen next. c) Recommend a pregnancy test after acknowledging the client's distress. d) Question the client about her feelings and possible parental reactions.

c) CORRECT ANSWER Recommend a pregnancy test after acknowledging the client's distress. Reason: Before any interventions can occur, knowing whether the client is pregnant is crucial in formulating a plan of care. Asking the client about what things she had thought about doing, giving the client some ideas about what to expect next, and questioning the client about her feelings and possible parental reactions would be appropriate after it is determined that the client is pregnant.

A man of Chinese descent is admitted to the hospital with multiple injuries after a motor vehicle accident. His pain is not under control. The client states, "If I could be with my people, I could receive acupuncture for this pain." The nurse should understand that acupuncture in the Asian culture is based on the theory that it: a) Purges evil spirits. b) Promotes tranquility. c) Restores the balance of energy. d) Blocks nerve pathways to the brain.

c) CORRECT ANSWER Restores the balance of energy. Reason: Acupuncture, like acumassage and acupressure, is performed in certain Asian cultures to restore the energy balance within the body. Pressure, massage, and fine needles are applied to energy pathways to help restore the body's balance. Acupuncture is not based on a belief in purging evil spirits. Although pain relief through acupuncture can promote tranquility, acupuncture is performed to restore energy balance. In the Western world, many researchers think that the gate-control theory of pain may explain the success of acupuncture, acumassage, and acupressure.

A client was hospitalized for 1 week with major depression with suicidal ideation. He is taking venlafaxine (Effexor), 75 mg three times a day, and is planning to return to work. The nurse asks the client if he is experiencing thoughts of self-harm. The client responds, "I hardly think about it anymore and wouldn't do anything to hurt myself." The nurse should make which judgment about the client? a) The client is decompensating and in need of being readmitted to the hospital. b) The client needs an adjustment or increase in his dose of antidepressant. c) The depression is improving and the suicidal ideation is lessening. d) The presence of suicidal ideation warrants a telephone call to the client's primary care provider.

c) CORRECT ANSWER The depression is improving and the suicidal ideation is lessening. Reason: The client's statements about being in control of his behavior and his or her plans to return to work indicate an improvement in depression and that suicidal ideation, although present, is decreasing. Nothing in his comments or behavior indicate he is decompensating. There is no evidence to support an increase or adjustment in the dose of Effexor or a call to the primary care provider. Typically, the cognitive components of depression are the last symptoms eliminated. For the client to be experiencing some suicidal ideation in the second week of psychopharmacologic treatment is not unusual.

Before an incisional cholecystectomy is performed, the nurse instructs the client in the correct use of an incentive spirometer. Why is incentive spirometry essential after surgery in the upper abdominal area? a) The client will be maintained on bed rest for several days. b) Ambulation is restricted by the presence of drainage tubes. c) The operative incision is near the diaphragm. d) The presence of a nasogastric tube inhibits deep breathing.

c) CORRECT ANSWER The operative incision is near the diaphragm. Reason: The incisions made for upper abdominal surgeries, such as cholecystectomies, are near the diaphragm and make deep breathing painful. Incentive spirometry, which encourages deep breathing, is essential to prevent atelectasis after surgery. The client is not maintained on bed rest for several days. The client is encouraged to ambulate by the first postoperative day, even with drainage tubes in place. Nasogastric tubes do not inhibit deep breathing and coughing.

A child with a poor nutritional status and weight loss is at risk for a negative nitrogen balance. To help diagnose this problem, the nurse anticipates that the physician will order which laboratory test? a) Total iron-binding capacity b) Hemoglobin (Hb) c) Total protein d) Sweat test

c) CORRECT ANSWER Total protein Reason: The nurse anticipates the physician will order a total protein test because negative nitrogen balance may result from inadequate protein intake. Measuring total iron-binding capacity and Hb levels would help detect iron deficiency anemia, not a negative nitrogen balance. The sweat test helps diagnose cystic fibrosis, not a negative nitrogen balance.

A client received chemotherapy 24 hours ago. Which precautions are necessary when caring for the client? a) Wear sterile gloves. b) Place incontinence pads in the regular trash container. c) Wear personal protective equipment when handling blood, body fluids, and feces. d) Provide a urinal or bedpan to decrease the likelihood of soiling linens.

c) CORRECT ANSWER Wear personal protective equipment when handling blood, body fluids, and feces. Reason: Chemotherapy drugs are present in the waste and body fluids of clients for 48 hours after administration. The nurse should wear personal protective equipment when handling blood, body fluids, or feces. Gloves offer minimal protection against exposure. The nurse should wear a face shield, gown, and gloves when exposure to blood or body fluid is likely. Placing incontinence pads in the regular trash container and providing a urinal or bedpan don't protect the nurse caring for the client.

During CBI (continuous bladder irrigation)

catheter is taped to thigh so leg should be kept straight. No other positioning restrictions.

Aspirin

cause Reyes syndrome (encephalopathy) when given to children

coarctation of the aaorta causes?

causes increased blood flow and bounding pulses in the arms

Isoniazid

causes peripheral neuritis

Where are most spinal cord injuries?

cervical or lumbar regions

Nepphrotic syndrome?

characterized by massive proteinuria (looks dark and frothy) caused by glomerular damage. Corticosteroids are the mainstay. Generalized edema common.

Antacids that contain Mag

check for DVT

Menigitis Ck for which sign?

check for Kernig's/ Brudzinski's signs.

Signs to look for in meningitis?

check for Kernig's/ Brudzinski's signs.

Cat scan

check for allergies, warm flush feeling, post procedure fluids encourage, during the procedure the dye is injected

PT

check for anticoagulant time of 12 seconds is this

if patient with NG tube becomes nauseous with decrease flow

check placement, check pH, 0-4 is good

Digoxin

check pulse, less than 60 bpm hold, check dig level & potassium level

Bryant's traction=

children <3y, <35 lbs with femur fx

Pylonephritis

chills, fever, flank pain,malaise,CVA tenderness

Brudzinski sign

chin to chest, change in LOC, seizures

BULIMIA

chipmunk face

Common S/S of Bulimia?

chipmunk face

Alzheimers?

chronic, progressive, degenerative cognitive disorder that accounts for more than 60% of all dementias

MRI-

claustrophobia, no metal, assess pacemaker

MRI?

claustrophobia, no metal, assess pacemaker

Ileastomy

clear stool, avoid nuts, low residue/fiber, change ileastomy 2/4 times a day

autologous blood

comes from self, can be used up the 5 weeks before surgery

More info on intussception?

common in kids with CF. Obstruction may cause fecal emesis, currant jellylike stools (blood and mucus). A barium enema may be used to hydrostatically reduce the telescoping. Resolution is obvious, with onset of bowel movements.

cognitive therapy think

counseling

caput succedaneum= diffuse edema of the fetal scalp that _______the suture lines. Swelling reabsorbs within ______.

crosses and reabsorbs 1 to 3 days

The client is taking risperidone (Risperdal) to treat the positive and negative symptoms of schizophrenia. Improvement of which of the following negative symptoms indicate the drug is effective? a) Abnormal thought form. b) Hallucinations and delusions. c) Bizarre behavior. d) Asocial behavior and anergia.

d - Asocial behavior, anergia, alogia, and affective flattening are some of the negative symptoms of schizophrenia that may improve with risperidone therapy. Abnormal thought form is a positive symptom of schizophrenia. Hallucinations and delusions are positive symptoms of schizophrenia. Bizarre behavior is a positive symptom of schizophrenia.

A client was talking with her husband by telephone, and then she began swearing at him. The nurse interrupts the call and offers to talk with the client. She says, "I can't talk about that bastard right now. I just need to destroy something." Which of the following should the nurse do next? a) Tell her to write her feelings in her journal. b) Urge her to talk with the nurse now. c) Ask her to calm down or she will be restrained. d) Offer her a phone book to "destroy" while staying with her.

d - At this level of aggression, the client needs an appropriate physical outlet for the anger. She is beyond writing in a journal. Urging the client to talk to the nurse now or making threats, such as telling her that she will be restrained, is inappropriate and could lead to an escalation of her anger.

A 56-year-old client is receiving chemotherapy that has the potential to cause pulmonary toxicity. Which of the following symptoms indicates a toxic response to the chemotherapy? a) Decrease in appetite. b) Drowsiness. c) Spasms of the diaphragm. d) Cough and shortness of breath.

d - Cough and shortness of breath are significant symptoms because they may indicate decreasing pulmonary function secondary to drug toxicity. Decrease in appetite, difficulty in thinking clearly, and spasms of the diaphragm may occur as a result of chemotherapy; however, they are not indicative of pulmonary toxicity.

Which of the following is an early symptom of glaucoma? a) Hazy vision. b) Loss of central vision. c) Blurred or "sooty" vision. d) Impaired peripheral vision.

d - In glaucoma, peripheral vision is impaired long before central vision is impaired. Hazy, blurred, or distorted vision is consistent with a diagnosis of cataracts. Loss of central vision is consistent with senile macular degeneration but it occurs late in glaucoma. Blurred or "sooty" vision is consistent with a diagnosis of detached retina.

While assessing a male neonate whose mother desires him to be circumcised, the nurse observes that the neonate's urinary meatus appears to be located on the ventral surface of the penis. The primary health care provider is notified because the nurse suspects which of the following? a. phimosis b. hydrocele c. epispadias d. hydrospadias

d - The condition in which the urinary meatus is located on the ventral surface of the penis, termed hypospadias, occurs in 1 of every 500 male infants. Circumcision is delayed until the condition is corrected surgically, usually between 6 and 12 months of age. Phimosis is an inability to retract the prepuce at an age when it should be retractable or by age 3 years. Phimosis may necessitate circumcision or surgical intervention. Hydrocele is a painless swelling of the scrotum that is common in neonates. It is not a contraindication for circumcision. Epispadias occurs when the urinary meatus is located on the dorsal surface of the penis. It is extremely rare and is commonly associated with bladder extrophy.

A client diagnosed with pain disorder is talking with the nurse about fishing when he suddenly reverts to talking about the pain in his arm. Which of the following should the nurse do next? a) Allow the client to talk about his pain. b) Ask the client if he needs more pain medication. c) Get up and leave the client. d) Redirect the interaction back to fishing.

d - The nurse should redirect the interaction back to fishing or another focus whenever the client begins to ruminate about physical symptoms or impairment. Doing so helps the client talk about topics that are more therapeutic and beneficial to recovery. Allowing the client to talk about his pain or asking if he needs additional pain medication is not therapeutic because it reinforces the client's need for the symptom. Getting up and leaving the client is not appropriate unless the nurse has set limits previously by saying, "I will get up and leave if you continue to talk about your pain."

Nurses teach infant care and safety classes to assist parents in appropriately preparing to take their neonates home. Which statement about automobile restraints for infants is correct? a) An infant should ride in a front-facing car seat until he weighs 20 lb (9.1 kg) and is 1 year old. b) An infant should ride in a rear-facing car seat until he weighs 25 lb (11.3 kg) or is 1 year old. c) An infant should ride in a front-facing car seat until he weighs 30 lb (13.6 kg) or is 2 years old. d) An infant should ride in a rear-facing car seat until he weighs 20 lb and is 1 year old.

d - Until the infant weighs 20 lb and is 1 year old, he should ride in a rear-facing car seat.

A client has refused to take a shower since being admitted 4 days earlier. He tells a nurse, "there are poison crystals hidden in the showerhead. They'll kill me if I take a shower." Which nursing action is most appropriate? a. dismantling the showerhead and showing the client that there is nothing in it b. explaining that other clients are complaining about the client's body odor c. asking a security officer to assist in giving the client a shower d. accepting these fears and allowing the client to take a sponge bath

d - by acknowledging the client's fears, the nurse can arrange to meet the client's hygiene needs in another way. Because these fears are real to the client, providing a demonstration of reality by dismantling the shower head wouldn't be effective at this time.

To prevent development of peripheral neuropathies associated with isoniazid administration, the nurse should teach the client to: a. avoid excessive sun exposure b. follow a low-cholesterol diet c. obtain extra rest d. supplement the diet with pyridoxine (vitamin B6)

d - isoniazid competes for the available vitamin B6 in the body and leaves the client at risk for developing neuropathies related to vitamin deficiency. Supplemental vitamin B6 is routinely prescribed to address this issue. Avoiding sun exposure is a preventative measure to lower the risk of skin cancer. Following a low-cholesterol diet lowers the individual's risk of developing atherosclerotic plaque. Rest is important in maintaining homeostasis but has no real impact on neuropathies.

A 16-year-old academically gifted boy is about to graduate from high school early, because he has completed all courses needed to earn a diploma. Within the last 3 months, he has experienced panic attacks that have forced him to leave classes early and occasionally miss a day of school. He is concerned that these attacks may hinder his ability to pursue a college degree. What would be the best response by the school nurse who has been helping him deal with his panic attacks? a) "It is natural to be worried about going into a new environment. I am sure with your abilities you will do well once you get settled." b) "You are putting too much pressure on yourself. You just need to relax more and things will be alright." c) "It might be best for you to postpone going to college. You need to get these panic attacks controlled first." d) "It sounds like you have a real concern about transitioning to college. I can refer you to a health care provider for assessment and treatment."

d) CORRECT ANSWER "It sounds like you have a real concern about transitioning to college. I can refer you to a health care provider for assessment and treatment." Reason: The client's concerns are real and serious enough to warrant assessment by a physician rather than being dismissed as trivial. Though he is very intelligent, his intelligence cannot overcome his anxiety. In fact, his anxiety is likely to interfere with his ability to perform in college if no assessment and treatment are received. Just postponing college is likely to increase rather than lower the client's anxiety, because it does not address the panic he is experiencing.

A 16-year-old primigravida at 36 weeks' gestation who has had no prenatal care experienced a seizure at work and is being transported to the hospital by ambulance. Which of the following should the nurse do upon the client's arrival? a) Position the client in a supine position. b) Auscultate breath sounds every 4 hours. c) Monitor the vital signs every 4 hours. d) Admit the client to a quiet, darkened room.

d) CORRECT ANSWER Admit the client to a quiet, darkened room. Reason: Because of her age and report of a seizure, the client is probably experiencing eclampsia, a condition in which convulsions occur in the absence of any underlying cause. Although the actual cause is unknown, adolescents and women older than 35 years are at higher risk. The client's environment should be kept as free of stimuli as possible. Thus, the nurse should admit the client to a quiet, darkened room. Clients experiencing eclampsia should be kept on the left side to promote placental perfusion. In some cases, edema of the lungs develops after seizures and is a sign of cardiovascular failure. Because the client is at risk for pulmonary edema, breath sounds should be monitored every 2 hours. Vital signs should be monitored frequently, at least every hour.

A nurse is performing a baseline assessment of a client's skin integrity. What is the priority assessment parameter? a) Family history of pressure ulcers b) Presence of pressure ulcers on the client c) Potential areas of pressure ulcer development d) Overall risk of developing pressure ulcers

d) CORRECT ANSWER Overall risk of developing pressure ulcers Reason: When assessing skin integrity, the overall risk potential of developing pressure ulcers takes priority. Overall risk encompasses existing pressure ulcers as well as potential areas for development of pressure ulcers. Family history isn't important when assessing skin integrity.

Based on a client's history of violence toward others and her inability to cope with anger, which of the following should the nurse use as the most important indicator of goal achievement before discharge? a) Acknowledgment of her angry feelings. b) Ability to describe situations that provoke angry feelings. c) Development of a list of how she has handled her anger in the past. d) Verbalization of her feelings in an appropriate manner.

d) CORRECT ANSWER Verbalization of her feelings in an appropriate manner. Reason: Verbalizing feelings, especially feelings of anger, in an appropriate manner is an adaptive method of coping that reduces the chance that the client will act out these feelings toward others. The client's ability to verbalize her feelings indicates a change in behavior, a crucial indicator of goal achievement. Although acknowledging feelings of anger and describing situations that precipitate angry feelings are important in helping the client reach her goal, they are not appropriate indicators that she has changed her behavior. Asking the client to list how she has handled anger in the past is helpful if the nurse discusses coping methods with the client. However, based on this client's history, this would not be helpful because the nurse and client are already aware of the client's aggression toward others.

On the second postpartum day a gravida 6, para 5 complains of intermittent abdominal cramping. The nurse should assess for: a) endometritis. b) postpartum hemorrhage. c) subinvolution. d) afterpains.

d) CORRECT ANSWER afterpains. Reason: In a multiparous client, decreased uterine muscle tone causes alternating relaxation and contraction during uterine involution, which leads to afterpains. The client's symptoms don't suggest endometritis, hemorrhage, or subinvolution.

A nurse is providing care for a pregnant client in her second trimester. Glucose tolerance test results show a blood glucose level of 160 mg/dl. The nurse should anticipate that the client will need to: a) start using insulin. b) start taking an oral antidiabetic drug. c) monitor her urine for glucose. d) be taught about diet.

d) CORRECT ANSWER be taught about diet. Reason: The client will need to watch her overall diet intake to control her blood glucose level. The client's blood glucose level should be controlled initially by diet and exercise, rather than insulin. Oral antidiabetic drugs aren't used in pregnant clients. Urine glucose levels aren't an accurate indication of blood glucose levels.

LLQ -

diverticulitis , low residue, no seeds, nuts, peas

autonomic neuropathy

damage to nerves supplying the internal body structures that regulate functions such as blood pressure, heart rate, bowel and bladder emptying, and digestion

FVE

decr. NA, lasix, bumex, aldactone, thiazide

Dilaudid SE

decrease BP, orthostatic hypotension, bradycardia

Hypothyroidism /Myxedema

decrease activity level, sens to cold, weigh gain/loss, bradycardia, constipation

hypotension

decrease cardiac output

Morphine other uses then for pain?

decrease preload and after load pressure and cardiac workload

MS

demylination of white matte in brain and spinal canal, charges in vision, sensation, motor function

HyperMg?

depresses the CNS, hypotension, facial flushing, muscle ewakness, absent deep tendon reflexes, shallow respirations, emergency

MG

descending muscle weakness

late adolecence 14-21yrs

develop intimate relationship with person of opposite sex is this not about communication?

Hirschsprungs is dx how?

diagnosed with rectal biopsy looking for absence of ganglionic cells. Cardinal sign in infants is failure to pass meconium, and later the classic ribbon-like and foul smelling stools.

caput succedaneum=

diffuse edema of the fetal scalp that crosses the suture lines. Swelling reabsorbs within 1 to 3 days.

Potassium potentiates

dig toxicity.

Digitoxin, digoxin

digibind

Theophylline increases the risk of

digoxin toxicity and decreases the effects of lithium and Dilantin

Peritoneal dialysis?

does not require that the patient's blood be pumped outside of his body. Instead, the blood is cleaned while still inside the body. The organs in the abdomen are surrounded by the peritoneum, which is a membrane that allows waste products to pass through it. Peritoneal dialysis uses a Tenckhoff catheter to run the dialysis fluid, a sugar solution with some salts, into the abdominal, or peritoneal, cavity. This allows the patient's blood to be filtered without pumping it through a dialysis machine. A Tenkhoff catheter is usually used in peritoneal dialysis

Coomb's test?

don't need to give Rhogam cuz she has antibody only give if negative coombs

Mevacor (anticholesterol) must be given with:

evening meal if it is QD (per day)

Beta blockers

ex. atenolol, decrease excitability of heart, decrease cardiac workload , decrease BP, Do not abruptly stop

HYPERTHYROIDISM/GRAVE'S DISEASE

exopthalmus.

The person who hyperventilates is most likely to

experience respiratory alkalosis.

Informed consent

explaining of treatment and expected results, risks and benefits, possible alternatives, answer to questions, can be withdrawn any time

prone

extension of hip joint

incident report

falls, burns, break in aseptic technique, incorrect sponge count, change in condition, medical legal incident

Crons

fat in stool, all the layer are affected

S/S of decreased RBC

fatigue , pallor,

Hyperparathyroid?

fatigue, muscle weakness, renal calculi, back and joint pain (increased calcium), low Ca, high phosphorus diet

Fat metabolism problem( Hepatobiliry disease)

fatty stool, low fat, high protein diet.

trendenburg

feet elevated for 20 used for shock

Russell traction=

femur or lower leg

Russel Traction =

femur or lower leg instability

Pancreatitis pts?

fetal position, NPO, gut rest, prepare antecubital site for PICC cuz they'll probably be getting TPN/Lipids

More info on lumbar puncture?

fetal postion. post- nuero assess q15-30 until stable, flat2-3hr, encourage fluids, oral anlgesics for headache, observe dressing

Turner's sign

flank grayish blue (turn around to see your flanks) pancreatitis

Turner's sign?

flank grayish blue (turn around to see your flanks) pancreatitis

Treatment for peritonitis

fluids and antibiotics

Push fluids with Allopurinol

flush the uric acid out of system

Push fluids with Allopurinol -

flush the uric acid out of system

Below the knee amputation?

foot of bed elevated for first 24 hours, position prone daily to provide for hip extension.

Morphine sulfate

for Mi used for pain and decrease oxygen demand

Rifampin

for TB, dyes bodily fluids orange

versed

for conscious sedation....watch for respiratory depression & hypotension

Zocor

for hyperlipidemia, take on empty stomach to enhance absorption, report any unexplained musle pain, especially if fever

Zocor,

for hyperlipidemia, take on empty stomach to enhance absorption, report any unexplained musle pain, especially if fever

Levodopa

for parkinsons, contraindicated in pts with glaucoma, avoid B6

Levodopa,

for parkinsons, contraindicated in pts with glaucoma, avoid B6

Sinemet

for parkinsons, contraindicated with MAOI's

Two situations can occur in the endocrine system:

hypersecretion or hyposecretion of hormones from the gland or organ. When hypersecretion occurs, treatment is aimed at blocking the hormone release through medication or surgery. When hyposecretion occurs, treatment is aimed at replacement therapy.

Thyroid gland disorders

hyperthyroidism and hypothyroidism

Addisons?

hypoNa, hyperK, hypoglycemia, dark pigmentation, decreased resistance to stress, fractures, alopecia, weight loss, GI distress

Don't use Kayexalate if patient has

hypoactive bowel sounds

Don't use Kayexalate if patient has

hypoactive bowel sounds.

TETANY

hypocalcemia (+) Trousseau's sign/carpopedal spasm; Chvostek sign (facial spasm).

After removal of the pituitary gland what should you watch for?

hypocortisolism and temporary diabetes insipidus.

Addison

hypoglycemia, hyponatremia, hyperkalemia increase protein and increase carbs and increase sodium. During the crisis the person is confused, you know that you are treating the patient when they are not confused

Ampho B causes

hypokalemia (amongst many other things..gotta premedicate before giving. Pts will most likely get a fever)

chronic alcohol use

hypomagnesium

dopamine (Intropine)

hypotension, shock, low cardiac output, poor perfusion to vital organs...monitor EKG for arrhythmias, monitor BP

Synthroid

hypothyroidism..may take several weeks to take effect...notify doctor of chest pain..take in the AM on empty stomach..could cause hyperthyroidism.

Morphine is contraindicated

in Pancreatitis. It causes spasm of the Sphincter of Oddi. Therefore Demerol should be given.

Depression manifests itself?

in somatic ways, such as psychomotor retardation, gi complaints, and pain.

Bells palsy

inability to close eye, distorted side of the face( asymmetric), speech difficulty. Use eye drops to prevent problems with eye. Use isometric exercise

Post appendectomy

incision must be splinted, take deep breath hold then cough deeply

When discussing the Food Pyramid with a 75-year-old client, the nurse should remember that the pyramid has been modified for older people. Unlike the standard Food Pyramid, the version for elderly individuals:

includes eight 8-oz glasses of water at the base of the pyramid.

When using a bronchodilator inhaler

inconjuction with a glucocorticoid inhaler, administer the bronchodilator first

Hypovolemia?

incrased temp, rapid/weak pulse, increase respiration, hypotension, anxiety, urine specific gravity >1.030

Implementation for leg cramps

increae ca, dorsiflex feet, local feet

hepatitis C

inflammation of the liver caused by the hepatitis C virus (HCV), which is transmitted by exposure to infected blood; this strain is rarely contracted sexually, can be contracted through blood transfusions or needles sticks ie drug users

phelibitis

inflammation of the vein form devices pain, edema warmth and redness, stop and sart on another extremeity, wrap and raise in warm towel for 20 min

Peritonitis

inflammation or rupture of appendix, ectopic pregnancy, trauma

Pancreas

influences carbohydrate metabolism, indirectly influences fat and protein metabolism, and produces insulin and glucagon.

-Multiple Sclerosis

is a chronic, progressive disease with demyelinating lesions in the CNS which affect the white matter of the brain and spinal cord. Motor S/S: limb weakness, paralysis, slow speech Sensory S/S: numbness, tingling, tinnitus Cerebral S/S: nystagmus, ataxia, dysphagia, dysarthria

Med of choice for SVT

is adenosine or adenocard

Med of choice for Vtach

is lidocaine

Positioning with pneaumonia?

lay on the affected side to splint and reduce pain. But if you are trying to reduce congestion the sick lung goes up. (Ever had a stuffy nose, and you lay with themstuff side up and it clears?)

Gerd again?

lay on their left side with the HOB elevated 30 degrees.

Pulmonary sarcoidosis?

leads to right sided heart failure..Sarcidosis is basically scar tissue build up

early adolescence12-14yrs

learn independence and how to relate to opposite sex

juvenile 6-9 years

learn to relate to peers

Discharge teaching for the the hip replacement

maintain abduction, avoid stooping, bending, do not sleep on operated side, never cross legs,resume activities in 3 month

Following thyroidectomy:

maintain the client in a semi-fowler's position. Monitor the surgical site for edema and for signs of bleeding and check the dressing anteriorly and at the back of the neck.

crede manuever

manual pressure on the bladder, helps push out urine

if mother is is 3rd trimesters and experiencing epigastic pain

may be impending convulsion

PCA

may have itching

Carafate (Sulcrafate) before

meals (mucosal barrier; constipation)

First sign of pyloric stenosis in a baby?

mild vomiting that progresses to projectile vomiting. Later you may be able to palpate a mass, the baby will seem hungry often, and may spit up after feedings.

Therapies?

milieu therapy= taking care of patient/environment cognitive therapy= counseling crisis intervention=short term.

Alkaline Ash diet?

milk, veggies, rhubarb, salmon

Glaucoma intraocular pressure is greater than the normal (22 mm Hg), give ...

miotics to constrict (pilocarpine) NO ATROPINE

melanoma

mole gone bad is aka

TB patients are at risk for hepatic dysfunction

monitor for fatigue and dark urine.

Organic Nitrates: Proper use of Nitroglycerin (Nitro-Bid) skin Patch

monitor for othostatic hypotension, apply to area with few hairs, and do not apply near the area for defibrillation to avoid skin burns. *applied once a day, in am, sites rotated *HEADACHE is most common side affect

chronic cocaine use symptom

nasal septum disruption

Hyponatremia?

nausea, muscle cramps, increased ICP, muscular twitching, convulsion; osmotic diuretics, fluids

DMI

never walk bear foot

for the client with hyperparathyroidism:

notify the physician immediately if a precipitous drop in the calcium level occurs; assess for tingling and numbness in the face and extremities and for other signs of hypocalcemia.

Cardiac cath

npo 8-12hr, empty bladder, pulses, tell pt may feel heat palpitations or desire to cough with dye injection. Post- Vital signs keep leg straight bedrest 6-8hr.

Cath lab?

npo 8-12hr, empty bladder, pulses, tell pt may feel heat palpitations or desire to cough with dye injection. Post- Vital signs keep leg straight bedrest 6-8hr.

post myleogram

nurse should encourage fluids to facilitate dye excretion.

When a pt comes in and she is in active labor?

nurses FIRST action is to listen to fetal rate/tone

Yogurt has live cultures- dont give to

o immunosuppressed pt

School aged kids and five year olds?

old enough, and should have an explanation of what will happen a week before surgery such as tonsillectomy.

Common S/S of pyloric stenosis?

olive like mass

PYLORIC STENOSIS

olive like mass.

During internal radiation?

on bedrest while implant in place

non reactive is not good may need c-section

on non stress test of OB pt. if there is no reaction or increase HR during fetal activity then ...

POSt bolus feeding position

on right side, HOB UP

Epispadias:

opening of the urethra on the dorsal (front) surface of the penis

Infancy 0-18 months

others will satisy needs

Risk factor that are associated with osteoporosis

over 60, small frame, Caucasian, Asian, use of alcohol and smoking

Gastroenteral Feedings: Inserting an NG tube

passed through the nares (nostril), down the esophagus and into the stomach. This type of feeding tube is generally used for short term feeding, usually only 2 weeks maximum -prior to feeding assess the bowel sounds and residual content -Check tube placement= Introduce 5-20ml of air into tube & auscultate to hear gurgling. Aspirate gastric content, immerse tip of tube into water, no bubbles must be produced. -Height of feeding =12 in. above pt point of insertion -position fowlers or sitting

PDA?

patent ductuous arteriousus in infants! Valve doesn't close!

Pathological jaundice?

pathological occurs before 24hrs and last7 days. Physiological occurs after 24 hours.

PT/PTT are elevated when

patient is on coumadin

Isoniazid causes

peripheral neuritis

INH can cause____, take______ to prevent also hepatotoxic

peripheral neuritis Vit B6

INH can cause

peripheral neuritis, take Vit B6 to prevent also hepatotoxic

Glaucoma patients loose

peripheral vision. Treated with meds

Pheochromocytoma

persistent hypertention, hypeglycemia, pounding headache, palpitation, visual disturbance, high secretion of cotacalamine

Lung biopsy

person pre medicated but should be awake to help during the procedure, person should be asked to hold the breath

DENGUE

petechiae or (+) Herman's sign.

Common S/S of Dengue

petechiae or + Herman's sign

The nurse inspects a client's back and notices small hemorrhagic spots. The nurse documents that the client has:

petechiae.

s/s of a fat embolism?

petechiae. Treated with heparin.

Dwarfism

pituitary disorder, delicate feature, height below normal, possibly give growth hormone

Acromegally

pituitary disorder. body size enlarged, poor coordination, monitor blood sugar

With lower amputations?

place in prone position

heparin prevents

platelet aggregation.

For a lung biopsy,

position pt lying on side of bed or with arms raised up on pillows over bedside table, have pt hold breath in midexpiration, chest x-ray done immediately afterwards to check for complication of pneumothorax, sterile dressing applied

For a lung biopsy?

position pt lying on side of bed or with arms raised up on pillows over bedside table, have pt hold breath in midexpiration, chest x-ray done immediately afterwards to check for complication of pneumothorax, sterile dressing applied

What to do in the case of tube feeding with decreased LOC?

position pt on right side (promotes emptying of the stomach) with the HOB elevated (to prevent aspiration)

Thoracentisis

position the patient with arms over the bed or lying side

dumping syndrome prevention

post gastrectomy, no fluids 1h before, with, or 2 hr after a meal. should decrease intake of carbs since they are the first digested. undigested food is dumped into the jejunum resulting in distention, cramping, pain, diarrhea 15-30 mins after eating, causes diaphoresis, diarrhea, and hypotension. 5-6 small meals, increase fat and proteins, lie down after eating to delay gastric emptying time, avoid high fiber foods

tigan

postop, & for nausea associated with gastroenteritis

Complication of amniocentesis

premature labor, infection, abruption of placenta, RH immunization

Protonix is given prophylactically to

prevent stress ulcers.

Fluid Imbalances: Recognizing fluid volume Deficit

primarily fluid deficit= (hypertonic)more water than salt is being lost( diuretic use, infections, fever, diabetes inspidus) *Primarily sodium deficit( hypotonic)= more salt than water is being lost ( diuretics, or salt-wasting renal disease *combined 50-50= isotonic, (diuretics, prolonged diarrhea or vomiting

patient bills of right

privacy, confidentiality, respect care, current information

patient in mania.

private room and eat in room

# 1 sign of surgically induced hypothyroidism

progressive weight gain

Pyloric stenosis

projecting vomiting,Give IV fluids, NG tube to decompress, monitor hydration, hypokalemia

Amputation immediate care

prosthesis filling, rigid dressing help prevent bleeding, observe for bleeding, if blood stains appear, mark the area and observe every 10 min

heparin antidote

protamine sulfate

Common S/S of down syndrome

protruding tongue

DOWN SYNDROME

protruding tongue.

Hydrocele?

provide ice bags and scrotal support.

Implementation for Alzemers'

provide safety, reduce anxiety and agitation, improve communication, promote independence, good nutrition balance activity and rest.

DIPTHERIA

pseudo membrane formation

DIPTHERIA -

pseudo membrane formation

For a lumbar puncture?

pt is positioned in lateral recumbent fetal position, keep pt flat for 2-3 hrs afterwards, sterile dressing, frequent neuro assessments

Signs of hypoxia

restless, anxious, cyanotic tachycardia, increased resps. (also monitor ABG's)

Basophils

release histamine during an allergic response.

For PVD remember?

remember DAVE (Legs are Dependent forArterial & for Venous Elevated)

hypo CA

renagel phoslo os-ca iv CA put on herat monitor

Complication after hip

replacement, dislocation of prothesis, excessive wound drainage, thromboembolis, infection

The nurse is evaluating a client who is complaining of shortness of breath. The client's respiratory rate is 26 breaths/minute, so the nurse documents that he's tachypneic. The nurse understands that tachypnea means

respiratory rate greater than 20 breaths/minute

Eating Disorders : Understanding Bulemia:

s/s Repeatedly eating large amounts of food in a short amount of time -purging, fasting, exercising to much, misusing laxatives, diuretics, &enemas -feeling ashamed of how much your eating, hides food, does not eat around others, irregular menstrual cycles *low levels of potassium, dry skin, tooth decay, mouth sores, bloating, BUT= stay with in normal weight range

Nephrotic syndrome?

s/s edema + hypotension. Turn and reposition (risk for impaired skin integrity)

Mono

s/s fatigue, flu-like symptoms, sore throat, don't share drinks

More info on droplet precautions?

sepsis, scarlet fever, streptococcal pharyngitis, parovirus B19, pnuemonia, pertusis, influenza, diptheria, epiglottis, rubella, mumps, meningitis, mycoplasma and adenovirus. Door open, 3 ft distance, private room or cohort, mask

SARS precautions?

severe acute resp syndrome - airborne + contact (just like varicella)

crisis intervention

short term.

ICP?

should be <2. measure head circonference.

compazine administration

should be drawn in separate syringe

Dunlap traction=

skeletal or skin

You can petal the rough edges of a plaster cast with tape to avoid .

skin irritation

Uremic fetor

smell urine on the breath

Sleep apnea syndrome

snores loudly, daytime sleepness, morning headache

Infant

solitary play

examples of antipyschotics

sparine navane, haldol

If you see a nurse make a mistake? Chain of command?

speak to her before going to management. If the situation persists, then take it higher.

LIVER CIRRHOSIS

spider like varices.

liver biopsy position

supine with arms raised above head. needle goes through two lower ribs.

bethamethasone (celestone)

surfactant. Med for lung expansion.

Asthma and arthritis best excercise?

swimming

How to treat phobic disorders?

systematic desensitization

Glomerulonephritis?

take vs q 4 hrs + daily weights

tagament

take with food

Shilling test?

test for pernicious anemia/ how well one absorbs Vit b12

Hba1c?

test to assess how well blood sugars have been controlled over the past 90-120 days. 4- 6 corresponds to a blood sugar of 70-110; 7 is ideal for a diabetic and corresponds to a blood sugar of 130.

Guthrie Test

tests for PKU, baby should have eaten source of protein first

dysplasia of hip

uneven gluteal folds and thing, limited abduction of hip, ortolani's sign, shortened limb on affected side , use pavlik harness thatshould be changed regarding the child growth

Placenta should be in the _______ part of the uterus

upper

Knee replacement?

use continuous passive motion machine.

pinworm specimen sampling

use tape around anus in AM

Hemovac

used after mastectomy, empty when full or q8hr, remove plug, empty contents, place on flat surface, cleanse opening and plug with alcohol sponge, compress evacuator completely to remove air, release plug, check system for operation.

Hemovac?

used after mastectomy, empty when full or q8hr, remove plug, empty contents, place on flat surface, cleanse opening and plug with alcohol sponge, compress evacuator completely to remove air, release plug, check system for operation.

Dexedrine

used for ADHD, may alter insulin needs, avoid taking with MAOI's, take in morning (insomnia possible side effect)

Dexedrine

used for ADHD, may alter insulin needs, avoid taking with MAOI's, take in morning (insomnia possible side effect)

Diamox

used for glaucoma, can cause hypokalemia

Diamox,

used for glaucoma, can cause hypokalemia

Cytovene

used for retinitis caused by cytomegalovirus, pt will need regular eye exams, report dizziness, confusion, or seizures immediately

Sengstaken blakemore tube

used for tx of esophageal varices, keep scissors at bedside

Sengstaken blakemore tube

used for tx of esophageal varices, keep scissors at bedside.

pitocin med used for

uterine stimulation

Prevention for Meningitis is

vaccination

c02 builds up and causes?

vasoconstriction.

Immunizations rules?

vastus lateralis is IM administration site for 6month infants For toddlers above 18 months ventrogluteal The deltoid and gluteus maximus are appropriate sites for children

age : 6mths, 18mths above, children location of injection

vastus lateralis is IM administration site for 6month infants For toddlers above 18 months ventrogluteal The deltoid and gluteus maximus are appropriate sites for children

Continue CPR until

victim response, someone else takes over, victim is transferred to to ther unit, rescuer is unable to continue

IV pylogram

visualization of renal structure, dye is used

Coumadin antidote

vitamin k

Informed consent should be

voluntary, understandable terms, client cannot give consent if the client is drinking or is pre medicated, minor can 't give consent

After cast care removal

wash area gently, apply baby powder, cornstarch, elevate limb, apply elastic bandage

Best indicator of dehydration?

weight

peronial dialysis

weight befor and after, monitor BP, monitor BP, monitor B sounds, us sterile technique. if problem with outflow reposition the client

cerebral angio prep-

well hydrated, lie flat, sire shaved, pulses marked post- keep flat 12-14hr, check site, pulses,force fluids

Cerebral angio prep?

well hydrated, lie flat, sire shaved, pulses marked post- keep flat 12-14hr, check site, pulses,force fluids.

18%

what % of burn for front chest/torso

PTT is prolonged and PT is normal

what are coagulate findings for hemophilia

A child with a ventriculoperitoneal shunt

will have a small upper-abdominal incision. This is where the shunt is guided into the abdominal cavity, and tunneled under the skin up to the ventricles. You should watch for abdominal distention, since fluid from the ventricles will be redirected to the peritoneum. You should also watch for signs of increasing intracranial pressure, such as irritability, bulging fontanels, and high-pitched cry in an infant. In a toddler watch lack of appetite and headache. Careful on a bed position question! Bed-position after shunt placement is flat, so fluid doesn't reduce too rapidly. If you see s/s of increasing icp, then raise the hob to 15-30 degrees.

Post Cholecystectomy

will have bile drainage.

How will CSF look in meningitis?

will have high protein, and low glucose.

Cardiac Catherization ( R-sided and L-sided)

with R side cardiac cath=look for valve problems with L side in adults look for coronary complications

Amphogel and Renegal take

with meals


Related study sets

Section 1.4 Credit and Debt (Borrowing)

View Set

Harry Potter, All about class of 2026, Spanish Learning

View Set

Periodic Table Choice Board - Who Am I?

View Set

FUNDS EAQ Exam 2 practice questions

View Set

Physical Science Ch.15 Sections 1 & 2

View Set

California Real Estate Law Ninth Edition 2019 Unit 5-8

View Set

Legal and Ethical Responsibilities Test

View Set

Astronomy Terms 1: History of Astronomy

View Set